BUSINESSMATHEMATICS
BUSINESSMATHEMATICS
net/publication/368586054
Business Mathematics
CITATIONS READS
0 1,345
1 author:
Keshab Bhuyan
Jahangirnagar University
242 PUBLICATIONS 483 CITATIONS
SEE PROFILE
All content following this page was uploaded by Keshab Bhuyan on 17 February 2023.
1
Business Mathematics
K.C.Bhuyan
2
Preface
Mathematics typically used in commerce and industry includes elementary arithmetic, elementary
algebra, statistics, and probability. These mathematical tools are important for business management and
commercial enterprises use these to record and manage business operations. Beside these mathematics ,
some more advanced mathematical tools, viz. calculus, matrix algebra, linear programming are used by
the commercial organizations. The organizations use higher mathematics in accounting, inventory
management, marketing, sales forecasting, and financial analysis.
These mathematical tools all together are the topics of Business Mathematics. It is also called
commercial mathematics or consumer mathematics which is used as a group of practical subjects used in
commerce and everyday life. The emphasis in this topic is on computational skills and predominant
application in practical field.
Whatever be the level or depth of the above mentioned topics of mathematics, these are indispensable part
of scientific development of business and economics. The subject , therefore, is very essential to the
planners of industrial, economics and commercial development. Considering its importance, some of the
topics used in business and commerce are introduced in the syllabus of higher secondary certificate
examination for the students studying in the field of commerce. Some higher mathematical tools used in
commerce are taught to the graduating students of the Faculty of Business Administration and to the
students of Economics.
Considering the importance and need of the students to be involved in commerce and industry, in future,
the topics function, limit, continuity, set theory, probability, determinants, matrix algebra, solution of
equation(s), etc. are included in different chapters of this book. The topics calculus and linear algebra are
also included in two separate chapters. Some aspects of compounding, discounting and annuity are also
included in three different chapters. These latter topics are the topics of financial mathematics. In total,
there are 12 chapters in the book.
In preparing the book, I have consulted the book of many foreign authors. I am indebted to all of them. I
feel confident that the book will meet the need of graduating students in the field of commerce and
Economics. My attempt will be successful if the book is helpful to the students and researchers in the
field of Economics, commerce and industry. I am thankful to Professor Khondoker Abdul Maleque for his
invalueable suggestion in preparing the manuscript of the book.
Any suggestion from any of the user of the book and of the expert will be cordially entertained to improve
and modify the book.
K.C.Bhuyan
3
Contents
Chapter 2: Determinant
2.1: Introduction
2.2: Minor and Cofactor of Determinant
2.3: Adjoint of a determinant
2.4: Properties of Determinant
2.4.1: Product of Two Determinants
2.4.2: Reciprocal of a Determinant whose Value is not Zero
2.5: Further Application of Determinant
Supplementary Problems of Determinant.
Chapter 3: Matrix
3.1: Introduction
3.2: Different Types of Matrices
3.3: Elementary Operation of Matrix
3.4: Matrix Operation
3.4.1: Addition and subtraction of Two Matrices
3.4.2: Equality of Two Matrices
3.4.3: Properties of Matrix Addition
3.4.4: Multiplication of a Matrix by a Scaler
3.4.5: Multiplication of Matrices
3.4.6: Properties of Matrix Multiplication
3.4.7: Transpose of a Matrix
3.4.8: Properties of Transpose of a Matrix
3.4.9: Rank of a Matrix
3.4.10: Matrix Inverse
3.5: Solution of System of Equations Using Matrix
Supplementary Problems of Matrices.
Chapter 4: Solution of Equations
4.1: Introduction
4.1.1: Linear Equations
4.1.2: Quadratic Equations
4.1.3: The Relation between Roots and Coefficients of Quadratic Equations
4.1.4: Formation of Quadratic Equations
4.1.5: Conditions of Common Roots in Two Quadratic Equations
4.2: Cubic Equations
4.2.1: Nature of the Roots of Cubic Equations
4.3: Simultaneous Equations
4
Chapter 5: Methods of Counting
5.1: Introduction
5.2: Factorial
5.3: Permutation
5.4: Combination
5.4.1: Combination with Repetition of Objects
5.5: Supplementary Problems
Chapter 6: Probability
6.1: Introduction
6.2: Terms Related to Probability
6.3: Definition of Probability
6.4: More Terms Related to Probability
6.5: Examples on Probability
6.6: Laws of Probability
6.7: Bayes Theorem
6.8: More Examples on Probability
Supplementary Problems
Chapter 7: Compounding
7.1: Definition
7.2: Compound Interest Payable at Different Time Periods
7.3: Examples on Compound Interest and Simple Interest
Supplementary Problems
Chapter 8: Annuity
8.1: Annuity
8.2: Life Annuity
8.2.1: Breaking Down Life Annuity
8.3: Amount of Annuity
8.4: Present Value of an Annuity
8.5: Sinking Fund
8.5.1: Advantages and Disadvantages of Sinking Fund
8.5.2: Doubling Option
8.6: Deferred Annuity
8.6.1: Deferred Perpetuity
8.6.2: Amount of an Annuity Due
8.6.3: Present Value of an Annuity Due
8.6.4: Amortization
8.7: Miscellaneous Examples
Supplementary Problems
Chapter 9: Discounting
9.1: Discounting
Supplementary Problems
Chapter 10: Geometry
10.1: Introduction
10.1.1: Coordinate System
10.1.2: Cartesian Coordinate System
10.1.3: Polar Coordinate System
10.1.4: Spherical Coordinate System
10.1.5: Relation Between Cartesian and Polar Coordinate
10.1.6: Distance Between Two Points
ii
5
10.2: Equation of Straight Line
10.2.2: Conditions for Parallel and Perpendicular of Two Lines
10.3: Circle
10.3.1: Analytical Results Related to Circle
Supplementary Problems.
Chapter 11: Calculus
11.1: Introduction
11.2: Differential Calculus
11.3: Integral Calculus
11.4: Rules for Differentiating Functions and Some Standard Results of Derivatives
11.5: Differentiation
11.6: Techniques of Differentiation
11.7: Applications of Rules of Differentiation
11.8: Derivative of Implicit Functions
11.9: Derivative of Parametric Functions
11.10: Successive Differentiation
11.11: Maxima and Minima
11.12: Optimization Problem
11.13: More Examples on Optimization
Supplementary Problems
11.14: Integration
11.15: Some Important Results of Integration
11.16: Examples on Integration
11.17: Practical Applications of Integration
11.18: Definite Integral [ Area under a Curve]
Supplementary Problems
Chapter 12: Linear Programming
12.1: Definition
12.2: Graphs of System of Linear Programming
12.3: Different Areas of Applications of Linear Programming Problems [ LPP]
12.4: Formulation LPP
12.5: Method of Solving Linear Programing Problems
Supplementary Problems.
iii
6
Chapter – 1
Mathematical Preliminaries
1.1 Introduction
Business mathematics is the application of mathematical tools in handling data collected either
by census or by sample survey from the population units related to business and economics. The
data are collected from a randomized study and these are analysed to establish or to verify any
hypothesis regarding the unknown characteristics of the population units. The analysis is based
on mathematical procedures. The mathematical procedures involve number, function, limit,
continuity and different other topics of algebra and calculus. In this chapter some preliminary
topics related to algebra and calculus are discussed.
1.2 Number
Number theory is a branch of pure mathematics devoted primarily to the study of the integers.
Originally number theorists study prime numbers as well as the properties of objects made out
of integers ( e.g. rational number ) or defined as the generalization of the integers (e.g. algebraic
integers viz. 1 , 2 , 3, …….). Integers can be considered either in themselves or as solutions to
equations.
The integers are, say a , b, and c, such that a2+b2 = c2 gives an identity. Another identity is of the
form
1 1 1 1
[ ( x )] 2 1 [ ( x )] 2
2 x 2 x
c
The triplets a , b, c are first constructed and then reordered by for actual use.
a
The task of arithmetic is to find rational solutions to the system of polynomial equations, usually
of the form
f(x, y) =z2 or f(x, y, z ) = w2
If the function is of the form (say) f( x1, x2, x3)= 0, one may need to find three rational functions
g1, g2 and g3 such that, for all values of r and s, setting xi = gi( r,s) for I = 1 , 2, 3 gives a solution
of f(x1, x2,x3 )=0.
Algebraic Number Theory:
An algebraic number is any complex number that is a solution to some polynomial equation
f(x)=0 with rational coefficients; for example, every solution x of
x5 + 5.5 x3 – 7 x2 + 9=0
is an algebraic umber.
Real Number:
A real number is a value of a continuous quantity that can present a distance along a line. Real
numbers can be thought of as points on an infinitely long line called the number line or real line,
where the points corresponding to the integers are equally spaced. Any real number can be
determined by a possibly infinite decimal presentation, such as 4.21, where each consecutive
digit is measured in units one-tenth of the size of the previous one. The real line can be thought
of as a part of the complex plane, and complex numbers include the real numbers. The real
numbers can be thought of as points on an infinitely long number line as shown below:
_________________________________________________
-5 -4 -3 -2 -1 0 1 2 3 4 5
7
Rational Number:
a
A rational number is any number that can be expressed as the quotient or function of two
b
integers a and b, where b is the denominator. The denominator b may be equal to 1, and hence
every integer is a rational number. For example, 1 , 2, 3, ……. are rational numbers. Again, as
a
is a rational number, the decimal expansion of a rational number always either terminates
b
after a finite number of digits or begins to repeat the same finite sequence of digits over and
over.
Rational numbers can be formally defined as equivalence classes of pairs of integers ( a , b ) such
that b≠ 0, for the equivalence relation defined by ( a1 , b1 ) ~( a2, b2) if, and only if a1b2=a2b1.
a
With this formal definition, the function becomes the standard notation for the equivalence
b
class of [ a2, b2].
Rational numbers together with addition and multiplication form a field which contains the
integers and is contained in any field containing the integers. The rational numbers form a dense
subset of the real numbers. The real numbers can be constructed by completion , using infinite
decimals or Cauchy Sequences , Dedekind cuts, or infinite decimals.
Some Aspects of Rational Numbers are :
( i ). Irreducible Fraction: Every rational number may be expressed in a unique way as an
a
irreducible fraction , where and b are co-prime integers, and b > 0. This is often called the
b
a
canonical form. Starting from a rational number , its canonical form may be obtained by
b
dividing a and b by their greatest common divisor, and if b< 0 , changing the sign of the resulting
numerator and denominator.
(ii). Embedding of Integers: Any integer a can be expressed as the rational number a / 1, which is
its canonical form as a rational number.
a c a c
(iii). Equality : if and only if ad = bc. If both fractions are in canonical forms, then
b d b d
,if and only if a=c and b = d.
(iv). Ordering: If both the denominators are positive, and if, in particular if both fractions are in
a c
canonical forms, then if and only if ad < b c.
b d
If either denominator is negative, each fraction with a negative denominator must first be
converted into an equivalent from with a positive denominator by changing the signs of its
numerator and denominator.
(v). Addition: Two fractions are added as follows :
a c ad bc
b d bd
If both fractions are in canonical forms, the result is in canonical form if and only if b and d are
co-prime integers.
8
a c
If , ,........ give the integers 1 , 2, 3, ………,n , then
b d
a c
.........upto n numbers is
b d
n(n 1)
1 + 2 + 3 + ………+n =
2
Also, we have
n(n 1)( 2n 1)
12+22+32 + ……….+ n2=
6
n(n 1) 2
13 2 3 33 .......... n 3 [ ] .
2
a c ad bc
(vi). Subtraction:
b d bd
If both fractions are in canonical forms, the result is in canonical form if and only if b and d are
co-prime integers.
a c ac
(vii). Multiplication:
b d bd
If both fractions are in canonical forms, then the result may be reducible fraction.
a
(vii). Inverse: Every rational number has an adaptive inverse , often called its opposite
b
a b
( ) 1
b a
a a
If is in canonical form, the same is true for its opposite. A non-zero rational number has a
b b
1
a b 1
multiplicative inverse, also called its reciprocal, . For example, inverse of 2 is ,
b a 2
1
inverse of 3 is , etc.
3
a b b
If is in canonical form, then the canonical form of its reciprocal is either or depending
b a a
on the sign of a.
a c
(ix). Division: If both b and c are non-zero and if and are two rational numbers, then
b d
a
b ad
c bc
d
9
(x) Exponential to Integer Power: If n is a non-negative integer, then
n
a an
n
b b
0
a a
The result is in canonical form if is in canonical form. In particular, 1 , if a ≠0, then
b b
n
a bn a bn
n . If is in canonical form, the canonical form of the result is n if either a > 0 or n
b a b a
bn a c
is even. Otherwise, the canonical form of the result is . For two fractions , we have
a n
b d
a ad bc c
, where b and d are positive.
b 2bd d
Irrational Number:
The irrational numbers are all the real numbers which are not rational numbers. These can be
constructed from rations ( or fractions) of integers.
Square Roots: The square root of 2 is irrational. The square root of all natural numbers which
are not perfect squares are irrational. For example,
2 1.414; 3 1.732; 5 2.236; 6 2.449. All these are irrational numbers. Again,
4 2; 9 3 are rational numbers. In general, if an integer is not an exact k-th power of
another integer, then its k-th root is irrational. For example, 3 4 1.589, 4 5 1.495,.... .
Logarithms: The irrational numbers are available from certain logarithms. For example,
log23=1.58 > 0. However, if it is assumed that log23 is rational for any integers m and n, then we
can have
n
m
m
m
Log2 3= 2 n 3 2 n 3 n 2 m 3 n
n
Irrational number can also be found out within the countable set of real algebraic numbers as real
solutions of polynomial
P(x) = an xn + an-1xn-1 + ……. + a1x + a0 = 0
where the coefficients ai are integers and an≠ 0.
Relation Between Rational and Irrational Numbers:
According to the definition of rational and irrational numbers, we an write
10
Rational Numbers Irrational Numbers
------------------------ ----------------------------------
From the above results it can be seen that the system of rational number can be divided into two
classes, say, c1 and c2 such that all numbers of c1 class have their squares less than 2
[ 2 1.414213 .. ] and those of the c2 class is greater than 2. Hence every number of c2 class is
greater than the number of c1 class. Thus
1, 1.4, 1.41, 1.414, 1.4142, ……………… belongs to c1 class
2, 1.5, 1.42, 1.415, 1.4143, …………….. belongs to c2 class.
The differences of the corresponding numbers of two classes are respectively
1, 0.1, 0.01, 0.001, 0.0001,……….
Proceeding in this way we can find a member of c1 class ad a member of the c2 class which differ
from one another as small as possible. From this we can say that there exists a number x and a
corresponding point P on the axis, such that P divides the class c1 from class c2. But this number
x is neither belongs to c1 class nor c2 class and x2 is neither > 2 nor < 2 . Let us consider that x2 >
2 , x2 = 2 + e. Then whatever be the value of e , we can get rational numbers of the c2 class. Such
rational numbers of the c2 class will lie to the left of x and so the assumption that x is the point
dividing the two classes in untenable.
Prime Number:
A prime number is a natural number greater than 1 which cannot be formed by multiplying two
smaller natural numbers. A natural number greater than 1 but is not prime is called composite
number. For example, 3 is a prime number as it cannot be obtained by multiplying any two
natural numbers less than 3. The number 3 can be written as 1 x 3, or 3 x 1, where 3 is itself
present and one of the components of the product is not less than 3. Again, 6 = 2 x 3 or 3 x 2,
where both the natural numbers are 2 and 3 and these are less than 6. So, 6 is not a prime
number. Every natural number greater than 1 is either prime itself or can be factorized as a
product of primes that is unique up to their order. This property of prime number is called
11
primality. The primality of any number n is checked by trial division if n is a multiple of any
integer between 2 to n . The other prime numbers are 2 , 3, 5, 7, 11, 13, 17,……..
Co-prime Integers: In number theory, two integers a and b are said to be relatively prime ,
mutually prime or co-prime if the only positive integer that divides both of them is 1.
Consequently, any prime number that divides one does not divide the other. The numerator and
denominator of a reduced fraction are co-prime. For examples, 7 and 11 are co-prime because
both of them are only divisible by 1, while 7 and 14 are not co-prime as both of them are
divisible by 7.
Properties of Co-prime:
The numbers 1 and -1 are the only integers co-prime to every integer, and they are the only
integers that are co-prime with 0. A number of conditions are equivalent to a and b being co-
prime.
(i). No prime number divides both a and b.
(ii). There exists x and y such that ax + by = 1.
(iii). The integer b has a multiplicative inverse modulo a, meaning that there exists an integer y
such that b y 1 ( mod a ).
(iv). Every pair of congruence relations for an unknown integer x of the form x k (mod a ) and
x m (mod b), has a solution; in fact the solutions are described by a single congruence
relation modulo ab.
(v). The least common multiple of a and b is equal to their product ab.
Complex Number: A complex number is a number that can be expressed in the form z = a + i b,
where a and b are real numbers, and i is a solution of the equation x 2 = -1. Because no real
number satisfies this equation. Here i is called imaginary quantity and a is the real part of the
complex number z = a + i b, and i b is the imaginary part.
The imaginary i is introduced to overcome the problem of taking square root of the (-ve) value,
i.e (ve) . This square root cannot be expressed by real number. But as i2 = -1, the square root
of – 1 is taken as i . For example, 25 = i5.
A complex number can be visually represented as follows:
Figure 1
12
It is a diagram representing as a pair of real numbers ( a, b) forming a vector. This diagram is
called Argand Diagram. It represent the complex plain. Here Re is the real axis and Im is the
imaginary axis and i2=-1.
Complex numbers allow solutions to certain equations that have no solution in real numbers. For
example,
( x + 1 )2 = -9
Have no solutions in real numbers. Here (-9) can be written as ( 3i)2 and hence the equation can
be written as
( x +1)2 = (3i)2, or ( x 1) (3i) , or x = -1 + 3i and x = -1 – 3i
Addition ,Subtraction, Multiplication and Division in Case of Complex Number:
(i).( a+ i b) + (c – id ) = (a+ c) + i (b+ d).
(ii) (a+i b) – ( c +id) = ( a – c) + i ( b – d )
Multiplication:
(a + ib) ( c +id) = (ac – bd ) + i( bc+ ad).
Let us consider that z = a + ib be a complex number and z = a – ib .Then
z. z = (a +ib) (a - ib) = a2 + b2= z , or z a 2 b 2 and it is non-negative real number.
2
Division:
a ib ac bd bc ad
2 i 2 .[ Multiplying numerator and denominator by conjugate of
c id c d 2
c d2
denominator ]
Reciprocal: Let us consider that
1 z a ib a ib
2 2 , where z = a +ib.
z z z (a ib)(a ib) a b 2
a b2
Square Root: The square root of ( a +ib ) is given by
a a2 b2 a a2 b2
( i ) , where b≠ 0 and sgn(b) , where sign is
2 2
the signum function.
Complex Conjugate: The complex conjugate of a complex number is the number with an
equal real part and an equal imaginary part equal in magnitude but opposite in sign. For example,
if
( a +ib ) is a complex number , then (a – ib) is complex conjugate number. If ( 2 +i3) is a
complex number, then ( 2 – i3) is the complex conjugate number.
Further Properties of Complex Number:
( i ). If a and b are real numbers and (a + ib) =0 , then a = 0, b= 0.
(ii). If a, b and c are real numbers, and (a +ib) = (c +id) , then a = c and b = d.
(iii). The sum of two conjugate complex numbers is real. Let z = (a +ib) be a complex number.
Then conjugate of z is z = a – ib . Now, z+ z = (a +ib) + ( a – ib) = 2a, which is real.
(iv) For two complex numbers z1 and z2 z1 z 2 z1 z 2 .
1.3 Function
A function is idealization of how a varying quantity depends on another quantity. For example ,
the diameter of a circle is measured by d = 2r, where r is the radius of the circle. Here the
diameter d depends on the value r of the circle; d depends on r. The quantity 2 is not changed
whatever be the value of r. This 2 is a constant and r is the variable. Depending on the value of r,
13
the value of d is changed, so d is also a variable. Here d is the dependent variable and r is the
independent variable.
Constant: A symbol having a fixed numerical value is called a constant. For example, y = a,
where a is a fixed numerical value and it is not changed. If we write y = 5 or y = 50, these are
given numerical values; there is no scope of change of these values. Here 5 is a constant. The
other constants are, generally denoted by a b , c, ….. etc. Here y = 5 is a constant variable or
fixed value. The function y = a is a constant function as it has the same output value no matter
what your input value is.
Variable: A symbol which takes different numerical values in different similar units or in
different time periods is called a variable. For example, let us consider that income of families
varies from family to family at a particular time. The production of an industry, the number of
workers in different time periods may vary. So, production of an industry in different periods or
number workers of industry in different periods may take different numerical values .Thus,
amount of production and number of workers are variables.
The variable is of two types, viz. ( a) Discrete variable, and (b ) Continuous variable. In the
above mentioned examples, the variable Number of Workers is an example of discrete variable
as it takes numerical values which are integral. The discrete variable can take values 0 , 1, 2, 3,
……. The other variable Amount of Production is an example of continuous variable as it can
take any values in the range to . For example, if we measure the change in production
of an industry in different days of a month compared to the production of first day of the month,
the change may be
Figure 2
Change in Production ( in tons): 1.5. 1.8, 0.0, -0.5, 1.2, 0.0, 1.8, -0.6, ……..
One may be interested to formulate a relationship between number of workers ( x ) and amount
of Production (y) of an industry over a period of time. This relationship can be formulated as
y = Bo + B1x + e,
The above is a relation which associates each element of x of a set X, the domain of the function,
to a single element y of another set Y, the codomain of the function. The relation is formulated as
y = f(x), where the element x is the argument or input of the function, and y is the value of the
14
function, the output or the image of x by f. Here f is a function of the variable x. This function is
uniquely presented by its graph which is the set of all pairs of values { x, f(x) }. When the
domain and codomain are sets of numbers, each such pair may be considered as the Cartesian co-
ordinate of a point in the plane.
In general, these points from a curve, which is also called the graph of the function. For
example, let y = 2 +x. Then, for the values of x = 1 or 2 or 3, we have y values as follows:
x 1 2 3
y 3 4 5
The graph of the pairs of values {x, f(x)} is shown above:
15
The values of x and y are listed in numerical order. There should not be any duplicate of x or y
values. The domain and range may not be ordered numerically.
Consider another set of values of (x, y) as follows:
(x,y): { (-3,5), ( -2,5),(-1,5) ,(0,5), (1,5), (2,5)}
Here Domain: { -3,-2,-1,0,1,2} and Range : {5}.
Usually, the domain and range are found out as follows:
(a). Domain is found out by looking for those values of the independent variable ( usually x),
(b). There should not be negative values under a square root sign,
(c ). There should not be any zero values in the denominator of a fraction,
(d) Invert log never be (-ve) or zero,
Range is found out by finding the resulting y-values after substituting the x-values in the
function.
Example 1.1: Determine the domain and range of the function
x2 x 2
y= 2
x x2
Solution: The domain is all the values that x is allowed to take on. Let the denominator be set to
equal to 0. Then x2 – x – 2 =0. It gives (x – 2 ) ( x+ 1) = 0. Therefore, x=2 and x = -1. It implies
that the domain of x is all the values except -1 , 2. Here range is all real numbers except the
numbers for x = 2 and x = -1. The graph of the function is shown below:
Figure 3
Example 1.2: Determine the domain and range of the function given below:
y = 2x 3
Solution: Let – 2x + 3 ≥ 0, or -2x ≥ -3. , or 2x ≤ 3 . It implies that x ≤ 3/2= 1.5.
Thus, the domain is all x ≤ 3/2 .The range is all y ≤ 0.
The graph of the function is shown below:
16
Figure -4
Range is also defined by the difference of highest value and lowest value . For example, let us
consider the set of values { 8, 5, 8, 3, 2, 10, 12 }. Here highest value is 12 and lowest value is 2.
So, the range is (12 – 2 ) = 10. This is the statistical range. It is used as a measure of dispersion.
This range is misleading also. For example, let a set of values be 5, 3, 12, 18, 809. Here range is
(809 – 3 ) = 806. This range is a misleading value of measure of dispersion.
Single-Valued Function: A single valued function is a function that provides a unique value in
the range for each point in the domain. It is , therefore, one-to-one or many-to- one. For example,
let us consider the following set of values of X and Y as follows:
Figure – 5
The domain indicates that a single-valued function with domain (1 ,2, 3) . Each input, on the left,
maps to a single output.
Real Valued Point Function: Let the elements x X. Using any element of x a real number
f(x) can be defined. Here f(x) is the value of f for the point x. This f is the real valued function.
The set X is the domain of f and the set of values { f(x); x X } is the range of f. The range is
usually, denoted by R.
17
For each x, f(x) is different, the function is one-to-one ( 1:1). If two or more x‟s have the same
value f(x), the function is many-to-one. If f(x)= x2 , then f is the 2 – 1 function.
A function f which maps X to R assigns to each x X , a value f(x) R . This f(x) is called the
image of x under f and x is called the pre-image of f(x).
If R = { f(x) : x X } , f is said to be a mapping from X to R . If { f(x): x X} R, f is said to be
a mapping from X to R. Real valued point function s f1 and f2are equal if f1(x) = f2(x) xX .
The function f is a numerical function if R= R/, the real line { y }.
Inverse Function: A function y = f(x) is invertible , if exactly a function g(y) exists such that
g(y)=x. Here the function g(y) is called inverse function of f(x). If f(x) is invertible ( injective),
the function g is unique. If f(x) is the function of x , then the inverse function is written as f-1. For
example, let us consider the following two sets as show below:
3 a
A: 5 , B: b
7 c
Let us define the function f(x) , where y=f(x)
Figure – 6
Figure – 7
It implies that y=f(x) and g( y) = x.
18
Unless otherwise stated, a function is invertible if and only if its inverse relation is a function on
the range Y. In such case the inverse relation is the inverse function. Not all functions have an
inverse . Let y B and x A, the inverse function is applicable if y B must correspond to not
more than one of x A. A function f with this property is called one-to-one or injection. If f and
f-1 are functions on A and B respectively, then are bijections.
The function f(x) = x2 is not invertible if x A are all real numbers. Here except 0 y=f(x)
corresponds to two different starting points in x A. because, if x = -2 or +2, y =f(x)= x2=4, i.e y
corresponds to two x values. Such a function is non-injective. However, if x takes only non-
negative real values, then f(x) = x2 ix invertible ( injective).
More Examples of Functions:
2x3 7 x 2 2x 1
2
(i). Analytical Functions: Let y=f(x)= x , y=f(x)= 2 , y f ( x) , etc. be some
x 9 x2 1
functions. In general,
Y=f(x) = a0 xn + a1 xn-1 + a2 xn-2 +…….+ an-1 x + an
where n is a positive integer. These functions are analytical functions. The second and third
functions are rational algebraic functions.
The domain of these are generally the set of all real values; in the last case the zeroes of the
denominator are excluded. Because at x=0, the function can be defined.
(ii). f(x) = x, if x > 0
=0, if x=0
= -x, if x< 0
This function can be shown graphically as follows:
Figure – 8
The function shows two lines, viz. OA and OB and these two lines bisect the angles XOY and
X / OY , respectively. The above graph is also the graph of the function y=f(x)= x .
(iii).Let y=f(x) = x , then the graph of the function is
19
Figure – 9
This function is defined for x= 0 , and for all positive values of x, say, { 0, 1 , 2, ……., n} and
even for all the fractional value of x. The graph of this function is a continuous one and takes the
shape of a parabola in the first quadrant.
(iv). Let y = f(x) = x!
The graph of this function consists of series of isolated points.
x2
(v). Let y =f(x) = , for x > 0. We have y=f(x) = x , if x ≠ 0, where y is not known.
x
The graph of the function is given by
Figure – 10
The graph of the function y=x is straight line passes through the origin.
(vi). Let y=f(x) = x sin(1/x)
The graph of this function is as follows:
20
Figure - 11
(vii). Transcendental Function: The function y=f(x) = ex, y=f(x) = log x, y=f(x) = sin x ,
y= f(x)= cos x, y=f(x) = sin-1x, y=f(x) = cos-1x are transcendental functions. Some of these can
be shown by graph as follows:
y=sin x, the graph is
Figure – 12
y= cos x , the graph is
Figure – 13
21
y= x2, the graph is
Figure – 14
Bounded Functions: A function y =f(x) defined on some set of values X with a real or complex
values is said to be bounded above by A. On the other hand, if f(x) ≥ B for all x in X , then the
function is said to be bounded below by B. For example,
1
y=f(x)= 2
x 1
for all values of x except for x = - 1 or x = 1 is unbounded. Because, when x gets closes to -1or 1,
the values of this function get larger and larger in magnitude. However, the function can be made
bounded if one considers its domain to be, for example, [ 2, ∞) or ( -∞, -2].
The function
1
Y=f(x)= 2
x 1
Is bounded for all values of x. The graph of this function is
Figure – 15
22
The function y =f(x) = sin x is a bounded function for all real values of x. But for the complex
values of x the function is unbounded one. The graph of the function is as follows:
Figure – 16
Monotone Function: A monotonic function is a function between ordered sets that preserves or
reverses the given order. This type of function may be increasing or decreasing. It will be
monotonically increasing if for all x and y such that x ≤y , one gets f(x) ≤ f(y). Similarly, a
function is called monotone decreasing function, if for x and y such that x ≤ y , one gets f(x) ≥
f(y). Here the function reverses the order. If for x ≤ y , we get f(x) < f(y) , then the function is
called strictly monotonic increasing function. But for x < y or for x > y , either we have f(x) <
f(y) or f(x) > f(y) , the function is strictly decreasing function. The opposite to these conditions
lead to the conclusion of weakly increasing or decreasing function.
3x 5 3 2 5 11
Let f(x) = for all value of x. Then for xi < xj we have f(2) = 2.2 , f(3)= 2,
2x 1 2 2 1 5
3 5 5 20
f(4) = 1.89, f(5) = 1.82 . Here 2 >3 > 4> 5. We have f(2)> f(3)> f(4)> f(5). It is
2 5 1 11
seen that for x > y , we have f(x)> f(y). Therefore, the function is decreasing function.
x 1
Example 1.3: Given f(x) = 2 , find (i) f(0), (ii) f(1), (iii) f( -1), (iv) f ( 2), (v) f(-2) , (vi)
x 2
f(1/x),
(vii) f(x +h), (viii) f(2a).
0 1 1 11 11 2
Solution: (i) f(0)= 2 , (ii) f(1) = 1 0 , (iii) f(-1)=
0 2 2 1 2 (1) 2
2
3
1
1
2 1 1 2 1 1 1 x x x2
(iv) f(2) = 2 , (v) f(-2) = , (vi) f( )
2 2 6 (2) 2 2 2 x 1 1 2x 2
2
x2
x h 1 x h 1 2a 1
(vii) f( x + h) = 2 , (viii) f( 2a) = .
( x h) 2 x 2hx h 2
2 2
4a 2 2
23
15 f ( x 3)
Example 1.4: If f(x)= 2x, show that (a) f(x+3) – f(x – 1) = f ( x ) , and (b) f ( 4)
2 f ( x 1)
Solution: Proof
1 16 1 15 15
(a). f(x+3) – f(x – 1 )= 2x+3 – 2x – 1= 2x [ 23 – 2-1] = 2x [ 8 - ] = 2x ( 2x f ( x) .
2 2 2 2
f ( x 3) 2 x 3 2 x 2 3
(b) 2 4 16 , Again f(4)= 24=16, hence proved.
f ( x 1) 2 x 1 2 x 2 1
Example 1.5: Determine the domain of the following functions :
1 1 x
(i). f(x) = 4 x 2 , (ii). f(x) = x 2 16 , (iii) f(x) = , (iv) f(x) = 2 , (v) f(x)= 2
x2 x 9 x 4
Solution: (i) Since f(x) is real, 4 – x ≥ 0, or x ≤ 4, x ≤ 2 . Therefore, the domain of the
2 2
function ix - 2 ≤ x ≤ 2.
(ii). f(x) = x 2 16 . Here x2 – 16 ≥ 0 or x2 ≥ 16, or x ≥ 4 . Therefore, the domain consists of
the interval - 4 ≤ x ≤ 4.
1
(iii) f(x) = , This function can be defined for all values of x except 2 . Hence, the domain of
x2
the function is all values except 2.
1 1
(iv) f(x) = 2 . The function does not exist for x = 3 or x = - 3. Hence, the
x 9 ( x 3)( x 3)
domain of the function is all values except x = - 3 and x = 3.
x
(v) f(x) = 2 . This function exists if x2 + 4 ≠ 0 for all values of x. So, the domain is the set
x 4
of all real values of x.
Example 1.6: (a) Given f(x) = x2 – 2 x + 3, (b) f(x) = x2 – 3 x +7.
Evaluate (i) f( 4),(ii) f(-4), (iii) f(-x), (iv) f(x+3), (v) f(x – 2 ), (vi) f( x + h),(vii ) f( x + h ) – f( x).
Show that [ f(x+h) – f(x) ] / h = 2x +h – 2 for (a).
Solution (a)
(i). f(4)= 42 – 2 x 4 + 3 = 11, ( ii ) f(-4) = ( -4)2 – 2 ( -4) +3 = 27,
(iii) f( - x) = ( -x)2- 2 ( -x) +3 = x2 + 2 x +3,
(iv) f( x+3) = (x+3)2 – 2 ( x+ 3) +3 = x2+6x+9 – 2x – 6 +3= x2+4x+6
t(v) f(x – 2 ) = (x – 2)2 – 2 ( x – 2 ) +3 = x2 – 4x +4 – 2x +4 +3 = x2 – 6x + 11
(vi) f(x +h) = (x +h)2 – 2 (x +h) +3 = x2 + 2xh +h2 – 2x – 2h + 3 = x2 +2x(h – 1) + h(h – 2 ) +3
(vii) f(x +h) – f(h) = (x +h)2 – 2 (x +h) +3 – f(h) + 2 h +3 = x2+2xh +h2 – 2x – 2h+3 – x2+2x – 3
= h2 + 2hx – 2h = h( h+2x – 2 ).
(viii) Proof: We have f( x +h) – f(h) = h( h + x – 2
f ( x h) f (h) h(h 2 x 2)
h 2x 2
h h
(b) f(x) = x2 – 3x+7
(i) f(4)= 42 – 3 x 4 + 7 = 11, (ii) f( -4 )= ( -4)2 – 3 (-4) +7= 35,
(iii) f(-x ) = (-x)2 – 3(-x) + 7 = x2+3x +7
(iv) f(x+3) = (x+3)2 – 3 (x+3)+7= x2+6x+9 – 3x – 9+7 = x2+3x +7
(v) f(x – 2 )= ( x – 2)2 – 3(x – 2 ) +7= x2 – 4x +4 – 3x +6 + 7= x2 – 7x +17
(vi) f(x +h) = (x + h )2 – 3 (x +h) +7= x2+2hx +h2 – 3x – 3h +7= (x2 – 3x +7) +h ( h+2x – 3 )
(vii) f(x +h) – f(x) = (x2 – 3x + 7) + h ( h +2x – 3) – (x2 – 3x +7) = h ( h+ 2x – 3 ).
24
f ( a h) f ( a )
Example 1.6: Given f(x) = x2 + 2x, find and interpret the result.
h
Solution: We have f(a +h)= (a +h)2+ 2(a +h)= a2+h2+2ah+2a+2h
f(a)= a2+2a; f(a +h) – f(a) = a2+2ah+h2+2a+2h – a2 – 2a= h( h+2a+2).
f ( a h) f ( a )
Hence =h+2a+2.
h
f (a h) f (a) difference of ordinates
Here
h difference of abscissas
Example 1.7: Sketch the graph of the function given as
(i). f(x) = 4, when 0 ≤ x<1,
= 8, when 1 < x ≤2
= 12, when 2 < x ≤ 3
=16, when 3 < x ≤ 4.
Solution:
Figure – 17
2
Example 1.8: Given f(x) = x – 10 x +3; find (i) f(0), (ii) f(-2).
Solution: (i ) f(0) = 02 – 10 x 0 + 3 = 3
(ii). f(-2) = (-2)2 – 10 (-2) +3 = 27.
xa xb
Example 1.9: Given f(x) = b a , show that f(a)+f(b) = f(a+b)
ba ab
aa a b ba bb
Solution: Here f(a) = b a a and f(b) = b a b
ba a b ba ab
f(a+b)= a
aba abb b 2
a 2
b 2
a 2
b a
2 2
(b a)(b a)
a ba
ba ab ba ab ba ba ba ba
Again, f(a) + f(b) = a +b= f( a+b).
x 2 5x 6
Example 1.10: Show that for x = 2 the function f(x)= 2 is not defined. Find f(-5) and
x 8 x 12
f(6).
Solution: The denominator is x2 – 8x +12
25
or x2 – 6x – 2x +12
or x(x – 6 ) – 2 (x – 6)
or (x – 6 ) (x – 2 )
If x = 2, the denominator becomes zero. So, the function cannot be defined. Now, the numerator
is
x2 – 5x +6
or x2 – 3x – 2x +6
or x(x – 3 ) – 2 ( x – 3 )
or (x – 3 ) ( x – 2 )
x 2 5 x 6 ( x 3)( x 2) x 3
Therefore, f(x) = 2
x 8 x 12 ( x 6)( x 2) x 6
53 8
Now, f(-5) =
5 6 11
The function is not defined if x = 6.
Example 1.11: Draw the graph of the function f(x)= 4 x 2 and find the domain and range of
the function.
Solution: We have y=f(x)= 4 x 2
or y2 = 4 – x2, or x2 + y2 = 4
which is the equation of a circle with centre origin (0.0) and radius r = 2. Since y = 4 x2 0
the graph of the function is upper half of the circle as shown below:
Figure – 18
From the figure it is seen that the domain is – 2 ≤ x ≤ 2 and the range of the function is the
interval 0 ≤ y ≤ 2.
x
Example 1.12: Given f(x) = . Show that for x > 0 the function is monotone increasing.
x 1
26
1 1 2 2 3 3
Solution: We can write f(1)= , f(2) = , f(3) = 0.75 , f(4)=
11 2 2 1 3 3 1 4
4 4
0.8 . Here 1 < 2< 3< 4 and we have f(1)> f(2)> f(3) > f(4). Therefore, the function is
4 1 5
monotonic increasing function.
Codomain: The codomain of a function is the set Y into which all of the output of the function
is constrained to fall. For example, if y=f(x) = x2, then the codomain of f is the range space , but f
does not map to any negative number. The image of f is the interval [ 0, )
Figure – 19
It has already been explained that a function relates an input to output and the domain is all the
values of input that go into a function and range is all the values that come out.
Let us consider that the income of an officer is increased by Tk.2000.00 per year. So, the salary
of the officer is related to the service length. If L is the service length, then after 10 years of
service the income will be f(10)= L x 2000.00 = 10 x 2000.00 = 20, 000.00.
For example, let y = 2x +1, then for different values of x = 1,2,3,4.. we have
27
Figure – 20
Here set A is the domain and the set B is the codomain.
The set of elements that get pointed to in B ( the actual values produced by the function ) are the
range. These elements are also called image. Thus, for the function y =f(x) = x2 if
A [ 1,2,3 ] is the domain, then the codomain is B [ 1, 4, 9 ].
Again, for the function y =f(x) = x2 if the domain is
A[ -3,-2,-1,0,1,2,3 ] then the domain is B [ 0,1,4,9,].
Supplementary Problems
1.Draw the graph of the following functions;
a) y=1, if x>0
=0, if x=0
= -1, if x <0
b) y=x2, if x ≠1
= 2, if x= 1
c) f(x) =1, if x is an integer
=0, if x is not an integer.
2.Given f(x) = x2 – 4x +4, find (i) f(2), (ii) f(3), (iii) f(4)
Ans. (i) 0, (ii) 1, (iii) 4.
x 1
3.Given f(x) = , find (i) f(a), (ii) f(0), (iii) f(3), (iv) f(-2)
x 1
a 1
Ans. (i) , (ii) -1, (iii) 0.5, (iv) 3.
a 1
4.Given f(x) = x2 – x , show that f(x+1) = f(-x).
1 ab
5. Given f(x) = , show that f(a) – f(b) = f( )
x ba
5x 3
6.Given y= f(x) = , show that x = f(y),
4x 5
7.t Determine the domain of the following functions:
x 2x
(a) y = x2+4, (b) y = x 2 4 (c) y x 2 4 (d ) y , (e) y=
x3 ( x 2)( x 1)
28
1 x2 1 x
(f) y = , (g) y , ( h) y
9 x2 x2 1 2 x
Ans. (a), (b), (g) all values of x, (c) x 2 , (d) x≠ -3, (e) x≠ -1, (f) -3 < x < 3, (h) 0 ≤ x < 2.
8.(i) Show that f(x) = ( 1 + 1/x)x for all x>0 is a monotone increasing function.
(ii) x 2 5 x 6 is not defined for 2 < x < 3.
(iii) Show that f(x) = 2x2 +4x +6 in the interval 0<x<1 has the lower bound 6 and the upper
bound 12.
1.4 Limit
Limit is the value that function approaches as the input approaches same value. It describes what
happens to a function f(x) as the variable (input) x approaches to a particular number c. In that
sense, the limit can be defined mathematically as follows:
Lim f(x) = L
xc
If f(x) is a real valued function and c is a real number, then
Lim f(x) =L
x c
means that f(x) can be made to be as close to L as desired by making x sufficiently close to c.
The above definition is true even if f(c) ≠ L. Indeed, the function f [f(x)] need not even be
defined at c. For example, let
x2 1
f(x) =
x 1
then f(x) is not defined for x = 1, yet x moves arbitrarily to 1 , f(x) correspondingly approaches
to 2. Let us see the cases for x<1.
f(0.8) f(0.85) f(0.9) f(0.95) f(0.99) f(1) f(i.01) f(1.1)
1.80 1.85 1.90 1.95 1.99 undefined 2.01 2.10
It is seen that by making x sufficiently close to 1 f(x) can be made arbitrarily close to 2. This
result can be expressed mathematically
x2 1
Lt f(x) = 2
x 1
x 1
It can also be shown that
x 2 1 ( x 1)( x 1)
f(x) = x 1, if x ≠1
x 1 x 1
Now, since (x+1) is continuous in x at 1, we can now plug in1 for x giving
Lim f(x) = 1 +1 = 2
x 1.
Again, let us consider another function
2x 1
f(x)=
x
Here f(10)= 1.90, f(100)= 1.99, f(1000) = 1.999, f(10000) = 1.9999, f(100000)= 1.99999 …..
It is seen that as x becomes extremely large, the value of f(x) approaches to 2 and the value of
f(x) can be made close to 2. In such a case, we can write
29
2x 1
Lt f(x) = 2
x
x
General Interpretation of Limit: Geometrically, the limit statement
Lim f(x) =L means that the height of the graph of y=f(x) approaches L as x
x c
approaches c. Let us see this phenomena in the following graph:
Figure – 21
x2 x 2
For example, let f(x) = is a line with a hole [ as shown below ] at point 1.3 and the
x 1
points (x, y)
Figure – 22
On the graph approach this hole as x approaches 1 from either side.
Limit of Sequence: Consider the following sequence 1.79, 1.799, 1.799 …… It can be
observed that the numbers are approaching 1.8, the limit of the sequence. Formally, suppose a1,
a2, ……..an … is a sequence of real numbers. It can be stated that the real number L is the limit
of the sequence, viz.
Lim an =L
n
This means that for every real number > 0, there exists a natural number N such that for all n >
N , we have a n L . Intuitively, this means that eventually all elements of the sequence get
arbitrarily close to the limit, since the absolute value a n L is the distance between an and L.
30
Not every sequence has a limit, if it does, it is called convergent, and if it does, it is divergent. It
is also known that a convergent sequence has one limit. Thus we can say that a limit L of a
function f(x) as x goes to infinity, if it exists, is the same as the limit of any arbitrary sequence an
1
that approaches L, and where an is never equal to L. One such sequence would be L + .
n
Properties of Limit:
Limits obey some algebraic rules which are used in evaluation a limit of a function f(x). These
are important because they simply the calculation of limits of algebraic functions. These are
If Lim f(x) and Lim g(x) exist, then
x3
x3 x3 1 1
Solution: lim = lIm = Lim =
x 3 x 18
2
( x 3)( x 6) x6 9
x3 x3 x3
Here as x 3 (x-3) can never be zero.
x5
(ii) Evaluate the limit Lim 2
x 25
x 5
Solution: As x 5 , x – 5 can never be zero.
31
x5 x5 1 1
Lim = Lim = Lim
x 25
2
( x 5)( x 5) x 5 10
x5 x5 x 5
x 27
3
(iii) Evaluate the limit LIm 2
x 9
x 3
x 3 27 ( x 3)( x 2 3 x 9) x 2 3x 9 9
Solution: Lim 2 Lim = Lim
x 9 ( x 3)( x 3) x3 2
x3 x3 x3
(iv) Evaluate the limit Lim (3x 4 x 8)
3
x 1
Solution: We can write
Lim 3x 3 4 x 8 Lim 3x3 – Lim 4x + Lim 8 = -3 +4 +8 = 9
x 1 x 1 x 1 x 1
( x h) 2 x 2
(v) Evaluate the limit Lim
h
h0
x 2 2hx h 2 x 2 h( 2 x h)
Solution: Lim =Lim = Lim(2x+h)= 2x
h h
h0 h 0 h 0
3x 8
3
(vi) Evaluate the limit Lim
x2
x0
Solution: Lim (x -2 ) ≠0. Therefore, we can write
x0
3x 8
3
Lim = Lim 3x3 – 8 Lim (x – 2 )= - 8 (-2)= 4
x2
x 0 x0 x 0
1
(vii) Evaluate Lim { (1 x) (1 x)
x
x0
1
Solution: Here Lim 0
x
x0
Let us multiply the numerator and denominator by (1 x) (1 x) . Then
{ (1 x) (1 x)}{ (1 x) (1 x)} 1 x (1 x)
Lim =Lim
x{ (1 x) (1 x)} x{ (1 x) (1 x)}
x 0 x 0
2 2
=Lim 1
(1 x) (1 x) 2
32
x 0
3x 2
(viii) Evaluate Lim
9x 7
x
2 2
x(3 ) 3
3x 2 x =Lim x =3
Solution: Lim =Lim
9x 7 7 7 9
x(9 ) 9
x x
x x x
x2 1
(ix) Find Lim
x 2 3x 2
x 1
Solution: if x =1, both numerator and denominator approaches zero. Hence no conclusion can be
drawn about the size of the quotient. It is noted that when x =1, the function is not defined. But
for other values of x <1 , we can proceed to find the limit . Let us proceed for as follows:
x2 1 ( x 1)( x 1) x 1
, hence
x 3 x 2 ( x 1)( x 2) x 2
2
x2 1 x 1
Lim =Lim 2
x 3x 2
2
x2
x 1 x 1
The graph of the function is shown in the following figure:
Figure – 23
Remark: If both the numerator and denominator approach zero, the function does not exist. In
that case we need to simplify the function so that it does not approach zero.
6x 2 2x 1
(x) Find the limit of when x .
6 x 2 3x 4
2 1
x 2 (6 2 )
Solution: Lim x x Lim 6 0 0 =1
3 4 600
x 2 (6 2 )
x x
33
x x
x 1
(xi) Find the limit of when x 1
x 1
Solution: Here both numerator and denominator approach zero when x 1. The function does
not exist. To avoid the problem, let us simplify the function as follows:
( x 1)( x 1) x 1 1
.
( x 1)( x 1) ( x 1)( x 1) x 1
x 1 1 1
Therefore, Lim =Lim
x 1 x 1 2
x1 x1
x x2
2
(xii) Evaluate Lim
( x 1) 2
x1
( x 1)( x 2) x2
Solution: Lim = Lim , no limit exists.
( x 1) 2
x 1
x 1 x1
x x2
2
(xiii) Evaluate Lim
4x3 1
x
1 1 2
x3 ( 2 3 )
x x2
2
x x x 0 =0
Solution: Lim =Lim =
4x 1
3
1 4
x 3 (4 3 )
x
x x
2x3
(xiv) Evaluate Lim 2
x 1
x
2 2
Solution: Lim Lim , no limit exists.
1 1 0
3
x x
x
4 x2
(xv) Evaluate Lim
3 x2 5
x 2
(4 x 2 )(3 x 2 5 ) (4 x 2 )(3 x 2 5 )
Solution: Lim = Lim Lim
(3 x 2 5) (3 x 2 5) 9 x2 5
(4 x 2 )(3 x 2 5)
4 x2
x 2 x 2 x 2
34
= Lim 3+ x 2 5 =6 .
x 2
f ( x h) f ( x )
Example 1.14: (i) Given f(x) = x2 – 3x , show that Lim 2x 3.
h
Solution: Since f(x) = x2 – 3x, f(x +h) =(x +h)2 – 3(x +h)= x2+2hx+h2 – 3x – 3h
f ( x h) f ( x) x 2 2hx h 2 x 2 3 x 3h 3x
h 2 x 3 . Hence
h h
f ( x h) f ( x )
Lim 2x 3
h
h 0
f ( x h) f ( x ) 5
(ii) Given f(x) = 5 x 1 , show that Lim
h 2 5x 1
h 0
Proof: We have f( x+h) – f(x )= 5 x 5h 1 5 x 1 .
f ( x h) f ( x ) { 5 x 5h 1 5 x 1}{ 5 x 5h 1 5 x 1}
Lim =Lim
h h{ 5 x 5h 1 5 x 1}
h 0 h 0
(5 x 5h 1) (5 x 1) 5h 5
=Lim =Lim =
h{ 5 x 5h 1 5 x 1 h{ 5 x 5h 1 5 x 1} 5x 1 5x 1
h 0 h 0
5
=
2 5x 1
1 1
(iii) Show that Lim 1
3
3 2x
x0
1
1 1 1
Proof: Let x 0 , then , 2 x 0 Lim 1
x 3
3 2x
x 0
Supplementary Problems
1 2 x 1
1.Lim 1
, Ans.
3
3 2 x
x 0
2. Evaluate the following limits:
(i). Lim 3x2 – 5x +2, Ans.4; (ii) Lim (x – 1)2( x+1), Ans. 16;
x 2 x3
x 1 3 x3
(iii). Lim , Ans. ; (iv) Lim , Ans. Limit does not exist;
x2 4 5 x
35
x 2 x5
x 1
2
x 3 x 10
2
( x 1)( x 4) 5
(v) Lim , Ans. 2; (vi) Lim , Ans. 7 ; (vii) Lim , Ans.
x 1 x5 ( x 1)( x 4) 3
x 1 x5 x 4
x x6
2
x 2 1 x 1
(viii) Lim 2 , Ans. 5 ; (ix) Lim , Ans. ; (x) Lim , Ans. 2
x 3x 2 x4 4 x 1
x 2 x 4 x 1
2x 3 1 x x 2 5x 6
(xi) Lim , Ans. ; (xii) Lim 2 , Ans. 0 ; (xiii) Lim , Ans.
4x 5 2 x 5 x 1
x x
x
3 3
x
x3 (3x 1) 2 1
(xiv) Lim x x
, Ans. -1 ; (xv) Lim 2 , Ans. 0; (xvi) Lim , Ans.
3 3 x 5x 6 ( x 1) 3
2
x x x 1
1.5 Continuity
The function f(x) is said to be continuous if the following three conditions are satisfied:
(i). f(x0) is defined, (ii) Lim f(x) exists ( i.e. finite), (iii) Lim f(x) = f(a)
t x x0 x a
in that case y+ f(x) is a continuous function at point x=a. Function f[f(x)] is continuous in the
interval I if f(x) is continuous at each point x in I. The first condition given above implies that a
function can be continuous only at points of its domain.
Properties of Continuous function:
(i). The sum of the continuous functions is continuous.
(ii). The difference of continuous functions is continuous.
(iii) The product of continuous functions is continuous.
(iv) The quotient of continuous functions is continuous at all points x , provided the denominator
is not zero.
(v) The functional composition of continuous functions is continuous at all points x where the
composition is properly defined.
(vi) Any polynomial is continuous for all values of x.
(vii) Function ex and trigonometric functions sin x and cos x are continuous functions for all
values of x.
Let us now consider a simple function f=f(x)=x2+1.This function is continuous at x = 2 since
Lim x2+ 1 = 5, where f(2)= 5.
x 2
Let us verify some other cases of continuous functions. For example,
f(x)= 3x – 5 , if x ≠1
= 2 , if x=1
This function f(x) is defined at x = 1 since
(i). f(1)=2, Lim f(x)= Lim (3x – 5 ) = 3 x 1 – 5 = -2
x 1 x 1
(ii) Lim f(x) = - 2
x 1
(iii) But Lim f(x) ≠f(1)
36
x 1
Here condition (iii) for continuous function is not satisfied. Hence the given function is not
continuous. Similar is the case if f(x) = 4 x 2 at x =3. Since f(3) is imaginary. Let us consider
another function f(x), where
f(x)= x2 +2x, if x≤ -2
=x3 – 6x, if x > -2
Here f(-2) = (-2)2+2x =0
Lim f(x) = Lim (x2+2x) = (-2)2 + 2(-2)=0
x 2 x 2
The right hand limit Lim f(x) = Lim (x3 – 6x) = (-2)3 – 6(-2) = 4
x 2 x 2
since the left ha limit and right hand limit are not equal Lim f(x) does not exist.
x 2
This means that second condition for continuous function is not satisfied at x = -2. Again, let us
consider another function
x6
f(x)= ,x 0
x3
= 2, if x=0
= 4 x 2 , if x >0
f(0) = 2 . The left-hand limit is
x6 6
Lim f(x) = 2
x3 3
x 0
The right hand limit Lim f(x) = Lim 4 x 2 4 0 2 2
x 0 x 0
Thus Lim f(x) exists with x 0 .
Lim f(x) = 2, Lim f(x) =2=f(0) .
x 0 x0
Here all 3 conditions for continuity are satisfied. Hence the given function is continuous at x = 0.
x2 1
Let f(x)= , if x ≠ 1
x 1
= 1 , if x = 1
x 2 1 ( x 1)( x 1)
Here f(x) = = x+1
x 1 x 1
Lim f(x) = Lim (x+1) = 2
x 1 x 1
x2 1
But f(x)= is not defined at x =1. Therefore, the function f(x) is not continuous at x = 1.
x 1
1
Again, f(x) = x , if 0 ≤x ≤ (i)
2
= 0, if x=0 (ii)
37
1
= x – 1 , if x 1x (iii)
2
1 1 1 1 1 1 1
Here Lim f(x) = Lim f( h) = Lim ( h) = [ h and from (i) f( h) h ]
2 2 2 2 2 2 2
1
x h 0 x 0
2
1 1 1 1
Again, Lim f(x) = Lim f( h) =Lim{ h) 1}= (1) = (v)
2 2 2 2
1
x h0 h0
2
1 1 1 1
[ h and from (iii) f ( h) h 1 ]
2 2 2 2
Here from(iv) and (v) it is seen that the left and right limits are not same. So, the function does
not exist and hence the function is not continuous at x = 1/ 2.
1
Given f(x) =
x2
Let us investigate the continuity of this function. Here f(x) is not defined at x =2 [ i.e. f(2) is not
defined ] . So, f(x) is discontinuous at x=2. Again, let us consider another function f(x), where
x2 4
f(x) =
x2
This f(x) is also discontinuous at x=2 because f(2) is not defined [ here both numerator and
denominator are zero]. Moreover,
Lim f(x) = x+2 = 4 ≠ f(2)
x2
But this discontinuity can be removed if the function f(x) is redefined, where
x 2 4 ( x 2)( x 2)
f(x) = x2
x2 x2
Here Lim f(x) =Lim (x+2) = 4, i.e f(2) =4
x2 x2
x2 4
Let us see the graph of f(x) = and g(x)= x+2.
x2
38
Figure – 24 Figure – 25
In the second graph if the whole(0) is removed , the function will be continuous.
Example1.15: Show that
(i). f(x) = 3x3 – x +5 is continuous at x = 1.
(ii). f(x) =4x2 – 8x +3 is continuous at x = 2.
x 1
(iii). f(x) = is continuous at x=3.
x2
1
(iv) f(x) = is continuous for all x≠ 0.
x
Proof:
(i). We have f(1) = 3 .13 -1 +5= 7
Again, Lim f(x) = Lim 3 x3 – x +5 =7=f(1)
x 1 x 1
Hence f(x) is continuous.
(ii). We have f(2) = 4 . 22 – 8 .2 +3= 3
Again, Lim f(x) = Lim 4x2 – 8x +3 = Lim (2x – 3) ( 2x – 1 ) = 3
39
Lim x 1
x3 4
= 4 f (3)
Lim x 2 1
x3
Hence f(x) is continuous at x =3.
1
(iv) f(x) = can be defied every where except at x=0, and thus the function is continuous for all
x
:
Figure – 26
x ≠ 0. The graph of the function is shown above.
40
(ii). Lim x = Lim x = 0, and Lim f(x) = Lim x = 0
x 0 x 0 x 0 x 0
Thus, Lim f(x) = Lim f(x) = f(0) = 0
x 0 x 0
Hence f(x) is continuous at x = 0.
Example 1.17: Investigate the continuity of the function
f(x) = 5 – 3x, if x < 1
= x2 – 3x +4, if x ≥ 0
Solution: Since 5 – 3x and x2 – 3 x +4 are both polynomials , it follows that f(x) will be
continuous everywhere except possibly at x =1. Here
Lim 5 – 3x = 2 . Here Lim f(x) = Lim f(x) = 2
x 1
x 1 x 1
and Lim x2 – 3x +4 = 2 ( 1-0
x 1
The limit exists. Moreover, f(1) = 5 – 3 . 1 = 2 and f(1) = 12- 3 .1 +4 = 2 ( 1+) . Therefore, f(x) is
continuous at x = 1.
Example 1.18: Examine the continuity of the function
x2
f(x)=
x3
on the open interval – 2 < x < 3 and on the closed interval -2 ≤ x ≤ 3.
Solution: The rational function f(x) is continuous for all x except x = 3. Therefore, it is
continuous on the open interval – 2 < x < 3 but not on the closed interval - 2 ≤ x ≤ 3 since the
denominator becomes zero if x = 3.
Example 1.19: Examine the continuity of the function
3x 3 8
f(x) = at x = 0
x2
Solution: Here Lim (x – 2 ) ≠ 0. the limit exists. Now,
3x 3 8
Lim f(x) =
x2
x 0
Lim 3 x 3 8
x0 8
= 4
Lim( x 2) 2
x0
8
Again, f(0) = 4 . Hence the function is continuous at x = 0.
2
Example 1.20: Evaluate the limit of the function
f(x) = 3 x3 – 4x +8
at x = -1 and examine the continuity of the function.
Solution: Here Lim f(x) = 3 ( -1)3 – 4 (-1) + 8 =9
x 1
41
Again, f(-1) = 3(-1)3 – 4 (-1)+8 = 9. Hence the function f(x) is continuous at x = -1.
Example 1.21: Examine the continuity of the function
x2 1
f(x) = , if x < -1
x 1
= x2 – 3 , if x ≥ -1
x 1
Solution: Lim f(x) = Lim = Lim x – 1 = -2
x 1
x 11 x 11 x 1
Lim x2 – 3 = (-1)2- 3 = -2. Here Lim f(x) = Lim f(x)
x 1 x 11 x 1
Here limit exists and f(-1) = -2 for both the limits . Therefore, f(x) is continuous at x = -1.
Supplementary Problems
1.Investigate the continuity of the function at the specified value of x and find the limit of the
function.
(i). Lim ( x5 – 6 x4 +7), Ans. 7, continuous at x =0.;(ii) Lim (x – 1 )2(x+1), Ans.16,continuous at
x=3
x 0 x 3
x 1 3
(iii). Lim , Ans. ; continuous at x =2;
x2 4
x2
x3
(iv). Lim , Ans. Limit does not exist , discontinuous.
5 x
x 5
x2 1 x2 x 6
(v) Lim , Ans. 2, discontinuous. (vi) Lim 2 , Ans.5, discontinuous.
x 1 x 3x 2
x 1 x 2
x 3 x 10
2
42
3. Evaluate the limits.
4x5 9x 7 x x6 5
(i) Lim , Ans. 7 ; (ii) Lim , Ans. 4; (iii) Lim 2 , Ans.
3x x 1
6 3
x x2 3
x 1 x 1 x2
x 2 5x 6 2x 2 x 3 1 x 3 1
(iv) Lim 2 , Ans. -1; (v) Lim 2 , Ans. ; (vi) Lim , Ans.
x 3x 2 x 2x 3 2 x9 6
x 1 x 1 x 9
2 3x 2 5 x 1 x a 1
(vii) Lim , Ans. ; (viii) Lim , Ans. .
4x 2 xa 2 a
x 0 xa
x 3
1 x e2x e x 2
(ix) Lim , Ans. 0; (x) Lim , Ans. -1; (xi) Lim , Ans. 3
3 9 x2 1 x x
x 0 x x 0
1 8 1
(xii) Lim [ ] , Ans.
x 4 x 2 16 8
x 4
4. Show that
1 1
a
(i). Lim x{ x a x } ; (ii) Lim { ( x 1) x } 0
3 3
2
x x
ax 2 b
4 , Find the values of a and b; Ans. a=1, b= 4
5. If Lim x2
x2
6. Examine the continuity of the following functions at indicated value of x.
x2 9
(i) f(x)= x 3 , if x≠3 at x = 3, Ans. 6; continuous; (ii)f(x) = x+1, if≥ 1 Ans. 3, discontinuous
= 6, if x = 3. = 2x+1, if x<1 at x =1
(iii) f(x) =t 4, if x = 2; at x = 2; Ans. continuous at x = 2; (iv) f(x) = 4x+3 , if x ≠4, at x=4
= 2x, if x<2 = 3x=7, if x=4 Ans. continuous
at x =4
x 2 16
(v) f(x)= x 4 , if x ≠4 at x = 4 Ans. Not continuous (vi) f(x) = 2 – x , if x > 0 ; at x = 0
= 10, if x = 4 = 2 , if x = 0
= 2+ 3x, if x < 0; Ans
continuous, at x =0
1.6 More examples on Limits, Continuity and Graphs:
(i). show that y=f(x)= 3x+2 is continuous at x =4. Also, draw the graph of the function.
Solution: Let us consider that
y 3x 3
x 4 . We have y = 3x +2 or y – 3x = 2 or 1 x , y 0 if x = 4 , y = 14.
2 2 2
It indicates that the line has no break any where. It is seen that at x =4 , y= 14. The graph of the function
is shown below:
43
Figure – 27
We have Lim f(x) = Lim ( 3x+2) = Lim {3 ( 4+h )+2 } = Lim (14+3h) = 14
x 4 x 4 h 0 h 0
Again, Lim f(x) = Lim 3x + 2 = Lim { 3(x – h ) +2} = Lim ( 14 – 3 h ) = 14
x 4 x 4 h 0 h 0
Here , left limit = right limit; Lim f(x) = Lim f(x)
x 4 x 4
Moreover, f(4) = 3 x 4 + 2 = 14 . Hence f(x) is continuous at x = 4.
(ii). Evaluate
e3x e 2 x 2x log(1 x) x 3 125
(a) Lim , (b) Lim , (c) Lim
x ex 1 x 4 625
x 0 x 0 x 5
Solution:
e3x e 2 x 2x (e 3 x 1) (e 2 x 1) 2 x e3x 1 e2x 1
(a). Lim = Lim = Lim - Lim +2
x x x x
x 0 x 0 x 0 x 0
Let 3x = y and 2x = z. Then, we can write
e y 1 ez 1
Lim - Lim +2 = 3 x 1 – 2 x 1 + 2 = 3
y z
3 2
y0 z 0
x2 x3 x x2 x3
x ...... x(1 .....)
log(1 x) 2 3 2 3 4
(b). Lim = Lim = Lim
ex 1 x2 x3 x x2
1 x ..... 1 x(1 .....)
2! 3! 2 3!
x 0 x 0 x
44
x x2 x3
Lim (1 .....)
2 3 4
x0
= =1
x x2
Lim (1 ....)
2 3!
x0
x 3 125 ( x 5) ( x 2 5 x 25) x 2 5 x 25
( c ). Lim 4 = Lim = Lim
x 625 ( x 5)( x 5)( x 2 25) ( x 5)( x 2 25)
x 5 x 5 x 5
Lim ( x 5 x 25)
2
x5 3
= = .
Lim ( x 5)( x 25) 20
2
x5
x2
x2 x 2 ( x 1)( x 2) ( x 2)
(d) Lim = Lim = Lim . Limit does not exist
( x 1) 2
( x 1) 2
x 1
45
x 1 x 1 x 1
3
2x 2
( e) Lim 2 =Lim . Limit does not exist.
x 1 1 1
x x2
x x
1 1 2
2 3
x2 x 2 x 0 0
(f) Lim = Lim x x
4x 1
3
1 4
4 3
x
x x
(iv) Prove that
1 x x2
(a). Lim ; (b) Lim 1 ; (c) Lim
( x 2) 3 x 1 x 1
x 2 x x
Proof:
1
(a). Let A be any non-negative number. Let us take a value which is equal to the minimum of 1 and
A
1
. Assume that x < 2 and 0 < x 2 . Then x 2
3 3
. Hence
A
1 1 1 1
A A . But (x-2)3<0. Therefore, A . Hence Lim
( x 2) 3
( x 2) 3
( x 2) 3
( x 2) 3
x 2
1
(b). Let w be any positive number and let A = . Assume that x > A. Then
w
x 1 1 1 1
1 . Again, w . Hence
x 1 x 1 x 1 x A
Lim x
x x A
Lim = Lim 1.
x 1 Lim x 1 A
x
x2 1
(c). Lim = Lim
x 1 1 1
2
x x
x x .
f ( x h) f ( x )
(v). Given f(x)= x2 – 5 x, find Lim
h
h0
Solution: Since f(x) = x2 – 5x ; f( x +h) = (x +h)2 – 5 (x +h)= x2+2hx+h2 – 35x - 5h,
f(x +h) – f(x) = x2+2hx+h2 – 5x – 5h – x2+5x= h( h+2x – 5 ). Hence
46
f ( x h) f ( x )
Lim Lim (h+2x – 5)= 2x – 5
h
h0 h0
x2 a a x2
(vi) Evaluate Lim
x2
x 0
[ x 2 a a x 2 ][ x 2 a a x 2 ] ( x 2 a) (a x 2 )
Solution: Lim =Lim
x2[ x2 a a x2 ] x2[ x2 a a x2 ]
x 0 x 0
2 2 1
= Lim
x a ax
2 2
a a a
x 0
(1 x) 2 1 (1 x ) 2 2x x 2
(vi) Find Lim . Solution: Lim = Lim = Lim (2+x)= 2+0 = 2
x x x
x 0 x 0 x 0 x 0
1.7 Elements of set
Definition: A set is a collection of pre-defined and pre-identified group of elements. For example, the set
of first 5 natural numbers. Let us write these numbers by showing a pictorial method, where diagram is
named by A
A={ 1, 2, 3, 4, 5 }
A contains 5 elements, each one is independent of others and they are pre-identified [ first 5 natural
numbers] . Here the diagram is named A. Similarly, the other diagrams can be named B, C, D …. etc.
according to the contents of elements in the diagram. For example, let us consider another set of first 5
odd natural numbers. Here the diagram can be named by B, where
B= { 1, 3, 5, 7, 9 }
Again, consider another set of first 5 natural even numbers, where the set is
C = { 2, 4, 6. 8, 10 }
Here the elements in B or in C are pre-defined and all the elements in B or in C are independent of each
other. No elements either in B or in C is repeated. The pictures A and B and C are called Venn Diagram.
In each diagram there are 5 elements. Each element in a set is called a member of the set. The
membership of the set is shown ,usually, in the notation x A or x B or so on, where x is used to
denote the member of the set. Here the set A can be shown as follows:
A = { x : x= 1, 2, 3, 4, 5 }
Similarly , B ={ 2x+1 : x = 0, 1, 2, 3, 4 } and C = { 2x : x= 1, 2, 3, 4, 5 }. Let us write another set
D,where
D= { 2x +1 : x = 1, 2, 3, 4, 5 }
Here 11 is a member of D [ 11 D] but 1 is not a member of D [ 1 D ].
Subset : If every element of a set A is the element of another set B , then A is called the sub set of B. For
example, let B = { 7, 8, 9, 10, 11, 12, 13 } and A = { 7, 8, 9, 10, 11 }. Here all element of A are the
elements of B. Hence, by definition, A is the subset of B. It is written as A B.[ A is contained in B ].
But all the elements of B are not the elements of A . It is written as B A [ B is not contained in A ,
rather B includes A or B contains A]. In such a case B is called super set of A. Here A is the proper sub
set of B as all members of A are the members of B. But A is not equal to B. By definition, A is itself a
sub set of A.
47
Equal or identical set: Two sets A and B are said to be equal or identical, if and only if all members of
A are all the members of B. It is denoted by A B or B A. It is also written as A = B or B = A.
Here A is a sub set of B and B is a sub set of A. For example, let A = {3, 5, 7, 9, 11} and B = { 11, 9, 7, 5,
3 } . It can be written A= B = {2x +1; x = 1, 2, 3, 4, 5 ] . Here A is a sub set of B and B is a sub set of A
and A is called a real sub set of B as all the elements of A are the elements of B. The sets A and B are not
equal, if at least one element of A does not contain in B .
A portion of a set S is a set of nonempty subsets of X such that every element x in X is in exactly one of
these sub sets .
Finite set: If a set A contains a definite number of elements , the set is called finite set. The members of
the set are countable. For example, let us consider the set A as follows:
A = { 5x : x = 1, 2, 3, 4, 5 } Here A = { 5, 10. 15, 20, 25 }
There are 5 members in the set A. This number is countable finite and hence the set A is finite set. Here
counting of the members of the set is ended.
Infinite set: If the counting of the number of members of any set is not ended , then the set is known as
infinite set. For example, let us consider the set B as follows:
B ={ x : 2 < x < 5 }
Here the members of the set B are all values within the limit 2 to 5. There is no end of counting of the
members of the set. This type of set is called infinite set.
Null set: If there is no element in any set, then the set is called null set or empty set. It is denoted by
.For example, let us consider a set with element H H H when two coins are tossed, where the set of the
result of the experiment is S = { HH, HT, TH, TT }. In this set there is no element HHH. This element is
impossible for the said experiment. Thus , the set with element HHH in tossing two coins is an impossible
result and hence it will form an empty set, ={ x: x =HHH, if two coins are tossed }. Consider another
set = { x: x2+1=0, where x is real }. This set is also empty set as x2+1=0 has imaginary roots. However,
an empty set may be considered as a subset of any set. For example, let S= {x: x= 1,2,3},
then ={x: x= 4} is a sub set of S , as 4 is an impossible element of S.
Universal set: If a set contains all possible elements of a specified property , the set is called universal
set. For example, the set of experimental results in throwing a die is a universal set, where the set is
U or S = { 1,2,3,4,5,6 ] as the results are either 1 or 2 or3 or 4 or 5 or 6 or 7. These 7 results are the
possible elements of a set. So, it forms a universal set. Universal set is denoted either by U or S. in
selecting a card from a pack of 52 card the universal set will be
U = { x : x = 1,2,3,4,……….,52 }.
Power set: The power of a set U is the set of all sub sets of U. For example, let us consider the universal
set U= {x: x= first 3 even natural numbers }, i.e. U = { 2,4,6 }. Now, the sets [{ 2,4,6}, {2.4},{2.6},{4,6),
{2}, {4}, {6}, ] are the power sets of the universal set U. The power set is usually written as P (U) or
P(s).
The power set of a finite set with n elements has 2n elements. As an example, we can mention the power
set of the set U={2,4,6}, where n = 3 elements in U . So, the elements in P(U) are 8 = 23. This relationship
is one of the reason s for the terminology power set.
The power set of an infinite set, countable or uncountable, is always uncountable. Moreover, the power
set of a set is always strictly bigger than the original set in the sense that there is no way to pair every
element of U with exactly one element of P(U).
Every partition of a set U is a sub set of the power set U.
Cardinality: The cardinality U of a set U is the number of members of U. As an example, we can
mention the cardinality of the set U= {2,4,6} as mentioned above is U =3.
There is a unique set with no numbers and zero cardinality, which is called empty set and is denoted by
. For example, the set of all three-sided coin has zero members and thus is the empty set. Some set has
infinite cardinality . For example, the cardinality of the set U= {2x +1: x = 0, 1, 2, 3, 4, …..n} is infinite.
1.8 Basic operation of sets.
48
Union of sets: Let A and B be any two sets . The union of these two sets A and B , denoted by, A U B , is
the set of all elements those are either elements of A or B. For example, let A ={ 1,2,3},and B = { 4,5,6 }.
Then, the union of the sets A and B is AUB= { 1,2,3,4,5,6 }. Again , let C ={ 1,2,3,4 } and D={ 3, 4, 5, 6
}. Then, the union of two sets C and D is CUD= { 1,2,3,4,5,6 }. Further, let E = {1,2 } and D = { 1,2 }.
Then,
EUD= {1,2 }. In general, AUB= { x : x A or x B }.
Some basic properties of unions:
(i) AUB = BUA, it is commutative law,
(ii) AU(BUC)= (AUB)UC, it is associative law,
(iii) A (AUB),
(iv) AUA= A, it is idempotency,
(v) AU =A,
(vi) A B, if and only if AUB = B.
Intersections of sets:
Let A and B be two sets. The intersection of A and B , denoted by A B, is the set of all common
elements of both A and B. For example, let A = { 1,2,3,4 } and B { 3,4,5,6 }, then A B= { 3,4}. Again,
let A = { 7,8, 9} and B = { 7,8,9 }, then A B={7,8,9 }. Further, let us consider that A= { 2,4,6} and B
={ 1,3,5}, then A B = , empty set as there is no common elements in the sets A and B. Here A and B
are called disjoint sets.
Some basic properties of intersections:
(i) A B=B A, it is commutative law,
(ii) A ( B C) = ( A B ) C, it is associative law,
(iii) A B A,
(iv) A A=A, idempotency
(v) A ,
(vi) A B , if and only if A B =A .
Difference of set: Difference of set S and A , denoted by S \A , is the set of all elements of S those are
not elements of A. For example, let S = { 1,2,3,4 } and A = { 3,4,5,6 ]. Then the difference of set S and A
is
S\A = {1,2 }. Again, the difference of and S is given by A\S = {5.6 }.
Symmetric difference: For two sets A and B the symmetric difference , denoted by A B, is the set of
all elements those are members of exactly one of A and B [ elements which are in one of the sets, but not
in both ]. For example, let A = { 1,2,3 } and B={ 2,3,4 }. Then the symmetric difference of A and B is
given by A B= { 1,4 ]. It is the set difference of the union and the intersection.
(A UB)\ ( A B) or( A\B) U ( B\A).
The difference of two sets A and B can also be denoted by A – B . From the example cited above
A–B≠B–A
Also, ( A – B ) C , B – A B.
Clearly, A – B and B – A are disjoint.
Complements: It has been seen that the difference of set A – B is the set of all elements those are
elements of A but not elements of B. If S is the universal set and A is a subset of S , then S – A is called
the absolute complement of or simply complement of A. For example, if S={ 1,2,3,4,} and A ={ 1,3],
where A is a subset of S , then S – A : { 2,4 } . Here S – A is a complementary set of A and its denoted by
A/ or Ac or A . Here
{1,3} {2,4}
S . Again, S= 1,2,3 4 5,6
A A
A B
49
Figure – 28
Cartesian Product: If A and B are any two sets, then their cartesian product, denoted by A X B, is the set
whose elements are all possible pairs (a, b) where a is any element of A and b is any element of B. Let us
consider that A = {1,2}, B = {C1, C2}, where c1= colour -1 and c2= colour – 2. Then
the cartesian product of A and B is {1, c1}, {1, c2}, {2, c1} and (2, c2}.
50
= { x ; x belongs to at least one of An, An+1, ………..}
A A B B
2 4,6,8 10,12
Figure – 29
51
Here A B=B A and each of A and B contains A B as a subset, i.e. A A B and B A B.
(ix). Disjoint set: Let A = {1,2,3,4} and B= {5,6,7,8,9}. Here A and B has no common elements, i.e.
A B= , the empty set. The sets A and B are called disjoint sets. We can show it by Venn diagram:
A B
5,6,7,8,
1,2,3,4
9
Figure – 30
(x). Difference of sets : Let A = { 2,4,6,8,10} and B = { 1,2,3,4,5,6}. Then A – B = { 1,3,5}.
(xi). Product of sets: Let A= { 2,3,} and B= { 4,5,6}. Then
A x B = { (x, y): x }. We can write
A x B = { (2,4), (2,5),(2,6),(3,4),(3,5), (3,6) }
(xii). Power set: Given A ={ 4,5,6}, the power set of A. As there are 3 elements in the set A, the number
of power set will be 23=8. We have
P(A): { , {4}, {5}, {6}, {4,5}, { 4,6}, { 5,6}, {4,5,6} }
(xiii). Given a set A = { 1,2,3,4,5,6,7,8,9}, find a subset B of A such that B: { x : x2 A }.
By definition of B , the elements of B should be the square root of the elements of A. Such elements of A
are { 4,9} and the square root of these two elements are {2,3}. Therefore, the subset is B={ 4,9}. Here we
can write B: { x: x2 A }.
(xiv). Given AUB= {2,4,6,8}, AUC={4,6,8,10}, A ={ 4,6]} and A C = { 4,8}. Find the elements of
the set A,B and C.
Solution: We have AUB=[2,4,6,8} ……(1); AUC={ 4,6,8,10}………(2); A =[,6}…..(3) and A C
={4,8}… (4)
From (3) and (4) , we have the elements 4,6,8 in A , i.e, {4,6,8} . Similarly, {4,6} and {4,8} C.
From (1) and (2), it is seen that 2 and 10 A. But 2 B and 10 C. Hence, the sets are
A= { 4,6,8}, B ={ 2,4,6}, and C =[ 4,8,10}.
(xv). Let A={1,2,3}, B={1,2}, C={1,2,4}, D={3,4}, E={4}. Explain the sets (a) B A, (b) D E, (c) D
B,
and (d) {1} A.B
Solution:
(a). Here the elements of B are the elements of A also. Hence B is the subset of A, i.e. B A.
(b). Here E has only one element and this element is 4 and it is also an element of D. So, D E.
( c ). The elements of B are not the elements of D . Again, the elements of D are not the elements of B.
Hence D B is not the correct statement. Here B and D are disjoint sets.
(d). The elements of A are 1, 2, and 3. Here 1 is an l element of A. Here, {1} is a proper subset of A.
Hence {1} A is correct.
(xvi). If U={1,2,3,4,5,6} be the universal set and A,B, and C are three subsets of U , where A={1,3,4,6},
B C ={1,2,6}, find ( A U B ) (AUC), and ̅ U ̅ .
Solution: By distributive laws of sets, we have
( A U B ) (AUC) = AU ( B C)= {1,3,4,6} U {1,2,6}= {1,2,3,4,6}.
By Morgan‟s law , we have
52
̅ U ̅ = ̅̅̅̅̅̅̅ = U – ( B C ) = {1,2,3,4,5,6} – { 1,2,6}={3,4,5}.
(xvii). Given A={ 2,4,6,8}, B= { 4,6,8,10}, C={2,6,8,10,12,14}, show that A
53
Solution: Let n(.) be the cardinal number of any set. Given n(T1) = 40, n(T2)=60 and n(T1 T2)= 35. We
need n (T1U T2). We can have
n (T1 UT2) = n(T1) + n(T2) – n (T1 T2) = 40 + 60 – 35 = 65.
(xxiv). A distributor supplies 150 pieces of Samsung television, 65 pieces of Philips television in a week
to retail shops. Some shops take 50 pieces of both the televisions. How many pieces of televisions are
supplied in a week?
Solution: Let S = Samsung, P = Philips; n(.)= cardinal number of any set. We need n (SUP). We can write
n (SUP) = n(S) + n(P) – n(S P); given n(S)= 150, N(P)= 65 and n(S P)= 50.
= 150 + 65 – 50
= 135.
(xxv). 60% customers, male or female, are satisfied with the service of the office of a mobile operator.
70% female customers and 55% male customers are satisfied. Find percentage of customers of both sexes
who are satisfied.
Solution: Let n(.) be the cardinal number of any set; M be the male customers and F be the female
customers. Given n(M)=55, n(F)=70 and n(MUF)= 60. We need , n(M F). We know
n (MUF) = n(M) + n(F) - n(M F)
or n(M F) = n(M) +n(F) – n(MUF)= 55 + 70 – 60 = 65, or 65%.
54
Chapter II
Determinant
2.1 Introduction
Let us consider that a garments industry has two units located in two places. The owner of the industry
observed the production after every two days. The first unit produced 250 pieces in an hour of two days,
where the production of each day is 125. The second unit produced 200 pieces in an hour of first day and
150 pieces in the second day. There are 4 observations of productions of an hour in two days from two
units which can be shown in rows and columns as follows:
125 200
A=
125 150
The above arrangement of 22 = 4 observations in two rows and in two columns is known as determinant
of order 2 x 2. In a similar way, the production data of n days from n units can be shown in a determinant
B having n rows and n columns as follows:
Here aij ( i = 1, 2, 3, ………,n ; j = 1, 2, 3,……..,n ) is the production of i-th day from j-th unit. The
value of n may be 2 or 3 or 4 or ……….. Here for determinant A the value of n = 2 and A is called a
determinant of order 2 and this order is written as 2 x 2. The determinant B has n2 elements in n rows and
n columns and B is known as determinant of order n [ n x n ].
The determinant has a particular value. For example, the value of the determinant A shown above is
55
Braches of the bank Service hour - 1 Service hour – 2 Service hour – 3
1 2 3 2
2 1 2 3
3 1 1 1
Now, let us represent these information by a determinant C as shown below:
2 3 2
C= 1 2 3
1 1 1
This determinant C is of order 3 [ 3 x 3] and it has a value, where the value is calculated as follows:
2 3 1 3 1 2
ICI=2 -3 +2
1 1 1 1 1 1
=2(2x1–3x1)–3(1x1-3x1)+2(1x1–2x1)
= 2( - 1 ) – 3 ( -2 ) + 2 ( - 1 ) = 2
Here the determinant C is of order 3 and in calculating the value of C we have formed another 3
determinants of order (3 – 1) = 2. These new determinants are say, C1 , C2 and C3 , where C1 is formed
with the elements except the elements of first row and first column, that is
2 3
C1 =
1 1
Here C1 is formed with respect to the elements of first row and first column. In a similar way, C2 is
formed corresponding to the elements of first row and second column and C3 is formed corresponding to
the elements of first row and third column. Thus, we have
1 3 1 2
C2 = and C3 =
1 1 1 1
Thus ,there are ( 3 -1 ) = 2 new arrangements of 22 elements in 2 rows and 2 columns. These new
arrangements are known as minors formed from the original determinant C. These minors are also
determinants of order 2. If we have a determinant of order n, we can have n2 minors of order ( n – 1 ).
Minor: The minor of the elements of a determinant of order n x n is the arrangement of element (
n – 1 )2 in ( n – 1 ) rows and ( n – 1 ) columns. Thus the minor is a newly formed determinant of order
( n – 1 ). However, it has no particular value like the value of a determinant. In section 2.1 we have
formed 3 minors C1 , C2 and C3 corresponding to the elements of first, second and third columns,
respectively of first row of the determinant C. As ai j is the element of i-th row corresponding to j-th
56
column [ i =1, 2, ……….,n; j =1, 2, 3, ………,n] of a matrix A, we can have minor Ai j corresponding
to the element of i-th row and j-th column of the original determinant. The other minors of the
determinant C shown in section 2.1 are shown below:
3 2 2 2 2 3 3 2 2 2 2 3
C21 = , C22 = , C23 = , C31= , C32= and C33 = .
1 1 1 1 1 1 2 3 1 1 1 2
Cofactor: The definite value of the minor of any element of a determinant is called cofactor. Thus, if A11
is a minor of element of first row and first column of the determinant A , then cofactor of the minor A11 is
IA11i. For example the cofactor of C21 observed above is I A21I = 3 x 1 – 2 x 1= 1. But in calculating the
cofactor the value I Ai j I is multiplied by ( - 1 )i+j , where Ai j is the minor of the element ai j of the
determinant A. Hence the cofactor corresponding to C21 = ( - )2 + 1x 1= - 1. Finally, if ai j is the element of
i – th row corresponding to j – th column of a determinant A , then minor of ai j is A I j and cofactor of ai j
is ( - 1 )i+j I Ai j I.
Example 2.1: A company produces electric bulbs using 3 plants P1, P2 and P3. Each plant produces
defective bulbs. The number of defective bulbs are recorded in 3 consecutive days by the Quality Control
Department of the company. The recorded information are shown below.
6 8 3
A = 4 10 5
3 6 4
The elements of first row are 6, 8, and 3. The minors and the cofactors corresponding to these elements
are
10 5 4 5 4 10
A11 = , A12= , A13= , IA11I = ( - 1 )1+1 ( 10 x 4 – 5 x 6 ) = 10.
6 4 3 4 3 6
Let A be a determinant of order n x n with elements ai j in j-th column corresponding to i –th row, where
A is given below:
57
a11 a12 a13 ............. a1n
a 21 a 22 a 23 ..............a 2 n
.......................................
a n1 a n 2 a n 3 ..............a n n
n xn
Let Ai j be the cofactor of the element ai j of the determinant A , where elements of Ai j are n x n. Then
adjoint of the determinant of A is defined by
adj A = I Ai j In x n
Here the new determinant formed with the cofactor values Aij in transpose form is the adjoint determinant
of A. For example, let us consider the following determinant A so that we can find adjoint of A:
2 4 6
A=3 5 7
1 2 3
A13 = ( - 1 )1 + 3 ( 3 x 2 – 5 x 1 )= 1
Example2.2: In an office of a multinational company there are some male and female employees. Some
of the employees remain absent every day. In two consecutive days the number of absent employees are
1 6 4
2 7 6
Total 13 10
58
Arrange the number of absent employees in a determinant A and hence find the adjoint of A.
6 4 a11 a12
A
7 6 a21 a22
A11 = ( - 1 )1 +1 Ia22I= 6, A12= ( - 1 )1 +2Ia21I= -7, A21= ( - 1 )2 +1Ia12I= -4, A22 = ( -1 )2 +2Ia12I = 6. Hence,
6 7
A/
4 6
Example 2.3: The production ( in million tons ) of rice in 3 places in three consecutive years are shown
below:
1 12 9 11
2 10 12 14
3 15 10 16
Represent the data by a determinant and find the corresponding adjoint determinant.
Solution: Let A be the determinant to represent the production data of 3 places in 3 consecutive years,
59
12 9 11
A 10 12 14
15 10 16
Let Mi j be the minor corresponding to the element ai j of the determinant A. The minors are shown below:
12 14 10 14 10 12 9 11 12 11
M 11 , M 12 , M 13 , M 21 , M 22
10 16 15 16 15 10 10 16 15 16
12 9 9 11 12 11 12 9
M 23 , M 31 , M 32 , M 33
15 10 12 14 10 14 10 12
Each and every determinant has a definite value. For example, the value of the determinant A given in
Example 2.3 is
60
12 9 11
A 10 12 14
15 10 16
IAI = 12 ( 12 x 16 – 14 x 10 ) – 9 ( 10 x 16 – 14 x 15 ) + 11 ( 10 x 10 – 12 x 15 ) =194
It is seen that I A I = a11A11 + a12 A12 + a13 A13. For the determinant of order n x n the value of the
determinant is given by
61
4 3 1
Solution: Consider that the determinant A = 2 5 3 . The value of this determinant is
6 2 1
I A I = 4 ( 5 x 1 – 3 x 2 ) -3 ( 2 x 1 – 3 x 6 ) + 1 ( 2 x 2 – 5 x 6 ) = 4( - 1 ) – 3( - 16 ) + 1 ( - 26 ) = 18
Let us form a new determinant by interchanging two rows of A mutually. The new determinant is
2 5 3
B= 6 2 1
4 3 1
Here I B I = 2 ( 2 x 1 -1 x 3 ) – 5 ( 6 x 1 – 1 x 4 ) + 3 ( 6 x 3 – 2 x 4 ) = 2( - 1 ) – 5 ( 2 ) + 3 ( 10 ) = 18
Due to even number of mutually interchanged rows the value of the determinant is not changed.
( iii ) If two rows ( columns) of a determinant are identical, then the value of the determinant is zero.
5 8 9
For example, let us consider the determinant A = 3 4 10 . Two rows of the determinant A are
5 8 9
identical [ same ]. Here
I A I = 5 ( 4 x 9 – 10 x 8 ) – 8 ( 3 x 9 – 10 x 5 ) + 9 ( 3x 8 – 4 x 5 ) = 5( - 44) – 8 ( -23 ) + 9 ( 4 )= 0
( iv ) If every element of any row or column of a determinant is multiplied by a constant k, then the value
of the determinant is k I A I [ k multiple of the value of the original determinant.]. For example, let us
consider the determinant A as given below
3 5 7
A= 2 4 6 ,
4 2 8
I A I = 3 ( 4 x 8 – 6 x 2 ) – 5( 2 x 8 – 6 x 4 ) + 7 ( 2 x 2 – 4 x 4) = 3( 20) – 5( -8 ) + 7( -12) = 16
Let us multiply the first row of the determinant A by k =5, where
15 25 35
B =5 A = 2 4 6
4 2 8
I B I = 15 ( 4 x 8 – 6 x 2 ) – 25 ( 2 x 8 – 6 x 4 ) + 35 ( 2 x 2 – 4 x 4 ) = 15( 20 ) – 25 ( - 8 ) + 35( - 12 ) =
80
It is seen that I B I = 5 I A I.
In a similar way, it can be said that if every element of every row ( column) of a determinant is
multiplied by a constant k, then the new determinant value becomes kn multiple of the original
determinant value , where the order of the determinant is n x n.
( v ) If the elements of any row ( column) are multiplied by the cofactor of the elements of any other row (
column) and the products are added , the result will be zero. For example., let us consider the determinant
A as given below:
62
4 6 8
A= 3 5 7
2 4 9
Let Ai j [ i = 1 2 3 ; j= 1,2,3 ] be the cofactor of the element ai j in determinant A. Then
A11 = ( - 1 )1 + 1( 5 x 9 – 7 x 4 ) = 17, A12= ( -1 )1 + 2 ( 3 x 9 – 7 x 2 ) = - 13, A13 = ( - 1 )1 + 3( 3 x 4 – 5 x 2 )=
2
Now, let us calculate a21 A11 + a22 A12 + a23 A13 = 3 x 17 + 5 x (-13 ) + 7 x 2 = 0. This implies that
∑ ai j Al j = 0 ,when i l= 1,2,3,….,n.
( vi ) If each element of any row ( column) of a determinant is the sum ( difference ) of two observations,
then determinant can be expressed as sum ( difference ) of two determinants when other elements remain
unchanged. For example, let us consider the determinant A as follows
53 7 4
A=
4 6
Here, we can write A = A1 + A2 or A = A1 – A2 . Let us verify these identities. We have, by statement,
5 7 3 4
A1 = , A2 = . Here IA1I = 5 X 6 – 7 X 4 = 2, I A2I = 3 X 6 – 4 X 4 = 2.
4 6 4 6
2 3
Again, I A I = 8 x 6 – 11 x 4= I A1I + I A2 I= 2 + 2. Again, I A1I – I A2I = 2 – 2 = 0. Again, if A =
4 6
when the elements of first row are the differences of two observations, then I A I = 2 X 6 – 3 X 4 = 0.
( vii ) If the elements of any row ( column) are added with the corresponding elements of any other row
( column), then the determinant value is not changed. Also, if the elements of any row ( column ) are
multiplied by a constant and added the result with the corresponding element of any other row ( column),
then the value of the determinant remains unchanged. For example, let us consider the determinant A as
given below
7 10
A= where I A I = 7 X 8 – 10 x 4 = 16
4 8
By adding the elements of second row with the corresponding elements of first row, we have
11 18
A1 = , where I A1I = 11 x 8 – 18 x 4 = 16
4 8
Again, let us add the 3 times of the elements of second row with the corresponding elements of first row
to get a new determinant, say A2 , where
19 34
A2 = , where I A2 I = 19 x 8 – 34 x 4 = 16. Hence the result.
4 8
( viii ) If a determinant can be transformed to diagonal form, then the determinant value is the product of
the diagonal elements of the determinant. For Example, let us consider the diagonal determinants shown
below:
5 0 0
A = 0 4 0 , where I A I = 5( 4 X 6 – 0X 0 ) – 0 ( 0 X 6 – 0 X 0 ) + 0 ( 0 – 4 X 0 )= 5 X 4 X6 = 120
0 0 6
63
This result is nothing but the product of the diagonal elements of the determinant. In a similar way, we
can write the determinant value of an n x n determinant as follows:
a11 0 0 ........0
0 a 22 0...........0
A= 0 0 a33 ..... ...0 , where I A I = a11 x a12 x ………….x an n
................................
0 0 0............a nn
( ix ) Cramer’s Rule:
This rule is used to solve the linear equations using the values of the determinant. Let us see how the
method is used in solving two linear equations in two variables ( unknown values ) shown below:
a11 x1 + a12 x2 = b1
a21 x1 + a22 x2 = b2
In matrix notation, the equations can be shown as follows:
AX=B
a11 a12 x1 b1
Here A = , X , B
a 21 a 22 x2 b2
By Cramer‟s rule, the value of xj [ j = 1,2 ] is given by
Aj
xj =
A
where determinant Aj is formed with other columns except the j –th column and the vector B of the
constant. In the above case
a12 b1 a11 b1
A1 = , A2 =
a 22 b2 a12 b2
Let us apply this rule to solve the following two simultaneous equations for the variables income ( y ) and
rate of interest ( i ).
0.5 y + 25 i = 5175
0.8 y + 50 i = 8350
The problem is to find the value of y and i. Here we can apply the Cramer‟s rule to solve the equations.
We get
0.5 25 25 5175 0.5 5175
A= , A1 , A2
0.8 50 50 8350 0.8 8350
I A I = 0.5 X 50 – 25 X 0.8 = 5, I A1I = 25 X 8350 – 5175 X 50 = - 50000, I A2I = 0.5 X 8350 – 5175 X
0.8 = 35
According to the solution proposed by Cramer‟s rule, the value of y (income ) will be negative due to the
value I A1 I is negative. But income cannot be negative. To solve this problem, we have to consider the
positive value of I A1 I as value of a determinant can be positive or can be negative [ A ]. Now
A1 50000 A 35
y= 10000 , i 2 7
A 5 A 5
64
Let us consider the solution of 3 linear equations involving three variables [ unknown ] , where are
equations are
a11 x + a12 y + a13 z = b1
a21 x + a22 y + a23 z = b2
a31 x + a32 y + a33 z = b3
In matrix notation, the equations are
A X = B , where determinant A , and vectors X and B are given
below:
a11 a12 a13 x b1
A = a 21 a 22 a 23 ,X y , B b2 . Assume that = IA I 0
a31 a32 a33 z b3
Let us multiply the first column of by x to get
a11 x a12 a13
x = a 21 x a 22 a 23
a31 x a32 a33
In a similar way we can multiply second column of by y and third column of by z and add to the first
column of x to get the following equation
a11 x a12 y a13 z a12 a13 b1 a12 a13
x a 21 x a 22 y a 23 z a 22 a 23 b2 a 22 a 23 1 A1 ( say)
a31 x a32 y a33 z a32 a33 b3 a32 a33
1 A1
Then x = , provided 0
A
In a similar way, multiplying second column of by y, we get
a11 a12 y a13
y = a 21 a 22 y a 23
a31 a32 y a33
Again , multiplying first column of by x and third column by z and add to the second column of y to
get the following equation
a11 b1 a13
y a 21 b2 a 23 2 , say
a31 b3 a33
a11 b1 a13
A2
There fore, y = 2 = , where A2 = a 21 b2 a 23
A
a31 b3 a33
65
a11 a12 b1
3 A3
Similarly, we have z = , where A3 = a 21 a 22 b2 , and A3 3
A
a31 a32 b3
x y z 1
From these results, we can write and if 0 , we can get unique solution of the
1 2 3
equations and the equations are consistent for solution. Also, the equations are independent.
If 0 , and if at least one of 1 , 2 3 is not zero, then the solution of the equations is not available.
In that case the equations are called inconsistent.
If 0, and 1 2 3 0 , the solution of the equations may or may not be available. The
equations may or may not be consistent. In such a case if unique solution is available, then the equations
may be consistent and independent. However, if infinite solutions are available, the equations may be
consistent but not independent.
Example 2.5: Solve the following system of linear equations using Cramer‟s rule.
X + y +2z = 5 , 3x + y – 3z = 2 , 2x + y – z = 4
Solution: The equations in matrix notation is A X = B, where
1 1 2 x 5
A = 3 1 3 , X y , B 2 .
2 1 1 z 4
5 1 2 1 5 2 1 1 5
Also, we can have A1 2 1 3 , A2 3 2 3 , A3 3 1 2
4 1 1 2 4 1 2 1 4
Here IA I = = 1 [ 1x(-1) – (-3)x 1] – 1[3 x(-1) – (-3) x 2] + 2[ 3 x 1 – 1 x 2] =-1(-1+3) -1(-3+6) +2(3-
2)= 1
IA1I = 1 = 5( -1+3) -1( -2+12) + 2( 2 – 4 ) = -4, IA2I = 2 = 1(-2+12) -5( -3+6) + 2 ( 12 – 4 )= 11
IA3I = 3 = 1( 4-2) -1( 12-4) + 5( 3 – 2 ) = -1. Now
1 4 11 1
x= 4, y = 2 11 , and z = 3 1 .
1 1 1
Example 2.6: An equilibrium condition of labour input ( x ), capital input ( y ) and raw material ( z )
along with output is given below:
x + y + z = 3 , 2x – 3y + 5z = 4 and x + 2y – 4z = -1
Solve the above system of equations by Cramer‟s rule.
Solution: The determinant A, A1, A2, and A3 to apply Cramer‟s rule are shown below:
1 1 1 3 1 1 1 3 1 1 1 3
A 2 3 5 , A1 4 3 5 , A2 2 4 5 , A3 2 3 4
1 2 4 1 2 4 1 1 4 1 2 1
Here IAI = !( 12 – 10 ) -1 ( -8 -5 ) + 1( 4 + 3 ) = 22, I A1 I = 3( 12 – 10 ) – 1 ( -16 + 5 ) + 1 ( 8 – 3 ) = 22
I A2 I = 1( -16 + 5 ) -3 ( -8 – 5 ) = 1 ( -2 – 4 ) = 22, I A3I = 1 ( 3 – 8 ) -1 ( - 2 – 4 ) +3 ( 4 + 3 ) = 22
66
A1 22 A2 22 A3 22
Therefore, x 1, y 1, z 1
A 22 A 22 A 22
Example 2.7: The equilibrium condition of demand ( Dc) of a commodity, the demand of a substitute (Ds )
of that commodity and the supply ( S ) of that commodity is given by
5 Dc + Ds – 3 S = 3
4 DC + 2 Ds – 5 S = 1
Dc - Ds + S =1
Find the values of Dc , Ds and S using Cramer‟s rule.
Solution: From the system of equations and to apply Cramer‟s, we can get the different determinants as
follows:
5 1 3 3 1 3 5 3 3 5 1 3
A 4 2 5, A1 1 2 5 , A2 4 1 5 , A3 4 2 1
1 1 1 1 1 1 1 1 1 1 1 1
The values of these determinant are as follows:
I A I = 5 { 2 x 1 – ( - 5 x -1) } – 1 { 4 x 1 – ( -5 x 1) } + ( - 3 e) { 4 x (-1 ) – 1 x 2 } = - 6.=
I A1 I = 3{ 2 x 1 – (-5) ( -1 )} – 1 { 1 X 1 – ( - 5 ) x 1 } + ( - 3 ) { 1 x ( -1 ) – 1 x2 } = - 6 = 1
I A2 I = 5 { 1 x 1 – 1 ( - 5 ) } – 3 { 4 x 1 – ( - 5 ) x 1 } + ( - 3 ) { 4 x 1 – 1 x 1 } = - 6 = 2
I A3 I = 5 { 2 x 1 – 1 x ( - 1 ) } – 1 { 4 x 1 – 1 x 1 } + 3 { ( - 1 ) x 4 – 1 x 2 } = - 6 = 3
A1 6 A 6 A 6
Therefore, Dc 1, Ds 2 1, S 3 1 ; DC = Ds = S.
A 6 A 6 A 6
Example 2.8: A company produces bulbs in three plants in each place. The plants are established in two
places. The production of bulbs ( in 000 pieces ) are recorded in 3 days in both the places. The production
data of both the places are shown below:
Find the total production of the company in 3 days for each plant. Show that the total production in two
places are not same.
Solution: Let us arrange the production of each place by two different determinants, say, A1 and A2 for
places- 1 and places-2, respectively. Then, we need A1 + A2 . Also, we need to show that IA1 I I A2 I.
Here I A1 I= 3 ( 1 x 2 – 2 x 2 ) – 2 ( 2 x 2 – 1 x2) +2 ( 2 x 2 – 1 x1 ) = -4
67
3 2 2 2 2 1
A1 = 2 1 2 , A2 = 2 3 2
1 2 2 1 1 1
3 2 2 2 2 1 5 4 3
Here A1 + A2 = 2 2 1 3 2 2 4 4 4
11 2 1 2 1 2 3 3
Example 2.9 : The amount of sales ( in million Taka ) of two products A and B of a company in two
departmental stores S1 and S2 at the end of a year and at the end of first 3 months are shown below
separately.
A B A B
S1 8 5 S1 3 2
S2 9 6 S2 4 2
Solution: Let A be the determinant of the amount of sales in a year and B be the amount of sales in first 3
months of that year. We need A – B . Here
8 5 3 2 83 52 5 3
A= , B , A B
9 6 4 2 94 62 5 4
Example 2.10: A buying house purchased three items A, B and C from three garments industry G1 , G2
and G3. Every item ( in 000 pieces ) is purchased from each industry. The information of number of items
purchased and the price ( in Taka ) per piece are given below:
Solution: Let A be the determinant of the number of items purchased and B be the vector of price per
piece of the item. We need B A and the sum of the elements of rows of BA separately. Here
68
10 8 6 150 150 10 8 6 10 150 8 150 6 150 1500 1200 900
A 5 3 3 , B 160 , BA 160 5 3 3 5 160 3 160 3 160 800 480 480
4 6 5 100 100 4 6 5 4 100 6 100 5 100 400 600 500
Therefore, item wise total costs are
For item A = 1500 + 1200 + 900 = 3600, For item B = 800 + 480 + 480 = 1760 and
Example 2.11: Find the value of x in terms of a, b and c using the equation given below:
xa b c x a a
(i) c xb a 0, (ii ) a x b =0
a b xc b b x
Solution: ( i ) By adding second and third column with first column, we get
xabc b c 1 b c
c x b a x b a 0 or ( x + a +b + c ) 1 x b a 0
ab xc b xc 1 b xc
1 b c
( x + a +b + c ) 0 x ac 0 or ( x + a +b + c ) x2 = 0
0 0 c
x a a
( ii ) Given a x b 0
b b x
69
1 0 0
( x+a+b ) a xa ba 0
b 0 x b
Or ( x + a + b ) ( x –a ) ( x – b ) = 0
Either x + a + b = 0, or ( x – a ) ( x – b) = 0
Therefore, x = - a – b ; x = a ; x = b.
1 2 0
0 2 x 0
1 0 2
1 2 0
A= 0 2 x
1 0 2
4 + 2x = 6 or x = 1.
x2 2 2
2 x2 2 0
2 2 x2
Solution: Subtracting second row from first row and third row, we get
x x 0
0 x x 0
2 2 x2
70
x 0 0
0 x x =0
2 0 x2
Therefore, x [ x ( x +2 )] = 0
Either x =0 or x + 2 = 0 or x = - 2, i.e. x = - 2, 0, 0
a b
A=
c d
a b a b a 2 b 2 ac bd
2
Solution: We can write A = A . A =
c d c d ac bd c 2 d 2
x y a b
Let A = and B = . We need the product AB.
z w c d
In finding the product of two determinants the elements in a row of first determinant is multiplied by the
corresponding elements of the row of second determinant and the products are added. This addition of the
products is done for every row of second determinant. For example, if we find the product AB for the
above given A and , x ix multiplied by a and y is multiplied by b and these two products are added to get
the first element of first row of the determinant AB. For the second element of first row of AB the
elements of first row of first determinant are multiplied by the corresponding elements of second row of
second determinant and these products are added. For example x of A is multiplied by c of B and y of A
is multiplied by d of B and then these two products are added. Thses give the two elements of first row of
the determinant AB. In the above case the first elements of the first row of AB are ax+ by and cx + dy.
The process of multiplication and addition is continued until all elements of the product determinant are
found out. According to the product and addition rule of elements discussed here , we get the determinant
AB which is shown below:
ax by cs dy
AB =
az bw cz dw
5 3 2 5 5 2 3 5 5 3 3 2 25 21
For example, if A = and B = , then AB =
4 7 3 2 4 2 7 5 4 3 7 2 43 26
71
Example 2.4: Find the product of the two given determinants A and B as below:
1 2 3 3 2 1
A= 5 6 2 and B= 2 6 5
4 5 1 1 5 4
1 2 3 3 2 1
Solution: We have AB = 5 6 2 2 6 5
4 5 1 1 5 4
1 3 2 2 3 1 1 2 2 6 3 5 1 1 2 5 3 4
= 5 3 6 2 2 1 5 2 6 6 2 5 5 1 6 5 2 4
4 3 5 2 1 1 4 2 5 6 1 5 4 1 5 5 1 4
10 29 23
= 29 56 43
23 43 33
1 a 1 1 a 1
Example 2.15: Given A = and B =
1 1 a 1 1 a
Solution: We have
1 a 1 1 a 1 1 a2 1 1 a 1 a 2 a2 2
AB =
1 1 a 1 1 a 1 a 1 11 a2 2 a 2 a2
Example 2.16: If x = - 9 is a root of the following equation, then find the other two roots.
x 3 7
2 x 2 0
7 6 x
x 3 7
Let A = 2 x 2 . Then I A I = x ( x2 – 12 ) – 3 ( 2x – 14 ) + 7 ( 12 – 7x )= x3 - 67 x + 126
7 6 x
72
As x = -9 is a root of the given equation ( x + 9 ) is one of the factor of the cubic function. Hence, we can
write
( x + 9 ) ( x2 – 9x + 14 ) = 0
Or ( x + 9 ) ( x – 7 ) ( x – 2 ) = 0 , thus x = -9, 2, 7.
Let A be the determinant of order n x n and let ai j be the element of j- th column corresponding to i – th
row. Further, consider thAat Ai j is the cofactor corresponding to ai j. Then the adjoint ( adjugate ) of A is
given by
Using the adjoint determinant [ A/ ] and I A I we can have a new determinant A- 1 such that each element
Ai j is divided by I A I , then the new determinant A- 1 is known as reciprocal of the determinant A. If we
consider a 3 x 3 determinant A , then reciprocal of A is given by
2 4
A=
5 12
12 5
12 5 A 4 3 1.25
Adjoint A = A/ = , A 1 4
4 2 A 4 2 1 0.5
4 4
73
4 3
A=
2 4
4 2
4 2 A 10 10 0.4 0.2
Adjoint A = A/ = A 1
3 4 A 3 4 0.3 0.4
10 10
4 8 6
A= 2 5 2
3 4 3
Let Ai j be the cofactor of ai j element of the determinant A , where the cofactors are
5 2 2 2 2 5
A11 (1)11 15 8 7, A12 (1)1 2 (6 6) 0, A13 (1)13 8 15 7
4 3 3 3 3 4
8 6 4 6 4 8
A21 (1) 21 (24 24) 0, A22 (1) 2 2 12 18 6, A23 (1) 23 (16 24) 8
4 3 3 3 3 4
8 6 4 6 4 8
A31 (1) 31 16 30 14, A32 (1) 3 2 (8 12) 4, A33 (1) 33 20 16 4
5 2 2 2 2 5
Now, we can find the inverse of the determinant A, where it is given
7 0 7 7 0 7
14 14 14 0.5 0 0.5 A 0 6 8
A 0 6 8 1 3 4 14 4 4
A 1 . Here, A 0 ,
A 14 14 14 7 7
14 4 4 2 2
1
14 14 14 7 7
( a ) Finding an Equation of Straight: Let us consider that a straight line is passed through two points
( x1,y1) and ( x2, y2 ). We need to find the equation of this straight line.
74
The line is given by the equation
x y 1
I A I = 0 , where A = x1 y1 1
x2 y 2 1
Example 2.20: Find an equation of straight linewfich passes through the points ( 2 , 3 ) and ( -1, 2 ).
x y 1 x y 1
I A I = 0 , where A = x1 y1 1 2 3 1
x 1 1 2 1
2 y2
I A I = x { 3 x 1 – 1 x 2} – y { 2 x 1 – 1 ( - 1 )} + 1 { 2 x 2 – 3 ( - 1 )} = x ( 3 – 2 ) –y ( 2 + 1 ) + 1 ( 4 + 3)
I A I = x – 3y + 7.
Example 2.21: The production of cement ( y in m.tons ) and the number of laborers ( x in 00 ) in two
days in an industry are ( 2 , 4 ) and ( 2.5, 6 ) . Assuming linear relationship of production ( y ) and number
of laborers ( x ) establish the relationship of x and y.
Solution: Let the linear relationship of x and y be straight line y = a + b x. We need to establish this
relationship. It is given by
x y 1
y1 1
I A I = 0 , where A = x1
x2 y 2 1
x y 1
2 4 1 , IA I = x ( 4 x 1 – 1 x 6 ) – y ( 2 x 1 – 1 x 2.5 ) + 1 ( 2 x 6 – 4 x 2.5 )
In our case A =
2.5 6 1
75
x1 y1 1
2 A , where A x2 y2 1
x3 y3 1
Example 2.22: Find the area of the triangle which is formed with the coordinates ( - 1 , 2 ), ( 1 , 2 ) and
( 0, 4). Do you think that the coordinates are collinear?
1
Solution: The area of the triangle is given by 2 A ,
x1 y1 1 1 2 1
Here A = x2 y2 1 . In our case A = 1 2 1 . IA I = -1 ( 2 – 4 ) – 2 ( 1 – 0 ) + 1 ( 4 – 0) = 4
x3 y3 1 0 4 1
1
Therefore , the area of the triangle is 2 4 2 0 . So, the coordinates are not collinear.
Example 2.23: Find the area of the triangle which is formed with the coordinates ( 2 , 5 ), ( -3 , 0 ) and
( 2, 0).
x1 y1 1
2 A , where A x2 y2 1
x3 y3 1
2 5 1
In our case A = 3 0 1 , I A I = 2 ( 0 - 0 ) – 5 ( -3 – 2 ) + 1 ( 0 – 0 ) = 25.
2 0 1
1
Therefore, the area is 2 25 12.5 .
4 6 7 1 7 6 9 7 6 3 8 9
5 9 12 10
(a) 2 3 8 , (b ) 2 4 8 , ( c ) 1 3 5 , ( d ) 5 3 8 , ( e ) , ( f )
7 13 8 7
5 3 9 4 8 16 2 6 10 7 4 9
76
2 . Show that
0 x y 1 1 1
(i ) x 0 z 0, (ii ) x y z ( x y )( y z )( x z )
y z 0 x2 y 2
z 2
a b c
(iii ) a 2 b 2 c 2 (a b)(b c)(c a )( ab ac bc),
bc ac ab
1 a bc 1 a bc
(iv ) 1 b ac (a b(b c)(c, (v) 1 b c a 0
1 c ab 1 c ab
bc a a
2 1 2 / 3 1/ 3
(vi ) b c a b 4abc, (vii ) If A , then A 1
1 2 1/ 3 2 / 3
c c ab
2 1 1 0
(viii ) If A , then A 2 4 A 3I , where I
1 2 0 1
2 1 5 4
(ix ) Given A , then A 2 ,
1 2 4 5
1 4 0 1/ 3 2 / 3 2 / 3
1
( x) If A 1 2 2 , then A 1 / 6 1 / 6 1 / 6
0 0 2 0 0 1/ 2
( a ) 2x + 3y = 8 , ( b ) 2x – y = 1 , ( c ) 5x + 2y = 11, ( d ) 4x + 3y = 2, ( e ) 2x + 3y = 4
x – 2y = - 3 3x + 2y = 5 3x + 4y = 1 x + 2y = 3 x–y=7
( f ) 2x + y + z=0 ( g ) 2x – 3y +4z = 3 ( h ) 2x + y – 2z = 10 ( i ) 2x – 3y + 4z = 3
3x +2 y - 3 z = 1 5x – y + z =5 5x + 4y +3z = 4 3x + 6y – z = 8
77
4 3 25 23 5 2
2 / 2
4 . ( i ) If A = 2 5 , then find A and A . Ans. A = 23 29 , A/ = 34 .
1 2 2 1 0 5
( ii ) If A = 1 1 and B= 1 2 , then find AB, Ans. AB = 3 1
1 1 1 4 3 2 1 4 3
a b
(i ) A 1 2 3 , (ii ) A , (iii ) A 5 4 3 , (iv ) A 2 6 4
b a
3 1 5 3 3 5 3 5 1
13 14 5 16 16 3 26 14 8
a b
Ans. (i ) 6 8 2 , (ii ) , (iii ) 21 26 3 , (iv ) 11 10 17
b a
1 2 1 20 22 14 34 2 14
5 3 12 8 8 10 9 10
(i ) , (ii ) , (iii ) , (iv )
7 5 10 7 4 6 7 8
1.25 1.75 1.75 2.5 0.75 0.5 4 3.5
Ans. (i ) , (ii ) , (iii ) , (iv )
0.75 1.25 2 3 1.25 1 5 4.5
78
Chapter III
Matrix
3.1 Introduction
The theory of matrix was introduced by Cayley in 1858. After his work, many mathematicians did the
work in developing the theory of matrices. Among them, notable mathematicians are Sylvester, Hamilton,
Heisenberg and others. Matrix theory helps in presenting the complex mathematical process into a
simpler one. For example, an industry produces cement ( xi j in tons ) in different days. The production
data of first week of first 3 months are, say, 500. The same information for second, third and fourth week
of first three months are, say, 610, 650 and 550, respectively. Similarly, the production data of every 4
weeks of each quarter of a year are 750, 650, 525, 650; 700, 600, 550, 600; and 725, 650, 750, 700,
respectively. These information of production data cab presented in a tabular form as follows:
Table: The production of Cement ( xi j tons ) in Different weeks of each Quarter of a Year
Definition of Matrix: The arrangement of m x n elements in m rows and n columns is called a matrix. If
m = n , then the arrangement is called a square matrix, otherwise the matrix is called a rectangular
matrix. Usually, the matrix is denoted by A, the elements of i-th row in j-th column is denoted by ai j .
Thus, we can write
A = [ ai j ]m x n , I = 1, 2, 3, ……., m ; j = 1, 2, 3, ………,n
Thus, the production data as shown above can be arranged in a 4 x 4 matrix , say A, where
This is a matrix of order 4 x 4 and as it has same number of rows and columns, it is called a square matrix
of order 4.
The matrix may be rectangular in which number rows is not equal to the number of columns. The number
of rows may be more than number columns or number of rows may be less than number of columns. In
general, a matrix of order m x n is written as below:
79
a11 a12 a13 . a1 j ...........a1n
a 21 a 22 a 23 ..a 2 j ..........a 2 n
............................................
mxn
A = [ ai j ]m x n = ai1 ai 2 ai 3 ..aij .............aim
...............................................
a m1 a m 2 a m3 ..a m j ..............a m n
.
For example, let us consider that the number of customers of a bank in different working hours of two
days are as follows:
42 38
55 50
65 68
A =
40 35
35 32
42 45 6 x 2
This information can also be presented in a matrix of order 2 x 6, i.e. in a matrix, say, B of order 2 x 6,
42 55 65 40 35 42
where B =[ aj I ]2 x 6 =
38 50 68 35 32 45 2 x 6
The customers in queue in Day -1 can be presented in a row ( column ) matrix as follows:
C = [ ci j ]1 x 6 =[ 42 55 65 40 35 42 ]1 x 6
80
42
55
65
40
D= [ di j ]6 x 1 =
35
42 6 x1
Square Matrix: A matrix which contains m2 elements in m rows and m columns is called a square
matrix.
In general, this matrix is written as A = [ ai j ]m x m. For example , let us consider a matrix of order 3 x 3 as
follows:
5 8 12
A = [ ai j ]3 x 3 = 3 7 10
9 6 6 3 x 3
Let us consider that the number of boys and girls under age 18 who are working in a garments industry
are 15 and 32, respectively. The corresponding number of workers in another industry are 22 and 45. This
information of number of workers of under age 18 can be shown in a 2 x 2 matrix [ row for industry and
column for gender] as follows:
15 22
A=[ai j ]2 x2 =
32 45 2 x 2
This matrix A is a square matrix of order 2 x 2. Similarly, we can write a square matrix A of order 3 x 3,
where
12 18 11
A = [ ai j ] = 5 8 3
10 12 17 3 x 3
Rectangular Matrix: If number of rows and number of columns of any matrix are not equal, then the
matrix is called rectangular matrix. For example, let us consider the demand of sugar ( in million tons) in
two years in 6 different big cities are as follows:
81
Let us denote the matrix by A to represent the demand of the years in rows and demand of cities in
columns. Here
50 45 30 55 60 40
A=
45 48 42 50 44 45 2 x 6
Again, the data of demand of sugar of cities can be presented in rows and that of year in columns as
follows:
50 45
45 48
30 42
B=
55 50
60 44
40 45 6 X 2
Here A is a matrix of 2 rows and 6 columns and B is a matrix of 6 rows and 2 columns. Number of rows
and number of columns of both the matrices are not equal. These matrices are known as rectangular
matrices.
Null Matrix: If all the elements of a matrix are zero, then the matrix is called null matrix. For example,
0 0 0 0
0 0
A = 0 0 0 ,B , C 0 0 0 01 X 4 , D 0
0 0 0 3 X 3 0 0 2 X 2 0 3 X 1
Transpose Matrix: If the row ( column ) elements of a matrix A are presented as column (row) elements
of another matrix, then the latter matrix is called a transpose matrix of A. The transpose matrix of A is
denoted by A/ . For example, let
4 5 9 4 3 2
5 4 5 3
A = 3 7 10
, A 5 7 4 ,C , C
2 4 6 3 x 3 9 10 6 3 x 3 3 2 2 x 2 4 2 2 x 2
Here the row elements of the matrix A are the column elements of the matrix A/ ; row elements of the C
matrix are the column elements of the C/ matrix. A/ is the transpose of A matrix and C/ is the transpose of
the C matrix. Further example of transpose matrix is
82
a1 b1
a1 a 2 a3
A= , A a 2 b2
b1 b2 b3 2 x 3 a3 b3 3 x 2
Here row elements of A matrix are the column elements of A/ matrix. So, A/ is the transpose matrix of A.
Symmetric Matrix: If the elements in row ( column ) of a square matrix are the elements of column
( row ) of another matrix, then the latter matrix is called the symmetric matrix. For example, let us
consider a matrix A, where
7 3 4 7 3 4
A = 3 6 9 , The transpose matrix of A is A/ = 3 6 9 . Here elements in rows
4 9 8 4 9 8 3 X 3
of A are the elements in columns of A/. So, A/ is the transpose matrix of A.
Skew Symmetric Matrix: If a matrix A/ becomes – A , then the matrix A is called skew symmetric
matrix. For example, let
0 4 0 4
A= , A
0 2 x 2 ,
, Here A = - A, so A is a skew symmetric matrix.
4 0 2 x 2 4
o g h 0 g h
Further example of skew symmetric matrix is ,A = g 0 i i A
as A g 0
h i 0 3 x 3 h i 0 3 x 3
Adjoint ( Adjugate ) Matrix: Let A = [ ai j ]n x n . Consider that Ai j are the cofactor of the element ai j ,
where the cofactor matrix is
83
A11 A21 ......An1
A A22 .......An 2
C Adj A [ A ji ] 12
.................................
A1n A2 n .........Ann nxn
5 9 a11 a12
For example, let us consider a 2 x 2 matrix A, where A= = . For A matrix the
3 7 a 21 a 22 2 x 2
cofactor of a11 is A11 = ( -1 )1+1 7 = 7, the cofactor of a12 is A12 = ( - 1 )1+2 3 = -3,
cofactor of a21 = A21 = ( - 1 )2+19 =- 9 , cofactor of a22=A22 =(-1)2+2 5=5. The cofactor matrix is
A11 A12 7 3 7 9
C = [ AI J ]= . Here C/ = adj A = [ AJ I]= .
A21 A22 2 x 2 9 5 2 x 2 3 5 2 X 2
1 Adj A
Inverse Matrix: For a matrix A the inverse matrix is defined by A . If A-1 is the inverse of
A
the Matrix A , then A-1 A = A A-1 = I, where I is an identity matrix of order n x n if A matrix is of order n
x n. For example, let us find the inverse matrix of the matrix given above. Here
7 9
5 9 7 9 1
8 8
A= , Adj A , A 7 5 9 3 8, A .
3 7 2 x 2 3 5 2 X 2 3 5
8 8 2 X 2
7 9 7 3 9 5
5 9 5 9
5 9 8 8 8 8 8 8 1 0
Here A A 1 I 2 , where I2 is the
3 7 3 5 3 7 7 3 3 9 7 5 0 1 2 X 2
8 8 8 8 8 8
identity matrix of order 2 [ 2 x 2].
Identity Matrix: A square matrix In x n is called an identity matrix if all of its diagonal elements are 1 and
the off-diagonal elements are zero. Above shown I2 matrix is an identity matrix of order 2 ( 2 x2 ). This
identity matrix is also called unitary matrix. Further examples of identity matrices are
1 0 0 0
1 0 0 0
1 0 1 0 0
I2 , I 3 0 1 0 , I 4 0
0 1 0 1 0
0 0 1
0 0 0 1
84
An important property of the identity matrix is that its determinant value is 1. It is a symmetric matrix and
transpose of I matrix is also same I matrix. It is a diagonal matrix also. The inverse matrix of identity
matrix is that identity matrix.
Diagonal Matrix: Let A be a square matrix such that its diagonal elements are non-zero and off-diagonal
elements are zero, then the A matrix is called a diagonal matrix. For example.
5 0 0 5 0 0 a 0 0
2 0
A , B 0 6 0 , C 0 5 0 , D 0 b 0
0 3 2 X 2 0 0 8 3 X 3 0 0 5 3 X 3 0 0 c 3 x 3
An important property of this matrix is that its determinant value is the product of the diagonal elements
of the matrix. The inverse of a diagonal matrix is the diagonal matrix with reciprocal of diagonal elements
of the matrix. For example, the inverse of the A matrix given above is
1
0
2
A 1
0 1
3 2 x 2
Scalar Matrix: A diagonal matrix with all equal diagonal elements is called a scalar matrix. In the above
example C matrix is a scalar matrix.
Singular Matrix: If the determinant value of a matrix is zero, i.e. I A I = 0, then the matrix A is called
singular matrix. On the other hand if I A I 0 , the matrix A is called non-singular matrix. For example,
let
3 5 8 1 4 2
5 10
A , B 2 6 4 , C 2 4 1
4 8 2 x 2 7 3 2 3 x 3 3 12 6 3 X 3
I C I = 1 ( 24 – 12 ) – 4 ( 12 – 3 ) + 2 ( 24 – 12 ) = 12 – 36 + 24 = 0. C is a singular matrix.
Triangular Matrix: If in a square matrix there exist non-zero elements above or below the diagonal
elements, then the matrix is called a triangular matrix. If elements exist below the diagonal elements, the
matrix is known as lower triangular matrix. The upper triangular matrix is one in which elements exist in
the upper part of the diagonal elements. For example, let us consider the following matrices
85
a b c g 0 0
a b 5 8
A , B 0 2 , C 0 d e , D h i 0
0 c 2x2 2x2 0 0 f 3 x 3 j k l 3 x 3
In the above examples matrices A , B and C are upper triangular matrices and D is a lower triangular
matrix.
Conjugate Matrix: If the elements of a matrix are replaced by the complex conjugate elements , then the
new matrix is called complex conjugate matrix. Thus the elements of a complex conjugate matrix are
complex quantities. For Example,
1 i i 1 i i
A ,B
2 3 4i 2 3 4i
Conjugate Transpose Matrix: If A is a matrix, then A* is called conjugate transpose matrix if the ( i, j
)th element of A* are the complex conjugate of (i,j)th element of A matrix. For example,
aij a ji i, j 1, 2, 3, ........,n
If the matrix A is real, then the Hermitian matrix is the same as real symmetric matrix. On the other hand,
a skew Hermitian matrix will be skew symmetric matrix if the matrix is real. For example, the
1 2 5i 5 6i
2 5i 2 7 9i
5 6i 7 9i 3
matrix given above is a Hermitian matrix. For this matrix A A and A* =A.
i 1 3i 4 5i
A 1 3i 0 6 7i
4 5i 6 7i 3i 3 x 3
86
∑ ai j =1
This matrix A is called a regular stochastic matrix, if ai j > 0 but ai j < 1, where ∑ ai j =1. For example,
1 a 0
A , 0 a 1, 0 b 1
b 1 b 2 x 2
1 1
A
1 1 2 X 2
1 1 1 1 2 0
Here A A 2 I2
1 1 1 1 0 2
Hadamard matrix is a square matrix. However, the matrix is not available for square matrix of any order.
If A is a matrix of order m x m , the Hadamard matrix is available when m =2, m = 4, m = 8 , i.e.
m 0 ( mod 2m )
a1 a2 a3 ..........an1 0
b1 b2 0 ...........................0
A 0 b2 0.............................0
.................................................
0 0 0...................bn1 0
nxn
I A - In I = 0
Then the equation has one real positive root, say 1 . This is a simple root and the characteristic vector
corresponding to this root is, say X1. All elements of X1 are positive.
( ii ) If the unique positive characteristic root of the characteristic equation of Leslie matrix A is 1 , then
for the other complex roots i I i I 1 .
87
( iii ) For any Leslie matrix A if the consecutive two elements of first row are non-zero ,then the only
positive characteristic root is dominant, i.e. if ai and ai – 1 are non-negative , then the only positive
characteristic root is dominant.
Infinite Matrix: Let A = [ ai j ] be a matrix of order m x n. If the values of m and n related to this matrix
is countable infinite, then the matrix is called infinite matrix.
If infinite matrix is available and if the elements of the matrix are real, then for that infinite matrix the
rules of addition, multiplication, equality and transpose of the matrix are valid. However, If
A = [ ai j ] and B= [ bi j ] , then a
k 1
ik bik
may not be available. Because this summation may not be converged and hence the product AB may not
be available.
( i ) For infinite matrix the commutative law of multiplication does not exist.
( ii ) If the elementary operations for infinite matrix are feasible, then distributive law exists, i.e
A + ( B + C ) = ( A + B ) + C or A ( B + C ) = AB + AC.
( iii ) For infinite matrix the associative law may not be valid always. For example, let
1 1 0 0 0 0
0 1 1 0 0 0
A 0 0 1 1 0 0
0 0 0 1 1 0
.....................................
and B is an infinite matrix with every elements one, then B( AB) = 0, but (BA)B = B.
( v ) If A , B, and C are three infinite matrices such that C > 0 and A B, then
Partition Matrix: Let A = [ ai j ]n x n be a matrix of order n x n. If using some of the elements of ai j the
matrices of smaller orders [ nl < n , l = 1, 2, 3, …., k < n ] can be formed and which can be shown in
partition form within the matrix A, then the matrix A is called a partition matrix. For example, let
88
4 3 6 9
5 8 5 4
A=
7 6 7 6
4 5 8 9 4 x 4
4 3 6 9 7 6 7 6
Let B1 , B2 , B3 , B4
5 8 2 x 2 5 4 2 x 2 4 5 2 x 2 8 9 2 x 2
B1 B2
A=
B3 B4 4x4
Here the matrix A is partitioned into 4 matrices and hence A is called a partition matrix. The matrices B1,
B2 ,B3 , and B4 are known as sub-matrices.
Idempotent Matrix: Let A = [ ai j ]n x n be a square matrix such that A2 = A. Then the A matrix is known
as idempotent matrix. For example,
1 0
A= ,
0 1 2 x 2
1 0 1 0 1 0
Here A2 A A . So, A is an idempotent matrix. From the above example
0 1 0 1 0 1
of idempotent matrix it can be said that identity matrix of any order is an idempotent matrix. Let us
consider another example of idempotent matrix B, where
2 2 4
B 1 3 4 . This B = B2 and hence B is an idempotent matrix.
1 2 3 3 x 3
Rank of this idempotent matrix is the Trace of A = ∑ diagonal elements of the matrix A. For B matrix
Rank is Rank ( B ) = 2 + 3 – 3 = 2 = Trace ( B )[ Rank ( A ) = Trace( A )]. If matrix A = A1 + A2 such that
Rank ( A ) = Rank ( A1 ) + Rank ( A2 ), then A1 and A2 are idempotent matrices and A1 A2 = 0. Here rank
of the matrix is the number of independent rows or number of independent columns of the matrix A.
89
1 2 3
A 1 2 3 . We need to show that A m 0 when Am1 0 . Let us show that A2 = 0. Here
1 2 3
1 2 3 1 2 3
A 1
2 2 3 1 2 3
1 2 3 1 2 3
1 1 2 1 3 1 1 2 2 2 2 3 1 3 2 3 3 3 0 0 0
1 1 2 1 3 1 1 2 2 2 3 2 1 3 2 3 3 3 0 0 0
1 1 2 1 3 1 1 2 2 2 3 2 1 3 2 3 3 3 0 0 0 3 x 3
Non-negative Matrix : Let A = [ ai j ]n x n be a matrix such all ai j elements are positive [i = 1, 2, 3, …..,n;
j = 1, 2, 3, …..n ] or zero [ ai j 0 ]. In that case the matrix A is known as non-negative matrix. For
example, let
2 0 5 8
5 0 8
3 5 3 7 9 , C 3 9 0 7
A , B
4 7 2 x 2 0 1 4 3
0 5 6 3 x 3
2 0 0 5 4 x 4
The non-negative matrices are decomposable if a matrix P is available such that P A P-1 takes the shape
A11 0 0 0 ........0
A A22 0 0.........0
A11 0
, P A P 1
21
P A P-1 =
A12 A22 2 x 2 ........................................
An1 An 2 ...................Ann nxn
Here Ai j are not decomposable or Ai j will be of order 1 x 1. If i j for all i and j Ai j = 0, then
cos sin
A=
sin cos 2 x 2
90
Then
cos sin cos sin cos2 sin 2 cos sin sin cos 1 0
AA I2
sin cos sin cos sin cos sin cos sin 2 cos2 0 1 2 x 2
Involutory Matrix: Let A = [ ai j ]n x n be a square matrix of order n x n such that A2 = In , where In is an
identity matrix. For example, let
1 0 1 0 1 0 1 1 0 0 1 0 0 1 1 0
0 1 0 1 0 1 1 0 0 0 1 1 0 1 I 2
2
A= . Then A =A A=
0 1 2x2
For transformation of a matrix A to a new matrix B rows ( columns ) can be interchanged, with rows
( columns) of a matrix same rows ( columns ) or multiple of rows ( columns ) can be added, from same
rows ( columns ) another rows ( columns ) can be subtracted , same rows ( columns ) can be multiplied by
a scalar quantity ,etc. These types of operation on rows ( columns ) of a matrix A are called Elementary
Operation or E – Operation.
The E – operation can be done on row or on column. When E – operation is done on row, it is called Row
operation. The column operation is done to change the elements of a column. Thus E –operation can be
divided into 3 classes. These are
( c ) The addition of multiple of elements of rows ( columns ) can be done with another row ( column ).
E – operation change the shape of a matrix but determinant value of the matrix is not changed in
magnitude. This operation is needed to find the inverse of a matrix, to transform a matrix to a diagonal
form, etc. Let us consider an E – operation by example.
Example 3.1: Let us find the value of the determinant for the given matrix by elementary operation,
where
1 2 3
A = 0 5 2 .Given I a I = 1 ( 5 – 8 ) – 2 ( 0+ 6 ) + 3 ( 0 + 15 ) =
3 4 1 3 x 3
30
91
1 0 0
A1 = 0 5 2 , I A1 I = 1 ( 5 x 10 – 2 x 10 ) = 30
3 10 10
Here the determinant value of the matrix A1 remains same as that of matrix A due to E - operations . We
get A1 after performing E – operationS on the matrix A.
Let us do the E- operation on the column – 2 of matrix A. The operation is C2 C 2 2C1 . This gives
new matrix , say A2 , where
1 0 3
A2 = 0 5 2 , now I A2 I = 1 ( 5 x 1 – 2 x 10 ) + 3 ( 0 x 10 +5 x 3 ) = 30
3 10 1
Again, let us perform E – operation by interchanging row – 1 [ R1] and row – 2 [ R2 ], i.e. R1 R2. It
gives new matrix, say A3 , where
0 5 2
A3 = 1 2 3 . Here I A3 I = - 5 ( 1 + 9 ) + 2 ( 4 + 6 ) = - 30
3 4 1
Due to interchange of one row the magnitude of the value of determinant of the matrix A remains same
but its direction is opposite.
Let us interchange row- 1 to row – 2 and row – 1 to row – 3 [ R1 R2 , R1 R3 . It gives new matrix,
say A4 , where
1 2 3
A4 = 3 4 1 , IA I = ! ( 8 – 5 ) – 2 ( - 6 – 0 ) + 3 ( - 15 – 0 ) = - 30
0 5 2
Here odd number of rows are interchanged and the value of the determinant of the matrix is changed to
negative value though the magnitude remains same.
Again, let us do the E – operation by changing row – 1 to 3 times of row – 1 [ R1 3R1 ]. We get a new
marix, say A5 , where
3 6 9
A5 = 0 5 2 , I A5 I = 3 ( 5 – 8 ) – 6 ( 0 + 6 ) + 9 ( 0 + 15 ) = 90
3 4 1
92
It is seen that the determinant value of the matrix A5 becomes 3 times of the determinant value of the
matrix A as one row of this matrix is multiplied by 3.
Example 3.2: Transform the given identity matrix A to elementary matrices by E – operation.
1 0 0
A = 0 1 0
0 0 1 3 x 3
1 2 0
B= 0 1 0 This B is an elementary matrix.
0 0 1 3 x 3
Transform column -1 by multiplying it by 3 [ C1 3C1 ] to get a new elementary matrix, say c, where
3 2 0
c 0 1 0
0 0 1 3 x 3
This operation can also be done on the matrix A to get another elementary matrix , say c1 , where
3 0 0
c1 = 0 1 0
0 0 1 3 x 3
3 4 0
D= 0 1 0
0 0 1 3 X 3
1 0 0
E = 0 1 0
3 0 3 3 x 3
93
Thus, we get 4 elementary matrices.
Example 3.3: Transform the matrix A into a triangular form by elementary operation, where
3 1 4
A = 1 2 5
3 1 2 3 x 3
1
Solution: Let us transform row-2 of A into ( row – 2 – 1/3 row-1)[ R2 R2 R1 ] . It gives the matrix ,
3
say A1 , where
3 1 4
A1= 0 5 / 3 19 / 3
3 1 2
3 1 4
A2= 0 5 / 3 19 / 3
0 0 2 3 x 3
The matrix operation is done by adding two or more matrices , multiplication of two matrices, subtraction
of one matrix from another matrix, etc. Let us perform this operation by example.
5 3 3 2 5 3 3 2 8 5
A= , B , then A + B = 4 4 7 6 8 13 . Again,
4 7 4 6
5 3 3 2 2 1
A–B= . From the results of addition and subtraction of two matrices it is seen
4 4 7 6 0 1
that for addition and subtraction the order of both the matrices must be same ,because in case of addition
or subtraction of two matrices the corresponding elements are added and the elements of one matrix are
subtracted from the corresponding elements of another matrix. In the above case to get A – B we have
subtracted the elements of B matrix from the corresponding elements of A matrix.
Example 3.4: A garments industry exports two types of garments in different amounts ( ooo pieces ) in
94
two countries in two shipments of two years. The information of the pieces exported are given below:
Find the total amount of pieces of exported garments of each type in two years and in two shipments.
Again, let C be the matrix to represent exported garments of shipment – 1 and D be the matrix to
represent the exported garments in shipment – 2, where
Here C + D matrix represent the total export of year – 1 and year – 2 ; C+ D matrix represents the total
exported garments in two shipments.
Example 3.5: A company produces cement ( in 000 bags ) per week in two units of production. The
production data in 4 quarters of a year are as follows:
Solution: Let A be the matrix to represent the production of different weeks of Unit – 1 and B be the
matrix to represent the production different weeks of Unit – 2. We need A + B and A – B . Here
5 8 7 5 6 6 7 7 5 6 8 6 7 7 5 7 11 14 14 12
6 9 6 6 8 8 8 6 6 8 9 8 6 8 6 6 14 17 14 12
A ,B , A B
7 8 7 7 9 9 8 9 7 9 8 9 7 8 7 9 16 17 15 16
8 8 8 9 4x4 8 7 8 8 4X 4 8 8 8 7 8 8 9 8 16 15 16 17 4 X 4
Similarly, we get A – B , the value is
95
5 6 8 6 77 5 7 1 2 0 2
6 8 9 8 68 6 6 2 1 2 0
A – B =
7 9 8 9 7 8 7 9 2 1 1 2
8 8 8 7 88 9 8 0 1 0 1 4x4
6 5 68 77 7 5 1 2 0 2
8 6 89 86 6 6 2 1 2 0
B–A= ( A B )
9 7 98 87 9 7 2 1 1 2
8 8 7 8 88 89 0 1 0 1 4 x 4
1 2 4 3 1 2
2 6 4 8 5 3 8 4 3 7
(i ) A ,B , (ii ) A ,B , (iii ) A 2 1 5 , B 4 2 5
7 9 3 5 2 6 7 6 2 4 3 4 5 5 1 3
Solution:
2 4 6 8 6 14 5 4 3 3 8 7 9 6 15
(i) A+B= . (ii ) A B
7 3 9 5 10 14 2 X 2 2 6 6 2 7 4 8 8 11 2 X 3
1 3 2 1 4 2 4 3 6
( iii ) A + B = 2 4 1 2 5 5 6 3 10
3 5 4 1 5 3 8 5 8 3 x 3
Example 3.7: An industry produces iron rod ( in m.ton ) per week and some of the rods ( in m.ton ) are
supplied to the selling agent per week. The production is done in two units of the industry and supply is
also done from two units. The information of production and supply for 4 weeks are given below:
Solution: Let A be the matrix of production of iron rod in different weeks and B be the matrix of supply
of rod to the selling agent, where A and B are given below. We need A – B .
96
55 65 54 60 50 56 50 52
A ,B
45 54 44 45 2 x 4 40 53 40 43 2 x 4
55 50 65 56 54 50 60 52 5 9 4 8
A B
45 40 54 53 44 40 45 43 2 X 2 5 1 4 2 2 X 4
Let A = [ ai j ]m x n and B = [ bi j ]m x n be two matrices of same order m x n. These two matrices A and B
will be equal if both the matrices are of same order and all corresponding elements of both the matrices
are equal, ai j = bi j for all i = 1 , 2, 3, ……..m; j=1, 2, 3, ……..,n. Under the condition of equality of two
matrices we can say that the matrices are comparable. For example, let us consider the two matrices A
and B ,
5 9 7 4 8 7
where A = 3 5 1 2 X 3
, B
3 6 4 2 x 3
Here both the matrices are of same order [ 2 x3 ] but all the elements in A and the elements in B are not
same. Here A and B are not equal. However, both the matrices are comparable as their orders are same.
Again, let us consider another two matrices , C and D, where
4 8
3 5 4 3 6 5
C= D
6 7 8 5 7 1 2 X 4
5 1 4X 2
Here elements in matrix C and D are same in magnitude but the corresponding elements in rows and
columns of both the matrices are not same. Moreover, the orders of the matrices are not same. Thus the
matrix C and the matrix D are not equal and they are not comparable.
5 8 9 4 5 8 9 4
E= F 3 9 6 2
3 9 6 2 2 X 4 2X 4
Here the matrix E and the matrix F are of same order [ 2 x 4 ] and the corresponding elements in rows and
columns are also same. These two matrices are comparable as both are of the same order. Again , they are
equal as their orders are same and the corresponding elements in both rows and columns are same. Hence,
E = F. Now, we can conclude that the two matrices will be equal if both of them are of same order and the
corresponding elements in rows and in columns are same.
Thus, from the equality property of two or more matrices, we can conclude that
97
( ii ) If A = B or B = A , then this property is known as symmetric property of the matrices.
( iii )If A = B , B = C , then the property is known as the transitivity of the matrices.
Example 3.8: A company produces electric bulbs ( in 00 units / hour ) using two plants, P1 and P2 . The
production of the company from each plant are 6 and 8 in one day, respectively. The production of
another day are7 and 9, respectively. Another company also produces the item using two plants, where
the production data of a day are 6 and 8 from plant P1 and P2, respectively. The corresponding data of
another day are 7 and 9. Are the companies similar in producing electric bulbs?
Solution: There are two companies producing electrical bulbs. The production data of each company can
be arranged in matrix notation. Let A be the matrix to represent the production data of two plants of
company – 1 in two different days and B be the matrix to represent the similar type of data of Company -
2. We need to show that A =B. Here
6 8 6 8
A ,B . It is seen that orders of both A and are same
7 9 2 x 2 7 9 2 x 2
and the corresponding elements of both the matrices are same. Hence A = B. The two companies are
similar in production capacity.
Example 3.9: In an area of a city there are two departmental stores. In each store there are 3 customers
service counters. In a random investigation of the service renders by the sales representatives the data of
different days are recorded and given below:
Solution: Let A be the matrix to arrange the number of customers served in different days by the
different sale representatives of Store -1 and B be the matrix for the similar information collected from
Store – 2. We need to show A = B. Here [ Counters are shown in columns and days are shown in rows ].
It is seen that the orders of the matrices are not similar. So, they are not comparable and there is no
question of equality of two matrices A and B. Moreover. The departmental stores are not similar in
providing service to the customers.
98
A+B +C = ( A + B ) + C= [ ai j + bi j] + ci j = ai j + bi j + ci j
A + ( B + C ) = a i j + ( bi j + c i j ) = a i j + bi j + c i j
2 4 3 2 4 5
For example, let A ,B ,C , then
5 7 4 3 6 8
2 3 4 4 2 5 9 11 3 4 2 5 7 7
A+B +C = .B+C= 4 6 3 8 10 11
5 4 6 7 3 8 2 x 2 15 18 2 x 2
2 4 7 7 2 7 4 7 9 11
A+(B+C)= A B C
5 7 10 11 5 10 7 11 15 18
2 4 3 2 2 3 4 2 5 6
A B
7 4 3 5 4 7 3 9 10
,
5
5 6 4 5 5 4 6 5 9 11
( A B) C A B C
9 10 6 8 9 6 10 8 15 18
5 3 5 3 5 5 3 3 0 0
A= ,B , then A + B = 6 6 8 8 0 0 O
6 8 6 8
( iv ) If A = [ai j ]m x n and B = [- ai j ]m xn, then A – B = 2 A. Thus, for the above given A and B , we have
A-B=
5 3 5 3 5 (5) 3 (3) 5 5 3 3 10 6 5 3
6 8 6 8 6 (6) 8 (8) 6 6 8 8 12 16 26 8 2 A
5 9 7 4
Example 3.10: Given A = , B= 8 5 ; show that A + B = B + A.
3 7
5 9 7 4 5 7 9 4 12 13
Solution: We have A + B = . Again
3 7 8 5 3 8 7 5 11 12 2 x 2
7 4 5 9 7 5 4 9 12 13
B+A= A + B.
8 5 3 7 8 3 5 7 11 12
Therefore, A + B = B + A.
99
4 8 3 5 9 6 3 2 1
Example 3.11: Given A ,B ,C
3 2 1 3 7 2 4 6 3
Show that A + ( B + C ) = ( A + B ) + C.
5 9 6 3 2 1 5 3 9 2 6 1 8 11 7
BC
3 7 2 4 6 3 3 4 7 6 2 3 7 13 5
4 8 3 8 11 7 4 8 8 11 3 7 12 19 10
A (B C)
3 2 1 7 13 5 3 7 2 13 1 5 10 15 6 2 x 3
4 8 3 5 9 6 4 5 8 9 3 6 9 17 9
A B
3 2 1 3 7 2 3 3 2 7 1 2 6 9 3
9 17 9 3 2 1 9 3 17 2 9 1 12 19 10
( A B) C
6 9 3 4 6 3 6 4 9 6 3 3 10 15 6 2 x 3
Hence ( A + B ) +C = A + ( B + C ).
Example 3.12: A company has the following amount of sales ( in 000 Taka ) of its two products P1 and
P2 in two centers in a year. Again, the sales data are available for the first 3 months of the year. Find the
amount of sales for the last 9 months of that year.
Product Sales of the year in Sales of the first 3 months of the year in
P1 75 68 28 18
P2 65 77 17 20
Solution: Let A be the matrix to represent the amount of sales of the products in two centers in a year and
B be the matrix to represent the amount of sales in first 3 months in the year, where
75 68 28 18
A ,B
65 77 17 20
100
k A = k [ ai j ]m x n = [ k ai j ]m xn is called a scalar multiplication of the matrix A by the scalar quantity k.
For
5 8
A=
3 7
5 8 4 5 4 8 20 32
kA=4
3 7 4 3 4 7 12 28 2 x 2
After scalar multiplication , the operation of addition and subtraction of two or more matrices can be
done. For example, Let A be the matrix of order 2 x 3 and B be the matrix of 2 x 3 . Consider that k=2
and l=3 are the two scalar quantities. At this stage, we can find k A lB , where
2 3 1 1 2 3
A= ,B
4 3 5 2 x3 3 1 4 2 x3
2 3 1 2 2 2 3 2 1 4 6 2
We have k A = 2A= 2
4 3 5 2 4 2 3 2 5 8 6 10 2 x3
1 2 3 3 1 3 2 3 3 3 6 9
LB = 3 B= 3
3 1 4 3 3 3 1 3 4 9 3 12 2 x3
After scalar multiplication both the matrices remain same in terms of order. So, addition and subtraction
on both the new matrices are possible[ the two new matrices are comparable]. Here
4 6 2 3 6 9 4 3 6 6 2 9 7 12 11
kA+lB=
8 6 10 9 3 12 8 9 6 3 10 12 17 9 22 2 x3
4 6 2 3 6 9 4 3 6 6 2 9 1 0 7
Similarly, k A – l B =
8 6 10 9 3 12 8 9 6 3 10 12 1 3 2 2 x 3
4 7 9
Example 3.13: Given a matrix A = 6 3 8 , find 4 A, - 2 A and 3A - 2A .
5 7 0 3 x 3
Solution:
101
4 7 9 4 7 9 4 4 4 7 4 9 16 28 36
A 6 3 8 , 4 A 4 6 3
8 4 6 4 3 4 8 24 12 32
5 7 0 5 7 0 4 5 4 7 4 0 20 28 0 3 x 3
4 7 9 2 4 2 7 2 9 8 14 18
2 A 26 3 8 2 6 2 3 2 8 12 6 16
5 7 0 2 5 2 7 2 0 10 14 0 3 x 3
4 7 9 3 4 3 7 3 9 12 21 27
3 A 36 3 8 3 6 3 3 3 8 18 9 24
5 7 0 3 5 3 7 3 0 15 21 0 3 x 3
12 21 27 8 14 18 12 8 21 14 27 18 4 7 9
3 A 2 A 18 9 24 12 6 16 18 12 9 6 24 16 6 3 8
15 21 0 10 14 0 15 10 21 14 0 0 5 7 0 3 x 3
Example 3.14: The amount of sales ( in million taka) in two different months in two different units of a
departmental store are given below:
Solution: Let A be the matrix to arrange the amount of sales in two months [ in rows ] in two units [ in
columns]. Then, we need to find 4A. Here
5 7 5 7 4 5 4 7 20 28
A ,4 A 4
8 12 2 x 2 8 12 4 8 4 12 32 48 2 x 2
1 2
Example 3.15: Given A= 2 4 , find a matrix B so that B + A = 0
3 6
1 2 1 2
Solution: B + A = 0 B = - A + 0 = 2 4 2 4
3 6 3 6 3 x 2
Remarks:
102
( i ) If A and B are two matrices and k is a scalar, then k ( A + B ) = k A + k B.
( ii ) If A and B are two comparable matrices and k and l are two scalars,
then ( k + l ) ( A + B ) = ( k + l ) A + ( k + l ) B = k A + l A + k B + l B.
If number of rows of a matrix is equal to the number of columns of another matrix, then the first matrix
can be multiplied by the second matrix. The (i,l )-th element of the product matrix is the sum of the
products of j –th element of first matrix and i-th element of second matrix. For example, let us consider
the two matrices A and B as given below:
1 2 4 3
A= , B
3 5 2 x 2 2 1 2 x 2
1 4 2 2 1 3 2 1 8 5
AB =
3 4 5 2 3 3 5 1 22 14 2 x 2
Here AB = [ a b
j
ij jl ]. Here A =[ ai j ]m xn , B = [ bj l ]n x m .
The above product matrix AB is possible if number of columns of A is equal to the number of rows of B.
In that case the two matrices are conformable for multiplication. If both the matrices are square
matrices, they are always conformable for multiplication and the products AB and BA are available , but
AB may not be equal to BA. Both the products will be equal if A = B. If a [ ai j ]m x n and B= [bj l ]n x m ,
then also AB and BA are available. The order of AB will be m x m and that of BA will be n x n.
However, if
A = [ai j ]m x n and B= [ bj l }]n x p , then AB of order m x p will be available but BA will not be available.
4 2 3 2
Example 3.16: Given A = ,B , find AB.
3 1 4 3
Solution: We have
4 2 3 2 4 3 2 4 4 2 2 3 20 14
AB =
3 1 4 3 3 3 1 4 3 2 1 3 13 9 2 x 2
103
5 3 8 4 2 6
Example 3.17: The two matrices A and B are A 10 6 4 , B 7 5 2
3 9 6 3 X 3 8 3 5 3 x 3
Solution: =
5 3 8 4 2 6 5 4 3 7 8 8 5 2 3 5 8 3 5 6 3 2 8 5
AB 10 6 4 7 5 2 10 4 6 7 4 8 10 2 6 5 4 3 10 6 6 2 4 5
3 9 6 8 3 5 3 4 9 7 6 8 3 2 9 5 6 3 3 6 9 2 6 5
57 49 76
= 114 52 92
123 69 66 3 x 3
4 2 6 5 3 8 4 5 2 10 6 3 4 3 2 6 6 9 4 8 2 4 6 6
BA = 7 5 2 10 6 4 7 5 5 10 2 3 7 3 5 6 2 9 7 8 5 4 2 6
8 3 5 3 9 6 8 5 3 10 5 3 8 3 3 6 5 9 8 8 3 4 5 6
58 78 76
= 91 69 88
85 87 106 3 x 3
It is that both AB and BA are comparable as both of them are of order 3 x3, but AB BA.
4 3 1 5 2
4 1 2
Example 3.18: Given the matrices A = 2 4 3, B 3 4, C
3 2 2
1 3 2 1 3
Solution:
4 3 1 5 2 4 5 3 3 11 4 2 3 4 1 3 30 23
AB = 2 4 3 3 4 2 5 4 3 3 1 2 2 4 4 3 3 25 29 .
1 3 2 1 3 1 5 3 3 2 1 1 2 3 4 2 3 16 20 3 x 2
As number of columns in B and number of rows in A are not same, BA is not available.
104
5 2 5 4 2 3 5 1 2 2 5 2 2 2 26 9 14
4 1 2
3 2 2 3 4 4 3 3 1 4 2 3 2 4 2 24 11 14
BC = 3 4
1 3 1 4 3 3 11 3 2 1 2 3 2 13 7 8
3 x3
5 2
4 1 2 4 5 1 3 2 1 4 2 1 4 2 3 25 18
CB = 3 4
3 2 2 1 3 3 5 2 3 2 1 3 2 2 4 2 3 23 20 2 x 2
Here BC and CB are not comparable as their orders are not same. BC CB.
4 3 1
4 1 2 4 4 1 2 2 1 4 3 1 4 2 3 4 1 1 3 2 2
CA= 2 4 3
3 2 2 1 3 2 3 4 2 2 2 1 3 3 2 4 2 3 3 1 2 3 2 2
20 22 11
=
18 23 13 2 x 3
Here number of columns in A and number of rows in C are not equal. A and C are not conformable for
multiplication. So, AC is not available.
( i ) Matrix multiplication is nor commutative, i.e. if A and B are two matrices, AB and BA are not, in
general, comparable and hence AB BA. Let us investigate this property by example. Let
5 3 4 3
A= , B= . Here
2 7 5 2
5 3 4 3 5 4 3 5 5 3 3 2 35 21
AB =
2 7 5 2 2 4 7 5 2 3 7 2 43 20 2 x 2
4 3 5 3 4 5 3 2 4 3 3 7 26 33
BA=
5 2 2 7 5 5 2 2 5 3 2 7 29 29 2 x 2
Here AB and BA are comparable but AB BA. Again, let us consider another two matrices A and B,
where
1 3
1 4 5 2 4
A= and B =
2 3 4 3 1
105
1 3
1 4 5
AB = 11 4 2 5 3 1 3 4 4 5 1 24 24
Here 2 4 22 2 x 2
2 3 4 3 1 2 1 3 2 4 3 2 3 3 4 4 1 20
1 3 11 3 2 1 4 3 3 1 5 3 4
1 4 5
2 3 4 2 1 4 2 2 4 4 3 2 5 4 4
BA = 2 4
3 1 3 1 1 2 3 4 1 3 3 5 1 4
8 13 17
= 10 20 26
5 15 19 3 x 3
( ii ) If AB =0 , then A =0 or B = 0 may not be true. Let us verify the fact by an example. Given
1 1 1 0
A = ,B ,
1 1 1 0
1 1 1 0 11 11 1 0 1 0 0 0
then AB = 0 . But A 0 and B 0.
1 1 1 0 11 11 1 0 1 0 0 0
( iii ) Matrix multiplication is associative, i.e. if A , B and C are 3 matrices and they are conformable for
multiplication, then A ( BC) = ( AB ) C. Let us investigate the phenomenon. Given
4 1
4 3 2 3 1
A ,B , C 5 2
2 1 3 4 2 1 4
4 3 2 3 1 4 2 3 3 4 3 3 4 4 1 3 2 17 24 10
Now, AB = 2 1 3 4 2 2 2 1 3 2 3 1 4 2 1 1 2 7 10 4 2 x 3
4 1
17 24 10
(AB)C= 17 4 24 5 10 1 17 1 24 2 10 4 198 105
5 2
7 10 4 1 4 7 4 10 5 4 1 7 1 10 2 4 4 82 43 2 x 2
4 1
2 3 1
BC= 2 4 3 5 1 1 2 1 3 2 1 4 24 12
5 2
3 4 2 1 4 3 4 4 5 2 1 3 1 4 2 2 4 34 19
106
4 3 24 12 4 24 3 34 4 12 3 19 198 105
A ( BC ) = 43 2 x 2
2 1 34 19 2 24 1 34 2 12 119 82
( iv) Matrix multiplication is distributive, i.e. if A, B, C and D are matrices which are conformable for
multiplication, then A ( B +C) = AB + AC, ( B + C ) D = BD + CD . Let us investigate this character by
example. Let
3 2 3 5 5 2 7 3
A , B , C , D
5 7 4 3 1 4 6 2
3 2 3 5 3 3 2 4 3 5 2 3 17 21
AB 43 46
5 7 4 3 5 3 7 4 5 5 7 3 2x2
3 2 5 2 3 5 2 1 3 2 2 4 17 14
AC
5 7 1 4 5 5 7 1 5 2 7 4 32 38 2 x 2
3 5 7 3 3 7 5 6 3 3 5 2 51 19
BD
4 3 6 2 4 7 3 6 4 3 3 2 46 18 2 x 2
5 2 7 3 5 7 2 6 5 3 2 2 47 19
CD
1 4 6 2 1 7 4 6 1 3 4 2 31 11 2 x 2
3 5 52 3 5 5 2 8 7
BC
4 4 1 3 4 5 7
,
4 3 1
7 3 8 2 5 3 7 2 7 34 35
3 2 8
A( B C )
7 5 8 7 5 5 7 7 7 75 84 2 x 2
5 7 5
17 21 17 14 17 17 21 14 34 35
AB AC
43 46 32 38 43 32 46 38 75 84 2 x 2
Therefore, A ( B +C ) = AB +AC.
8 7 7 3 8 7 7 6 8 3 7 2 98 38
(B+C)D= 6 2 5 7 7 6 5 3 7 2 77 29
5 7 2x2
Hence ( B+ C ) D = BD + CD.
( v ) Let A=[ ai j ]m x n be a matrix and Im and In be two identity matrices of order m and n , respectively.
Then Im A = A = A In .
107
Let us investigate this property by example. Let
5 8 1 0
A , I2
3 7 2 x 2 0 1 2 x 2
1 0 5 8 1 5 0 3 1 8 0 7 5 8
I2 A A
0 1 3 7 0 5 1 3 0 8 1 7 3 7
Now,
5 8 1 0 5 1 8 0 5 0 8 1 5 8
AI2 A
3 7 0 1 3 1 7 0 3 0 7 1 3 7
3 2
3 5 8 4 7 4 3
A= , A 5 6 ,B= , B
2 6 9 2 x 3 8 9 3 x 2 3 2 2 x 2 7 2 2 x 2
Here elements in rows of A are the elements in columns of A/. This A/ is called the transpose matrix of
A. Similarly, B/ is the transpose of B matrix.
A
3 5 2 4 3 5 2 4 3 2 5 5 3 4 5 3 31 27 31 43
4 7, B 5 , AB , ( AB ) /
3 4 7 5 3 4 2 7 5 4 4 7 3 43 37 27 37 2 x 2
2 5 / 3 4 / / 2 5 3 4 2 3 5 5 2 4 5 7 31 43
B/ , A 5 7, B A 4 3 5 7 4 3 3 5 4 4 3 7 27 37 ( AB)
/
4 3 2x2
1 4 3 1
3 2 1 2 3, B / 1 2 3, AB 10 19 , ( AB ) / 10 13
A , B 2 3 , A /
4 3 1 13 15 19 15
1 3 2 2 x 3 3 1 3 x 2 1 2 2x2
108
3 1
1 2 3
B A
/ / 1 3 2 2 3 1 11 2 3 3 2 10 13 ( AB ) / . Also, let
2 3
4 3 1 1 2 4 3 3 2 11 4 1 3 3 1 2 19 15 2 x 2
2 3 4 3
4 3 1 2 1 3
A 1 4 , B ,A
/
, B 3 2
/
3 5 3 x 2 3 2 3 2 x 3 3 4 5 2 x 3 1 3 3 x 2
4 3 4 2 3 3 4 1 3 4 4 3 3 5 17 16 27
2 1 3
B A 3 2
/ /
3 2 2 3 3 1 2 4 3 3 2 5 12 11 19
1 3
3 4 5
1 2 3 3 1 1 3 4 1 3 3 5 11 13 18 3 x 3
2 3 2 4 3 3 2 3 3 2 2 1 3 3 17 12 11
4 3 1
AB 1 4 1 4 4 3 1 3 4 2 11 4 3 16 11 13
3 5
3 2 3
3 4 5 3 3 3 5 2 3 1 5 3 27 19 18 3 x 3
17 16 27
( AB ) 12 11
/
19 B / A /
11 13 18 3 x 3
( ii ) Let A = [ ai j]m xn and B = [ bi j]m xn such that A and B are conformable for addition,
( A + B )/= A/+ B/
3 4 4 3 3 4 4 3 7 7
A ,B , A B ,
2 1 2 5 2 2 1 5 4 6
For example, let
7 4 / 3 2 4 2 3 4 2 2 7 4
( A B) / , A B/
7 6 4 1 3 5 4 3 1 5 7 6
3 2 0 3 0 6
, B 1 0 5 , find ( i ) A , ( ii ) B , ( iii ) ( A + B ) , ( iv)
/ / /
Example 3.19: Given A = 0 1 4
4 0 2 3 x 3 0 3 2
(A/)/
109
/
3 0 4 3 1 0 3 0 4 3 2 0
Solution: ( i ) A/ = 2 1 0 , (ii ) B / 0 0 3 , ( iv) ( A/)/= 2 1 0 0 1 4
0 4 2 3 x 3 6 5 2 3 X 3 0 4 2 4 0 2 3 x 3
3 3 2 0 0 6 6 2 6 6 1 4
( iii ) A B 0 1 1 0 4 5 1 1 9
.( A B) 2 1 3
/
4 0 0 3 2 2 4 3 4 3 x 3 6 9 4 3 x 3
3 2 0 9 6 0 3 2 0 3 0 6
( v ) KA= 3 A =3 30 1 4 0 3 12 ( vi) A – B = 0 1 4 1 0 5
4 0 2 12 0 6 3 x 3 4 0 2 0 3 2
3 3 2 0 0 6 0 2 6
= 0 1 1 0 4 5 1 1 1
4 0 0 3 2 2 4 3 0 3 x 3
3 0 4 3 1 0 3 3 0 1 4 0 6 1 4
( vii ) A + B = 2 1 0 0 0 3 2 0 1 0 0 3 2 1 3 ( A B) /
/ /
0 4 2 6 5 2 0 6 4 5 2 2 6 9 4 3 x 3
Let A = [ ai j ]m x n be a matrix of order ( m x n ). The matrix A has m rows and n columns. Some of the
rows of A or columns of A may be independent. The number of independent rows ( columns) of a matrix
is the rank of that matrix. For example, let
4 5
A= , where |A | = 4 x 7 – 3 x 5 = 13 0
3 7 2 x 2
Here the matrix A is a non-singular matrix . By elementary operation first row ( column) of the matrix is
not available from the second row ( column ). In this case rows ( columns) of matrix A are independent
and there are 2 rows ( 2 columns). Hence rank of A = 2 [ Rank( A) ( A) 2]
For the above given matrix A the determinant value of order 2 x 2 exists. This determinant value indicates
that the minor of order 2 x 2 formed from the matrix A exists. Hence, we can define the rank of a matrix
A is the order of a non-vanishing minor formed from the matrix A. Thus the rank of a matrix A is Rank (
A ) = ( A) min( m, n) , where m= number of rows, n = number columns. Let r = min ( m n ), then
Rank ( A ) = r , if r is the order of the non-vanishing minor formed from the matrix A, r = 1,2, , min(
m,n ) . Thus, unless the matrix is null, the rank of a matrix is at least 1. For example, let us consider the
matrices A , B, C and D, where
110
4 3 2
5 7 3 6 0 0
A , B 1 2 , C 0 0 , D 6 5 3
3 5 2 7 1 3 x 3
Here |A | = 5 x 5 – 7 x 3 = 4, Rank ( A ) = 2. For the matrix B the minor of order 2 x2 is |B|= 3x2 – 6 x
1=0. So, Rank ( B ) can not be 2. As there exists at least one element in the matrix B , the Rank ( B ) = 1.
As
C is a null matrix, no minor of at least order 1 x 1 exists , so rank of C is zero. Again, the determinant
value of the matrix D is
|D | = 4( 5 x1 – 3 x 7) – 3( 6 x 1 – 3 x 2 ) + 2 ( 6 x7 – 2 x 5 )= -4 x16- 3 x 0 + 2 x 32 = 0
The matrix D is a singular matrix [ the minor of order 3 x 3 formed from the D matrix does not exist] and
hence the rank of D cannot be 3. It will be less than 3. Now, let us investigate the existence of the minor
of order less than 3. One of the minor of order 2 x 2 formed from D matrix is [ minor corresponding to the
element of first row and first column, 4]
5 3
7 1
The value of this minor [ cofactor = ( - 1 )1+1(5 x1 – 3 x 7 ) = -16 ] is not zero and hence the rank of the
matrix D is 2. It is further seen that the elements of the first column of the matrix D are twice of the
elements of third column of D. So, at least two columns of D are dependent and hence the Rank ( D )
cannot be 3.
Remarks:
( b ) Rank ( A ) =r indicates that there exists at least a minor of order r formed for any element ai j of the
matrix A.[ i = 1, 2, 3, ….., m ; j = 1, 2, 3, ……,n].
( c ) For a matrix A if there exists at least a minor of order r , the Rank ( A ) cannot be less than
r= min (m,n).
( e ) For an identity matrix of order n x n , the rank is n [ Rank ( In ) = n , where In is an identity matrix of
order n x n.].
111
( h ) If A * is the conjugate transpose of the matrix A , then Rank ( A*) = Rank ( A ).
( j ) If or a square matrix A of order n x n rank of A is r , then ( n – r ) is the nullity of the matrix. This
( n – r ) is the complementary number to r. For a rectangular matrix both ( m – r ) and ( n – r ) are the
nullity, where the matrix is of order m x n.
4 6
7 4 3 6 1 0 4 5 8
(i)A= , (ii ) B , (iii ) C , (iv ) D , (v) E 3 5
8 5 5 10 0 1 3 9 6 5 7
( iv ) The matrix D is a rectangular matrix of order 2 x 3, so its rank cannot be more than 2 [ Rank =r=
4 5
min (2,3 )]. Here a minor of order 2 x 2 is . Its determinant value is ( 4 x 9 – 5 x 3 ) = 30. As the
3 9
minor of order 2 x 2 exists, so Rank ( D ) = 2.
( v ) The matrix E is a rectangular matrix of order 3 x 2, so its rank cannot exceed 2. Here one minor of
order 2 x 2 is
4 6
. Its determinant value is ( 4 x 5 - 6 x 3 ) = 2. As the minor of order 2
3 5
x 2 exists, the rank of E is 2. [ Rank( E ) = 2 ]
Example 3.21 Find the rank of each of the following matrices by usual method or by E- operation.
112
5 3 7 1 1 1 4 5 1
( i)A= 2 4 6 ,( ii ) B = 4 9 16 ( iii ) C = 8 7 2 ,(iv) D =
5 3 8 5 8 9 4 9 1
1 4 6 3 7 9
2 5 5, E 6 10 12
3 6 8 9 13 15
5 3 7
4 6 2 6 2 4
2 4 6 5 3 7
Solution: ( i ) |A | = 3 8 5 8 5 3
5 3 8
5 (4 8 6 3) 3(2 8 6 5) 7(2 3 4 5) 70 42 98 14 0
1 1 1
( ii ) | B | = 4 9 16 . By elementary operation [ E – operation ] we can transfer C2~C2 – C1 and
5 8 9
1 0 0
5 12
C3~C3-C1 . It gives | B | = 4 5 12 1 1(5 4 12 3) 16 0
3 4
5 3 4
4 5 1
( iii ) | C | = 8 7 2 . By elementary operation [ E – operation ]we can transfer column-1 to new
4 9 1
column with every element zero i.e. C1~C1- 4C3. It gives
0 5 1
| C | = 0 7 2 =0. Therefore Rank ( C) < 3. Now one of the minor of order 2 x 2 is
0 9 1
5 1
5 2 1 7 3 0 . As one minor of order 2 x 2 exists, the Rank ( C ) = 2.
7 2
1 4 6 1 3 5
( iv ) |D |= 2 5 5 2 3 3 ;C2~ C2 - C1 , C3~C3 – C1
3 6 8 3 3 5
113
2 0 0
3 3
= 2 3 3 2 , R1~R1 – R3 . We have - 2 ( 3 x 5 – 3 x 3) = -12 . The matrix D is
3 5
3 3 5
non – singular and hence Rank ( D ) = 3.
3 7 9 3 7 9
( v ) |E|= 6 10 12 3 3 3 , R2~R2 – R1 , R3 ~R3 – R2 . As two rows are identical after
9 13 15 3 3 3
3 7
3 10 7 6 12 0
Again, one minor of order 2 x 2 is 6 10
2 3 3 2 2 4
(i) A= , (ii ) B , (iii ) C
1 4 4 1 3 6
7 18
2 3 2 3 2 2 3 1 2 3 3 4
2
Solution: ( i ) We have A = A A= 6 19
1 4 1 4 1 2 4 1 1 3 4 4
3 2 3 2 3 3 2 4 3 2 2 1 17 8
( ii ) B2 = B B =
4 1 4 1 4 3 1 4 4 2 11 16 9
2 4 2 4 2 2 4 3 2 4 4 6 16 32 2 4
( iii ) C2 = C C = 8
3 6 3 6 3 2 6 3 3 4 6 6 24 48 3 6
114
2 4 1 2 0 4
2 3 3 0
Example 3.23: Given (i ) A 3 2 0 , B 1 3 5 , (ii ) A ,B
4 1 2 0 2 4 1 5 1 4
Solution:
2 4 1 2 0 4 2 2 4 1 1 0 2 0 4 3 1 2 2 4 4 5 1 4
(i ) AB 3 2 0 1 3 5 3 2 2 1 0 0 3 0 2 3 0 2 3 4 2 5 0 4
4 1 2 0 2 4 4 2 11 2 0 4 0 1 3 2 2 4 4 1 5 2 4
8 14 32
8 6 22
9 7 29
8 8 9 8 0 1
( AB ) 14 6 7 ,
/
( AB ) 8 8 1
/
32 22 29 9 10 7
Here | ( AB )/ | is found out by E – operation; R2~R2 – R1, R3~R3 – R2. The value of the determinant is
8 1 8 8
( AB ) / 8 1 8(56 10) 1(80 72) 376 0
10 7 9 10
2 3 3 0 2 3 3 1 2 0 3 4 9 12 9 8
( ii ) AB= , ( AB ) /
1 5 1 4 1 3 5 1 1 0 5 4 8 20 12 20
|( AB)/| = 9 x 20 – 8 x 12= 84. The matrix ( AB)/ is non-singular of order 2 x 2. Hence Rank ( AB)/ = 2.
B=[ bi j ]n x n.. If it is such that AB = BA = In , then B is called an inverse matrix of A. The inverse matrix
is calculated by the formula
adj A
A 1
A
where adj A is the transpose of the cofactor matrix of A and |A | is the determinant value of the matrix A.
The inverse matrix of A is available if A is a square matrix and | A | 0 . The adjoint matrix of A is given
by
115
/
A11 A12 ........A1n
A
21 A22 ........A2 n
............................
adj A = [ AI J] ,
/
or adj A = [ Aji ]n x n
Ai1 Ai 2 ..........Ain
.............................
An1 An 2 ..........Ann
5 8
where Ai j = ( -1 )I + j x minor of ai j. For example , let A = , then | A | = 5 x 13 – 8 x 8 = 1.
8 13
Now,
Example 3.24: Find inverse matrix of each of the following two matrices.
3 15 5
2 4
(i) A , (ii ) A 1 6 2
3 7 1 5 2
7 4 adj A 1 7 4 3.5 2
adj A = [ Ai j ]/ = , A1
3 2 A 2 3 2 1.5 1
6 2 1 2 1 2
A11= (-1 ) 1+1minor of 3 = 12 10 2 , A12 (1) 0 , Similarly, the other Ai j
5 2 1 2
values are calculated.
1 6 15 5 3 5 3 15
A13 (1)13 1, A21 (1) 21 5, A22 (1) 2 2 1, A23 (1) 23 0
1 5 5 2 1 2 1 5
116
15 5 3 5 3 15
A31 () 31 0, A32 (1) 3 2 1, A33 (1) 33 3
6 2 1 2 1 6
A11 A21 A31 2 5 0 2 5 0 2 5 0
adj A 1
adj A A12 1
A22 A32 0 1 1 , A 0 1 1 0 1 1
A A 1
13 A23 A33 1 0 3 1 0 3 1 0 3 3 x 3
3 4 8 6 6 8 3 6
(i) A , (ii ) B , (iii ) C , (iv ) D
5 7 5 4 4 6 5 10
1
Solution: Let A = [ ai j]n xn and Ai j be the cofactor of ai j . Then adj A = [Aj i] and A { A }1 adj A
.
adj A 1 7 4 7 4
adj A = [AJ I ] and A1
A 1 5 3 5 3 2 x 2
adj B 1 4 6 2 3
Aadj B = [ BJ I ] and B 1
B 2 5 8 2.5 4
( iii )|C|=6 x 6 – 8 x 4 = 4. C11=(-1)1+1 minor of c11=6, C12= (-1)1+2 minor of c12=-4, C21=(-1)2+1minor of
c21=-8
adj C 1 6 4 1.5 1
adj C = [ Cj I ] and C 1
C 4 8 6 2 1.5
117
1 2 3
Example 3.26: Find the inverse of the matrix A = 3 1 1 by the definition using adjoint matrix
3 2 3 3 x 3
3 15 5
and inverse by elementary operation of the matrix B , B= 1 6 2
1 5 2
A11 = ( -1)1+1 minor of a11 = (1 x 3 – 1 x 2 )= 1 , A12= (-1)1+2 minor of a12= (-1 ) ( 3 x 3 – 1 x 3 ) = -6,
A22 = (-1 )2+2 minor of a22 = ( 1 x 3 – 3 x 3 ) = -6, A23 = (-1 )2+3 minor of a23 = (-1) ( 1 x 2 – 2 x 3 ) = 4 ,
A31 = ( -1 )3+1 minor of a31 = ( 2 x 1 – 3 x 1 ) = -1, A32 = ( -1 )3+2 minor of a32 = ( - 1 ) ( 1 x 1 – 3 x 3 ) =8,
1 0 1 1 0 1 0.5 0 0.5
1
adj A = [ Aj I ] = 6 6 8 , A
adj A 1
6 6 8 3 3 4
A 2
3 4 5 3 4 5 1.5 2 2.5 3 x 3
IB=B
1 0 0 3 15 5 3 15 5
0 1 0 1 6 2 1 6 2 ,
0 0 1 1 5 2 1 5 2
We have to make elementary operation in such a way that the right hand side matrix in the identity
becomes inverse of B matrix and left hand matrix becomes identity matrix. Let us transform R1~R1 3
. Then , we have
1 / 3 0 0 1 5 5 / 3
0 1 0 B 1 6 2
0 0 1 1 5 2
118
R2~R2 +R1 and R3~R3 – R1 , we get
1 / 3 1 0 1 15 / 3 5 / 3
1 / 3 1 0 B 0 1 1 / 3
1 / 3 1 1 0 1 1 / 3
R3~3R3
1 / 3 1 0 1 5 5 / 3
1 / 3 1 0 B 0 1 1 / 3
1 0 3 0 0 1 / 3
R1~R1+5 R2
2 5 0 1 0 0
1 / 3 1 0 B 0 1 1 / 3
1 0 3 0 0 1
R2~R2+1/3(R3)
2 5 0 1 0 0 2 5 0
0 1 1 B 0 1 0 B 1 0 1 1
1 0 3 0 0 1 1 0 3
1 4 2 5 -1 -1 -1
Example 3.27: Given A = and B= , show that ( AB ) = B A .
2 10 3 9
A11 = ( -1 )1+1 minor of a11 = 10, A12 = ( -1 )1+2 minor of a12 = - 2, A21 = ( - 1 )2+1 minor of a21=-4,
B11 = ( -1)1+1 minor of b11 = 9, B12 = ( -1)1+2 minor of b12 = -3 , B21 = ( -1 )2+1 minor of b21 = -5,
119
9 5 1 adj B 1 9 5 3 5 / 3
adj B = , B B 3 3 2 1 2 / 3
3 2
1 4 2 5 1 2 4 3 1 5 4 9 14 41
AB = , | AB| = 14 x100 – 41 x 34 = 6
2 10 3 9 2 2 10 3 2 5 10 9 34 100
50 41
100 41 1 adj ( AB ) 1 100 41 3 6
In a similar way, adj AB =
6 34 14 17 7
, ( AB )
34 14 AB
3 3
50 41
3 5 / 3 5 2 3 5 (5 / 3) 1 3(2) (5 / 3) (1 / 2) 3 6
B 1 A1
1 1 / 2 1 5 (2 / 3) (1) 1 2 (2 / 3)(1 / 2 ( AB )
1
1 2 / 3 17 7
3 3
3.5 Solution of System of Equations Using Matrix
Let x1 , x2 , ……..xn be n unknown quantities. Let us consider some linear combinations of these
unknown quantities. The linear combinations are
…………………………………………………………………………
………………………………………………………………………..
A X = B ……………………(3.1)
120
There are n equations for the linear combinations of n unknown quantities. Let us consider that ai j values
and bi values are known [ i = 1,2 , 3, …….,n ; j = 1, 2, 3, ……..,n] . The matrix A in equation ( 3.1 ) is
called coefficient matrix of the linear combinations of X vector of unknown quantites, B is a column
vector of n known quantities. The problem is to find the values x1, x2 …….xn contained in the vector X.
The right hand side of equation ( 3.1 ) is the vector of constant. The elements of this vector may all be
zero i.e. B = 0. In that case the linear combinations of unknown quantities xi‟s can be shown in matrix
notation
AX= -B ………………………….(3.2)
This equation is known as homogeneous equation. If B 0, then the equation is known as non-
homogeneous equation. We need the solution of both homogeneous ( 3.2 ) and non-homogeneous
( 3.1) equations.
From equation (3.1) , we have X = A-1 B, provided A is a non-singular matrix. For example, let
x–y=2
2x + 3y = 14
Here
3 1
1 1 x 2 1 5 5 1 0.6 0.2 2 0.6 2 0.2 14
A , X , B ,A ,X A B
2 3 y 14 2 1 0.4 0.2 14 0.4 2 0.2 14
5 5
Therefore, x = 4, y = 2.
This is an example of non-homogeneous equation. Let us consider another pair of equations as given
below:
x + 4y + 2z = 9
2x + 4y + z = 12
3x + 12y + 6z = 39
A X = B ……… (3.3)
1 4 2 x 9
Here A 2 4 1 , X y , B 12
3 12 6 3 x 3 z 39
121
the equation (3 .3) represents a non – homogeneous equation. Again, consider another two simultaneous
equations as given below:
2x - 3y = 0
3x + 4y = 0
A X = B ..……… (3.4)
2 3 x 0
Here A , X , B
3 4 y 0
|A| = 2 x 4 – (-3) x 3 = 17 implies that A is a non – singular matrix and hence X = A-1B exists. But at
least one of the solution is x = y = 0. The equation (3.4) is an example of homogeneous equation and its
solution is known as zero solution or trivial solution.
From the above discussion it can be concluded that homogeneous equations may provide trivial solution
and non-homogeneous may or may not provide the solution of the equations. The availability of solution
of the equations depends on some other characteristic which is known consistency of the equations.
Rank ( A ) = Rank ( A |B ) [ ( A) ( A | B) ]
Here the matrix A|B is called the augmented matrix. This property of the rank of the coefficient matrix
and rank of the augmented matrix is known as consistency of the equations. If the equations are not
consistent, the solution of the equations is not available and for consistent equations the solution, unique
or infinite , is available.
2 3 x 5
Thus , when we have the equation of the type ,then | A |= 2 x 5 – 3 x 3 = 1 implies that
3 5 y 8
2 3 5
Rank (A) = 2 . Again , for the matrix [A|B]= the rank can not be more than 2 as minor of
3 5 8
order 3 x 3 cannot be formed. At least one minor of order 2 x 2 of [ A | B] exists. Hence Rank ( A ) and
Rank ( A|B) are equal and thus the equations are consistent and the solution of the equations is available.
The solution is
For the equation ( 3.3) given above Rank ( A ) =2 [ As |A| = 0] . The augmented matrix [ A | B ] is
122
1 4 2 9
[ A | B ] = 2 4 1 12
3 12 16 39 3 x 4
From this matrix the minor of order 4 x 4 cannot be formed and hence Rank ( A| B ) 3. Again, one of
the minor of ( A|B) of order 3 x 3 is
1 4 9
2 4 12 =1( 4 x 39 – 12 x 12 ) – 4 ( 2x39 – 12 x 3 ) )+ 9 ( 2 x 12 – 4 x 3 ) = - 48 0 .
3 12 39
Therefore, Rank ( A | B ) 3. From the above two results of Rank( A|B), we can conclude that
Rank (A|B)=3. The equations are not consistent and hence the solution of the equations is not available,
though the equations are non-homogeneous.
3x + 2y = 0
2x + 3y = 0
3 2 x 0
A , X , B
2 3 y 0
3 2 0
In this case | A | = 3 x 3 – 2 x 2 = 5. So, Rank (A) = 2. The augmented matrix is [ A |B ]= .
2 3 0
The Rank ( A|B ) can not be less than 2 and it can not be 3, i.e. Rank ( A ) = Rank ( A|B ) . The equations
are consistent , but the solution of the equations is trivial. It implies that ,even a set of homogeneous
equations are consistent , unique solution is boe available. For non-homogeneous equations there may be
unique or infinitely many solutions depending on the consistency of the equations. Unique solution is
available if Rank of coefficient matrix [ Rank (A ) ] is full. If the coefficient matrix is a rectangular one
, then Rank A < min ( m.n ). In that case A-1 does not exist and hence unique solution of the equation is
not available. However, if | A | = 0 , generalized inverse is available. Here generalized inverse is not
discussed.
Example 3.28: Solve the following system of simultaneous equations using matrix method.
2x – 3y = -1
4x + y = 5
123
Solution: We can write A X = B, where
2 3 x 1
A , X , B . | A | = 2 x 1 – ( -3) x 4= 14, Rank ( A ) = 2. The
4 1 y 5
2 3 1
augmented matrix is [ A|B] = . As minor of order 3 x 3 cannot be formed from this
4 1 5
augmented matrix, rank of the matrix cannot be 3. As two columns of A matrix exist in ]A|B], Rank (
A|B) cannot be less than 2, i.e. Rank ( A ) = Rank ( A|B). The equations are consistent and as |A| 0,
unique solution of the equations is available and the solution is X = A-1 B. Here
1 3
adj A 1 1 3
A1 14 14
A 14 4 2 4 2
14 14
1 3 1 1 3 5
1 14 14 1 14 14 1
Hence X A B x = y =1
4 2 5 4 1 2 5 1
14 14 14 14
Example 3.29: The equilibrium condition of supply and demand of a commodity is as follows:
Example 3.30: Find the solution of the system of equations given below by matrix method.
124
3 x + 4 y = 10
5 x - 10 y = 0
3 4 x 10
A , X , B . |A|= (-10) x 3 – 4 x 5 = -50 0 . Hence ,
5 10 y 0
3 4 10
Rank( A ) = 2. The augmented matrix is [A|B] = . As there is no scope to form minor of
5 10 0
order 3 x 3 using the elements in [ A |B], the rank of [A |B] cannot be more than 2. Again, first
two|columns of [ A |B] are the columns of matrix A and A is a non-singular matrix, the rank of [A|B]
cannot be less than 2. From these arguments, we can say tht Rank ( A |B ) = 2 and the equations are
1
consistent. The soluyion of the equations is X A B
10 4 10 10 4 0
adj A 1 10 4 10 50 50 10 50
50 2
X B x=2, y=1.
A 50 5 3 0 5 3 0 5 10 3 0 1
50 50 50 50
x + y + 2z = 5
3x + y - 3z =2
2x + y – z = 4
1 1 2 x 5
A 3 1 3 , X y , B 2
2 1 1 z 4
1 1 2
A 3 1 3
Let us make elementary operation C2~C2 – C1 , C3~C3 – 2C1. It gives
2 1 1
1 0 0
A 3 2 9 , |A | = (-2 ) x (-5) – 1 x 9 = 1 . Hence Rank ( A ) = 3. Now, let us investigate
2 1 5
the consistency of the equations. The augmented matrix is
125
1 1 2 5
3 1 3 2
(A|B)=
2 1 1 4 3 x 4
The augmented matrix is of order 3 x 4 and hence the minor of order 4 x 4 cannot be formed from this
augmented matrix. It implies that rank of ( A|B) cannot be 4 [ ( A | B) 3] . Again, the elements of
the columns in matrix A exist in the columns of the matrix ( A |B) and hence the rank of this matrix
cannot be less than 3 [ ( A | B) 3] . From these two conclusions about the rank , we can say thay the
Rank ( A | B ) = 3. So, the equations are consistent and as |A | 0 , the uique solution of the equations is
1
available and the solution is X A B . Here adjoint of matrix is given by
2 3 5 2 3 5 5 2 5 3 2 5 4 4
adj A 3 5 9 , X A 1 B
adj A
B 3 5 9 2 3 5 5 2 9 4 11
1
A 1
1 1 2 1 1 2 4 1 5 1 2 2 4 1
Therefore, x = - 4, y = 11 , z = -1.
2x–y +z=8
x + y – z =-2
x – y + 4 z = 15
x
2 1 1 y 8
A 1 1 1 , X , B 2
z
1 1 4 15
2 0 1
A 1 0 1 . Again, by elementary operation R1~R1 +R2, we have
1 3 4
3 0 0
A 1 0 1 , | A | = 0 x 4 + 1 x 3 = 3 0 Rank (A) = 3. The augmented matrix (A|B) is
1 3 4
126
2 1 1 8
1 2 ,
( A | B ) = 1 1
1 1 4 15 3x 4
Here no minor of order 4 x 4 can be formed and hence rank of ( A|B) cannot be more than 3
[ ( A | B) 3 ]. Again, the columns in matrix A exist in columns of ( A|B) and hence rank of ( A | B)
cannot be less than 3 [ ( A | B) 3 ]. From these two conclusions on rank we can say that
Rank (A|B)=3 = Rank ( A ) and hence the equations are consistent and as A matrix is of full rank , the
unique solution of the equations is available, where the solution is X = A-1 B . Now, by elementary
operation the solution can be found out. The steps of operation are shown below:
AX B
2 1 1 x 8
1 1 1 y 2
1 1 4 z 15
2 1 1 x 8
0 3 3 y 12
0 1 7 z 22
2 1 1 x 8
0 3 3 y 12
0 0 18 z 54
2x–y+z=8
Or 2 x – ( - 1 ) + 3 = 8
Or 2 x = 4
Or x = 2.
Therefore, x = 2, y = -1 , z = 3.
Example 3.33: The market equilibrium condition of supply ( S ) and demand ( D ) of a commodity is
given by
127
0.4 S + 0.3 D = 180
Here | A | = 0.4 x 0.5 – 0.3 x 0.6 = 0.02 and hence Rank ( A ) = 2. The matrix A is of order 2 x 2 and the
augmented matrix ( A | B) is of order 2 x 3. So, rank of (A|B) cannot be more than 2 and cannot be less
than 2, i.e. Rank ( A|B) = 2. The equations are consistent. Therefore, unique solution of the equations is
1
available and the solution is X A B. Here
Supplementary Problems
4 2 3
6 2
1 . Given A 4 5 4 , find |A |. Ans. 91; 2 . Given A
5 6
, find |A|. Ans. 26 .
3 2 10
2 3 1 6 3 4 1 3
3 . Given ( i ) A ,B ; ( ii ) A ,B .Show that ( AB ) = B A
/ / /
5 7 3 4 5 2 4 2
1 6 2 2 3 0
4 . Given A 3 4 0 , B 1 1 2 , find ( i ) A + B , ( ii ) A – B
5 1 3 4 2 1
128
3 9 2 1 3 2
2 3 2
Ans . ( i ) A + B= 4 5 2 , ( ii ) A – B =
9 3 4 1 1 2
5 2 1 4
5 . Given A = , B 2 10 , show that AB = I2 .
1 0.5
5 8 0
0 , show that A2 = I3.
6 . Given A = 3 5
1 2 1
1 5 6 2 3 1 0 0 2
7 . Let A 3 4 0 , B 0 4 2 , C 1 4 1 , show that ( A + B ) + C = A + ( B +C)
0 1 1 1 0 2 0 2 1
0 1 1 0 3 3
2 4 0
8 . Given A = , find 3 A . Ans . 6 12 0
4 2 3 12 6 9
4 2 1 3 5
9 . Given A = ,B , find AB and show that BA does not exist.
1 5 2 x 2 2 4 6
8 20 32
Ans. AB =
11 23 35 2 x 3.
1 2 3 6 2 3 1 0 0
1 3 3 and B 1 1 0 , show that BA = 0 1 0
10. If A=
1 2 4 1 0 1 0 0 1
3 4 1 2 13 20
11. If A = , find A1 and A2 . Ans. A1 , A2
1 2 0.5 1.5 5 8
3x – 4y = 5
2x + 3y = 9 Ans. x = 3 , y = 1
2 1 1 1 1 1 5 2
13 . Given A and B , find B A . Ans. 3.5 2.5
4 3 1 2
129
8 4 2 74 68 48
36 80 68
14 . Given A= 2 8 4 , find |A |, A2. Ans. | A |= 392, A2=
1 2 8 20 36 74
1 4 0 1 / 3 2 / 3 2 / 3
1
Ans.1 / 6 1 / 6 1 / 6
15. Given A = 1 2 2 , find A .
0 0 2 0 0 1/ 2
1 8 5 1 / 2 1 / 2 1 / 2
3 , find A 1 . Ans. 4 3 1
16. Given A = 1 6
1 2 1 5 / 2 3 / 2 1 / 2
2 1 4 2 1 1 2 7 / 2 1/ 2
1 0 2 1 1 2
, find ( AB ) . Ans. 11 / 3 119 / 18 17 / 18
-1
17. Given A = and B =
2 3 1 3 2 1 7 / 3 73 / 18 13 / 18
2 1 4 2 11 14
3 6 0 .
18. If A = 1 4 2 , find |A| and adjoint of A, Ans. |A |= -21, adj A =
2 3 1 3 4 6
2 1 1 2 0 7
3 3 6
19. Find rank of the ( i ) matrix A = 0 3 2 and ( ii ) matrix B = . Ans.( i ) 2, ( ii ) 2.
2 4 3 2 2 4
0 1 2 3 4 5 6
0 0 1 1
20. Given ( i ) A = and ( ii) B = 5 6 7 , find rank of A and B. Ans.( i ) 2, ( ii ) 2.
0 0 0 0 7 8 9
1 3 5
1 3 5 2 1 4 3
4 2 and C = 3 4 ,show that ( i ) A = A. ( ii ) ( BC) =C B .
2 / / /
21. If( i ) A = , ( ii ) B=
1 3 5
( i ) 4 x1+ 8 x2 + 6 x3 = 0 , ( ii ) x + 2 y + 3 z = 0 , ( iii ) 2 x + 3y +z = 6 , ( iv ) 2 x – 3 y + z =0
5 x1 +9 x2 + 10 x3 = 6 7x + 13 y + 9 z = 0 x+4y–3z=2 4x–y–2z=0
130
(v)x+2y+z=4 ( vi ) 2x + 3y + 4 z = 4 ( vii ) x - 3 y + z = -1 ( viii ) 2 x + y + z = 5
2x+3y–z=1 3 x + y + 6 z = -1 6x – 7 y + 8 z = 7 2x + y – z = 1
( ix ) 3 x + y + z = 1 ( x ) 5 x – 3 y + 4 z = 11 ( xi ) 2 x + y – 3z = 0 ( xii ) 6 x – 5 y + z = 2
20 1 22
(v) x , y ,z ( vi ) x = 1, y = 2 , z = - ; ( vii ) x = 6.5, y= 4, z = 4.5; ( viii ) x=y=1, z = 2;
9 3 9
(i)2x–y=3 , ( ii ) 2 x - y = 1 , ( iii ) 4 x + 3 y = 2 , ( iv ) 3 x – 7 y = -1 , ( v ) 5 x + 3 y = 13
-x + 2 y = -3 3 x – 4 y = -1 3 x + 4 y = -2 2x+3y=7 3x–2y=4
4 3 2 3 5 1 5 6
3 5 4 5 3
(i ) A 5 2 1 , (ii ) B , (iii ) C , D 4 9 , E 5 3 7
3 5 6 7 9 2 4 9 2 3 6 7 2
4 5 3 4 2
3 7 5 4 , (iv ) D / 3 4 2 ,
(i ) A 3 2 5 , (ii ) B
/ /
, (iii ) C /
5 9 3
2 1 6 5 9 3 9
Ans .
1 5 6
(v) E 5 3 7
/
6 7 2
2 1 4 2
25. Given A = and B = , find AB , BA and A .
2
3 2 1 6
131
9 10 14 8 7 4
Ans. AB = , BA= and A =
2
12 7 .
14 18 20 7
2 5 3
4 7 3 8
2 . B= , C =
5 14
A = 3 1
1 2 1 3 5
5 11 7
11 B 1 5 7 1
7 4
Ans. A 5 5 5 C 2.5 1.5 .
30 . ,
5 1 13 3 4
132
Chapter IV
Solution of Equations
4.1 Introduction
An equation is a claim that two quantities are equal. It is a string of mathematical symbols stating the
equality of two expressions.
x
For Example,( i ) 3x – 9 = 0 , (ii) 2 x + 5 = 9 , and (iii) 3 -3=2
These are examples of equations as there are equality of expressions , where these expressions are true for
some values of unknown x . For a particular equation the value of x is known as solution of the equation.
For example, equation (i ) can be written as
3x = 9
or x=3
The value of x = 3 when is replaced in equation (i) ,we have both sides of the equation same. Here x = 3
satisfies the equation (i). So, x = 3 is a solution of the equation. Similarly, the equation (ii) can be written
as
2x = 9 - 5
or 2x = 4
or x = 2
Here x = 2 is a solution to the equation (ii). There is no other value of x which satisfies the equation (ii).
If there is only one unknown variable x with power one in an equation, then the equation is known as
simple linear equation. For example, in the previous section the equations (i), (ii) and (iii) contain one
unknown variable x with power one in each case, so these equations are known as simple linear
equations.
In any equation there may be more than one unknown variables, each of which has power one, Then the
equation is known as multiple linear equation. For example, let x1 , x2, …….., xn be n unknown values of
n variables . Let us consider a linear combination of these variables as a1x1 +a2x2+ --------- + anxn . Now, if
this linear combination is expressed as equal to a constant quantity, say c, such that
then combination of x „s equals to c and hence this expression of x‟s is known as equation. This is an
example of multiple linear equation. It is also known as linear polynomial as each of xi‟s has power one.
133
The power of unknown variables x1 , x2, …….., xn may be k and in that case their combination can be
equated to a constant value , say d., i.e.
This latter equation is known as k – degree polynomial equation. Here ai‟s ( i= 1, 2, 3, -----,n) are the
known ( unknown ) constants and are known as coefficients of xi‟s . For example, let us consider the
equation of two variables x1 and x2 such that
These two equations are examples of second degree polynomial equation. These are also called non-linear
equations. More non-linear equations are
3x2 + 5x + 2 = 0
ax3 + bx2 + cx = 8
4x3 + 3x – 7 = 0
Again, let us consider that there are 2 variables x and y and their linear combination is ax + by + c. If
this linear combination equates to zero , then we can write
ax + by + c = 0
This equation is a linear polynomial of degree one provided a and b are not both zero.
Using the laws of elementary algebra , linear equations in two variables may be formulated in several
standards forms . One of the form is
where a and b both are not equal to zero. The equation is usually written so that a 0. The graph of this
equation is a straight line. Every straight can be represented by the above standard form of equation. If a
is non-zero, then the x-intercept , that is, the x-coordinate of the point where the graph crosses the x –axis
( where y is zero) is c/a. If b is non-zero, then the y-coordinate of the point where the graph crosses the y-
axis ( where x = 0 )is c/b and the slope of the line is( – a/b).
y = mx + c
where m is the slope [ it measures the change in y for unit change in x ] and c is the y-intercept which is
the y-coordinate of the location where the line crosses the y – axis. If we plot the graph for x = 0 , we
immediately get y = c [ c is the value of y when x =0].
134
y – y1 = m ( x – x1 )
where m is the slope of the equation and ( x1 , y1 ) is a point on the line [ the line passes through the pairs
of values ( x1 , y1 )]
The point-slope form expresses the fact that the difference in the y-coordinate between two points on a
line [ that is y – y1] is proportional to the difference in the x – coordinate [ that is x – x1]. The
proportionality constant is m.
The equation given above is one which passes through one point. But equation may pass through two
points ( x1 ,y1) and ( x2 , y2 ) such that x1 x2 . Here the equation is written as
y 2 y1
y – y1 = ( x – x1 ) ---------------------------------------------(1)
x 2 x1
This equation is equivalent to the point- slope form of the equation discussed above, where the slope is
y 2 y1
explicitly given as . Now, multiplying both sides of the equation (1) above by ( x2 – x1) ,we
x 2 x1
have
( x2 – x1 ) ( y – y1 ) = ( y2 – y1 ) ( y – y1 ) -----------------------------------------------(2)
On simplification, we have
x y 1
x1 y1 1 0
x2 y2 1
x y
1
a b -------------------------------------------------------------------------- (3)
where a and b must be non-zero. The graph of the equation has x – intercept a and y-intercept b . This
A 1 B 1
form is similar to standard form of equation discussed above if and , where
c a c b
Ax + By + c = 0
Lines that pass through the origin or which are horizontal or vertical violate the non-zero condition on a
or b and cannot be represented in this form.
135
The matrix form of the equation can be derived from the standard form of the equation
ax + by = c
a b
x
c
y
Here there is one linear equation. If there are more than one linear equations, we call them system of
linear equations. Let us consider a system of linear equations having two equations
a1 x + b1 y = c1
a2 x + b2 y = c2
a1 a2 x c1
b b y
1 2 c 2
This form can be extended for a system of equations containing n equations. In matrix form general n
simultaneous equations can be presented as follows :
x1 c1
a11 a12 a13........a1m c
a a a ........a x2 2
21 22 23 2m
.. ..
............................
.. ..
an1 an 2 an 3 ........anm x c n
m
where a1 , a2, ---------an are coefficients of x , x2 , --------xn respectively and b is a constant. The solution
of this equation is given by
b a1 x1 a2 x2 a x
n n
a
xi = i
ai ai ai
This is possible if ai 0. One may choose arbitrary values for all xi except one. There will be infinite
number of solutions and one can choose (n – 1) values of xi.
136
4.1.2 Quadratic Equation
A quadratic equation is one in which the unknown value of the variable has power maximum 2. For
example, let us consider that x is an unknown value of the variable. Then, we can have an equation of the
form
ax2 + bx + c = 0
Here a , b, and c are known constants such that a is not equal to zero [ a 0]. If a = 0 , then the equation
becomes linear.
The quadratic equation is uni-variate, where there is only one variable x [ or y or z …]. It is called
quadratic equation as s has power maximum 2 which is a non-negative integer. It is also called a
polynomial of degree 2 [ second degree polynomial equation ].
Solution of Quadratic Equation :The solution of the equation is done by factorizing the left-hand side
of the equation. For example, let us consider a quadratic equation 4 x2 + x – 3 = 0 .This equation can be
written as 4 x2 + 4x – 3x – 3 = 0
3
or 4 x ( x + 1 ) – 3 ( x + 1 ) =0; or (4x – 3 ) ( x + 1 ) = 0 ;or x = - 1 and x= .
4
After factorization, we write two linear equations because either of the factor can be taken as zero. Hence
write either x +1 =0 or 4x – 3 = 0 . From these two equations have the two values of x.
The above solution of the quadratic equation is done by factorization of the left-hand side of the equation.
But factorization may not always possible. Then the following steps are to be followed to solve the
quadratic equation.
2 x2 + 4x – 4 = 0
137
Solution from Quadratic Formula: The quadratic formula is given by
b b 2 4ac
x= , when the quadratic equation is ax2 + bx +c = 0.
2a
b b 2 4ac
x
2a 2a
3x2 – 2x – 8 = 0.
or 3x ( x – 2 ) + 4( x – 2 )=0
or (x – 2 ) ( 3x + 4 ) = 0
4
or 3x + 4 =0, giving x = - The above solution is available from
3
factorization of the quadratic equation. The solution can also be done using quadratic formula. From the
equation, we have a =3 ,b=- 2 and c= -8. The quadratic formula for the values of x is
b b 2 4ac (2) (2) 2 4(3)( 8) 2 4 96 2 10
2a 23 6 6
4
or x and x 2
3
2c 2(8) 16 16 8
b b 4ac
2
2 (2) 3(3)(8)
2
2 4 96 2 10 1 5
4
This formula gives x = 2 and x = .
3
ax2 +bx +c = 0
138
x2 + px + q = 0 , where p = ( b/a) and q = ( c/a)
p
2
p
x= q
2 2
9x2 + 24x + 16 = 0
The reduced form of this equation is [ obtained by dividing both sides of the equation by 9 ]
2
8 8
2
16 4 4 16 4
x 3 3
2 2 9 3 3 9 3
Here both the values of x are –(4/3). The equation can be written as ( 3x + 4)2 =0,or (3x +4) =0.
D = b2 – 4ac
This D is used to decide number and nature of the roots of the quadratic equation. The quadratic equation
with real coefficients can have either one or two distinct real roots or two distinct complex roots. The
decision regarding he nature and number of roots is taken as follows:
(i) If D = b2 – 4ac is positive, then there are two distinct roots. These are
b D b D
x and x
2a 2a
(ii) If the value of D is negative , then then there is no real root . Rather, there are two distinct
complex roots which are complex conjugates of each other. These are
139
b D b D
x i , and x i , Here i is the imaginary quantity.
2a 2a 2a 2a
b
(iii) If D = 0, then both the rots are same and these are 2a and the roots real and rational
number.
(iv) If D = b2 – 4ac is a perfect square, the two roots are real, rational but are different.
(v) If a root becomes irrational , then the other root is complex conjugate.
(vi) If D = b2 – 4ac = q , where q is rational but not perfect square, then the roots are
b q b q
, and
2a 2a 2a 2a
Let ax2 + b x + c = 0 is a quadratic equation, where a 0 and a and b are coefficients of x2 and x ,
respectively and c is constant. Let us consider that α and β be the roots of the equation. Then the two
factors of the quadratic equation are ( x – α ) and ( x – β ).
b c
Here x x 0 ( x – α ) ( x – β ) = x2 - ( α +β )x + αβ .
2
a a
b c
( ) and
a a
b c
or ( ) and
a a
Let us consider that the roots of a quadratic equation which is to be formed are α and β. Then, the
equation will be
(x–α)(x–β)=0
or x2 – (α + β )x + α β = 0
For example, let us consider that the two roots of a quadratic equation are 2 and 3. Then the equation is
x2 – ( 2 + 3 ) x +2 x 3 =0
or x2 – 5x + 6 = 0
140
As an another example, let the roots be ( 2 + 3 i ) and ( 2 - 3 i). The second root is the conjugate of first
root. Then , the quadratic equation will be
x2 – ( 2 + 3 i + 2 – 3 i) x + ( 2 +3 i) ( 2 – 3i) = 0
x2 – 4 x + 13 = 0 ,as ( 2 + 3 I ) ( 2 – 3 i) = 13
Let a1x2 + b1 x + c1 = 0 and a2x2 + b2 x + c2 =0 be two quadratic equations. Let the common root of
these two equations be α . Then, we can replace x by α in both the equations. This will give
a1 α 2 + b1 α + c1 = 0 and a2 α 2 + b2 α + c2 = 0
2 1
b1 c2 b2 c1 c1a2 c2 a1 a1b2 a2b1
b1c 2 b2 c1 c1 a 2 c 2 a1
. Again, we have .
c1 a 2 c 2 a1 a1b2 a 2 b1
b1c2 b2 c1 c1a2 c2 a1
c1a2 c2 a1 a1b2 a2b1
b1 c
( ) and 1 .This is from the first equation. From the
c1 a1
b2 c
second equation, we have (α + β )= ( ) and 2
a2 a2
b1 b2 a b
1 1 (ii )
a1 a2 a2 b2
c1 c2 a c
Again, 1 1 (ii )
a1 a2 a 2 c2
a1 b1 c1
From (i ) and (ii ), we have
a2 b2 c2
141
Example 4.1: Find the solution of the following quadratic equations :
( i ) 3x2 – 8x - 28 = 0, (ii) 10x2 - 31x +15 = 0, (iii) x2 – 10x +23 = 0, (iv) x2 – 6x +34 =0
or 3x ( x + 2 ) – 14 ( x + 2 ) =0 or (2x – 5 ) ( 5x – 3 )=0
or (x + 2 ) ( 3x – 14 ) =0 Either 2x – 5 =0 or 5x – 3 = 0
14 5 3
Therefore, we have x = -2 and x x= x and x = x
3 2 5
b b 2 4ac
x
2a
(10) (10) 2 4 1 23
x
2 1
10 8
x 5 2, x 5 2 , x 5 2
2
( iv ) x2 – 6x + 34 = 0,
Here a = 1 , b = -6 , c = 34
142
( v ) 5x2 – 7x – 12 = 0 (vI) 3x2 – 5x – 8 =0
or 5x ( x + 1 ) -12 ( x +1) =0 or 3x ( x +1 ) - 8 ( x + 1 ) =0
or (x +1) ( 5x – 12) =0 or ( x + 1 ) ( 3x - 8 ) =0
8
Therefore, we have Therefore, we have x = -1 and x = 3
12
x= and x = -1.
5
Example 4 2:
( i ) The two roots of a quadratic equation are 2 and 3, form the equation.
( ii ) The two roots of a quadratic equation are 5 and 6, form the quadratic equation.
( iii ) For a quadratic equation one of the root is 3 + 2 i, form the quadratic equation.
( iv ) Sum and product of two roots of a quadratic equation are 7 and 10, respectively, form the quaratic
equation.
( v ) Sum and product of two roots of a quadratic equation are 6 and 13,respectively. Form the quadratic
equation. Also, find the roots.
Solution: (i) Let the roots be α and β Given α= 2 ( say) and β= 3. Then α +β= 2 +3 =5 and αβ = 2 x 3 = 6.
x2 – (α +β)x + αβ =0
or x2 – 5x + 6 = 0
( ii ) Let the roots be α= 5 and β= 6 . Then α +β =5 +6 = 11 and αβ = 5 x 6 = 30. The required quadratic
equation is
x2 - (α +β)x + αβ = 0
or x2 – 11x + 30 = 0
( ii ) Let α= 3 + 2 i be one of the root. As it is a complex root, another root is its conjugate and it is
143
Hence the equation
x2 - (α +β) x + αβ = 0
or x2 – 6x + 13 = 0
x2 - (α +β)x + αβ = 0
or x2 - 7x +10 =0
( v ) Let α = a + ib be one of the root , it is complex. Then another root is β = a – ib, as it is the complex
Therefore, b = 2. Now, the roots are (a + ib) = (3 + 2i ) and ( a – ib)=( 3 – 2i). The quadratic equation is
x2 - (α +β)x + αβ =0
or x2 – 6 x + 9 + 4 = 0
or x2 - 6x +13 =0
Example 4.3: Find the solution of the following quadratic equations by factorization:
144
Therefore, x =2 and x = 1.5 Therefore, x =-2 and x= -3
1
Therefore, x = 2 , and x = 14 Therefore , x = (3 8 ),
2
1
and x = ( 3 8 )
2
or x( x – 10 ) – 2 (x – 10 ) =0 or (2x – 8 )2 - (i 7)2 = 0
Example 4.4: For quadratic equations the sum and products of roots are given . Form the quadratic
equation. Also, find the values of the roots.
Solution:
Let α and β be the two roots of the quadratic equation ax2 + bx +c =0 . If the values of (α +β) and αβ are
given, then the quadratic equation is written as
x2 - (α +β) x + αβ =0
x2 - (α +β) x + αβ =0 x2 - (α +β) x + αβ =0
145
or x2 – 16x + 64 = 0 or x2 – 5x + 6 =0
or ( x – 8 )2 =0 or x2 – 3x – 2x +6=0
or x=8,8 x( x – 3) – 2 ( x – 3 ) =0
x2 - (α +β) x + αβ =0 x2 - (α +β) x + αβ =0
or x2 – 10x + 25 = 0 or x2 – 9x + 18=0
or x2 – 5x – 5x + 25=0 or x2 – 3x – 6x + 18=0
or ( x – 5 )2 = 0 or x (x – 3) – 6 (x – 3) =0
Hence, x =3 , 6
x2 - (α +β) x + αβ =0 x2 - (α +β) x + αβ =0
or x2 – 2x +1 =0 or x2 – 6x + 5 =0
or ( x – 1 )2 =0 or x2 – x – 5x +5 =0
or (x – 1) ( x – 5) =0
Hence, x =1 , 5
Example 4.5: Using the coefficients of the given quadratic equations find the roots of the equations and
hence find the sum and product of roots for each quadratic equation.
Solution:
Let α and β be the two roots of the quadratic equation ax2 + bx +c =0 , where
146
b c
,
a a
If the values of (α +β) and αβ are given, then the quadratic equation is written as
x2 - (α +β) x + αβ =0
(3) 4
3 , 4
1 1 . The roots of the equation is
8 12
8 , 12
1 1
or x ( x + 2) + (x +2) =0
(7) 7 8
,
3 3 3 .
b b 2 4ac (_ 7) (7) 2 4 3 8 7 47 7 i 47
x
2a 23 6 6
(5) 5 4
2.5 , 2
2 2 2
b b 2 4ac (5) (5) 2 4 2 4 5 7 5 i 7
x
2a 2 2 4 4
147
( v ) Given a = 1, b = -10 and c= 15
(10) 15
10 , 15
1 1
x2 – 10x +15 =0
or x2 – 10x +25 – 10 =0
( x – 5 )2 - ( 10 ) 0
2
or
or (x – 5 + 10 ) ( x – 5 - 10 ) =0
or x=5- 10 , x = 5 + 10
( vi ) Given a =4 , b =- 12 and c = - 27
4x2 – 12x – 27 =0
or ( 2x – 3 )2 – 62 = 0
or ( 2x – 3 +6 )( 2x -3 – 6 )=0
or ( 2x + 3 ) ( 2x – 9 )=0
Example 4.6: Comment on the nature of the roots of the following quadratic equations and find the roots
of each of the equation:
Solution:
As b2 – 4ac =0, the roots are same and these are As b2 – 4ac =0, the roots are same and these are
b (12) 3 b (8)
4
x=
2 a 2 x 4 2 x = 2a 2 1
148
Therefore, x = 1.5 , 1.5 Therefore, x = 4 , 4
( iii ) x2 + 10x +25 = 0;here a = 1,b=10 and c=25 ( iv ) x2 + 2x +1=0; here a =1 , b= 2 and c=1
AS b2 – 4ac = 0, the roots are same and these are As b2 – 4ac =0, the roots are and these are
b 10 b 2
5 1
x = 2a 2 1 x= 2a 2 1
b 2 4ac 24
As >0, the roots are real but
b 2 4ac (7) 2 4 1 4 33 0
( v i ) x2 – 7x + 4 = 0; here a =1 , b= - 7 and c = 4;
( vii ) x2 – 5x +14=0; here a = 1, b= -5 and c =14, b2 – 4ac = (-5)2 – 4 x 1x 14 = - 31 <0. Therefore, the
roots are complex numbers. These are
149
( viii ) x2 + 8x + 16 =0 ; here a =1 , b= 8 and c= 16. b2 – 4 ac = 82 – 4 x 1 x 16 = 0. Therefore, the roots
or x ( x +4 ) + 4 ( x +4) =0
or (x+4) (x+4) =0
Therefore, x = - 4 , -4.
ax3 + bx2 + cx +d =0
The problem is to get the solution of the above form of cubic equation. In the above form of cubic
equation any of the coefficients b or c or the constant d may be zero. For example,
( i ) 3x3 - 4x2 + 1 =0
( ii ) 3x3 – 4x + 1 =0
( iv ) 2x3 – 3x +1 =0
For the above type of equations , we need the solution, i.e. we need the values of x for each equation. The
general strategy for solving a cubic equation is to reduce the power of x in the equation so that the left-
hand side of the equation transforms to components , where one component may be a quadratic form and
another component is in linear form . As left-hand side of cubic equation is transformed into two factors,
either of the factor will be equated to zero and hence solving equations obtained from two components
we get the values of x. For example, let us consider the following cubic equation
x 3 + x2 – x - 1 = 0
This is a cubic equation and the left-hand side of it can be expressed as the product of two components,
where the equation transforms to
( x – 1 ) ( x2 +2x +1 ) =0
In the above form either of the component is equal to zero. That is, we can write
x – 1 =0 or x2 +2x +1=0
150
Here the cubic term transforms into product of linear and quadratic terms. At this stage we can solve both
the linear and quadratic equations by te method which has already been discussed.
Factor Theorem : The solution of the cubic equation which has already been shown above is done by
factor theorem . The theorem states that if x = 1 is a solution of the cubic equation, then (x – 1) is a factor
( component) of the left-hand side of the equation.
n. As an another example, let us consider the cubic equation 3x3 – 4x +1 =0. This equation is satisfied if x
= 1. Then (x – 1 ) is a factor of the left-hand side of the equation. The other factor will be a quadratic
form of the type ax2 +bx + c. Then, we can write
( x – 1) ( ax2 + bx + c ) =0
So, we need to find the values of a and b. The quantity a and b or the quadratic form of the equation is
found out by dividing the original cubic equation by the factor ( x – 1) . Thus , in ou r example, we can
have the values of a and b as follows:
3x 3 4 x 1
ax2 + bx +c = = 3x2 +3x - 1
x 1
x – 1 = 0 or 3x2 +3x -1 =0
From the first linear equation ,we have x = 1 and from the quadratic equation, we have
b b 2 4ac 3 3 4 3 (1) 3 21
2
x=
2a 23 6
x3 – 5 x2 – 2x +24 =0
This equation is satisfied for x = 3 and hence (x – 3 ) is a factor of the left-hand side of the equation.
Hence, we can write (x -3 ) ( ax2 +bx + c ) = x3 – 5x2 – 2x + 24 =0. Now, we need the values of a, b, and
c.
Here the coefficients and constant of the original equation are 1 , -5, -2, and 24. This can be written as
1 -5 -2 24
________________________| x = 3
Here x = 3 is a root of the equation. This value of x is written to the right of the vertical line . Another
vertical line remains empty. Then the following steps are needed to fill up the empty vertical positions:
( i ) In the first step coefficient of x3 is written in the first place of the empty vertical line.
151
( ii ) in the next step the value of x is written in the second place of the empty vertical line. Thus, we
have
1 -5 -2 24
3 -6 -24
________________________| x =3
1 -2 -8 0
Now, the numbers in the second column are added, which gives
( iii ) This number – 2 is to be multiplied by x = 3 . It gives the value – 6. This value is placed in the third
( iv ) The numbers in the third column are added. It gives -8 . It is written in the against the third
column.
( v ) The value -8 is to be multiplied by 3 and place the result in the last empty place of the vertical line.
Add the values of fourth column. It gives zero. So the coefficient b = -2 and c = -8.
Thus, we have
x2 – 2x – 8
( x – 3 ) ( x2 – 2x – 8) =0
Here the second component of the equation is a quadratic form. Here either
x – 3 =0 or x2 – 2x – 8 =0
x2 – 4x +2x – 8 =0
or x ( x – 4 ) + 2 (x – 4 ) =0
or (x – 4 ) ( x + 2 ) =0
or x = 4 , x = - 2.
Therefore, x= -2, 3, 4.
152
4.2.1 Nature of the Roots of Cubic Equation
The cubic equation may have three real roots, just like a quadratic equation has two real roots. In some
cases a quadratic equation may not have any real solution. But in case of cubic equation there exists
always at least a real root. The cubic equation has three roots and all three or two of them may be same.
Example 4.7: Find the solution of the following equations :
( i ) x3 +3x2 – 6x – 8 =0, ( ii ) x3 + 2x2 – 21x +18 =0, ( iii ) x3 +4x2 +7x +6 =0, ( iv ) 2x3 + 9x2 + 3x – 4
=0
Solution :
( i ) x3 +3x2 – 6x – 8 =0 .
It is seen that the equation is satisfied for value x = 2. So, ( x – 2) is a factor of the left –hand side of the
equation. Now, the coefficients of the equation can be written as follows:
1 3 -6 -8
2 10 8
__________________________| x = 2
Therefore, the quadratic equation for second factor of the left-hand side of the equation is
x2 +5x +4 =0
or x2 +4x +x +4 =0
or x ( x + 4 ) +1 (x +4 )=0
or (x +1) ( x +4) =0
Finally, the equation can be written as ( x – 2 ) ( x +1 ) ( x +4 ) =0. Hence ,the solutions are x = - 1, - 4,
2.
( ii ) x3 + 2x2 – 21x +18 =0. It is seen that for x = 1 the equation is satisfied. Hence, ( x – 1 ) is a factor of
the left-hand side of the equation. The equation can be written as
( x – 1 ) ( ax2 + bx + c ) =0
To find the values of a , b, and c , let us write the coefficients from the equation as follows:
153
1 2 - 21 18
1 3 - 18
__________________________|x = 1
1 3 -18 0
x2 + 3x - 18 =0
or x2 + 6x – 3x – 18 =0
or x ( x + 6 ) – 3( x + 6 ) =0
or ( x + 6 ) ( x – 3 ) =0
( x – 1 ) ( x – 3 ) ( x +6 )=0
Therefore, x=1,3,-6.
( x +2 ) ( ax2 + bx +c ) =0
To find the values of a, b, and c the coefficients from the equation can be written as follows:
1 4 7 6
-2 -4 -6
____________________|x = - 2
1 2 3 0
x2 +2x + 3 =0
b b 2 4ac 2 2 2 4 1 3 2 i 8
x= 1 i 2 =
2a 2 1 2
154
Finally, the equation stands
( x +2 ) ( x2 +2x +3 ) = 0
The two solutions are imaginary and these are x = -2, (-1 + i 2 ),( - 1 - i 2 ).
( iv ) 2x3 + 9x2 + 3x – 4 =0. It is seen that for x = - 4 the equation is satisfied. So, (x + 4 ) is a factor of the
left-hand side of the equation. The other factor is a quadratic equation of the type ax2 +bx +c =0. The
given equation is then stands as
( x +4 ) ( ax2 + bx + c ) = 0
2 9 3 -4
-8 -4 4
___________________|x = -4
2 1 -1 0
2x2 + x – 1= 0
or 2x2 + 2x – x – 1 =0
or 2x ( x + 1 ) - 1 ( x + 1=0
or ( x +1 ) ( 2x -1 )
( x +4 ) ( (x+1) ( 2x – 1) = 0
( i ) x3 + 4x2 +x – 6 =0, ( ii ) x3 +4x2 – 6x – 20=0, ( iii) 2x3 – 3x2 – 4x +20 =0, ( iv ) x3 – 3x2 – x + 6 =0
( v ) x3 – 3x +2=0.
Solution:
155
( i ) x3 + 4x2 +x – 6 =0. It is seen that for x = 1 the equation is satisfied. Hence, (x - 1 ) is a factor of the
left-hand side of the equation. So, we have
( x – 1 ) ( ax2 + bx +c ) = 0
We need the values of a, b, and c. For this, let us write the coefficients of the left-hand side of the
equation
1 4 1 -6
1 5 6
_____________________| x =1
1 5 6 0
x2 +5x +6 =0
or x2 + 3x +2x +6 =0
or x ( x +3 )+ 2 ( x +3)=0
or ( x +2 ) ( x + 3 ) =0
( x – 1 ) ( x + 2 ) ( x + 3 ) =0
( ii ) x3 +4x2 – 6x – 20=0. It is seen that for x = - 2 the equation is satisfied. Hence ( x + 2 ) is a factor of
the left –hand side of the equation. We have
( x +2 ) ( ax2 +bx + c ) =0
To find the values of a , b, and c let us write the coefficients of the left-hand side of the equation as
follows:
1 4 -6 - 20
-2 -4 20
______________________|x = -2
1 2 - 10 0
x2 +2x – 10 =0
156
The solution of this equation is
b b 4ac 2 2 4 1 (10) 2 4 40
2 2
x= 1 11
2a 2 1 2 1
( x +2 ) ( x2 +2x - 10 ) =0
( iii ) 2x3 – 3x2 – 4x +20 =0. It is seen that for x = - 2 the equation is satisfied. Hence ( x + 2 ) is a factor
of the left-hand side of the equation. The equation can be written as
( x +2 ) ( ax2 +bx +c ) =0
To find the values of a, b, and c, we can write the coefficients of left-hand side of the equation as follows:
2 -3 -4 20
-4 14 - 20
_______________________|x = -2
2 -7 10 0
( x + 2 ) ( 2x2 – 7x + 10 ) =0
x=
2a 2 2 4
7 31 7 31
Hence, the roots are x = -2, ,
4 4
( iv ) x3 – 3x2 – x + 6 =0. It is seen that for x = 2 the equation is satisfied. Hence, ( x – 2 ) is a factor of
the left-hand side of the equation. We can write
( x – 2 ) ( ax2+ bx +c ) =0
To find the values of a , b, and c, let us write the coefficients of the left-hand side of the equation as
follows:
1 -3 -1 6
157
2 -2 -6
________________________|x = 2
1 -1 -3 0
( x – 2 ) ( x2 – x – 3 ) =0
x=
2a 2 1 2
1 13
Finally, the solutions are x= 2,
2
( v ) x3 – 3x +2=0. It is seen that for x = 1 the equation is satisfied. So, ( x – 1 ) is a factor of the left-hand
( x – 1 ) ( ax2 +bx +c ) =0
To get the values of a , b and c , we can write the coefficient of the left-hand side of the equation as
follows:
1 0 -3 +2
1 1 -2
____________________|x = 1
1 1 -2 0
( x – 1 ) ( x2 + x - 2 ) =0
x2 +x – 2 =0
or x2 + 2x – x - 2 =0
158
or x ( x + 2) - 1( x +2 )=0
or ( x – 1 ) ( x + 2 ) =0
or x =1 and x =- 2
A set of equations , also known as a system of equations , is a finite set of equations for which common
solutions are sought. Such set of equations is known as simultaneous equations. There are different types
of system of equations. These are
In this section, we shall discuss the common solution of the system of linear equations. Simultaneous
linear equations are used mostly in Econometrics to formulate the models for different economic
variables. For example, let us consider the Supply and Demand function of a commodity marketed by a
company or by the general producers. Let the functions be
a1 1 s + b1 1 p = d ------------------------------------------------------------------------------------( i )
a2 1 s + b21 p = d -------------------------------------------------------------------------------------( ii )
where s = quantity of the commodity supplied , p= price of the commodity and d = quantity of the
commodity demanded. The producer will be interested to maintain the market equilibrium . The market
equilibrium will exist if both the equations are satisfied by one set of values of a1 1 , a2 1, b1 1, and b2 1 .
Here a1 1 , a2 1, b1 1, and b2 1 are the coefficients in the equations. The equation( i ) and ( ii ) show the
quantitative relationship of three variables supply ( s ), demand ( d ) and price ( p ).
The equations ( i ) and ( ii ) are algebraic equations. These types of quantitative expression are used to
express many relationships in science, economics , business, social science , biological science , medical
science, life science, etc. For example, let us consider that a garments industry has two wings . Two wings
produce two items , namely A and B. Let x units of A and y units of B are produced. Each unit of A
needs 4 hours to be produced through wing-1 . The time of production of B in wing -1 is 3 hours. Wing –
1 can produce 60 units . Wing -2 can produce 42 units. The production time for each unit of A and B are
159
3 and 2 hours respectively. What is the value of x and y ? Here, we can formulate two equations for two
wings. These are
3x + 2y = 42 ----------------------------------------------------------------------------------( iv )
Now, to get the values of x and y ,we need the solution of these two equations. For the solution, let us
multiply both sides of equation ( iii ) by 3 and both sides of equation (iv) by 4 . This gives
Subtracting ( vi ) from ( v ) , we get y = 12. Replacing the value of y= 12 in equation ( iii ), we get x = 6.
Here there are two unknown variables x and y and there are two equations .
Simultaneous equations are applicable when there are at least two variables. For example, two variables x
and y may be related by the mathematical equations of the following type:
y = 2x +10, or y – 2x = 10 -------------------------------------------------------------------------(vii )
y = x + 22 , or y – x = 22 ----------------------------------------------------------------------- ( viii)
These two equations may be generated in two occasions or in two places for the same occasion. We need
the values of x and y [ solution of the equations].
( i ) Solve the simultaneous linear equations by substitution. For example, let us substitute the value of y
from equation ( viii )[ y = x + 22 ] in equation ( vii ). It gives x +22 - 2x = 10 or - x = 10 – 22 or x = 12.
Then put the value of x in equation ( viii ). It gives y – 12 = 22 or y = 22 + 12 = 34.
( ii ) Solve simultaneous linear equations by elimination. The method is applied to solve the equations
In this section, we shall apply methods ( i ) , ( ii ) and ( iii ). Methods ( iv) has been discussed in the
chapter containing Determinant and matrix. As an example, let us consider two simultaneous linear
equations as follows:
d = 20 –2 p
s = - 10 + 2p
160
where d = quantity of commodity demanded, s = quantity of commodity supplied, and p = price of the
commodity. The equilibrium condition will exist if the equations for d and s are equal, i.e.
d=s
or 20 – 2p =2 p – 10
or 20 + 10 = 2p +2 p
or p = 7.5
d= 20 – 2 x 7.5 = 5
We have demand = 5 .In a similar way, we get supply ,s = 2p – 10 = 2 x 7.5 – 10 = 5. As d= s=5, the
market equilibrium exists.
5x – y = 11 x +y = 0 13x + 2y = 69 3x – 2y = 19 5x +2y = 7 4x +
3y =11
Solution :
( i ) 7x + y = 25 ……( a ) ( ii ) 8x + 9y = 3 ……( a )
5x – y = 11 ……..( b ) x+ y =0 ……( b )
Adding ( a ) and )(b ), we get Multiplying (b) by 8 and subtracting the result
x =3 y=3
7 x 3 + y = 25 x+3=0
y = 25 – 21 = 4 x = - 3.
Therefore, x = 3 , y= 4 Therefore, x = - 3 , y= 3.
161
Multiplying (a ) by 13 and (b) by 2, we get Multiplying (a) by 2 and subtracting it
from
y=2 7 x 1 – y = 15
13 x = 69 – 4= 65
x= 5
Therefore, x =5 and y = 2.
( v ) 2x – y = 1 ……..(a) ( vi ) 3x – y = 5 ……………(a)
Multiplying ( a ) by 2 and adding the result with (b), Multiplying ( a ) by 3 and adding the
result
x=1 x=2
Putting the value of x in (a), we get Putting the value of x in (a), we get
2x 1 – y = 1 3x2–y=5
y=1 y =1
( i ) 2x – 3y + z = 0 , ( ii ) x + 2y +3 z =6 ,( iii ) x + 2y – z = 9 , ( iv ) x + y – z = - 3, ( v ) 2x + 3y -
4z = 3
5x – 3y – z = 1 2x + 4y +z = 7 x +y +z=9 2x +3y +z = 2 3x – 2y + z =5
162
3x +2y + z = 6 3x + 2y +9z = 14 3x + y – z = - 1 8y + 3z = 1 x + y – 3z =0
Solution:
5x – 3y – z = 1 ……( b ) 2x + 4y + z = 7 ……. .( b )
3x +2y – z = 6 … (c ) 3x + 2y + 9z = 14 …..( c )
Subtracting (e) from (d), we get Subtracting (e) from (d), we get
Multiplying (b) by 3 and (c) by 5, we get Multiplying (b) by 3 and (c) by 2, we get
we get 2x – 3 x1 +1 = 0, or –x = -1 or x = 1. Therefore,
x = y =z =1.
x + y +z = 9 …..( b ) 2x + 3y + z = 2 ……( b )
3x +y – z = -1 ……( c ) 8y + 3z = 1 …..( c )
163
y – 2z = 0 ….. ( d ) 2x + 2y – 2z = -6…..( d )
x + 2 x 7 – 3.5 = 9 , or x = - 1.5
( v ) 2x + 3y – 4z = 3 ……( a )
3x – 2y + z = 5 ……( b )
x + y – 3z = 0 …..( c )
6x – 4y + 2z = 10 ……( e ) 3x + 3y – 9z = 0 …….( g )
65y – 70z = -5 …. ( j )
164
Putting the values of y and z in (a ) , we get 2x + 3 (1) – 4 ( 1 ) = 3, or 2x = 3 -3 +4, or x = 2,
Example 4.11: The supply (s) and demand (d) are function of price (p). One set of such function is
d = 30 + 0.5 p
s = -20 + p
30 + 0.5p = -20 +p
or 0.5p = 50
50
p= = 100
0 .5
Solution:
( i ) 3x +y = 13 …..( a ) ( ii ) x + 2y = 5 …….( a )
5x – 2y = 7 …..( b ) 2x + y = 4 ……..( b )
5x – 2 ( 13 – 3x ) = 7 2( 5 – 2y ) + y = 4
or 11x = 33 or -3y = -6
or x=3 y=2
5 x 3 – 2y = 7 x+2x2=5
or -2y = 7- 15 or x = 5 – 4
or y=4 or x =1
165
Therefore , x =3 , y = 4. Therefore, x -1 , y = 2
x – 2y = 4 ……( b ) x + 8y = 30 ……….( b )
7 + 5y – 2y = 4 6 – 2y + 8y = 30
or 3y = 4 – 7 or 6y = 30 – 6
or 3y=-3 or 6y = 24
or y = -1 or y=4
x – 5 (- 1) = 7 x+2(4)=6
or x = 7 – 5 or x=6–8
or x = 2 or x=-2
x = 2, y = -1 x = -2, y = 4
( v ) x - 2y = 5 ……..( a ) ( vi ) 3x + y = 5 ……..( a )
-4 ( 5 + 2y ) + 5y = -26 10x -3 ( 5 – 3x ) = 4
or 5y – 8y = 20 – 26 or 10x + 9x = 4 + 15
or - 3y = -6 or 19x = 19
or y=2 or x=1
x – 2( 2 ) = 5 3(1)+y=5
or x=5+4 or y = 5 -3
or x=9 or y =2
166
x = 9, y= 2 x=1,y=2
3x – 2y = 1 3x + y = 5 3x + 2y = 5 2x – 3y = 5 2x + 4y = 10 4x + 3y = 7
a1x + b1 y = c1
a2x + b2 y = c2
a1 b1 x c1
In matrix notation we can write the equations A X = B , where A = , X ,B
a 2 a 2 y c 2
1 c1 b1 b2 c1 b1c 2 a1c 2 a 2 c1 1 a1 c1
Then , x = ,y = =
A c2 b2 a1b2 a 2 b1 a1b2 a 2 b1 A a2 c2
Therefore, x =1 , y=1
167
b2 c1 b1c2 (3)(3 ) (1)5 4 a c a2 c1 1 5 2 3 1
x 4, y 1 2 1
a1b2 a2b1 1 (3) 2(1) 1 a1b2 a2b1 1 (3) 2(1) 1
Therefore, x =4 , y=1.
Therefore, x =1 , y = 2
Therefore, x =1, y =1
Example 4.14: Solve the following simultaneous equations of three variables using Cramer‟s rule :
A X = B, where
a1 b1 c1 x d1
a2 b2 c2 , X y , B d 2
a b c z d 3
A= 3 3 3 .
b2 c 2 a2 c2 a 2 b2
Here |A | = a1 b1 c1 a1 (b2 c3 b3 c 2 ) b1 (a 2 c3 a 3 c 2 ) c1 (a 2 b3 a3b2 )
b3 c3 a 3 c3 a3 b3
d1 b1 c1 a1 d1 c1 a1 b1 d1
1 1 1
By Cramer‟s rule x = d 2 b2 c 2 , y a2 d 2 c2 , z a 2 b2 d 2
| A| | A| | A|
d 3 b3 c3 a 3 d 3 c3 a3 b3 d 3
Let A1 be the matrix in the numerator in finding the value of x. Similarly, the matrices in the numerator in
finding the values of y and z are , say A2 and A3 . Here
168
A1 = d1 ( b2c3 – b3 c2 ) - b1 (d2c3 – d3c2) + c1 ( d2b3 – d3b2 ). Similarly,
( i ) 2x – y + z = 6, x + 2y +3z = 3, 3x +y – z = 4
2 1 1 x 6
, X y , B 3
A = 1 2 3 , |A|= 2(-2 – 3 ) +1(-1 – 9 ) +1 ( 1 – 6 ) = -25
3 1 1 z 4
6 1 1
A1= 3 2 3 , | A1 | 6(2 3) 1(3 12) 1(3 8) 50
4 1 1
2 6 1
A2 = 1 3 3 , | A2 | 2(3 12) 6(1 9) 1(4 9) 25
3 4 1
2 1 6
A3 = 1 2 3 , | A3 | 2(8 3) 1(4 9) 6(1 6) 25
3 1 4
| A1 | 50 |A | 25 | A | 25
x 2, y 2 1, z 3 1
| A | 25 | A | 25 | A | 25
Therefore, x = 2 ,y= - 1 ,z = 1
( ii ) 5x + 3y =65, 2y – 5z = -25 , 3x + 4z = 57
5 3 0 x 65
0 2 5, X y , B 25
A= , |A| = 5( 8 – 0) -3 ( 0 + 15) + 0 ( 0 – 6 ) = -5
3 0 4 z 57
65 3 0
2 5 , |A1|= 65 ( 8 – 0) – 3 ( -100 + 285) + 0 ( 0 – 114 ) = -35
A1 = 25
57 0 4
169
5 65 0
A2 = 0 25 5 , |A2|= 5 ( -100 + 285 ) – 65 ( 0 +15 ) +0 ( 0 +75 ) = -50
3 57 4
5 3 65
A3 = 0 2 25 , |A3| = 5 ( 114 – 0 ) – 3 ( 0 + 75 ) +65 ( 0 – 6 ) = -45
3 0 57
| A1 | 35 | A | 50 | A | 45
x 7, y 2 10 , z 3 9
| A| 5 | A| 5 | A| 5
1 1 1 x 6
1, |A| = 1 ( 2 + 3 ) – 1 ( 4 – 9 ) +1 ( -12 -18 ) = - 20 , X = y , B = 11
A = 4 2
9 3 1 z 6
6 1 1
A1 = 11 2 1 , |A1| = 6 ( 2 + 3 ) – 1 ( 11 – 6 ) +1 ( -33 – 12 ) = - 20
6 3 1
1 6 1
A2 = 4 11 1 , |A2| = 1 ( 11 – 6 ) – 6 ( 4 – 9 ) + 1 ( 24 – 99 ) = - 40
9 6 1
1 1 6
4 2 11
9 3 6
A3 = , |A3| = 1 (12 + 33 ) – 1 ( 24 – 99 ) +6 ( -12 – 18 ) = -60
| A1 | 20 | A | 40 | A | 60
x 1, y 2 2,z 3 3
| A | 20 | A | 20 | A | 20
170
2 1 1 x 5
y , B 3
A = 3 2 2 , |A| = 2 ( 6 +6 ) – 1 ( -9 – 2 ) – 1 ( -9 + 2 ) = 42, X =
1 3 3 z 2
5 1 1
2 , |A1| = 5 ( 6 + 6 ) – 1 ( 9 +4 ) – 1 ( 9 – 4 )=42
A1= 3 2
2 3 3
2 5 1
3 3 2, |A2| = 2 ( 9 + 4 ) – 5 ( -9 – 2 ) – 1 ( -6 + 3 ) = 84
A2 =
1 2 3
2 1 5
3 2 3,
A3 = | A3| = 2 ( 4 – 9 ) – 1 ( -6 + 3 ) +5 ( -9 + 2 ) = -42
1 3 2
| A1 | 42 | A | 84 | A | 42
x 1, y 2 2,z 3 1
| A | 42 | A | 42 | A| 42
Therefore, x = 1, y = 2 , z = -1
2 3 1
1 1 1 ,
A= |A| = 2 ( 2 – 2 ) – 3 ( -2 – 3 ) +1 ( 2 + 3 ) = 20
3 2 2
17 3 1
3 1 1 ,
A1 = |A2| = 17 ( 2 – 2 ) – 3 ( - 6 – 4 ) +1 ( 6 +4) = 40
4 2 2
2 17 1
A2 = 1 3 1 , | A2| = 2 ( -6 – 4 ) – 17 ( -2 – 3 ) +1 ( 4 – 9 ) = 60
3 4 2
171
2 3 17
1 1 3 ,
A3 = |A3| = 2 ( - 4 – 6) – 3 ( 4 – 9 ) + 17 ( 2 + 3 ) = 80
3 2 4
| A1 | 40 | A | 60 | A | 80
x 2, y 2 3,z 3 4
| A | 20 | A | 20 | A | 20
Therefore , x = 2, y = 3 , z =4
Solution:
( i ) 3x – 2y = 1, 2x + 3y =5,
The system of equations can be presented in matrix form as A X =B, where the solution is X = A-1B. Here
( ii ) 4x – 3y = -2 , 3x +4y = 11.
The system of equations can be represented in matrix notation as AX = B, where X = A-1B. Here
4 3
4 3 x 2 1 1 4 3 25 25
A , X , B , A 4 4 (3)3 25, A1 AdjA
3 4 y 11 A 25 3 4 3 4
25 25
4 3 2 4 3 11
1 25 25 2 25
25 1
X A B x 1, y 2
3 4 11 3 2 4 11 2
25 25 25 25
The system of equations can be represented in matrix form as A X = B, where X = A-1B. Here
172
2 4
5 4 x 13 adj A 1 2 3 22
A , X , B , A 5(2) 3(4) 22, A1 22
3 2 y 1 A 22 4 5 3 5
22 22
2 4 2 13 4(1)
1 22 22 13 22 22 1
X A B x 1, y 2.
3 5 1 3 13 (5)( 1) 2
22 22 22 22
Example 4.16: Solve the following simultaneous equations by matrix inversion.
Solution:
The system of equations can be written in matrix form as AX = B and the solution of the equations is
Here
3 15 5
A 1 6 2, A 3(12 10) 15(2 2) 5(5 6) 1 0, Rank( A) 3, Again Rank( A | B) 3.
1 5 2
x 4 2 5 0 2 5 0 4 2
y , 2 , A1 Adj A 0 1 1 , X A1 B 0 1 1 2 1
A
z 1 1 0 3 1 0 3 1 1
X= B=
The system of equations can be presented in matrix notation as A X = B, where X = A-1B. Here
2 4 3 0 x
3 ,
A = 3 2 4 , B = X = y , |A| = 2(20 – 36) – 4 (30 – 20) +3( 27 – 10 )= - 21.
5 9 10 6 z
173
Hence, Rank ( A) = 3, Again, Rank ( A|B ) =3. Therefore, solution will be available.
16 13 10 16 13 10
21 0 3 6
13 10 21 21 0 21 21 21
16 1
1 10 5 1 10
5 1 B 0 3 6 1
1 Adj A 5 1
A B B 10 3
A 21 21 21 21 21 21 21
17 2 8 17 6 2
2 8 17 0 2 3 8 6
21 21 21 42 21 21
Therefore, x =-1,y=-1, z = 2.
2 4 3 4
Cofactor of a11 =( -1 ) 1+1
= 2 x10 – 9 x 4 = 16 ,Cofactor of a12= ( -1 ) 5 10 =-[3 x 10 – 5 x 4]] =
1+2
9 10
-10
4 3
3 2 9 10 =-[4 x 10 – 3x 9]=-
Cofactor of a13= ( -1)1+3 = 3 x 9 – 5 x 2 = 17, Cofactor of a21= ( -1 )2+1
5 9
13
2 3 2 4
Cofactor of a22= (-1) 2+2
=2 x 10 – 5 x 3 = 5, Cofactor of a23= (-1) 5 9 = -[2 x 9 – 5 x4]=2
2+3
5 10
4 3 2 3
2 4 3 4
Cofactor of a31 = (-1) 3+1
= 4 x 4 – 2 x 3 = 10, Cofactor of a32 = (-1) 3+2
= -[2 x 4 – 3 x 3] =1
2 4
3 2 =2x2–3x4=-8.
3+3
Cofactor of a33 = (-1)
These equations can be represented in matrix for as A X = B,and the solution of the equations is X = A-1
B.
Here
x
2 2 1 6 y
A 2 3 1 , B 9 , X
z
4 2 1 10
, |A| = 2 ( -3 – 2 ) – 2 ( 2 – 4 )-1 ( -4 – 12 ) = 10.
174
Rank A = Rank ( A|B)=3. Therefore , the solution of X is available. Now, let s find the Adj A matrix.
The different elements of Adj A are shown below:
3 1
Cofactor of a11 = ( - 1 )1+1 5, Cofactor of a12= (-1)1+2
2 1
2 1
[(1) (2 ) 1 4] 2
4 1
2 3 2 1
0
Cofactor of a13 = (-1)1+3 = -2 x 2 – 3 x 4 =- 16, Cofactor of a21= (-1 )2+1 2 1
4 2
2 1
2, 2 2
Cofactor of a22 = ( - 1) 2+2
4 1 Cofactor of a23 = (-1 )2+3 = 4
4 2
2 1 2 1
5 0
Cofactor of a31= (-1) 3+1
3 1 , Cofactor of a32= (-1) 2 1
3+2
2 2
10
2 3
3+3
Cofactor of a33= (-1)
X=A-1B
5 0 5 6 0.5 0 0.5 6 0.5 6 0 0.5 10 2
1
0 9 0.2 0.2 0 9 0.2 6 0.2 9 0 3
Adj A
B 2 2
A 10
16 4 10 10 1.6 0.4 1 10 1.6 6 0.4 9 1 10 4
Therefore, x = 2 , y = 3, z = 4
( iv ) 2x +3Y – 5z = 0, x –y + 3z = 0 , x +y – z =0.
The system of equations can be written in matrix form as A X = B, where X = A-1B. Here
2 3 5 x 0
1 1 3 , y X , B 0
A= ,|A|=2(1 – 3 )- 3 ( -1 – 3 ) – 5 ( 1 +1)= - 2
1 1 1 z 0
The Rank A =3. Again, Rank ( A |B ) = 3. Therefore, there exists the solution and it is X = A-1B =A-1 0=0.
175
Supplementary Problems
( i ) x2 – 4x +4 = 0 , ( ii ) 3x2 – 8x +4 = 0, ( iii ) x2 – 7x + 12 = 0, ( iv ) x2 – 2x – 15 =0 , ( v ) x2 – 8x
+16 =0
2
( vi ) x2 – x + 6 = 0. Ans: ( i ) x = 2,2; ( ii ) x =2 , ; ( iii ) x= 3,4; (i v ) x = - 3 , 5; ( v ) x = 4 ,4 ;
3
( vi ) x = - 2, 3.
2 4 1
( i ) 27x2 + 6x – 8 = 0; Ans. , . ( ii ) 3x2 + 5x – 8=0; Ans. [5 121 ] .
3 9 6
1
,2 1
( iii ) 2x2 – 3x + 2=0, Ans. 2 ; ( iv ) 5x2 – 7x +8 = 0, Ans. [7 i 111 ]
10
1
( v ) x2 – 3 x +4 = 0, Ans. [3 i 7 ] ; ( vi ) x2 – 6x +34 = 0, Ans. 3 + i 5, 3 – i 5.
2
3 . Given the sum and product of two roots of some quadratic equations as follows . Find the quadratic
Equations.
( iv ) 3x – 5y = - 11, x – y = -1; ( v ) 2x + 3y = 3, x + y = 2 ; ( vi ) 4x – 3y = 9, 2x + y = 7;
176
( iii )2x +3y +z = 6, x – y +2z = 2 , x +4y – 3z = 2 ( iv ) 3x + 2y -3z = 4, x – 3y +4z = -1,2x +3y – 4z =4
( i ) 2x – y + z = 6, x +2 y +3z = 3, 3x + y – z = 4; ( ii ) x + y –z = -3, 2x + 3y + z = 2, 8y + 3z = 1
177
Chapter V
Method of Counting
5.1 Introduction
Counting method is a technique to decide the groups of independent possibilities of two or more actions
when occurs jointly. The method is very important in calculating probability. The mathematical definition
of probability is based on counting of favourable cases, exhaustive and equally likely cases. The counting
of cases is easier in small scale random experiment where total outcomes can be shown in a sample space.
For example, a manufacturer produces two types of items, viz. I1 and I2 and supply the items to the market
through wholesaler and /or retail seller [ W/R ]. Then, the items can be supplied in the following possible
ways:
I1 W, I2 W, I1 R, I2 R
Two items can be supplied by two suppliers in 2 x 2 = 4 ways. If two items are supplied by 3 different
ways, then total number of ways to supply the items in the market will be 2 x 3 = 6 ways. This is an
example of simple experiment of supply of the industrial goods in the markets. Here we can count
possibilities simply by enumerating them exhaustively listing the out. But for random experiment having
many alternative ways to occur , it is not easier to list all the possible results of occurrence of the
experiment. In such a case we need to count the exhaustive cases by any short cut method. Method of
counting deals with such short cut way of counting the possibilities of outcomes of random experiment.
Rule of Sum: Let us consider that an action is performed in n1 ways and another action is performed in n2
ways, but both the actions are not performed simultaneously, then there are n1 + n2 ways to perform any
of the actions. For example, let us consider that in a community library there are two books of a subject.
Any of the books you can have for use. But there are two options to have any of these two books. The
options are either you can buy or you can borrow any one of the books. Therefore, you have 2 + 2 = 4
ways to have a book from the library of the subject.
Rule of Product: Let us consider that an action can be performed in a ways and another action can be
performed in b ways depending on the result of each of the a ways of the first action. Then the two
actions simultaneously can be performed in a x b ways. For example, let us consider that an industry
produces electric bulbs using two plants A and B. The product of any of the plant may be good ( G) or
defective ( D ). One day bulbs of both the plants are inspected one by one and noted the status of the
bulbs. The noted results are of the form as shown below:
G G, G D, D G, D D.
The results of the investigation are 4 when there are two plants. These results are observed from the count
of two types of possibilities of two events. The results are 2 x 2 = 22 = 4. In a similar way, if the good and
defective bulbs are inspected simultaneously from n plants, then the status of the bulbs will be noted in 2n
ways.
178
Let us consider another experiment of observing the consumers‟ satisfaction when they buy 3 items from
three shops or from the same shop. Each of the consumers may be satisfied ( S ) or dissatisfied ( D ) with
any of the items. The status of satisfaction can be shown as follows:
S S S
D D D
Item – 1: , Item – 2 : , Item – 3 :
Let us consider another example of selecting a shop for marketing by a customer. Consider that a man
will buy furniture. Furniture may be of wooden, or of steel or of partex. Wooden furniture are sold in 10
shops, steel furniture are sold in 3 shops and partex furniture are sold in 2 shops. The man can buy the
furniture from any of the shops. He has the options to choose the shop from ( 10 x 3 x2 ) =60 ways.
Let us consider an example of supply-chain management of cement produced in an industry and which is
marketed through different sellers for the consumers. The product of the industry is supplied through two
distributors. Each distributor supplies the product through 3 wholesalers. Each wholesaler supplies the
product through 4 retail sellers from where the consumers can buy the item. The diagram of the
distribution of cement is shown below:
179
Re tail seller 1
Wholesaler 1 Re tail seller 2
Re tail seller 3
Re tail seller 4
Re tail seller 1
Re tail seller 2
Distributor 1 Wholesaler 2
Re tail seller 3
Re tail seller 4
Re tail seller 1
Re tail seller 2
Wholesaler 3
Re tail seller 3
Re tail seller 4
Re tail seller 1
Re tail seller 2
Wholesaler 1
Re tail seller 3
Re tail seller 4
Re tail seller 1
Distributor 2 Wholesaler 2 Re tail seller 2
Re tail seller 3
Re tail seller 4
Re tail seller 1
Re tail seller 2
Wholesaler 3
Re tail seller 3
Industry : Re tail seller 4
It is seen that the cement is supplied to the consumers from 24 retail shops. The possible number of ways
to supply the product are ( 2 x3 x 4 ) = 24.
5.2 Factorial
This is a mathematical rule used in permutation and combination. The rule gives the product of all
probable n positive natural numbers arranged in an ascending or descending order. For example, let us
consider the first n positive natural numbers , where the numbers are 1, 2, 3, 4, …………..,n. The product
of these numbers is
1 x 2 x 3 x 4 x …………… x ( n – 3 ) x (n – 2 ) x ( n – 1 ) x n
180
n! = 1 x 2 x 3 x 4 x ………. x ( n – 3 ) x ( n – 2 ) x ( n – 1 ) x n
=(n–3!x(n–2)x(n–1)xn
= ( n – 2 )! X ( n – 1 ) x n
= ( n – 1 )! X n
7! = 1 x 2 x 3 x 4 x 5 x 6 x 7 = 5040
= 6! X 7
= 5! X 6 x 7
5! = 1 x 2 x 3 x 4 x 5 = 120
=4! X 5
4! = 1 x 2 x 3 x 4 = 24
= 3! X 4
3! = 1 x 2 x 3 = 6
=2! X 3
2! = 1 x 2 = 2
= 1! X 2
1!= 1
Again, n! = ( n – 1 )! X n
If n = 1, we have 1! = ( 1 – 1 )! X 1
= 0! X 1
As 1! = 1, we have 1 = 0! X 1
Thus, we have 0! = 1
n!
Also, we have ( n – 1 )! =
n
181
5.3 Permutation
It is a technique of arrangement of r distinct objects taken together from n distinct objects. For example,
let us consider that in an industry 3 managers are appointed to look after the different administrative
aspects of the industry. But all 3 managers do not perform duty at the same time. In general, two
managers are assigned duties in two different periods of the office work. Let the managers be M1, M2 and
M3. Two of them are assigned duties in two different periods without any repetition of the duty in the
same day. Their arrangements of duties will be of the form as follows:
There are 6 types of arrangement of managers for the duty. Here M1,M2 and M2,M1 are two separate
arrangements of duties. Because, M1M2 indicates that M1 will do the duty in the morning, say and M2 will
do the duty in the afternoon, say. But the arrangement M2 M1 indicates the opposite arrangement of the
duties. Similar are the cases of other arrangement of duties.
Here there are 6 arrangements of duties for the managers. This type of arrangement is known as
permutation, where order of arrangement is important. Any of the managers can be assigned duty in the
first period of duty and this can be done in n = 3 ways. For the duty of second hour the number of
managers left are ( n – 1) =( 3 – 1 ) =2 and they can be accommodated in 2 ways with any of the first
selected managers. Thus total number of ways of selection of managers for two duties in two periods is n
( n – 1 ) = 3 x 2 = 6. Here out of 3 managers 2 are selected to do duties in two periods. Mathematically,
the number of arrangements can be found out by the formula
n
Pr = n ( n – 1 ) ( n – 2 ) ………….. ( n – r + 1 )
n!
=
(n r )!
3
3!
P2 = =6
(3 2)!
If all 3 managers are assigned duties in 3 different office hours, then the number of arrangements will be
3
3!
P3 = =6.
(3 3)!
In general, if n managers are assigned duties in n different periods, then the number of arrangements will
be
n
n!
Pn = = n!
( n n)!
182
Example 5.1: In a container there are 16 pieces of items of games and sports. These items are to be
shown in 2 shelves each of which can contain 8 pieces. Find the number of arrangements of pieces of
items in a shelve.
Solution: Let the items be identified by serial number 1, 2, 3, 4, ………, 16. These items can be
accommodated in 2 shelves. Each shelve can accommodate 8 items. So, we need the number of
arrangements of 16 items taking 8 at a time. This can be done in
16
16!
P8 = = 518918400 ways.
(16 8)!
This number of arrangements is calculated following one of the rule of permutation. There are other rules
of permutation. These are:
Rule – 1 : The number of different permutations of n different objects taking r at a time with repetition is
n
Pr = nr
For example, in assigning duties for 2 managers out of 3, if any manager becomes agree to do duty in two
consecutive hours, then the number of 9 and these are
Rule – 2 : The number of arrangements of n different objects taken all together round a circle is ( n – 1 )!
Rule – 3 : The number of permutations of n objects taken all at a time , when n1 objects are of one type,
n2 objects are of second type , and nk objects are of k-th type, then the total arrangements of n objects
will be
n!
, where n = n1+n2 +n3 + ………….+ nk.
n1!n2 !..........nk !
Example 5.2: There are 7 digits 0, 1, 2, 3, 4, 5, and 6. How many 6 digits number can be formed with the
above 7 digits so that no digit is repeated? How many numbers will be less than 2000000? How many
numbers will be more than 5000000?
7! 1 2 3 4 5 6 7
7
P7 = = 5040
(7 7)! 0! 1
In these numbers 0 may take the first position in some numbers. But those numbers will not be
meaningful. Therefore, we need the numbers in which 0 must not in the first position. Except 0 there are 6
183
digits. Without repetition the numbers using these 6 digits will be 6! = 720. Therefore, total number of 7
digits number will be ( 5040 – 720 ) = 4320.
These numbers may start with one also. In that case the numbers will be less than 2000000. Without the
digit one the numbers can be formed with the remaining 6 digits in 6! = 720 ways. Hence total numbers of
7 digit number will be ( 4320 – 720 ) = 3600.
The value of the numbers will be more than 5000000 if the number begins with the digit 5. Except the
digit 5 the numbers can be formed with the remaining 6 digits in 6! = 720 ways. Therefore, total number
of 7 digits number will be ( 4320 – 720 ) = 3600.
Example 5.3: In an office there are 4 office assistants. Three of them are needed to work in 3 shifts of
office working hours. In how many ways the duty can be assigned to these office assistants, if (a) no one
works more than one shift, ( b ) there may be repeated duties for any one of the assistants?
Solution: We have n = 4 office assistants. Three of them are needed for office duty in three periods.
( a ) If none of them works repeatedly, the number of arrangements of duties can be assigned in
4! 1 2 3 4
n
Pr = 4P3= = 24 ways.
(4 3)! 1
(b) There may be repeated duties. Then, total number of arrangements of duties will be
n
Pr= 4P3 =43 = 4 x 4 x4 = 64.
Example 5.4: In an office there are 5 computers identified by serial number 1, 2, 3, 4, and 5. Two
computers are needed for two office assistants to work ( a ) at a time , ( b ) at different time periods.
In how many ways two computers can be given to the office assistants?
Solution: We have n = 5, r = 2. From 5 computers two can be taken ( a ) without repetition, ( b ) with
repetition.
5! 1 2 3 4 5
5
P2= = 20.
(5 2)! 1 2 3
In this case as two office assistants will work at the same time, they cannot use the computer of the same
serial number. Thus the distribution of computers for two assistants will be as follows :
184
1 2 21 31 41 51
13 5 2
23 32 42
1 4 24 34 43 5 3
15 25 35 45 5 4
(b) As the assistants will work at different office hours, both of them can use the computer of same
serial number. The number arrangements of n = 5 computers taking r = 2 at a time with repetition. The
number of arrangements are
n
Pr = nr or 5P2= 52 = 5 x 5 = 25
1 1 12 13 14 15
2 1 22 23 24 25
31 32 33 34 35
4 1 42 43 44 45
51 52 53 54 55
Example 5.5: In how many ways the letters in the word BUSINESS can be arranged ,where
n! 8!
4 x 5 x 6 x 7 x 8 = 6720
n1!n2 !n3 !n4 !n5 !n6 ! 1!1!3!1!1!1!
( b ) There are 3 S’s . These S’s can occupy the positions together in the following ways.
(i) S S S - - - - -
(ii ) - S S S - - - -
( iii ) - - S S S - - -
( iv ) - - - S S S - -
(v) - - - - S S S -
( vi ) - - - - - S S S
185
There are 6 ways in which the letters S can occupy the positions together. The remaining 5 positions will
be occupied by the remaining 5 distinct letters. These remaining positions can be occupied in 5! = 120
ways. This is true for every position of S’s . Therefore, total number of ways in which 3 S‟s can occupy
the positions together are ( 6 x 120 ) = 720.
Example 5.6: In how many ways the letters in the word STATISTICS can be arranged? In how many
ways all T will occupy the consecutive positions?
Solution: There are in total n = 10 letters in the word STATISTICS, A is 1, C is 1, I are 2, S are 3 and T
are 3. Thus, n = n1 + n2 + n3 + n4 + n5 = 1 + 1 + 2 + 3 + 3 = 10.
First Part: The 10 letters, some of which are same, can be arranged in
n! 10! 1 2 3 4 5 6 7 8 9 10
50400
n1!n2 !n3 !n4 !n5 ! 1!1!!2!3!3! 1 1 1 2 1 2 3 1 2 3
Second Part : There are 3 T‟s and they can take consecutive positions in the following ways:
(i) T T T - - - - - - -
( ii ) - T T T - - - - - -
( iii ) - - T T T - - - - -
( iv ) - - T T T - - - - -
(v) - - - T T T - - - -
( vi ) - - - - T T T - - -
( vii ) - - - - - T T T - -
( viii ) - - - - - - - T T T
It is seen that 3 T‟s can occupy consecutive positions in 8 ways. In each position, there are more 7 places
which will be filled in by the remaining ( 10 – 3 ) =7 letters. These are 3 S‟s , 2 I‟s, 1 A and 1 C. As all 7
letters are not distinct, these can be arranged in
7! 1 2 3 4 5 6 7
7
P7 = = 420 ways.
3!2!1!1! 1 2 3 1 2 1 1
These 420 arrangements will occur with any of the arrangement of T consecutively. Therefore, total
number of ways in which all 3 T‟s will occupy consecutive position are ( 420 x 8 ) = 3360.
186
Example 5.7: In how many ways the letters in the word MATHEMATICS can be arranged? In how
many arrangements all the vowels will occupy the odd positions [ 1 3, 5, 7, 9, 11 ]?
Solution: In the word MAMTHEMATICS there are n = 11 letters, but all of them are not distinct. There
are n1= 2 A‟s, n2= 1 C , n3= 1 E, n4 = 1 H , n5 = 1 I, n6 = 2 M‟s, n7= 1 S and n8 = 2 T‟s.
11! 1 2 3 4 5 6 7 8 9 10 11
11
P11 = = 6252800.
2!1!1!1!1!2!1!2! 1 2 1 1 1 1 1 2 1 1 2
Second Part: There are 4 vowels. These are 2 A‟s, 1 E and 1 I. These 4 vowels can occupy the 1st, 3rd,
5th, 7th, 9th and 11th . Six positions will be filled up by 4 vowels. This can occur in
6! 1 2 3 4 5 6
6
P4 = = 360 ways.
( 6 4) ! 1 2
Again, 7 consonants can be arranged among themselves in 7 ! = 5040 ways. Therefore, total number of
arrangements of the letters in the word MATHEMATICS ,where 4 vowels can occupy odd positions are
360 x 5040 = 1814400.
Example 5.8: In a city there are 4 government hospitals. Four persons will be admitted in the hospitals. In
how many ways 4 persons ( i ) can be admitted in the hospitals, ( ii ) can be admitted in the same
hospital?
Solution: ( i ) Four persons can be admitted in 4 hospitals in 44 = 256 ways as any two or more persons
can be accommodated in a hospital.
( ii ) There are 4 hospitals. Any one of the persons can choose any of the hospitals. Thus, they have 4
options to choose a hospital. Therefore, the required number of ways in which 4 persons can be admitted
in 4 hospitals is 4.
Example 5.9: There are 4 letters and 4 envelopes. In how many ways the letters ( i ) will be placed in the
correct envelopes, ( ii ) at least one will be placed in the correct envelope, ( iii ) no one will be placed in
the correct envelope?
( i ) As there are 4 envelopes, 4 letters will be placed in the correct envelopes in one way.
( ii ) As total number of ways of placing the letters in the envelopes are 24 and in one way the letters are
placed correctly to the envelopes, so in ( 24 – 1 ) =23 ways at least one letter is not placed correctly.
( iii ) No letter will be placed in correct envelope if these are placed as follows:
Placement of letters in 1 2 3 4
envelopes
187
__________________________ ______________________
1 2 1 4 3
2 2 3 4 1
3 2 4 1 3
4 3 4 1 2
5 3 4 2 1
6 3 1 4 2
7 4 3 2 1
8 4 3 1 2
9 4 1 2 3
____________________________ ______________________
These are the 9 ways in which no letter is placed correctly in the envelope..
Example 5.10: In an experiment an unbiased coin is tossed 4 times independently. From the experiment
how many probable results are expected and what are the results?
Solution: A coin has two faces, viz. head ( H ) and tail ( T ). The probable results from each toss are 2
and in each toss any of the results can be repeated. Hence, total number of results will be 24 = 16. The
results are shown below :
188
H
H
H T
H
T
T
H
H
H
T T
H
T
T
H
H T
H
H
T T
T
H
H T
T
H
T T
Example 5.11: How many 4 digits numbers can be formed using the digits 0 , 1, 2,and 3 so that the
number is divisible by 3 ?
Solution: The 4 digits can be arranged in 4! = 24 ways. All of these arrangements do not represent the 4
digits number as in these arrangements the digit „0’ also occupy the first position of the arrangement. In
that case the digital arrangements will not form 4 digit numbers. Except „0‟ there are 3 other digits and
these digits can be arrangement in 3!=6 ways. These 6 ways will give numbers of 3 digits. Hence , except
these 6 arrangements , there will be ( 24 – 6 )= 18 arrangements which give 4 digits number. But, all these
numbers are not divisible by 3. The numbers will be divisible if the last two digits are 03,30,12,21.
If 0 and 3 occupy the right sides, the remaining 2 digits can be arrangement in 2! = 2 ways. Similar is the
case for the digits 3 and 0. If 1 and 2 occupy the right sides of the arrangements , then only 3 [ 2-1=1] will
be in the extreme left to form to form 4 digits number. This can be done in 1! =1 way. Similar is the case
if 2 and 1 occupy the right sides. Therefore, there will be in total ( 2 + 2 +1 +1 ) = 4 numbers of 4 digits
which will be divisible by 3.
Example 5.12: In an office there are 5 male and 3 female officers. In how many ways the sitting
arrangements of these officers can be made so that all 3 female officers can sit together.
Solution: Let the female officers be represented by F1 , F2 and F3. These female officers can sit together
if their sitting arrangements are of the following types:
189
________________ _____________________________________
(i) F1 F2 F3 - - - - -
( ii ) - F1 F2 F3 - - - -
( iii ) - - F1 F2 F3 - - -
( iv ) - - - F1 F2 F3 - -
(v) - - - - F1 F2 F3 -
( vi ) - - - - - F1 F2 F3
These are the 6 types of sitting arrangements in which all female officers can sit together. However, 3
females can interchange their sitting position among themselves in 3!= 6 ways. Again, 5 male officers can
arrange their sitting arrangements among themselves in 5!= 120 ways. These arrangements of male and
female are 6 x 120 . Again, any one of these arrangements is true for each of the sitting arrangement of 3
females together. Hence, the total number of sitting arrangements of officers in which all 3 female
officers will sit together are 6 x 6 120 = 4320.
Example 5.13: In a bank 10 persons including a couple are in queue. In how many ways the couple will
be in queue together? In how many ways the male partner of the queue will be in the first position
wherever they are in queue?
Solution: First part: Let the two members of the couple be M and F . These two members of the couple
can stand in the queue in the following manner:
(i) M F - - - - - - - -
( ii ) - M F - - - - - - -
( iii ) - - M F - - - - - -
( iv ) - - - M F - - - - -
(v) - - - - M F - - - -
( vi ) - - - - - M F - - -
( vii ) - - - - - - M F - -
( viii ) - - - - - - - M F -
( ix ) - - - - - - - - M F
___________________________________________________________
190
Here in the above arrangements two positions in the queue are occupied by the couple. These two
positions ,again, can be filled in 2! = 2 ways. The other 8 positions can be filled in 8! = 40320 ways.
Hence, the total number of ways in which a couple can stand together in the queue are ( 2x 40320 x 9 )
=725760.
Second part: In the queue shown in the first part of the problem male ( M ) is seen first and female ( F )
is seen second. The position of the male ( M ) and female ( F ) will not be interchanged . This can occur in
1 way. The other 8 positions in the queue will be occupied in 8! = 40320 ways. Therefore, total number of
queues in which male partner of the couple will be in first position are ( 9 x 40320 ) = 362880.
Example 5.14:In a queue there are 8 persons including Mr.X and Mr.Y. Find the number of arrangements
of queue so that in the queue there are other two persons between Mr.X and Mr.Y.
Solution: According to the given condition the standing position of persons in queue will be as follows:
________________ ___________________________________
(i) X - - y - - - -
( ii ) - X - - Y - - -
( iii ) - - X - - Y - -
( iv ) - - - X - - Y -
(v) - - - - X - - Y
It is seen that in 5 different ways the queue can be formed so that between Mr.X and Mr.Y there are other
two persons. In each position of the queue there are ( 8 – 2 ) = 6 empty places which need to be filled in
by 6 other persons. These 6 persons can be accommodated in 6 places in 6! = 720 ways. This arrangement
of persons in 6 places is true for every positions shown above. Again, the positions of Mr. X and Mr. Y
can be interchanged in 2! = 2 ways. Hence, total number of arrangements of queue in which there are two
other persons between Mr.X and Mr.Y are ( 5 x 2 x 720)= 7200.
Example 5.15: In how many ways 8 persons can sit around a round table so that two persons will sit
between two specific persons x and y?
Solution: The two persons between x and y will sit in the following ways:
(i) x - - y - - - -
( ii ) - x - - y - - -
( iii ) - - x - - y - -
( iv ) - - - x - - y -
(v) - - - - x - - y
191
There are 5 types of arrangements in which two persons can sit between two specific persons x and y.
However, the position of x and y will not be interchanged as they are sitting around a round table. Again,
in each type of arrangement 6 persons can be accommodated in ( 6 – 1 )! = 5! = 120 ways. This is true as
the persons are sitting around a round table. Thus, the total number of sitting arrangements of 8 persons
such that between any two persons x and y there will be other 2 persons are 5 x 5! = 600.
Example5.16: Using the digits 1, 2, 3, 4, 5, and 6 how many 4 digits number can be formed so that the
numbers will fall between the limit 3000 to 5000?
Solution: As the numbers will fall between 3000 to 5000, the left side digit of the 4-digit numbers must
not be less than 3 and must not exceed 4. Except the digit 3 we have more 5 digits and these are 1, 2, 4, 5
and 6. Using these 5 digits 3-digit numbers can be formed in
5! 1 2 3 4 5
5
P3=
(5 3)! 1 2 = 60 ways
Now, if we add 3 in the left side of these 60 numbers , we shall have 4-digit numbers which are more than
3000. If we use 4 in the left side of the 3-digit numbers, then the numbers will be less than 5000. So,
except 4 we can form 3-digit numbers using the digits 1, 2, 3, 5, and 6. The 3-digit numbers can be
formed in
5
5!
P3 = = 60 ways
(5 3) !
Using 4 in the left side of these 60 numbers of 3-digit we shall get 4-digit numbers which are less than
5000. Hence, total number of 4-digit numbers will be ( 60 +60 ) = 120.
Example 5.17: A code has 4 digits in specific order and the code is formed using the digits 0, 1, 2, 3, 4, 5,
6, 7, 8, and 9. In how many ways the code can be formed if no digit is repeated in the code?
Solution: There are 10 digits and we need 4-digit numbers without repetition of any of the digits. This
can be done in
10 ! 6! 7 8 9 10
n
Pr = 10P4 = = 7 x 8 x 9 x 10 = 5040 ways.
(10 4) ! 6!
5.4 Combination:
Combination is a mathematical tool to select [ to make groups, to combine] some of items [ objects ] from
a big group of distinct items or objects so that the order of the combination or selection is ignored [ unlike
permutation]. For example, let us consider that there are 3 items A, B, and C and we need to select 2
items at a time from these 3 items. The selected items either will be AB, or AC, or BC. This type of
combination is selection of 2 items one by one without replacement from three items. This combination of
A, B, and C taking 2 at a time is different from the grouping of 3 taking 2 one by one when any of the
items may be selected either at first or at last. Thus, we may have the selected results as follows:
192
AB, BA, AC, CA, BC, CB
In the above arrangements of 3 objects taking 2 at a time, the order of selection of objects is considered.
This type of arrangement is known as permutation. The number of permutations of n objects taking r at a
time is evaluated by the formula
n
n!
Pr =
( n r )!
The value of nPr becomes 6 if n= 3 and r= 2. Here AB and BA is not the same arrangement. This is true
for other groups also. Moreover, in case of permutation items are selected one by one with replacement
and even with repetition. But in combination AB and BA is the same group. Similarly, AC and CA are
not two distinct groups and BC and CB are not two distinct groups. Thus, we have only 3 groups from 3
objects taking 2 at a time. The combination is done by selecting the item one by one without replacement
and there is no scope of repetition, in general, of any object during selection. The number of groups of 3
taking 2 at a time is evaluated by the formula
3
3!
C2 = = 3.
2!(3 2) !
In general, the number of combinations or groups of n objects taking r at a time is decided by the formula
n
n!
Cr =
r ! (n r ) !
n n! n n r 1
n
Cr = , if r 0
r r ! (n r ) ! r 1 r
n 1 n
= , if r n
r n r
n n 1 n
n
Cr =
r r 1 r
Example 5.18: In a well shuffled pack of cards there are 52 cards. Six cards are to be selected. In how
many ways this selection can be done?
n
52 ! 47 48 49 50 51 52
Cr = 52C6= = = 20358520 ways.
6! (52 6) ! 1 2 3 4 5 6
193
Example 5.19: In an office there are 10 male and 6 female officers. Five officers will be selected to form
a committee for supervising the official management. In how many ways the committee can be formed?
In how many ways the committee will consist of 5 male officers? In how many ways the committee will
be formed with 3 male and 2 female officers.
16
16 ! 12 13 14 15 16
C5 = = = 4368 ways.
5!(16 5) ! 1 2 3 4 5
Second part: The committee will consist of only 5 male officers if they are selected from 10 male
officers. This selection can be done in
10 ! 6 7 8 9 10
10
C5 =
5!(10 5 ) ! 1 2 3 4 5 =252 ways.
Third Part: The committee will be formed with 3 male and 2 female officers if 3 males are selected from
10 male officers and 2 female are selected from 6 female officers. This selection is done if the two
separate selections are combined as follows:
10
C3 x 6C2 = 1800 ways.
Let there be n objects. We need to select r ( n) objects with repetitions of any object. The number of
such selection is decided by the formula
n n r 1
r r
n r 1
If n 1 and r 0, we can write
r n 1 .
194
5 5 3 1 7 7! 5 6 7
3 3
3 7C3 = 3!(7 3) ! 1 2 3 =35
The number of r combinations for all r is the number of subsets of a set of n objects. These are several
ways to see that this number is 2n . In terms of combinations it is written as
r
2 n
n
0rn
n! n!
1
n
( I ). Co =
0 ! ( n 0) ! , ( ii ) nC1=
1!(n 1) ! =n = nC ( iii ) nCr + nCr – 1 = ( n + 1 )Cr
n–1
( iv ) nCr = ( n – 1 ) Cr – 1 + ( n – 1 ) Cr = nCn – r .
that these p objects will be included in any of the selection process ,then we have to
(n p) !
(n–p)
Cr – p =
(r p) ! (n r ) !
(vi) Let there be n objects out of which p objects are same. In selecting r objects when p objects
(n–p)
(n p) !
Cr=
r ! (n p r ) !
For example , let us consider that in a packet of 20 bulbs 4 bulbs are defective. Five bulbs from the
remaining ( 20 – 4 ) = 16 bulbs are to be selected so that no defective bulb is included in any selection.
Here we need to select 5 bulbs from the 16 non-defective bulbs. This can be done in
(n – p )
16 !
Cr = 16C5 = =4368 ways.
5! (16 5) !
195
Example 5.20: In a packet there are 20 pieces of cakes including 5 cakes which are to be supplied free of
cost to the retail sellers. To each retail seller 5 cakes including one which is free of cost are to be supplied.
In how many ways the cakes can be supplied?
Solution: There are n = 20 cakes, out of which p = 5 cakes are for free of cost. Five cakes including one
cake which is free of cost are to be given to the retail seller. Thus, the selection of 4 cakes from 15 cakes
is to be made. This can be done in
(n p) ! (20 5) ! 15!
(n–p)
Cr – 1 = = 1365 ways.
(r 1) ! (n p r 1) ! (5 1)! (20 5 5 1) ! 4! 11!
Example 5.21 : In an office of a multinational company there are 3 managers, 6 assistant managers and
11 officers. A committee of 6 members is to be formed to look into the economic transaction of the
company. In how many ways the committee can be formed so that there will be one manager, 2 assistant
managers and 3 officers in the committee?
Solution: We have n= n1+n2+n3=3 +6 + 11 = 20 employees in the office. The committee will be formed
with 1 from n1 , 2 from n2 and 3 from n3 . This can be done in
n
1C1 x n2C2 x n3 C3 = 3C1 x 6C2 x 11C3 = 7425 ways.
Example 5. 22: Two percent products of an industry are usually found defectives. In 15 minutes time the
industry produces 200 units of a product. One day from the production of a randomly selected 15 minutes
5 are inspected and found that there is one defective unit. In how many ways this type of event can
occur?
Solution: As 2 % are defective units and 200 are produced in any 15 minutes time period, the defective
units are 0.02 x 200 = 4 . The non-defective units are ( 200 – 4 ) = 196. Out of 5 one unit is defective. So,
among the 5 inspected units 4 are non-defectives and one is defective. The 4 non-defective units will be
observed from 196 non-defective units and 1 defective units will be observed from 4 defective units. This
can occur in
Example 5.23 : In an office of a multinational company there are 5 receptionists. They are identified by
R1, R2, R3, R4, and R5. Two receptionists are always engaged in turn to handle the visitors. None is
engaged in two consecutive times to handle the visitors. Find number of ways in which the visitors will be
served ( i ) by the receptionists of odd number first, ( ii ) by the receptionists of even number second.
Solution: From 5 receptionists one can be selected in 5C1=5 ways. As any receptionist is not selected for
two consecutive duties, the second receptionist can be selected from the remaining ( 5 – 1 ) =4 and this
can be done in 4C1 = 4 ways. Therefore, total number of ways the receptionists can be selected are 5 x 4
= 20. The arrangements of the receptionists are as follows:
196
R1 R2 R2 R1 R3 R1 R4 R1 R5 R1
R1 R3 R2 R3 R3 R2 R4 R2 R5 R2
R1 R4 R2 R4 R3 R4 R4 R3 R5 R3
R1 R5 R2 R5 R3 R5 R4 R5 R5 R4
( i ) The receptionist of odd identification number who serve the visitors first are
The total number of times the receptionists of odd identification number serve the visitors
are 12.
[ There are 3 receptionists identified by odd numbers. They are R1 R3 and R5. With any one of them the
remaining ( 5 – 1 ) =4 receptionists can be accommodated in 3 x 4 = 12 ways when first position is
occupied by the receptionist of odd numbers.]
( ii ) There are two receptionists for whom identification numbers are even. Any one of them can be
R1 R2 , R1 R4 ; R2 R4 ; , R3 R2 , R3 R4 ; R4 R2 ; R5 R2 , R5 R4 .
Example 5.24: There are 8 male and 6 male partners in the board of govornors of a bank. A sub-
committee of 8 members from the members of the board of govornors is to be formed. In how many
ways the sub-committee consists of 5 males and 3 females?
Solution: We have n = 8 + 6 + 14 members, out of which 8 are males and 6 are females. Five male
members will be selected from 8 in 8C5 ways and 3 females will be selected from 6 females in 6C3 ways.
But the committee will consist of male and females. Hence the number of ways in which the sub-
committee can be formed are 8C5 x 6C3 = 1120.
Example 5.25: The managing committee of a bank consists of one chairman, 3 vice-chairmen and 6
members. A sub-committee of 5 persons is to be formed so that in the sub-committee chairman, one vice-
chairman and 3 members are included. In how many ways such a sub-committee can be formed?
Solution: In the sub-committee chairman will be included. But 1 vice-chairman and 3 members are to be
selected from 3 vice-chairmen and from 6 members. Vice-chairman and members can be selected in 3C1 x
6
C3 = 3 x 20 = 60 ways. With this 60 arrangements one chairman will be included. Hence, total number of
ways in which the sub-committee can be formed are
1
C1 x 3C1 x 6C3 = 1 x 3 x 20= 60 ways
197
Example 5.26: In how many ways a sub-committee of 4 can be formed from 3 technicians, 4 mechanical
engineers and 5 electrical engineers so that in the sub-committee there will be
( i ) 2 mechanical and 2 electrical engineers, ( ii ) 4 electrical engineers, and ( iii ) one technician, one
mechanical engineers, and 2 electrical engineers
Solution: ( i ) Two mechanical engineers will be selected from 4 in 4C2= 6 ways. Two electrical engineers
will be selected from 5 in 5C2 = 10 ways. Total number of ways of sub-committee of 4 will be formed in
4
C2 x 5C2 = 6 x 10 = 60.
( iii ) One technician can be selected from 3 technicians in 3C1 = 3 ways; one mechanical engineer can be
selected from 4 in 4C1 = 4 ways; and 2 electrical engineers can be selected from 5 in 5C2 = 10 ways. Hence
, total number of ways to form the sub-committee of 4 are 3 x 4 x 10 = 120.
Example 5.27: In a box there are 5 balls identified by serial number 1, 2, 3, 4, and 5. One ball is selected
first from the box and is not replaced it in the box. Then another ball from the remaining ( 5 – 1 ) = 4 balls
is selected from the box.
( ii ) In how many ways the number of first selected ball is of odd number?
( iii ) In how many ways both the selected balls are of odd numbers?
( iv ) in how many ways the second selected ball is off odd number?
Solution :
( i ) The odd numbers of the balls are 1 or 3 or 5. If ball no. 1 is selected first, then with this 1 the
remaining ( 5 – 1 ) = 4 numbers will be occurred in 4 ways. This true for the ball of any number. Hence
total number of ways for this selection is 5 x 4 = 20 as any of the first selected ball can be selected in 5C1
=5 and the second selected ball can occur in 4C1 = 4. The selected balls can be shown as follows:
1 2 2 1 3 1 4 1 5 1
1 3 2 3 3 2 4 2 5 2
1 4 2 4 3 4 4 3 5 3
1 5 2 5 3 5 4 5 5 4
( ii ) There are 3 odd number balls. These are 1, 3, and 5. Any one of these 3 can be selected in 3C 1= 3
ways. If one is selected from either 1 or 3 or 5 , the second one will be selected from the remaining
( 5 – 1 ) = 4 balls. The second one is selected in 4C1 =4. Thus, the number of ways in which the first
selected ball is of odd number is 3C1 x 4C1 = 12. The number of selected balls are as follows
1 2 3 1 5 1
198
1 3 3 2 5 2
1 4 3 4 5 3
1 5 3 5 5 4
( iii ) There are three odd numbers and these are 1, 3, 5. Any one of these can be selected in 3C1= 3 ways.
The second one will be selected from the remaining ( 3 – 1 ) = 2 odd numbers. This can occur in 2C1=2
ways. Total number of ways for the selection of both odd number balls are 3C1 xx 2C1= 6. The selected
results are
1 3 3 1 5 1
1 5 3 5 5 3
( iv ) First ball can be selected in 5C1 = 5 ways. The second selected ball is of odd number. This can
happen in 2C1 = 2 ways. Total number of ways in which second selected ball is of odd number is
5
C1 x 2C1 = 10. The selected results are
1 3 2 1 3 1 41 5 1
1 5 2 3 3 5 4 5 5 3
Example 5.28: In an office there are 5 computers identified by serial number 1, 2, 3, 4, and 5. These
computers are used by two operators. In how many ways the operators can use the computers if they
Solution: ( i ) Both the operators work at a time. If first one gets a computer of any serial number, the
second one can not get that computer. Any one operator can get any one computer in 5C1 = 5 ways. The
second operator will get one computer from the remaining ( 5 – 1 ) = 4 computers in 4C1 =4 ways. Hence,
distribution of computers between two operators can be done in 5 x 4 = 20 ways. These ways are :
12 21 31 4 1 51
13 23 32 42 52
14 24 34 43 53
15 25 35 45 54
( ii ) As the operators will work at different time periods, both of them have the same chance to use any
one of the 5 computers. First operator can have any of the computer in 5C1 = 5 ways. The operator will
also get any of the computer in 5C1= 5 ways. Both of them will get the computers in 5 x 5 = 25 ways.
These ways are
199
11 21 31 41 51
12 22 32 42 52
13 23 33 43 53
14 24 34 44 54
15 25 35 45 55
Example 5.29: In a box there are 4 Samsung and 6 ACER computers. From the box 4 computers are
selected twice. In how many ways
( a ) first time 2 Samsung and ACER computers are selected, if the selection is done ( i ) with replacement
, ( ii ) without replacement?
Solution: ( a )
( i ) First time 2 Samsung computers are selected from 4 Samsung computers in 4C2 = 6 ways and 2
ACER computers are selected from 6 ACER computers in 6C2 = 15 ways. Therefore, first time 2 Samsung
and 2 ACER computers are selected in 4C2 x 6C2= 6 x 15 =90 ways. As the selection is done with
replacement, second time one Samsung and 3 ACER computers are selected in 4C1x 6C3 = 80 ways. Both
the selection are independent and hence total number of ways of selection will be 90 x 80 = 7200.
( ii ) First time 2 Samsung and 2 ACER computers are selected in 4C2 x 6C2 = 90 ways. As selection at
second time is done without replacement, the computers left in box are 2 Samsung and 4 ACER. Second
time one Samsung and 3 ACER computers are selected in 2C1 x 4C3 = 2 x 4 = 8 ways. Therefore, total
number of selection in both times are 90 x 8 = 720.
Example 5 30: In a packet there are 5 bulbs of 40 watt, 6 bulbs of 60 watt, and 8 bulbs of 100 watt. It
needs to select 2 bulbs of each type for a purpose. In how many ways it can done?
Solution: If 2 bulbs of each type are needed, total bulbs to be selected are 6. Among these 6 there will be
2 bulbs of 40 watt, 2 bulbs of 60 watt and 2 bulbs of 100 watt. Two bulbs of 40 watt can be selected from
5 bulbs in 5C2 = 10 ways, 2 bulbs of 60 watt can be selected from 6 bulbs in 6C2= 15 ways, and 2 bulbs of
100 watt can be selected from8 bulbs in 8C2=28 ways. Hence total number of ways of selection are 5C2 x
6
C2 x 8C2 = 10 x 15 x 28 4200.
Example 5.31: In a packet there are 6 non-defective and 4 defective spare parts of a machine. Three spare
parts are to be selected. In how many ways ( i ) all defective spare parts can be selected? ( ii ) no defective
spare part is selected? ( iii ) one defective and 2 non-defective spare parts are selected?
200
4! 1 2 3 4
4
C3 = = 4 ways.
3! (4 3) ! 1 2 3 1
( ii ) As no defective spare parts are selected, all three will be non-defective. These 3 can be selected from
6 in
6! 1 2 3 4 5 6
6
C3 = = 20 ways.
3! (6 3) ! 1 2 3 1 2 3
( iii ) One defective will be selected from 4 in 4C1= 4 ways. Two non-defective will be selected from 6 in
6
C2 = 15 ways. Total number of ways of selection of 1 defective and 2 non-defective spare parts will be
4
C1 x 6C2 = 4 x 15 = 60 ways.
Example 5.32: In an office of a multinational company there are 20 officers, 4 of them have MBA
degree, 6 of them have BBA degree and 10 of the rest are general graduates. The company has decided to
form a committee of 5 members under the chairmanship of an officer who have MBA degree. In how
many ways the committee can be formed so that maximum members of the committee are general
graduates?
Solution: As chairman of the committee will be an officer having MBA degree, the other 4 members will
consist of
The chairman will be selected from 4 officers having MBA degree. It can be done in 4C1= 4 ways.
The case ( i ) of the committee will be constituted with another officer of MBA degree and with 3 general
graduates. This can occur in
4
C1 x 4C1 x 10C3 = 1920 ways
201
Example 5.33: A student is asked to solve 6 questions out of 12 questions. But the questions are in two
parts, first part contains 6 questions and he needs to answer any 4 questions from these 6. From the
remaining 6 questions the student needs to answer 2 questions. In how many ways the student can answer
the required questions?
Solution: From first part the student can answer 4 questions out of 6. This can be done in 6C4=15 ways.
From the ( 12 – 6 ) = 6 questions he needs to answer ( 6 – 4 ) = 2 questions. This can be done in 6C2= 15
ways. Therefore, the student can answer the questions in
6
C4 x 6C2 = 225 ways.
Example 5.34: A company invites applications for two posts of managers. Five male and 3 female
candidates applied for the posts. In how many ways ( i ) 2 male candidates, ( ii ) 2 female candidates, ( iii
) one male and one female candidates, ( iv ) 2 managers, and (v ) two managers can be selected
irrespective of male or female?
( iii ) One male and one female can be selected in 5C1 x 3C1 = 15 ways.
8! 1 2 3 4 5 6 7 8
8
C2 = = 28 ways.
2! (8 2) ! 1 2 (1 2 3 4 5 6)
Example 5 35: In an office there are 10 employees. After Eid- ul-fitre they came back to the office and
they shake hand with each other. In how many ways they can shake hand?
Solution: An employee will shake hand with the remaining ( 10 – 1 ) = 9 employees. This is true for
every one. It can be done in 10 x 9 = 90 ways. But these 90 ways indicate that employee-1 shakes hand
with employee – 2 and vice versa. This means that an employee shakes hand twice with another one. But
this not true in case of shaking hand. This event will occur once only. Hence, an employee can shake hand
in
10 9
= 45 ways.
2
[ The shaking hand with each other is a case of combination n persons taking 2 at a time. This can be
done in
10 ! 8! 9 10
n
C2 = 10C2 = = 45 ways.]
2! (10 2) ! 2 ! 8!
202
Example 5.36: How many car number plates can be formed with 3 letters followed by 3 digits? How
many of these number plates begin with the letters ABC?
Solution:
First Part: There are 26 letters. Three of these are to be arranged and the arrangement can be done in
263ways. Again, there are 10 digits. Using these 10 digits numbers of 3 digits can be formed in 103 ways.
Therefore, total number of plates can be formed in 263 x 103 ways.
Second Part: The letters A, B, and C can be arranged among themselves in the order ABC in 1 x 1x 1 x 1
= 1 way. But with these 3 letters 3 digits from10 digits can be arranged in 1 x 103 = 1000 ways.
Example 5.37: In a race court there are 8 horses to start a race. In how many ways the race can be ended?
Solution: This is a problem of arrangement of 8 objects taking 8 at a time. This can be done in
8! = 40320 ways.
Example 5.38: In how many ways a basketball team of 5 players can be formed from 10 players?
10 ! 5! 6 7 8 9 10
10
C5 = = 252 ways.
5! (10 5) ! 5! (1 2 3 4 5)
Example 5.39: ( a ) In how many ways a poker hand gets cards from a regular pack of 52 cards?
( b ) In how many ways a poker hand will get 4 kings ? ( c ) In how many ways a player in a game of
bridge will get 7 spades, 4 diamonds and 2 clubs ?
Solution: ( a ) A poker hand gets 5 cards from a pack of 52 cards. Five cards can be distributed from 52
cards in
52 ! 47 ! 48 49 50 51 52
52
C5= = 2598960 ways.
5! (52 5) ! 5! 47 !
( b ) A poker hand gets 5 cards. He will get 4 kings from 4 kings and one card from the remaining ( 52 – 4
)= 48 cards. Four kings will be distributed from 4 in 4C4= 1 way. One card will be distributed from 48
cards in 48C1 = 48 ways. Hence, poker hand will get 4 kings in 1 x 48 = 48 ways.
( c ) In a game of bridge any player out of 4 players gets 13 cards from 52 cards. There are 4 suits of cards
and in each suit there are 13 cards. These are spade, heart, diamond and club. Any player can get 7
spades, 4 diamonds and 2 clubs in
13
C7 x 13C4 x 13C2 = 95701320 ways
203
This is true for any one of the player. This distribution will occur in 4 ways. So, total number of ways for
the distribution of 7 spades,4 diamonds and 2 clubs will occur in 4 x 2598960 ways.
Example 5.40: ( a ) In how many ways a poker hand will get 5 hearts?
( c ) In how many ways a poker hand will get a particular suit of card?
( d ) In how many ways from a pack of 52 cards 6 spades, 3 hearts, 1 club can be taken?
Solution:
( a ) A poker hand gets 5 cards. These 5 cards will be hearts if these are taken from 13 hearts.
13! 8! 9 10 11 12 13
13
C5 = =1287.
5! (13 5)! (1 2 3 4 5) 8!
( b ) A poker hand will get 3 hearts from 13 hearts and 2 diamonds from 13 diamonds. This can happen in
( c ) There are 4 suits of cards. In each suit there are 13 cards. The poker hand will get 5 cards from any
suit in
13! 8! 9 10 11 12 13
13
C5 1287
5! (13 5)! (1 2 3 4 5) 8! .
As there are 4 suits, total number of ways to get a particular suit are 4 x 1287 = 5148.
( d ) The cards in a packet are of 4 suits and in each suit there are 13 cards. Six spades, 3 hearts and 2
diamonds can be distributed in
13
C6 x 13C3 x 13C2 = 38280528.
Example 5.41: Twelve bulbs are distributed in 4 boxes. ( i ) In how many ways first box will contain 4
bulbs? ( ii ) any of the box will contain 4 bulbs?
Solution: ( i ) Out of 12 bulbs 4 bulbs can be distributed in the first box in 12C4 = 495 ways. The
remaining 8 bulbs will be distributed in 3 boxes in 38 ways. Therefore, the first box can contain 4 bulbs in
495 x 38 ways.
( ii ) There are 4 boxes. Any of the box can contain 4 bulbs in 4 x 12C4 = 1980 ways. The remaining 8
bulbs will be distributed in 3 boxes in 38 ways. Therefore, any of the box can contain 4 bulbs in 1980 x 38
ways.
204
Example 5.42: In an arrangement of a raffle draw in every hour in a departmental store 30 tickets are
distributed to the customers. The ticket numbers are 1, 2, 3, 4, 5, ……. 27, 28, 29, 30.
Five tickets are selected in a randomly selected hour and are arranged in ascending order such that
x1<x2<x3<x4<x5 on the understanding that x3 will be awarded the prize. In ho w many ways x3 = 20 ?
Solution: If the tickets x3 = 20, then above this ticket there are 10 tickets and below this ticket there are
19 tickets. As five are selected , below x3 two tickets can occur in 10C2 ways and above x3 two tickets can
occur in 19C2 ways. Hence, the total number of ways for x3 = 20 to get the prize are
Example 5.43: From a pack of 52 cards 4 cards are selected at random. ( i ) In how many ways one of the
cards will be knave, one will be queen , one will be king and one will be an ace?
Solution: ( i ) there are 4 knaves, 4 queen, 4 kings and 4 aces. From each type one will be selected. This
can occur in 4C1 x 4C1 x 4C1x 4C1 = 256 ways.
( ii ) There are 5 honors cards of each suit and there are 4 suits. From 5 cards 4 can be selected in 5C4 = 5
ways. This is true for one suit. But for 4 suits this occurrence of 4 honors cards can happen in 4 x 5 = 20
ways.
Example 5.44: In how many ways one hand in a game of bridge will get 8 cards of same suit?
Solution: In the game of bridge 52 cards are distributed among 4 players. The cards are of 4 suits and in
each suit there are 13 cards. If one hand gets 8 cards of same suit , the remaining 5 cards will be from
other 3 suits. This distribution of cards can occur in
13
C8 x 39C5 =
13! 39!
8!(13 8)! 5!(39 5)!
8! 9 10 11 12 13 34!35 36 37 38 39
740999259
8!1 2 3 4 5 34!1 2 3 4 5
Again, this can occur in 3 ways. Hence, total number of ways are 3 x 740999259.
Example 5.45: In a show room there are 8 Samsung, 5 ACER and 3 hp computers. A man will buy 3
computers. ( i ) In how many ways he can get one computer of each type? ( ii ) In how many ways he will
buy only Samsung computers?
205
( ii ) Out of 8 Samsung computers 3 can be bought in
8! 5!6 7 8
8 = =
C3 3!(8 3)! 1 2 3 5! 56 ways.
Supplementary Problems
1 . A distributor received 6 Walton and 5 Samsung refrigerators for selling. These refrigerators are to be
arranged in the show room of the distributor taking 4 Walton and 3 Samsung refrigerators. In how many
ways these can be arranged if ( i ) there is no restriction on the arrangement, ( ii ) 3 Samsung refrigerators
will remain together?
2 . How many different 8 letter words can be formed using the letters of the word SYLLABUS ? Ans.
10080.
4 . In a city the bus route numbers consist of a natural number less than 100 followed by one the letters A,
B,C, D and E. How many different bus routes are possible? Ans .594.
5 . There are 3 questions in a question paper. If the questions have 4,3 and 2 solutions, respectively, what
are the total number of solutions? Ans. 24.
6 . How many 3-digit number can be formed using the digits 1 , 4, 7 , 8 , and 9 so that no digit is
repeated? Ans. 60.
7 . A person visits from place A to place E. During his visit he moves from place A to place B by 3 buses,
B to C by 4 buses, C to D by 2 buses and D to E by 3 buses. In how many ways the travel will be
completed? Ans. 72
8 .In how many ways 6 employees of different posts of an office can be accommodated in a room? Ans.
720.
9 . In how many ways 8 new year‟s greeting cards can be sent to 5 friends? Ans. 6720.
10 .In how many ways the letters in the word BUSINESS can be arranged and in how many ways the
arrangement the letter B will be in the beginning? Ans. 6720 ; 1680.
11 . In a laboratory there are 15 non-defective and 5 defective computers. ( i ) In how many ways 5 non-
defective computers can be selected? ( ii ) In how many ways 3 non-defective and 2 defective computers
can be selected? Ans. ( i ) 3003; ( ii ) 4550.
206
12 . From a well shuffled pack of 52 cards 13 cards are to be selected at random. ( i ) In how many
ways this selection can be done? ( ii ) In how many ways 7 red and 6 black cards can be selected? ( iii ) In
how many ways 5 spades, 4 hearts, 2 diamonds and 2 clubs can be selected?
52!
Ans. ( i) ; ( ii ) 888030 ; ( iii ) 5598527320 ; ( iv ) 4 x 48C12 .
13! 39 !
13 . In a packet there are 5 Nokia mobile phone sets, 8 Samsung mobile phone sets, and 7 Walton mobile
phone sets. ( a ) In how many ways 6 mobile phone sets can be selected? ( b ) In how many ways 2 sets of
each type can be selected? Ans. ( a ) 38760; ( b ) 5880.
14 . In a multinational company‟s office there are 10 commerce and 6 arts graduates. A committee of 5
graduates is to be selected. ( a ) In how many ways the committee can be formed? ( b ) In how many ways
there will be 3 commerce and 2 arts graduates in the committee? ( c ) In how ways the committee will
consist only of commerce graduates?
15 . In an office there are 30 computers. These computers are to be distributed in 6 tables, were each table
can contain 5 computers. In how many ways the computers can be distributed in the tables? Ans. 142506.
16 . In a bank there are three administrative officers identified by O1 , O2 , and O3 . Two officers will
perform duties in two periods. In how many ways duty can be assigned ( a ) if any one will perform duty
repeatedly, ( b ) no one will perform duty repeatedly ?
Ans. ( a ) 9; (b) 6.
17 . In an office there are 3 computers identified by serial number 1, 2, and 3. In how many ways the
computers can be arranged ( a ) taking 2 at a time with repetition, ( b ) taking 3 at a time with repetition?
Ans. ( a) 9; ( b ) 27.
18 . In an office there are two receptionists to serve the visitors. In the office hour 20 visitors visit the
office. In how many ways the receptionists serve the visitors? Ans. 380.
19 . In how many ways the letters of the word COMPUTER can be arranged taking all together?
Ans. 40320.
20 . In a library there are 6 books of the same title of the same author, where books are identified by serial
number 1, 2, 3, 4, 5, and 6 . In how many ways the books can be given to two students to read the books
in the library, if they ( a ) visit the library at different time periods, ( b ) visit the library at the same time?
Ans. ( a ) 36 ; ( b ) 25.
21 . In how many ways the 3 – digit numbers can be formed using the digits 0, 5, 6, 7, 8, and 9 without
any repetition of any digit in the number? Ans. 130.
207
22 . In a packet there are 7 non-defective and 5 defective spare parts. ( a ) In how many ways 5 spare parts
can be selected? ( b ) In how many ways 5 non-defective spare parts can be selected? ( c ) In how many
ways 4 non-defective and one defective spare part can be selected?
23 . In an office of a company there are 3 vacant posts. Six male and 4 female candidates applied for the
posts. ( a ) In how many ways 3 post can be filled in? ( b ) In how many ways 2 males and one female
will be selected? Ans. ( a ) 120; ( b ) 60.
24 . In how many ways a poker hand can get 3 aces and 2 queens? Ans. 24.
25 . In how many ways a poker hand will get ace of spade, king of heart, queen of diamond, jack and 10
of club? Ans. 1
26 . In how many ways a poker hand will get 3 spades and 2 hearts? Ans.22308.
27 . In how many ways 4 cards of same suit can be selected from a pack of 52 cards? Ans.2860.
28 . Ten male and 5 female students have tickets to do social works. In how many ways 3 male and 2
female students can be selected to join a social work? Ans. 1200.
29 . In how many ways there will be two boys in a family of 3 children? Ans. 3.
30 . In how many ways a poker hand will get cards of same suit ? Ans. 5148.
31 . In how many ways a poker hand will get all honor cards? Ans. 4.
208
Chapter VI
Probability
6.1 Introduction
Probability can be considered as a measure of uncertainty of an event that can be defined from a random
experiment. Let us explain it by an example. Suppose a company produces electric bulbs and assures that
90% of its products are good. A researcher may be interested to measure the chance of observing a
defective bulb. For the purpose, he can select two bulbs one after another during production period. The
selected bulbs may be Defective (D) or Non-detective (N) or one of them are Defective. The probable
results of this investigation may be of the following type :
No one can claim that any of these 4 types of results will occur unless the investigation is completed.
These results are uncertain and we need to measure this uncertainty. The results of this investigation of
two bulbs, one by one, are not pre-determined. So, this work of investigation of bulbs is known as
„Random Experiment‟.
In the above investigation of two bulbs there are 4 possible outcomes (results). Each of the outcomes is
known as „Sample Point‟ and the total sample points of the random experiment constitutes a „Sample
Space‟ „S‟, where
These 4 out comes (sample points) are „Exhaustive‟ as we cannot expect any other results of the above
mentioned random experiment. Again, during investigation one can expect one of any of these 4 results.
No two results can be observed at the same time and hence the results are known as „Mutually Exclusive‟
outcomes. Here „DD‟ excludes „DN‟ or „DN‟ excludes „DD‟. Similar is the case for other sample points.
Any two outcomes can not occur together. Let us explain the outcomes of another random experiment.
Consider that in an office there are 50% male (M) and 50% female (F) officers. Two officers are assigned
duties in two periods. One of them will perform duty in the morning and one will do in the afternoon. The
probable outcomes of this experiment are
As there is no prior information about the occurrence of any of these 4 results, any of the outcomes can
occur with equal chance as there are 50% male and 50% female officers. These outcomes are equally
probable and they are termed as „Equally likely‟ outcomes. Here chance of occurrence of a male is 0.5
and chance of occurrence of a female is also 0.5. There are 4 equi-probable results. These 4 results are
equally likely and it is denoted by n. In the experiment of selection of bulbs if it is known that 10%
defective bulbs are produced in the industry, then the chance of a defective bulbs to be selected is 0.1 and
the chance of selection of a non-defective bulb is 0.9. The chance of selection of D and N are not equal.
The 4 cases of the sample space are not „Equally likely‟.
209
In the experiment of assignment of duties to the officers, one may be interested to measure the uncertainty
or to measure the chance of selection of one male and one female officers. The interest is centered on the
out comes {MF and FM} of the sample space. This statement about these two outcomes is known as
„Event‟. Event is generally denoted by A or B or C or …….. For the particular statement, let us wirte.
MF, FM
A:
This diagram is known as „Venn Diagram‟. A is constituted with 2 sample points of the sample space.
These outcomes are the „Favourable outcomes of the event A. The number of favourable outcomes is
denoted by m (≤ n), where n is the number of equally likely outcomes.
From the above discussion we have learned some terms. These are discussed below :
Random Experiment : This is an act which is repeated under homogeneous condition and provides any
of the probable results or outcomes. For example, if from some male candidates female (F) candidates
two are selected, then the probable selected results may be MM or MF or FM or FF. If any of these
probable results occur from an act, the act is called random experiment. More examples of random
experiments are selection of a committee from a group of persons, selection of an item from an industrial
output, selection of some labourers from an industry, selection of some industries from a group of
industries, etc.
Outcome : Each of the probable results of a random experiment is known as outcome. For example, if
two computers are selected from 4 computers identified by 1, 2, 3 and 4, then the probable outcomes are
Again, if two computers are given randomly to two persons to work at the same time, then probable
results are
12 21 31 41
13 23 32 42
14 24 34 43
There are probable 12 types of selection. Each of the above results is an outcome.
Again, if two computers are given randomly to two persons to work at different time periods, then the
probable results are
12 22 32 42
11 21 31 41
13 23 33 43
14 24 34 44
210
Each of the above 16 types of results is an outcome.
Sample Space : A set of all probable results of a random experiment constitutes a space known as sample
space. It is usually denoted by S. For example, if „Quality Control‟ department of an industry investigates
3 produced items, one by one, during production process, the probable results are shown in a sample
space as follows
Let there be 3 selection boards to select some employees for an office, where the applicants are males (M)
and females (F). Any of the board can ask the candidates to appear at the interview in the following
orders, where the probable results constitute a sample space.
Here MMM indicates that first board, second board and third board ask the male candidates, MMF
indicates that first and second board ask the male candidates and third board ask the female candidates for
interview. Similar meaning is for other outcomes also.
Sample Point : Each outcome of a sample space is a sample point. For example, in the above sample
space of male and female candidates, MMM is a sample point and there are 8 sample points in the above
sample space.
Exhaustive Outcomes : All probable outcomes of a random experiment are exhaustive outcomes. For
example, let us consider an experiment of selection of 2 books from 3 Economics [E] and 2 Accounting
[A] books. The sample space of this experiment is
S : { E1 E2, E1 E3, E2 E3, A1 A2, E1 A1, E1 A2, E2 A1, E2 A2, E3 A1, E3 A2}
There are 10 possible outcomes. All these outcomes are exhaustive outcomes.
Equally Likely Outcomes : If the proportions (chances) of occurrence of all exhaustive outcomes of a
random experiment are same, then the outcomes are known as equally likely outcomes. For example, if
two computers of an office are selected from 4 computers identified by 1, 2, 3 and 4 then the exhaustive
outcomes constitute the sample space
1
The proportion of occurrence of any of the outcomes in the sample space is 6 and this is true for all
outcomes. As chances of occurrence of any of the outcomes are equal, the outcomes are equally likely.
The equally likely outcomes are denoted by n and in the above example equally likely outcomes are n =
6.
Mutually Exclusive Outcomes : If any two outcomes can not occur together, they are called mutually
exclusive outcomes. For examples, if one saleperson is selected from some males (M) and females (F) to
work in the shop, then the sample space will be
211
S : {M, F}
Here, when M is selected, F can not be selected. M excludes F. Again, if F is selected, M cannot be
selected at the same time. F excludes M. M and F excludes each other. They are mutually exclusive
outcomes.
Event : Any statement regarding one or more of sample point(s) in a sample space of random experiment
is known as event. It is denoted by A or B or C or …… . For example, let there be two computers in a
merchant office, one of which is good (G) and one is not good (N). Two computers are given to two
persons to work at different periods of time. The probable sample space is
One may be interested to calculate the chance of getting a good computer by any one of the persons. Here
a statement is made regarding the points {GN, NG}. This statement to get a good computer by any one of
the persons is known as „Event‟. This event can be denoted by
A : (GN, NG)
Favourable Outcomes : The number of outcomes in favour of any event defined on a sample space of
the random experiment is called favourable outcomes. It is denoted by m (≤ n). In the above example of
event A, there are m = 2 favourable outcomes.
Mutually Exclusive Events : If two events or more events defined on the same sample space of a random
experiment have no common outcomes, then the events are known as mutually exclusive events. For
example, let us define two event A and B as follows using the sample points of the sample space
Here A and B have no common outcome. So, A and B are mutually exclusive outcomes.
Not Mutually Exclusive Events : If two or more events defined on a sample space of a random
experiment have one or more common sample points, then the events are not mutually exclusive events.
For example, let us define two events A and C as follows using the sample points of the sample space
Here A and C have two common outcomes. So, A and C are not mutually exclusive events.
In the example, A and C can occur simultaneously and the sample points common to A and C are
AC (GN, NG)
212
The event AC is usually written as AC.
Classical Definition of Probability: If a random experiment has n exhaustive, mutually exclusive and
equally likely outcomes and if m (≤ n) of these outcomes are in favour of an event A, then the probability
of A is measured by
m
P (A) = n
As an example, let us again consider the selection of two computers for two persons to work at different
periods. The sample space of the experiment is
If it is assumed that there are only two computers in the office and one is good and one is not good. Then,
the above 4 outcomes are exhaustive, mutually exclusive and equally likely
1 1
[n = 4, 2 and chance of (N) = 2 ]. Let A be the event that one of the persons will get a
good computer. Then
A : (GN, NG)
m 2 1
P(A) = n = 4 = 2
(i) if n is infinite,
(ii) if all exhaustive and mutually exclusive cases are not equally likely.
As an example, let us consider that in selecting two computers for two persons to work at different
periods, the selection is done from a lot of 80% good and 20% not good computers. Then P(G) = 0.8 and
P(N) = 0.2. So, good and not good computers are not equally likely. The probabilities of mutually
exclusive and exhaustive outcomes of the experiment of selection of two computers for two persons to
work at different time periods are
213
P(GN) = 0.8 × 0.2 = 0.16, P(NN) = 0.2 × 0.2 = 0.04
All 4 exhaustive cases are not equally likely. Here the classical definition of probability of A is not
applicable. In such a case
Statistical Definition of Probability : Let us consider that a random experiment is repeated n times
under homogeneous conditions and an event A is noted m times (m ≤ n), the probability of A is measured
by
lim m
P (A) = n n
This definition is also not free of defects as homogeneous conditions may not be maintained during
repetition of the experiment. Moreover, it is not easy to repeat the experiment infinite times.
0 ≤ P(A) ≤ 1
m
Since P(A) = n and m ≤ n, P(A) ≤ 1. Again, m cannot be negative as it indicates favourable outcome of
an event A. So, P(A) ≥ 0. Therefore, 0 ≤ P(A) ≤ 1.
The probability laws, formula and techniques were developed by Jacob Bernoulli (1654–1705), De
Moivre (1667–1754), Thomas Bayes (1702–1761) and others. The modern theory of probability was
developed by Russian mathematicians A, Markov, Liaponov, Nevenko, Kolmogoroff and Chebyshev.
Complementary Event : Let there be n exhaustive, mutually exclusive and equally likely outcomes in a
sample space of a random experiment. Consider that m of the outcomes are in favour of an event A. With
the remaining (n–m) outcomes one can define another event. This latter event is known as complementary
– , where
event of A. It is denoted by A
– ) = n–m = 1 – m = 1 – P(A)
P(A n n
– )=1
A
P(S) = 1
Independent Events : Two events A and B are said to be independent, if and only if
214
Here event AB indicates that A and B are not mutually exclusive events. If event A and B are mutually
exclusive events, then AB event is impossible and P(AB) = 0.
Marginal Probability : Let there be n exhaustive, mutually exclusive and equally outcomes
1
in a sample space of a random experiment. The probability of each of the outcome is n .
Consider that the sample points of the S can be divided into k mutually exclusive classes, where the
observations of i-th class (i = 1, 2, .......... , k) are ni = (i = 1, 2, ………… , k) and these are in favour of an
event Ai. Again, each ni observations are sub-divided into non-overlapping r classes having mij of
observations of each sub-class and these mij observations are in favour of another event Bj (j = 1, 2,
…….., ri). The outcomes of Ai and Bj can be shown as follows :
A/B B1 B2 Bj Br Total
… … … … …
… … … … …
Total r1 r2 rj rr n
Here mij outcomes are infavour of an event Ai Bj where A and B occur simultaneously. Therefore,
probability of Ai Bj is
mij
P (Ai Bj) = n
ni rj
Similary P (Ai) = n , and P (Bj) = n
Definition: If the favourable outcomes of an event are divided into several classes of another event, then
the ratio of total outcomes of another event to total equally likely outcomes is the marginal probability.
Let us explain it by example. Consider that in a merchant office there are 100 employees, which are
215
classified into 4 classes according to levels of salary (in Rs). Each class of employees of a particular
salary group are classified according to length of service. The classified results are as follows:
B1 B2 B3 B4
A4, 30,000 + 1 2 4 3 10 = n4
Total 28 = r1 33 = r2 24 = r3 15 = r4 100 = n
It is seen that 50 employees get salary less than Rs 10,000 and they are classified into 4 sub-classes
according to their length of service. Total employees of all sub-classes are the employees who get salary
less than Rs 10,000. The probability of a selected employee who get salary less then Rs 10,000 is
n1 50
P [an employee who gets <Rs 10,000 ] = n = 100 = 0.5
r1 28
Similarly, P [B1] = n = 100 = 0.28
Conditional Probability : Let A be an event which occurs under the condition that another event B
occurs first. Here P(A/B) is called probability A under the condition that B occurs first. This probability is
given by
P (AB)
P (A/B) = P (B)
Similarly, the probability of B under the condition that A occurs first is given by
P (AB)
P (B/A) = P (A)
216
and P (AB) = P (A) P (B/A).
In the cited example of employees of merchant office, one may be interested to measure the probability of
an employee whose length of service is 4–6 years (B3) under the condition that his/her salary is Rs 30,000
and above [A4]. By definition, we have
Example 6.1 : A company is marketing rice oil in plastic bottles. Eighty per cent of the bottles are usually
found in good condition. Once 3 bottles are randomly checked one by one. (a) Find the probability that
among the bottles there will be (i) 3 good bottles, (ii) 2 good bottles, (iii) no good bottles, (iv) at least one
good bottle, (v) at best one good bottle.
Solution : Let G = good and N = not good. The bottles, one by one, can be arranged in the following
ways :
There are 8 exhaustive and mutually exclusive outcomes. But, the outcomes are not equally likely as
= 0.384
217
(iv) Let D : at least one good bottle
–
– , P(D) = P ( C
But D = C )= 1 – P(C) = 1 – 0.008 = 0.992
= 0.2 × 0.2 × 0.2 + 0.8 × 0.2 × 0.2 + 0.2 × 0.8 × 0.2 + 0.2 × 0.2 × 0.8
= 0.104
1
(b) Given P(G) = P(N) = 2 . All 8 outcomes in the sample space are equally likely, we have n = 8.
m 1
P(A) = n = 8
m 3
P(B) = n = 8
m 1
P(C) = n = 8
m – 1 7
P (D) = n = [P(D) =P (C ) = 1 – P(C) = 1 – 8 = 8 ]
m 4 1
P(E) = n = 8 = 2
Example 6.2 : In an office there are 5 computers identified by serial number 1, 2, 3, 4, 5. Two computers
are randomly assigned for two office assistants to work (i) at different periods, (ii) at the same time. Find
the probability that (a) both of the office assistants will use the computer of same serial number, (b) one
who will use the computer in first period will use the computer of odd serial numbers.
Solution :
218
12 22 32 42 52
11 21 31 41 51
S : 13 23 33 43 53
14 24 34 44 54
15 25 35 45 55
Consider the first serial number is for the assistant who works in the first period. The equally like cases
are n = 25.
(a) Let A : both office assistants will use computer of same serial number.
m 5
Favourable cases to A, m = 5, P(A) = n = 25 = 0.2
(b) Let B : the computer of odd serial number will be used by first office assistant. The favourbale
outcomes of B are
B (11, 12, 13, 14, 15, 31, 32, 33, 34, 35, 51, 52, 53, 54, 55)
Here m = 15
m 15
P(B) = n = 25 = 0.6
13 23 32 42 52
12 21 31 41 51
S : 14 24 34 43 53
15 25 35 45 54
Here n = 20
m 0
P(A) = n = 20 = 0
B (12, 13, 14, 15, 31, 32, 34, 35, 51, 52, 53, 54), m = 12
m 12
P(A) = n = 20 = 0.6
Example : 6.3 : A book store receives a packet of books from a publisher, where there are 6 books for
selling and 4 books as complimentary copies. The owner of the book store selects 2 books (a) at random,
(b) one by one with replacement, (c) one by one without replacement. Find the probability that (i) both
books are for selling (ii) one book is for selling.
219
Solution :
(a) There are 6 + 4 = 10 books. Two books from 10 books can be taken in 10C2 ways. All these ways
are exhaustive, mutually exclusive and equally likely. We have
10!
n = 10C2 = 2! (10–2)! = 45
(i) Let A : both books are for selling. The event A will occur if 2 books are taken from 6 books
which are for selling.
m 15 1
P(A) = n = 45 = 3
(ii) Let B : one book is for selling and one is complimentary copy.
m 24 8
P(B) = n = 45 = 15
6C1 6C1
(i) P(A) = P(SS) = 10C1 × 10C1 = 0.36
Here as the books are selected from 10 books one by one with replacement, first time and second time one
is selected book from 10 books in 10C1 = 10 ways. One S book is selected in 6C1 = 6 ways and one C
book is selected in 4C1 = 4 ways.
6C1 5C1 30 1
(c) (i) P(A) = P(SS) = 10C1 × 9C1 = 90 = 3
First book is selected from 10 books in 10C1= 10 ways. As the book is not replaced, second book is
selected from (10–1) = 9 books in 9C1 = 9 ways second book is selected from (6–1) = 5 S books in 5C1 =
5 ways.
Example 6.4 : An industry supplies juice in packets. For packaging, the industry uses 3 machines A, B
and C and the machines make 500, 800 and 700 packages, respectively in an hour.
220
(a) A packet is randomly selected from the output of a day. Find the probability that it is a packed of
machine A.
(b) Two packets are selected (i) at random, (ii) one by one with replacement, (iii) without
replacement. Find the probability that (1) both packets are the products of B, (2) One is the product of A
and one is the product of C,
(c) It is known that 2% of machine A, 1% of machine B and 3% of machine C are defective packets.
If 5 packets are randomly selected, what is the probability that 1 of machine A, 2 of machine B and 2 of
machine C are defective packets.
Solution : The total packets produced are (500 + 800 + 700) = 2000. One packet can be selected in
2000c1=2000 ways. All these ways are mutually exclusive, exhaustive and equally likely.
m 500
P(A) = n = 2000 = 0.25
m 319600
P(B) = n = 1999000 = 0.1599.
m 350000
P(C) = n = 1999000 = 0.1751
(ii) If two packets are selected one by one, then the probable results are
{AA, AB, AC, BB, BC, BA, CC, CA, CB}, here A, B and C are used to identify the products of
A, B and C, respectively.
800C1 800C1
(1) P(B) = P(BB) = 2000C1 × 2000C1 = 0.16
221
500C1 700C1 700C1 500C1
(2) P(C) = P(AC) + P(CA) = 2000C1 × 2000C1 + 2000C1 × 2000C1 = 0.175
800C1 799C1
(iii) (1) P(B) = P(BB) = 2000C1 × 1999C1 = 0.1599
(c) There are 10 defective packets of A, 8 defective packets of B and 21 defective packets of C. Total
defective packets are (10 + 8 + 21) = 39. Now, 5 packet can be taken from 2000 packets in n = 2000C5 =
ways.
m 5880
P(D) = n = 2000c5
Example 6.5 : A company invites application for two posts of managers. Five males and 3 females apply
for the posts. Find the probability that the company will select(i) two males, (ii) one male and one female,
and (iii) two females.
Solution : The number of applicants are (5 + 3) = 8, two applicants from 8 applicants can be selected in
n = 8C2 = 28 ways.
(i) Let A : two males are selected. The favourable outcomes of A are
m = 5C2 = 10
m 10 5
P(A) = n = 28 = 14
m 15
P(B) = n = 28
m 3
P(C) = n = 28
222
Example 6.6 : In a container there are 15 Samsung and 10 ACER computers. Five computers are to be
selected for an office. Find the probability that (i) all selected computers are Samsung, (ii) all selected
computers are ACER, (iii) 3 are Samsung and 2 are ACER computers.
Solution : There are (15 + 10) = 25 computers in the container. From 25 computers 5 can be selected in
All these ways are equally likely, mutually exclusive and exhaustive.
m 3003
P(A) = n = 53130 = 0.0565
m 252
P(B) = n = 53130 = 0.00474
iii) Let C : 3 of the selected computers are Samsung and 2 are ACER computers.
m 20475
P(C) = n = 53130 = 0.3854.
Example 6.7 : Five engineers, 12 technicians and 4 technical assistants are serving in a multinational
company. The company wants to form a committee of 4 persons who will take care of the machineries of
the company. Find the probability that the committee will consist of (i) one engineer, 2 technicians and
one technical assistant, (ii) one engineer and 3 technical assistants, (iii) no engineer.
Solution : There are (5 + 12 + 4) = 21 persons who are serving in the company. From 21 persons 4 will
be selected in n = 21C4= 5984 ways.
All these ways are mutually exclusive, exhaustive and equally likely.
(i) Let A : the committee will consist of one engineer, 2 technicians and one technical assistants
m 1320
P(A) = n = 5985 = 0.221
(ii) Let B : the committee will consist of one engineer and 3 technical assistants.
223
m 20
P(B) = n = 5985 = 0.00334
C can occur in m = 12C4 + 12C3 × 4C1 + 12C2 × 4C2 + 12C1 × 4C3 + 4C4= 1820 ways.
m 1820
P(C) = n = 5985 = 0.3041
Example 6.8 : In an area there are 4 departmental stores. Four persons will visit departmental stores for
marketing. Find the probability that they will visit (i) same departmental store, (ii) 4 separate
departmental stores, (iii) any two departmental stores.
(i) Let A : Four will visit the same departmental store. They can do that in m = 4 ways.
m 4 1
P(A) = n = 256 = 64
m 24 3
P(B) = n = 256 = 32
iii) Let C : Four persons will visit any two departmental stories.
If they visit 2 stores, one will visit one store and another 3 will visit another store or 2 will visit
one store and another 2 will visit another one store. One or 2 can visit in 30 ways. But there are 4 stores.
So, total number of ways to visit 2 stores in m = 4× 30 = 120 ways.
m 120 15
P(C) = n = 256 = 32
Example 6.9 : There are 3 letters and 3 envelops with address. The letters are put in the envelopes at
random. Find the probability that all the letters are put in correct envelopes.
Solution : The letters can be put in n = 3! = 6 ways. Any letter will be put in correct envelop in m = 1
way.
m 1
Reqd. prob. = n = 6
Example 6.10 : In an office there are 3 sockets and 5 bulbs. Out of 5 bulbs 3 are defectives. Find the
probability that the office will get light after putting the bulbs in the sockets.
224
Solution : Let A : there will be no light in the office. There are 5 bulbs. They can be put
n = 3 sockets. From 5 bulbs 3 can be selected in n = 5c3 = 10 ways. If all 3 bulbs are defective, there will
be no light. Therefore,
1
P(A) = 10
– ) = 1 – P (A) = 1 – 1 = 9
P (A 10 10
Example 6.11 : Twelve sales persons are assigned duties in 4 departmental stores. Find the probability
that the 4 sales persons are assigned for the first store.
Solution : Any sale person can be assigned for any of the departmental store. Twelve sales persons can be
distributed duties in 4 stores in n = 412 ways. Let A be the event that 4 sales persons are assigned for first
store. Four can be assigned in 12c4 ways. The remaining 8 sales persons can be assigned in the remaining 3
stores in 38 ways. Therefore, total number of ways in which 4 sales persons are assigned for first store are
m = 12c4 × 38.
12
c 4 × 38
P(A) = 412 .
Example 6.12 : Find the probability that in a random arrangement of the letters in the word „BUSINESS‟,
three S‟s do not come together.
Solution : There are 8 letters in the word „ business‟. 5 letters are different and 3 are same. The
arrangement of letters can be done in
8!
n = 3! = 6720 ways.
(v) s s s ...
(vi) s s s
225
Three S‟s can be arranged in 6 ways. The other (8–3) = 5 letters can be arranged in 5! = 120 ways. Again,
3 S‟s can be arranged among themselves in 3! ways. Let A be the event that all 3 S‟s are arranged
together.
m 4320
P(A) = n = 6720 = 0.6429
Example 6.13 : From 20 applicants for a job 3 are to be selected. Find the probability that, out of 3, a
specific applicant is selected, (ii) there will be no specific one among 3 selected candidates.
Solution :
Therefore, A can occur in m = 19C2 ways. As remaining 2 will be selected from the rest. (20–1) =
19 applicants.
m 19C2 3
P(A) = n = 20C3 = 20
– ) = 1 – P(A) = 1 – 3 = 17
P(A 20 20
Example 6.14 : In a box there are 10 bulbs. Two bulbs are selected at random from the box. Find the
probability that (i) two specified bulbs will be selected; (ii) any one of the two specified bulbs will not be
selected.
m 1
P(A) = n = 45
–
(ii) Let A : none of the two specified bulbs will be selected.
– 1 44
P(A ) = 1 – P(A) = 1 – 45 = 45
226
iii) Let B : any one of the two bulbs will not be selected.
For the event B two bulbs can be selected from the remaining (10–2) = 8 bulbs. This can be done
in m = 8C2 = 28 ways.
m 28
P(B) = n = 45
Example 6.15 : In a container there are 4 non-defective and 3 defective computers. Three persons, one
after another, take a computer without replacement. The person who will take first non-defective
computers will be rewarded. What is the probability that the first, second and third person, respectively
will be rewarded.
Solution : Let us consider that A, B and C will be rewarded at first, second and third time, respectively.
Let N be non-defective and D be the defective computers. The sample space, if one computer is selected
without replacement, is
{N, DDDN}
4 3 2 1 4 63
P(A) = P(N) + P(DDDN) = 7 + 7 6 5 4 = 110
3 4 2
P(B) = P(DN) = 7 . 6 . = 7
3 2 4 4
P(C) = P(DDN) = 7 . 6 . 5 = 35
Example 6.16 : Twenty five applicants have applied for a post in a merchant office. The applicants are
identified by serial number 1, 2, 3, 4, ....... 25. Four applicants are to be selected at random and they will
be ranked according to their performance of the selection procedure. Find the probability that the second
position will be secured by the applicant number 1 to 19.
Let second position is secured by the applicant having serial number 20 and first position will be secured
by any one of the applicant bearing serial number 1 to 19. First position will be secured in 19C1 = 19
ways. Third and fourth position will be secured from the remaining 5 serial number in 5C2 = 10 ways.
Therefore, second position will be secured by the applicant having serial number 20 in m = 19 × 10 = 190
ways.
m 190 19
Reqd. Prob. = n = 12650 = 1265
227
Example 6.17 : If Sunday is the weekly holiday for offices, what is the probability that in a lip year there
will be 53 holidays.
Solution : In a lip year there are 366 days and so there are 52 weeks and more 2 days. If week starts on
Sunday and ends on Satureday, the next two days will be Sunday and Monday. If week starts on any other
day, the next two days may be Monday and Tuesday or Tuesday and Wednesday or Wednesday and
Thursday or Thursday and Friday or Friday and Saturday or Saturday and Sunday. Thus, there are n = 7
possible results and in favour of Sunday there are m = 2 possibility. If A is the event that there will 53
Sunday, the
m 2
P(A) = n = 7
Example 6.18 : Three persons are selected at random. Find the probability that (i) their birthday is Friday,
(ii) birthday of 2 persons is Friday, (iii) birth day of 2 is Friday and birth day of another one is Monday.
Solution : The birthday of any one may be one day out of 7 days. The probable cases of birth day are
n = 73 = 343
m 1
P(A) = n = 343
(ii) Let B : birthday of 2 persons is Friday. Birth day of another persons may be any other day of the
remaining 6 days. The birth day of third person can occur in 6 ways. But birth day of 2 persons in Friday
m 36
can occur in 3! = 6 ways. So, B can occur in m = 6 × 6 ways. P(B) = n = 343
iii) Let C : birthday of 2 persons is Friday and birth day of third person is Monday. C can occur in
m = 3 ways.
m 3
P(C) = n = 343
Theorem : If A and B are two not mutually exclusive events defined on a sample space, then
228
– –
If is seen that A B , AB and A B are 3 mutually exclusive events. It is seen that
– –
A = A B + AB and B = A B + AB
– –
A B = A – AB A B = B – AB
– –
P(A B ) = P(A) – (AB), P( A B) = P(B) – P(AB)
– –
Again, P (AB)= P (A B ) + P (AB) + P (A B) = P (A) – P(AB) + P( A B) + P(B) – P(AB)
= P(A) + P(B) – P(AB)
If there are n events A1, A2, .......... , An defined on the same sample space, then
n
P (A1A2 .......... An) = ∑ P (Ai) – P( Ai A j ) P( Ai A j Ak )
i=1 i 1 i jk
i j
n
P (A1 A2 ......... An) = ∑ (PAi)
i=1
Corollary : If A and B are defined on the same sample space S and if A B, then
229
P (A) ≤ B)
As A B, A B = A. So,
Corollary : If A and B are two not mutually exclusive events defined on a sample space, S, then
– –
P (A B) = 1 – P (A ) – P(B )
–––– – –
AB = A B [Venn diagram given above].
––––
P (AB) = 1 – P (AB )
– –
= 1 – P (A B )
– – – –
= 1 – {P (A ) + P(B ) – P (A B )}
– –
P (AB) ≥ 1 – P (A ) – P(B )
Example 6.19 : A company produces computers using 2 workshops. One workshop produces 50
computers per day and another workshop produces 30 computers per day. 10% of first workshop and 20%
of second workshop are usually found defectives. A computer is selected at random. Find the probability
that the selected computer (i) is either product of first workshop or defective, (ii) is defective under the
condition that it is the product of second workshop.
–
A : second workshop
B : Defective computer
–
B : Good computer
230
B – Total
B B
A
A 5 45 50
AB –
AB
– 6 24 30
A
– – –
A B A B
Total 11 69 80
One computer can be selected in 80C1 = 80 ways, A can occur in 50C1 = 50, B can occur in 11C1 = 11 ways
50 11 5 56
= 80 + 80 – 80 = 80
–
– P(A B) 6/80
(ii) P (B/A ) = – = 30/80 = 0.2
P(A )
Example 6.20 : Two computer scientists of an office are asked to develop a programme for solving
official problem. One of them has the record of 80% success and another one has the record of 70%
success. They work independently. Find the probability that the programme will be developed.
Solution : Let A be one of the computer scientists and B is another computer scientist. Probability of
success of A and B are, respectively P(A) = 0.8, P(B) = 0.7
Example 6.21: A company invites application to select an officer for the management of official work.
One hundred candidates have applied for the post. Among them 60 are males and 40 are females. Among
male applicants 45 have MBA degree. The corresponding figure among female applicants is 30. One
231
application is selected at random. Find the probability that the selected application is (i) of male or of an
MBA degree holder, (ii) of female under the condition that she has no MBA degree, (iii) of female having
MBA degree.
–
A : application of a female
–
B : application of a candidate without MBA degree.
B B – Total
B
A
A AB – 60
AB
45
15
– – – – 40
A AB AB
30 10
Total 75 25 100
We need,
60 75 45
= 100 + 100 – 100 = 0.9
– –
– – P(A B ) 10/100
(ii) P (A /B ) = – = 25/100 = 0.4
P(B )
– 30
(iii) P (A B) = 100 = 0.3
Example 6.22 : Three computer scientists have the experiences of success in developing programmes in
80%, 70% and 60% cases. They work independently. A company needs a developed programme and asks
the computer scientists to work for the development. Find the probability that the programme will be
develped.
232
Solution : Let A, B and C be the computer scientists. Assume tha P(A) = 0.8, P(B) = 0.7 and P(C) = 0.6.
The programme will be developed if either one of them can develop the programme. We need
= 0.8 + 0.7 + 0.6 – 0.8 × 0.7 – 0.8 × 0.6 – 0.7 × 0.6 + 0.8 × 0.7× 0.6
= 0.976
Alternative : The programme will not be solved if A, B and C fail. Probability of failure of them are
– – –
P (A ) = 1 – P(A) = 0.2, P(B ) = 1 – P(B) = 0.3, P(C ) = 1 – P|C) = 0.4
– – –
Failure of them is P (A B C ) = 0.2 × 0.3 × 0.4 = 0.024
– – –
1 – P (A B C ) = 1 – 0.024 = 0.976
Example 6.23 : A machine has two parts A and B. Machine works properly, if both A and B function
properly. It is known that A fails to work properly in 30% cases and B fails to work in 20% cases. Find
the probability that the machine will work properly.
Solution : Machine will work properly if both A and B work together properly. We need success of both
– – – –
A and B, ie. P (AB). Given P(A ) = 0.3 and P(B ) = 0.2, where A and B are the failure cases of A and B,
respectively. Therefore, P(A) = 0.7 and P(B) = 0.8
Example 6.24: A sales person has 60% chance of making sale to each customer. The behaviour of
successive customer is independent. If two customers A and B enter, what is the probability that the sales
person will make a sale to A or B?
Example 6.25 Two candidates A and B have applied for a post. The probability of selection of A is 0.7
and that of B is 0.6. Find the probability that (i) both of them will be selected, (ii) none will be selected.
233
P (AB) = P(A) + P(B) – P(AB)
Example 6.26 : In a city 60% people read Bengali paper 40% people read English paper,. 30% people
read both. One person is selected at random. Find the probability that the selected person reads one of the
paper.
We need.
Example 6.27 : In a merchant farm 70% officers have MBA degree and 60% officers are male. An
officer is selected at random. Find the probability that the selected one is (i) either male or having MBA
degree, (ii) female under the condition that she has no MBA degree, (iii) male having MBA degree.
–
A : officer without MBA degree
B : officer is male
–
B : officer is female
B B – Total
B
A
A AB – 0.7
AB
0.42 0.28
234
– – – – 0. 3
A A B A B
0.18 0.12
– –
– – P(A B ) 0.12
(ii) P (B /A ) = – = 0.3 = 0.4
P(A )
Example 6.28 : If two unbiased dice are thrown, find the probability that (i) the sum of the upper faces of
dice is neither 7 nor 11, (ii) sum of the upper faces of dice is 10 or first die shows 5.
11 21 31 41 51 61
12 22 32 42 52 62
S:
13 23 33 43 53 63
14 24 34 44 54 64
15 25 35 45 55 65
16 26 36 46 56 66
(i) Let A : Sum of the upper faces of the dice is 7.
6 2 8
= 36 + 36 = 36
8 28 7
We need 1 – P(AB) = 1 – 36 = 36 = 9
235
(ii) Let C : Sum of the upper faces of dice is 10
3 6 1 8 2
= 36 + 36 – 36 = 36 = 9
Example 6.29 : Suppose that one of the three men : a researcher, an educationist and a politician will be
selected as vice-chancellor of a university. The chances of their selection is 0.3, 0.4 and 0.6, respectively.
The probabilities that the university will be developed by these people if they are appointed are 0.6, 0.8
and 0.5 respectively. Find the probability that the university will be developed by the new vice-
chancellor.
236
Example 6.30 : A company wants to appoint a managing director for the welfare of the firm. The
managing director may be appointed from a politician, from an economist or from a retired manager of
any other company. The probabilities of these selection are 0.7, 0.2 and 0.1, respectively. If any one of
these persons is selected, the production of the company will be increased with probability 0.4, 0.6 and
0.8, respectively. Find the probability that the production of the company will be increased after the
appointment of the new managing director.
= 0.48
Example 6.31 : A research organisation conducted a survey on 1250 families to investigate the marketing
behaviour of the families. The families are classified according to yearly income (in $), marketing
frequency and possessing refrigerator. The classified results are shown below :
<20000.00 20000.00+
Frequency of market
If a famly is selected at random, what is the probability that the selected family possesses (i) refrigerator,
(ii) refrigerator under the condition that it has yearly income 20000.00, (iii) refrigerator under the
condition that it visits two times.
237
Solution : Let A : family possesses refrigerator,
m 750 3
(i) P(A) = n = 1250 = 5
P(AB) 280/1250 14
(ii) P(A/B) = P(B) = 500/1250 = 25
P(AC) 300/1250 3
(iii) P (A/C) = P(C) = 500/1250 = 5
Example 6.32 : A company wants to select a manager for the production sector of the company. Those
who applied for the post 30% of them have the MBA degree, 20% of them Ph.D holders and 19% of them
have both the degrees. One is selected at random. Find the probability that (i) the candidate has Ph.D
degree under the condition that he has MBA degree, (ii) the candidate has MBA or Ph.D degree.
P(AB) 0.19 19
(i) P (B/A) = P(A) = 0.30 = 30
Example 6.33 : Two persons A and B are asked to develop a computer programme under the condition
that he will be rewarded who can develop first. It is known that chances of success of A and B are equal
and it is 50%. If A starts to develop the programme, what is his chance to be rewarded?
1
Given P(S) for both A and B are 2 .
238
A : {S, FFS, FFFFS, ....................... }
1 1 1 2 1 1
= 2 + 23 + 25 + ....................... = 2
1 = 3 Example 6.34 : A computer is assembled from
1 – 22
three components A, B and C. The probabilities that the components are defective are 0.04, 0.05, 0.03,
respectively Find the probability that the assembled computer is good.
– – –
P(A ) = 0.96, P(B ) = 0.95, P(C ) = 0.97
– – –
The computer will be good if A B C occurs.
– – –
The occurrence any of the events A , or B or C are independent. Therefore,
– – – – – –
P (A B C ) = P (A ) P (B ) P(C ) = 0.96 × 0.95 × 0.97 = 0.88464
Example 6.35 : In an industry there are 10 machines made in China and 5 machines made in Korea. Five
machines made in China and 2 machines made in Korea are not working properly. Once a machine is
found out which is not working properly. Find the probability that (i) the machine is made in China or is
not working properly, (ii) the machine is made in China and is not working properly.
AB 2 B
239
We need P(AB) = P(A) + P(B) – P(AB)
10 7 5 4
= 15 + 15 – 15 = 5
5
(ii) We need P(AB) = 15
Example 6.36 : A manager of a company has sent 10 files to the higher authority of the company. Among
these files 6 are related to administration and 4 are related to finance. The authority has selected 2 files (i)
one by one without replacement, (ii) one by one with replacement, and (iii) at random. Find the
probability that one of the file is related to administration and another is related to finance.
6 4 4
We need P(AB) = P(A) P(B/A) = 10 × 9 = 15
4 6 4
We need P(CD) = P(C) P(C/D) = 10 × 9 = 5
4 4 8
P(AB) + P(CD) = 15 + 15 = 15
6 4 6
(ii) P(AB) = P(A) P(B/A) = 10 10 = 25
4 6 6
P(CD) = (C) P(C/D) = 10 . 10 = 25
6 6 12
P (AB) + P (CD) = 25 + 25 = 25
n 24 8
P(A) = m = 45 = 15
240
Example 6.37 : For a construction work 3 labourers are to be appointed. The leaders of the labourers are
asked to help in selecting the candidates. There are 3 leaders. There are 3 male and 1 female labourers
under first leader. There are 2 male and 2 female labourers under second leader. There are one male and 3
female labourers under third leader. If one is selected from each leader, what is the probability that one
male and 2 female labourers will be selected.
F : Female labourer
M and F can be selected from first, second and third leader in the following order; respectively
M F1 F2 or F1 M F2, F1 F2 M
3 2 3 1 2 3 1 2 1
=4×4×4+4×4×4+4×4×4
= 0.40625
Example 6.38 : A couple has applied for a post in a company. The male partner has the chance 1/6 to be
selected and for the female partner this chance is 1/8. Find the probability that (i) both of them will be
selected, (ii) either husband or wife will be selected, (iii) none of husband and wife will be selected.
1 1 1
(i) P(AB) = P(A) P(B) = 6 ×8 = 48
(ii) Either of them will be selected. In favour of this statement the events are
– –
A B Or, AB
– – – –
P (A B) + P(A B ) = P(A ) P(B) + P(A)( P(B )
5 1 1 7 12 1
= 6 ×8 + 6 × 8 = 48 = 4
– – – – 5 7 35
(iii) We need (A B ) = P(A ) P(B ) = 6 ×8 = 48
241
Example 6.39 : Two partners A and B have started a joint business on first of January of a year. The
probability that A will continue the business for one complete year is 0.7 and the same probability for B is
0.9. They can stop the business independently at any time. Find the probability that (i) any one of A and B
can stop the business before completion of one year, (ii) both of them will stop the business.
– – –
(i) We need P (A B) + P (AB ) = P(A ) P(B) + P(A) P(B)
= 0.34
– – – –
(ii) We need P(A B ) = P(A ) P(B ) = 0.3 × 0.1 = 0.0
Let H1, H2, ....... Hk be k mutually exclusive events defined on a sample space S, where P(Hi) ≠ 0, i = 1, 2,
3, ......k. Let E be the another event defined on the same sample space S such that E occurs as union of Hi
k
[E = Hi]. The event E and Hi can be shown by Venn diagram as follows :
i=1
H1 H2 ... Hk
S H1 E H2 E Hk E
It is seen that
E = H1 E + H2 E + .......... + (Hk E)
P(HiE)
P (Hi/E) = P(E)
242
This is a definition of conditional probability. Another conditional probability is given by
P(HiE)
P (E/Hi) = P(Hi)
Here E occurs under the condition that Hi occurs first. By multiplication rule of probability, we can write
Here P(Hi‟s) are apriori probabilities. If these two probabilities or the statement of the events Hi and E/Hi
k
are provided, one can calculate P(HiE) for all i = 1, 2, .................. , k, and hence P(E) = ∑ P(Hi E )
i=1
can be evaluated. At this stage the values of P(E) and P(HiE) can be replaced in the Bayes theorem.
Remarks :
(i) P(H1), P(H2), ............. P(Hk) are apriori probabilities because these exist before we get
information from the experiment itself.
(ii) The probabilities P(Hi/E), i =1, 2, .......... , k are called posteriori probabilities, because these are
evaluated after the result of the experiment is recorded.
(iii) The probabilities P(E/Hi), i=1, 2, ................ , k are called likelihoods because these indicate how
likely the event E under consideration is to occur, given each and every a priori probability.
Example 6.40 : A factory produces certain types of output by three machines A, B and C. Machine A
produces 3,000 units, machine B produces 2500 unit and machine C produces 4500 units. Past experience
shows that 1 percent of the outputs of A, 1.2 percent outputs of B and 2 percent outputs of C are
defective. An output is selected at random from the production of a day and found that it is defective.
Find the probability that it is produced by (i) machine A, (ii) machine B, (iii) machine C, (iv) machine A
or machine B.
Solution : Let
H1 : the production of A,
H2 : the production of B,
H3 : the production of C,
E : an output is defective.
243
Then,
P(H1E) 0.003
(i) P(H1 /E) = P(E) = 0.015 = 0.2
P(H2E) 0.003
(ii) P(H2 /E) = P(E) = 0.015 = 0.2
P(H3E) 0.009
(iii) P(H3 /E) = P(E) = 0.015 = 0.6
Example 6.41 : In a bolt factory machines A, B and C manufacture respectively 30%, 30% and 40%
bolts. Of the total of their output 5,4 and 2 percent are defective bolts. A bolt is selected at random and
found that it is defective. Find the probability that it is the product of (i) A, (ii) B, and (iii) C.
Solution : Let
H1 : production of A,
H2 : production of B,
H3 : production of C,
E : defective product.
P(H1E) 0.015
(i) P(H1 /E) = P(E) = 0.035 = 0.43
244
P(H2E) 0.012
(ii) P(H2 /E) = P(E) = 0.035 = 034
P(H3E) 0.008
(iii) P(H3 /E) = P(E) = 0.035 = 0.23
Example 6.42 : In a merchant office 60% are male and 40% are female employees. The probability of
getting promotion of a male employee is 0.8 and that of a female is 0.6. One employee is selected for
promotion. Find the probability that the selected one is female.
P(H2E) 0.24 1
P(H2/E) = P(E) = 0.72 = 3
Example 6.43 : In a merchant office of a multinational company 70% employees are male. 60% male and
50% female employees have MBA degree. An employee is selected at random and found out he/she has
MBA degree. Find the probability that the selected one is a male.
Solution : Let H1: a male employee, H2: a female employee, E: employee having MBA degree. Given
P(H1) = 0.7, P(H2) = 0.3, P (E/H1) = 0.6, P(E/H2) = 0.5
P(H1E) 0.42 42
P(H1/E) = P(E) = 0.57 = 57
Example 6.44 : A life insurance company classifies the driver when the company needs to do an
insurance for the vehicle. Those who have minimum accidents record, they are first class drivers, those
who face the accidents in 10% days of the year, they are medium category drivers and those who face
accidents more than 10% days in a year, they are the worst type of drivers. The percentages of these 3
types of drivers are 20%, 35% and 45% respectively. The first class drivers face accident in 5% days and
other two categories of drivers face accident 25% each. A driver did the insurance and within one year of
insurance he faced accident. Find the probability that he belongs to worst type of drivers.
H2 : worse driver
245
H3 : worst driver
E : faced accidents
P (H3 E) 0.1125
P (H3/E) = P(E) = 0.21 = 0.54
Example 6.45 : In a factory there are 4 machines of A class and 3 machines of B class. In an A class
machine 2 males and 2 females are working and in a B class machine 2 males and one female are
working. One worker is selected at random and found that she is female. Find the probability that she is
working in A class machine.
H2 : B class machine
E : Worker is female
4 3 2 1
Given P(H1) = 7 , P(H2) = 7 , P(E/H1) = 4 , P (E/H2) = 3
4 2 2
P(H1 E) = P(H1) P(E/H1) = 7 × 4 = 7
3 1 1
P(H2 E) = P(H2) P(E/H2) = 7 × 3 = 7
2 1 3
P(E) = P(H1 E) + P(H2 E) = 7 × 7 = 7
P (H1 E) 2/7 2
P (H7/E) = P(E) = 3/7 = 3
Example 6.46 : From two canons A and B it is fired to target. In a particular period it is fired from A 9
times and 10 times from B. Canon A hits the target 8 times out of 10 times. Canon B hits the target 7
times out of 10 times. A target is fired during firing. Find the probability that it is fired by canon B.
246
H2 : Canon B,
9 10 8 7
Given P(H1) = 19 , P(H2) = 19 , P(E/H1) = 10 , P (E/H2) = 10
9 8 72
P(H1 E) = P(H1) P(E/H1) = 19 × 10 = 190
10 7 70
P(H2 E) = P(H2) P(E/H2) = 19 × 10 = 190
72 70 142
P(E) = P(H1 E) + P(H2 E) = 190 × 190 = 190
P (H2 E) 70/190 35
P (H2/E) = P(E) = 142/190 = 71
Example 6.47 : A labourer of an industry uses bus or train to reach the industry. The ratio of using bus
and train is 3 : 7. The probability that the labourer reaches to the industry late is 0.25 and the same
probability if he uses train is 0.05. One day he reaches to the industry late. Find the probability that he
uses train.
H2 : uses train,
E : reaches late.
3 7
Given P(H1) = 10 , P(H2) = 10 , P(E/H1) = 0.25, P(E/H2) = 0.05
3
P(H1 E) = P(H1) P(E/H1) = 10 × 0.25 = 0.075
7
P(H2 E) = P(H2) P(E/H2) = 10 × 0.05 = 0.035
P (H2 E) 0.035
P (H2/E) = P(E) = 0.11 = 0.318
Example 6.48 : A company has decided to appoint a computer personel and arranged an advertisement
for the post. The advertisement has come out twice. The number of applicants after the two
advertisements are as follows :
247
Second advertisement : 8 computer scientists and 6 computer engineers.
Each group of applicants has equal chance of appointment and one candidate is selected from a group and
found out that the candidate is computer scientist. After the selection, the company has decided to appoint
another one. Find the probability that the next appointed one is a computer engineer.
5 8
Given P(H1) = 0.5, P(H2) = 0.5, P(E/H1) = 9 , P(E/H2) = 14
1 5 5
P(H1 E) = P(H1) P(E/H1) = 2 × 9 = 18
1 8 8
P(H2 E) = P(H2) P(E/H2) = 2 × 14 = 28
4 6
Given P(B/H1 E) = 8 , P(B/H2 E) = 13
5 8 142
P(E) = P(H1 E) + P(H2 E) = 18 + 28 = 252
5 4 8 6
18 × 8 + 28 × 13 0.270757
= 142/252 = 0.563492 = 0.48
Example 6.49 : A hotel gets 70% native guests and 30% foreign guests. Thirty percent native guests
reside in the hotel for one week and 60% foreign guests reside in the hotel for one week. A guest resides
in the hotel for one week. Find the probability that he/she is a foreign guests.
H2 : foreign guest,
248
Given P(H1) = 0.7, P(H2) = 0.3, P(E/H1) = 0.3, P(E/H2) = 0.6
P (H2 E) 0.18
P (H2/E) = P(E) = 0.39 = 0.46
Example 6.50: From a well shuffled pack of 52 cards 8 cards are selected at random. Find the probability
that, out of 8 cards, ( i ) all are of same colour, ( ii ) 4 are black and 4 are red cards , ( iii) 5 are honours
and 3 are other cards, ( iv ) 5 are spades and 3 are diamonds.
Solution: From 52 cards 8 cards can be taken in 52C8 ways. All these ways are mutually exclusive,
exhaustive and equally likely, i.e,
52 ! 44 ! 48 49 50 51 52
n= = =752538150
8! (52 8) ! 44 ! (1 2 3 4 5 6 7 8 (44 !)
( i ) Let A = all cards are of same colour. There are 13 cards of each colour and cards are of 4 colours.
From 13 cards 8 can be taken in 13C8 ways = 1287 ways. As there are cards of 4 colours, A can occur in m
= 4 x 1287 ways= 5148 ways. Hence
m 5148 4 13 C
P(A)= 0.00000684 [ P( A )= 52 8 ]
n 752538150 C8
( ii ) Let B = 4 are red and 4 are black cards. There are 26 red and 26 black cards in a pack. B can occur in
m = 26C4 x 26C4 ways. Hence
m 26
C 4 26 C 4
P(B)= 52 =0.297.
n C8
( iii ) Let C = 5 are honours cards and 3 are other cards . These 3 cards can be taken from the remaining (
52 – 5 ) = 47 cards in 47 C3 ways . Therefore, C can occur in m = 4 x 5C5 x 47C3 ways. Hence
( iv ) Let D = Five cards are spades and 3 are diamonds. D can occur in m = 13C5 x 13C3 ways. Hence
m
13
C5 13 C3
P(D)= 52
n C8
Example 6.51: Find the probability that a poker hand will get ( i ) all honours cards, ( ii ) three spades
and 2 hearts, ( iii ) cards of same suit, ( iv ) cards of same colour.
Solution: A poker hand gets 5 cards from 52 cards. It occurs in n = 52C5 ways.
249
( i) Let A be the event that all cards are honours cards in a poker hand. There are 5 honours cards in each
suit and there 4 suits in a packet of 52 cards. Therefore, 5 honours cards can be found in m = 4 x 5C5 = 4
ways. Hence
m 4
P(A)= 52
n C5
( ii ) Let B be the event that ,out of 5 cards, 3 are spades and 3 are hearts. This can occur in
m
13
C3 13 C 2
P(B)= 52
n C5
( iii ) Let C be the event that all 5 cards are of same suit. There are 4 suits and in each suit there are 13
13
cards. From 13 cards 5 cards can be distributed in
C 5 ways. This is true for one suit. But it can occur in
4 ways for 4 suits. Hence m = 4 x 13C5 . Therefore,
m 4 13 C5
P(C)= 52
n C5
( iv ) Let D be the event that all 5 cards are of same colour. There are 26 cards of each colour and cards
are of two colours. Hence D can occur in m = 2 x 26C5 ways. Therefore,
m 2 26 C 5
P(D)= 52
n C5
Example 6.52:From a pack of 52 cards 4 cards are taken at random. Find the probability that ( i ) all 4
cards are honours cards, ( ii ) one is ace,one is king, one is queen and one is jack, ( iii ) 2 are honours
cards and 2 are cards other than honours cards.
( i ) Let A be the event that all 4 cards are honours cards. This can occur in m = 4 x 5C4 = 20 ways as there
are 5 honours cards. Hence
m 20
P(A)=
n 270725
( ii ) Let B be the event that one is king, one is queen, one is ace and one is jack. There are 4 kings, 4
queens, 4 jacks and 4 aces. Hence B can occur in m = 4C1 x 4C1 x 4C1 x 4C1 = 256 ways. Hence
m 256
P(B)=
n 270725
250
( iii ) Let C be the event that there are 2 honours cards and 2 cards other than honours cards. This can
occur in m = 5C2 x 45C2 = 9900. Hence
Example 6.53: In a packet there are 5 bulbs of 40 watt, 7 bulbs of 60 watt and 9 bulbs of 100 watt. Three
bulbs are taken at random. Find the probability that one bulb of each type is selected. If two bulbs are
selected at random, what is the probability that one bulb of 60 watt and one bulb of 100 watt are selected?
In the packet there are 5 + 7 + 9 = 21 bulbs. From these 21 bulbs 3 bulbs can be taken in n = 21C3 = 1330
ways. Let A be the event that bulbs of each type are selected. The event A can occur in m = 5C1 x 7C1 x
9
C1= 315 ways. Hence
m 315
P(A)= = 0.23684
n 1330
Second part: From 21 bulbs 2 bulbs can be taken in 21C2= 210 ways. Let B be the event that one bulb is of
60 watt and one bulb is of 100 watt. The event B can occur in m = 7C1 x 9C1=63
ways. Hence
m 63
P(B)= = 0.30.
n 210
Example 6.54: In a packet there are 5 colgate and 7 pepsodent paste tubes. Three tubes are selected two
times at random. Find the probability in the first selection 3 colgate paste tubes and in the second
selection 3 tubes of pepsodent paste are found, if
Solution: ( a ) There are 5 + 7 = 12 tubes. From these 12 tubes 3 tubes can be selected in n= 12C3 = 210
ways. During first selection 3 colgate tubes can be selected in m =5C3 = 10 ways. Let this event of
selection of 3 colgate tubes in first selection be A . Then
m 10
P( A)
n 210
Let B be the event that 3 pepsodent tubes are selected at the second selection. B can occur in 7C3= 35
ways. As the tubes of first selection is replaced before the second selection, the event B is independent of
the event A . Here
m 35
P(B)=
n 210
251
10 35
We need P ( AB ). Here P ( AB ) = P ( A ) P( B )= =0.0079365.
210 210
( b ) After first selection the tubes are not replaced in the packet. Second selection depends on first
selection. During second selection 3 tubes are selected from ( 12 – 3 ) = 9 tubes. It can be done in 9C3 =
84 ways. The event B depends on the event A. Here
35
P ( B /A ) =
84
10 35
Thus, P ( A B ) = P(A ) P B /A) = = 0.019.
210 84
Example 6.55: In an office there 5 economists, 3 statisticians and 2 persons who are M.A. in English.
From these 10 persons a committee of 5 members is to be formed. Find the probability that the committee
( i ) is formed with 2 economists, 2 statisticians and one person who has an M.A. degree, ( ii ) is formed
with 3 economists and 2 persons having M.A. degree, and ( iii ) is formed with 3 economists and 2
statisticians.
Solution: There are 5 + 3 + 2 = 10 employees in the office. A committee of 5 can be fomed in n = 10C5 =
252 ways.
( i ) Let A be the event that in the committee there will be 2 statisticians, 2 economists and one having
M.A. degree in English. The event A can occur in 5C2 x 3C2 x 2C1 = 60 ways. Hence P(
m 60
A)= = 0.238
n 252
( ii ) Let B be the event that the committee is formed with 3 economists and 2 persons having M.A.
degree in English. The event B can occur in 5C3 x 2C2 = 10 ways. Hence
m 10
P(B)= =0.04.
n 252
( iii ) Let C be the event that the committee is formed with 3 economists and 2 statisticians. The event C
can occur in 5C3 x 3C2 = 30 ways. Hence
m 30
P(C)= = 0.119.
n 252
Example 6 56: In a rural cottege industry there are 50 machines . Among these machines 35 are
electrical and 15 are hand driven. Ten electrical machines and 5 hand driven machines are old. The
remaining machines are new. One machine is selected at random. Find the probability that the selected
one is ( i ) either electrical or old, ( ii ) hand driven under the condition that it is old.
Solution: Let A = electrical machine, A = hand driven machine, B = old machine, and
252
B = new machine. The machines are classified as follows:
B B Total
A A B =10 35
AB =
25
A A B5 A B 10 15
Total 15 35 50
35 15 10
(i) P ( A or B ) = P ( A ) + P ( B ) – P ( A B ) = = 0.8
50 50 50
P( A B) 5 / 50 1
( ii ) P ( A / B ) = = 0.33
P( B) 15 / 50 3
Example 6.57: In a factory of razor blades there are two plants X and Y . The plant X produces 70% and
plant Y produces 30% blades. In each day 2% products are found defectives. One razor blade is selected
at random. Find the probability that the selected one is ( i ) either the product of plant X or non-defective,
( ii ) defective under the condition that it is the product of plant Y.
B : Defective , B : Non-deective
The classified information along with probability of the products of the factory are as follows: Here A
and B are independent events and hence P ( AB ) = P( A ) P( B ) = 0.7 x 0.2 = 0.014
B B Total
253
P ( A B ) 0.006
( ii ) P ( B / A ) = 0.02.
P ( A) 0.300
Example 6.58: From a well shuffled pack of 52 cards one card is selected at random. Find the probability
that the seleced card is ( i ) an ace, ( ii ) a spade, ( iii ) a spade or an ace, ( iv ) an ace under the condition
that it is a spade.
m 4 1
P(B)=
n 52 13
m 13
P(A)= = 0.25
n 52
13 4 1 16
( iii ) P ( A or B ) = P ( A ) + P ( B ) – P ( A B ) =
52 52 52 52
P ( AB ) 1 / 52 1
( iv ) P(B/A )= =
P ( A 13 / 52 13
254
Supplementary Problems
1. (a) Write down examples of random experiment, sample space, mutually exclusive outcomes, event,
equally likely outcomes and exhaustive outcomes.
(b) In an office there are 6 Samsung and 4 ACER computers. Two computers are selected at
random. Find the probability that the selected computers are ACER. One of the selected computer is
Samsung and another one is ACER.
2 8
Ans. 15 , 15 .
2. (a) State the classical definition of probability and explain its limitations. Discuss the importance of
probability in taking decision in business.
(b) In an office there are 8 Samsung and 4 ACER computers. Two are taken one after
another without replacement. Find the probability that (i) both are Samsung computers, (ii) one is
Samsung and another is ACER computer.
14 16
Ans. 33 , 33 .
(c) The odds against a certain event are 5 to 2 and the odds infavour of another event are 6 to
5. Find the probability that at least one of the events will occur.
52
Ans. 77 .
4. (a) State addition law of probability for two events A, and B. Also, define conditional probability.
(b) Three programmers A, B and C are asked to try to develop a computer programme
related to accounting. They work independently and previous knowledge gives the idea that their chances
1 1 1
of success are 4 , 5 and 6 , respectively. Find the probability that the programme will be developed.
1
Ans. 2
5. (a) Define independent and mutually exclusive events. Can two events be mutually exclusive and
independent simultaneously? Justify your answer with an example.
255
(b) Among 12 persons, five are women and 7 are men. One person is chosen at random to
speak at a meeting first and then a second person is chosen. If the second person is a woman, what is the
probability that the first person is also a woman?
2
Ans. 9
6. (a) Critically examine the different approaches to probability. Define independent events and
marginal probability.
(b) In a survey of 100 readers, it is observed that 30 read English paper, 40 read Bengali
paper and 15 read both the papers. Find the probability that a reader at least read one of the paper. Ans.
0.55.
7. (a) Define independent events. State and prove addition law of probability.
(b) Three group of workers consist of 4 men and 2 women, 3 men and 3 women, and 2 men
and 4 women. One worker is selected at random from one group. Find the probability that the group
selected consists of 2 men and 1 women.
7
Ans. 21
8. (a) Explain the difference. (i) Simple probability and conditional probability. (ii) independent events
and mutually exclusive events.
(b) An article manufactured by a company consists of two parts A and B. In the process of
manufacture of part A, 9 out of 100 are defective. The same probability for B is 0.05. Find the probability
that the assembled article will not be defective.
Ans. 0.8645.
9. (a) Sate and prove the addition and multiplication rule of probability.
(b) There are 3 persons A, B and C who are contesting for a post. The probabilities of
appointment of the persons are 0.30, 0.25, and 0.40, respectively. The probabilities that the development
work will be promoted by these persons if they are appointed are 0.4, 0.5 and 0.6, respectively. Find the
probability that the development work will be promoted by the newly appointed person.
Ans. 0.485.
10. (a) Define conditional probability and independent events with example. State and prove Bayes
theorem.
(b) A company produces juice packets using machines A and B. Machine A produces 1000
packets per hour and machine B produces 1200 packets per hour. Ten packets produced by machine A
and 5 packets produced by machine B are found defective. A packet is randomly selected from the
production of an hour. Find the probability that the selected packet is produced by machine A or it is
defective.
256
201
Ans. 440
11. (a) Define random experiment, sample space, mutually exclusive events with examples. If A and B
are two mutually exclusive events, show that,
P(A)
P [A/(AB)] = P(A) + P(B) , provided P(AB) ≠ 0
31
Ans. 60
12. (a) Define probability. Mention the limitations of classical definition of probability.
(b) A company has advertised the post of managers. For the post 5 males and 3 females have
applied. Two managers will be appointed. Find the probability that the selected candidates are (i) males,
(ii) one is male and one is female.
13. (a) Define a priori and posterior probabilities. State and prove Bayes theorem.
(b) A factory produces electric bulbs by 3 plants. Plant–1 produces 3000 units, plant-2
produces 4000 units and plant-3 produces 3000 units per day. Past experience provides information that 2
per cent product of plant-1, 3 per cent of plant-2 and 1 per cent product of plant-3 are defectives. One
defective bulb is identified. Find the probability that it is the product of plant-1,
2
Ans. 7
14. (a) Explain with examples random experiment, sample space, event, equally likely outcomes,
exhaustive outcomes. State addition law of probability for 3 events.
(b) A publisher sends 40 books of 4 titles to a library for sell. There are 10 books of each title
and one out of 10 books is complimentary copy. A book is selected from the lot. Find the probability that
(i) the book is of first title or a complimentary copy, (ii) the book is a complimentary copy if it is of
second title.
13 1
Ans. (i) 40 , (ii) 10
15. (a) Define marginal probability, conditional probability and independent events. For two events A
and B, prove that
–
P (A /B) = 1 – P(A/B)
257
(b) A company learned that inventory shortages were associated with a loss of good will with
a probability 0.10. The company also knew that a loss of good will from all causes occurred with a
probability of 0.15. What is the probability of an inventory shortage, given a loss of good will?
Ans. 0.67.
(b) Ten per cent of the employees of a company have been to public school. Of these, 30%
hold administrative posts. Of those who have not been to public school, 30% of them hold administrative
posts. If an employee is selected at random and found that the selected one hold an administrative post,
find the probability that he was educated in a public school.
Ans. 0.1.
17. Two persons A and B have decided to open a departmental store on first of January of a year. The
probability that A will continue to run the store for one complete year is 0.6 and the same probability for
B is 0.7. They can close the store at any time independently. Find the probability that (i) any one of A and
B can discontinue the business before completion of the year, (ii) both of them will discontinue to run the
store.
18. (a) Two persons A and B start a game on the understanding that he will win the game who will get
head first by tossing an unbiased coin. If starts the game, what is his chance of winning?
Ans. 2/3.
(b) In a mobile operator‟s office there are 6 electrical and electronic engineers and 4
computer engineers. Out of 6 electrical engineers and out of 4 computer engineers 2 have experience of
20 years. One is selected from the office. Find the probability that (i) the selected one is either an
electrical and electronic engineer or is experienced of 20 years, (ii) the selected one is computer engineer
given that he is experienced of 20 years.
4 1
Ans. (i) 5 , (ii) 2 .
19. In a departmental store there are 5 sales persons. They are A, B, C, D and E. Their service
experiences are for 6, 4, 3, 5 and 2 years, respectively. One of them will be promoted and promotion will
be based on proportion of service experience. Find the probability that (i) one will be promoted who have
experience of 4 years or more, (ii) one will be promoted who has experience of less than 4 years.
3 1
Ans. (i) 4 , (ii) 4 .
20. (a) From 20 applicants for a post 3 will be selected. What is the probability that, of 3 applicants (i) a
special one is selected, (ii) special one will not be selected.
258
3 17
Ans. (i) 20 , (ii) 20 .
(b) In a petrol pump there are 3 petrol distributing points and in each point there are 2 petrol
supply pipes. Find the probability that a car will get petrol using any of the pipes.
Ans. 1/6.
21. (a) In an office there are 20 male and 15 female employees. Four of them will be given bonus. Find
the probability that bonus will be given to (i) 4 male employees, (ii) 2 male and 2 female employees.
(b) In a store there are 15 casio fx-80, 20 casio fx-100 and 40 casio. HL-122 calculators. One
calculator is selected at random. Find the probability that the selected one is either casio fx-80 or casio
HL-122.
11
Ans. 15
22. In a family of 3 children where the probability of a child to be a boy or to be a girl are equally
likely. The birth order of the children is not known. Find the probability that (i) two are boys under the
condition that elder one is a boy, (ii) there is no boy, (iii) at least one is a boy.
2 1 7
Ans. (i) 3 , (ii) 8 , (iii) 8 .
23. Two fair dice are thrown once. Find the probability that (i) first die shows dots of even numbers,
(ii) second die shows dots more than 4, (iii) sum of the number of dots in the upper faces of the die is 9,
(iv) both the dice will show same dots, (v) sum of the number of dots is 6 under the condition that first die
shows dot 3.
1 1 1 1 1
Ans. (i) 2 , (ii) 3 , (iii) 9 , (iv) 6 , (v) 6 .
24. In a box there are 10 red, 6 blue and 4 white balls. Four balls are taken at random. Find the
probability of the events. (i) all 4 are red balls, (ii) all 4 are white balls, (iii) 2 are red and 2 are blue balls,
(iv) at least one is white ball, (v) 2 are red, 1 is blue and 1 is white ball.
14 1 45 605 72
Ans. (i) 323 , (ii) 4845 , (iii) 323 , (iv) 969 , (v) 323 .
25. A machine has 3 components A, B and C. The machine runs well if all 3 components run well.
But during work the components A, B and C will not work properly with probability 0.12, 0.15 and 0.20,
respectively. Find the probability that the machine will not work in a day.
Ans. 0.4016.
259
26. Fifteen per cent workers of an industry are unskilled and 25% workers are specially trained. Five
per cent workers are unskilled but specially trained. One worker is selected at random. Find the
probability that the selected one is unskilled when the selection is made from the specially trained
workers.
Ans. 0.20.
27. A company produces an item using machine A and B. Machine A produces 60% of the item and
machine B produces 40% of the item. Eighty per cent of machine A and 70% of machine B are non-
defective. If an item is selected at random, what is the probability that it is non-defective. If the item is
non-defective, what is the probability that it is produced by A.
28. The labourers of an industry are skilled and unskilled. Seventy per cent are unskilled. Seventy per cent
of skilled labourers and 30% of unskilled labourers get special bonus from the owner of the industry. A
labourer is selected at random for bonus. Find the probability that the selected one is skilled labourer.
Ans. 0.50.
260
Chapter VII
Compounding
7.1 Introduction
Compounding is the process of the exponential increase in the value of an investment due to earning
interest on both principal and accumulated interest . The compound interest formula is given as follows
Then Pn = P0 ( 1 + r )n
1
Pn n
r = 1
P0
Here r is the constant rate of interest for every year. In practice, the interest may be compounded at
different rates in each time period. Let the successive rates are r1 , r2 , …….., rn . Then, we have
Pn = P0 ( 1 + r1 ) ( 1 + r2 ) …….. ( 1 + rn )
Compounding is often referred to as magic, because it is one of the most fundamental ways to build
wealth , yet takes the least amount of effort. Given time, earning interest on interest can exponentially
grow wealth.
Investors should also note that the rate of compounding may be increased or decreased, depending on
how often the interest amount is calculated and paid. The shorter the interval between interest calculator,
the faster interest will accrue and vice versa. Thus, an account which calculate and pays interest on a daily
basis will grow faster than the same account calculating interest on a monthly basis.
Due to compounding any investment is increased as its interest is added to the principal for calculation of
future interest. As investment base gets larger , it has the potential to grow faster. And the longer the
money is invested , the more gain occurs from compounding. For example, let us consider that a person
invested Tk. 10000.00 at a compound rate of interest 9.5% per year and the money is invested for 10
years. Due to compound interest the amount is reinvested annually. Then after 10 years the principal
becomes
On the other hand, if Tk.10000.00 is invested at the rate 9.5% interest per year at a simple interest, the
invested money will transform to Tk.19500.00.
261
Compounding Equity
In case of equity investments or growth assets , the market returns differ every year. Compounding can
happen on two fronts :
( 1) Price change itself is automatic compounding and if your reinvestment dividends or earnings they
also benefit from compounding . However, keep in mind that unlike the compounding interest on a bond,
debenture or fixed deposit ; returns in equity and growth assets are not linear. The invested money will
not be returned similarly every year.
If the invested money is losing value , just as price compounds gain , a fall in prices can compound the
losses. As a result, one can lose a lot of the capital value very fast if price is falling.. This means, any one
needs to pay attention to quality of the equity stock or portfolio or the asset what is going to buy.
(2) Given the non-linear nature of the assets, any capital need to be invested for long periods to get
benefit from compounding.
The example what has been considered above in the beginning of this section to calculate the compound
interest it is seen that the capital of Tk.10000.00 has been increased to Tk.24782.28. The compound
interest becomes :
Compound Interest = Total amount received – Invested capital = Tk.24782.24 – Tk.10000.00 = 14782.28
On the other hand on the basis of simple interest of 9.5% per annum, we see that the simple interest
becomes
Capital year r
Simple Interest = , where r is the interest rate in percentage . Therefore, we have
100
10000 10 9.5
Simple Interest = = 9500.00
100
There is a difference between compound interest and simple interest . This difference is given by
For this example the breakdown of the interest calculated for every year is as follows:
262
Capital after 3 years 13129.57 Interest during 9th year 1963.56
Interest during fourth 1247.31 Capital after 9 years 22632.63
year
Capital after 4 years 14376.88 Interest during 10th year 2150.10
Interest during 5th year 1368.80 Capital after 10 years 24782.73
Capital after 5 years 15742.68 Compound interest for 14782.73
the 10 years period
We have considered the compound interest when r is the rate of interest per annum. But interest may be
payable daily, monthly, quarterly, half-yearly. The formula for final payment if interest is payable half-
yearly is given by
r 2n
Pn = P0 ( 1 + ) ,where r is the interest on unit sum for one year.
2
r 4n
Pn = P0 ( 1 + ) ,
4
r 12n r 365 n
Pn = P0 ( 1 + ) and Pn = P0 ( 1 + ) , respectively.
12 365
For example, an investment of Tk.1000.00 at the compound interest 8% when interest is payable half-
yearly is calculated as follows:
r 2n
Pn = P0 ( 1 + ) = 1000 ( 1+ 0.04 )2 = 1081.6. Here n= 1 and r = 0.08
2
We have discussed the compound interest rate when rate is increased. But rate may be decreased also. In
that case r will be replaced by – r. Thus,
263
Pn = P0 ( 1 – r )n.
Example 7.1: A man invested Tk.5000.00 at the rate of 8% interest per annum for 5 years. What would
be his capital (a) under simple interest scheme, (b) under compound interest scheme ?
Example 7.2: A man invested Tk.6000.00 for 5 years and secured Tk. 8500.00 Find the average rate of
interest and calculate the capital after 10 years assuming the scheme of compound interest.
1
1
Pn
5
1
n 8500
1 1.0721 1 0.0721
Solution: The rate of interest , r = 0
P 6000
Example 7.3: A man invested Tk.60000.00 at a rate of interest 9.5% per annum. After one year the
amount received is reinvested at a rate of interest 10% per annum. After second year the total amount
received is reinvested at a rate of interest 11% per annum. Find the accumulated money after third year.
Also, find the average rate of interest per year.
Solution: The rates of interest are r1 = 0.095, r2 = 0.01 , r3 = 0.11, where r1, r2 and r3 are the rates of
interest of first, second and third year, respectively. Therefore, the accumulated capital after 3 years is
The invested taka , percentage of increase ( x ) in different years and the average percentage of increase
are shown below:
264
Average percentage of increase is
1
G = Antilog log X 110.21 where N = 3.
N
The average rate of interest is ( G – 100 ) = 10.21 % .
The initial investment is Tk 60,000.00 . The accumulated amount is 80,219.00 . This amount is 1.336995
times of initial investment after 3 years. The average interest is
Example 7.4 A man invested Tk.5000.00 for 10 years at an interest rate of 8% per annum. Find his
accumulated money after 10 years, if he is sanctioned (a) compound interest rate, (b) simple interest rate .
Solution: (a) The principal amount is , P0 = Tk. 5000.00 r = 0.08, n = 10 years. We need the accumulated
money , Pn, after 10 years. It is calculated by the formula
( b ) The accumulated after 10 years when interest rate is simple is calculate by formula
P0 n r 5000 10 8
Pn = P0 + I = 5000 + I = 5000 + 5000 9000.00 ; Here r = rate of
100 100
P0 n r
interest per annum = 8% and I = = 4000.00.= Interest.
100
Example 7.5 A man has invested Tk 10,000.00 in a business as a partner . After 10 years he decided to
discontinue the business. On understanding, he secured Tk.30,000.00 from his partner. Find the annual
rate of return.
Solution: The invested money was accumulated to Tk.30,000.00. There is an increased amount of
Tk.20,000.00 [ 30,000.00 – 10,000.00 ] in 10 years. The accumulated money is [ 30,000 ÷ 10,000.00 ] = 3
times of the invested money after 10 years. The annual rate of interest is calculated by geometric mean,
this mean is calculated by the formula
Example 7.6 : A person invested some amount of money at compound rate of interest of 8.5% per
annum. After 10 years he received tk.18,500.00. What was his principal amount?
265
Solution : Let Pn be the accumulated amount the person received after 10 years; the rate of interest was, r
= 0.085 and number of years, n = 10 . We need the value of P0 , the invested amount. We know
Pn = P0 ( 1 + r )n . Then
Pn 18500
P0 = = 8182.28 ( approx. )
1 r (1 0.085 )10
n
Example 7.7 : Find the rate of compound interest if a person received Tk.12,000.00 from an investment
of Tk.6000.00 for 10 years of time.
Solution : Let Pn be the accumulated amount the person received after 10 years; the rate of interest was, r
, number of years, n = 10, and P0 = Tk. 6000.00= the invested money. We need the value of r.
We know , Pn = P0 ( 1 + r )n . Then
1
Pn n
0.10
Pn 12000
( 1 + r )n = 1 1 1.1385 1 0.1385 or 13.85%
6000
, or r =
P0 P0
Example 7.8: A person invested Tk.5000.00 at the rate of compound interest 8 percent and received
Tk.15,000.00 as accumulated money after some years. How many years his money was in invested
condition ?
Solution : Given P0 = 5000, Pn = 15,000.00 , r = rate of interest = 0.08 . We need the value of n.
We have , Pn = P0 ( 1 + r )n . Then
Pn Pn
( 1 + r )n = , or n log ( 1 + r ) = log , or n log ( 1 + r ) = log Pn – logP0 . Therefore,
P0 P0
n log ( 1 + 0.08 ) = log 15000 – log 5000 or n x 0.03342 = 4.17609 – 3.69897 = 0.47712
0.47712
n= = 14.28 years.
0.03342
Example 7.9: A senior citizen invested Tk.5,00,000.00 for 5 years and received Tk.8,25,000.00 after 5
years. The total money is again invested for another 5 years at the same rate of compound interest. What
would be his accumulated money after 10 years ?
Solution : In the first investment period n =5, P0 = 500000, Pn = 8,25,000 . We know that
266
1
Pn n
Pn = P0 ( 1 + r )n, or 1+r = , or r =
P0
1
Pn n
0.20
825000
1 , or r `1 0.10534
P0 500000
and n = 5.
Example 7.10 : A senior citizen invested Tk.100000.00 at a rate of compound interest . After one year his
invested money transformed to Tk.1,08,000.00 . This amount is again invested at a rate of compound
interest and after one year he received Tk.1,12,000.00 . What was the average rate of interest over two
years ?
Solution : Given P0 = 100000, Pn = 108000, n =1. We need the value of r[ rate of interest]. We have,
1 1
Pn n Pn n 108000
Pn = P0 ( 1 + r )n , or 1 + r = , or r = 1 1 0.08 , As n = 1.
P0 0
P 100000
112000
r= 1 0.03704
108000
Let r for first year is r1 = 0.08 and r for second yea is r2 = 0.03704. Now, the average rate of interest is
calculated by geometric mean ( G ) , where
Example 7.11 : A company invested Tk.5,00,000.00 for one year and after one year this became
Tk.5,80,000.00 . This amount was again invested for another one year and after the second year the
invested money became 6,80,000.00 . What is the average rate of interest ? If this rate of interest prevails,
what will be his accumulated money after 10 years ?
Solution : During first year the initial amount was P0 = 500000 . After the end of one year the
accumulated money became Pn = 680000.00, where n = 1 . Let r1 be the rate of interest during first year,
where
267
1
Pn n
580000
r1 = `1 1 0.16 , as n = 1 . In a similar way , the rate of interest
P0 500000
during second year is , say r2 , and it is calculated by
1
Pn n
680000
r2 = 1 1 0.1724 , Here also n= 1. The capital in the beginning of
P0 580000
second year is P0 = 580000, and accumulated money after second year is Pn = 680000 .
The average rate of interest is calculated by geometric mean, this mean is measured by
If this rate prevails , then after another ( 10 -2 ) = 8 years the accumulated money will be
Example 7.12 : In an industrial unit there are 3000 employees . Every year the authority of the industry
appoints 2% new employees. Again, every year 1% employees are retired. If the current rate of
employment and retirement prevails, what would be the strength of employees in the industry after 10
years ?
Solution : The employees are increased at a rate of r = ( 2 – 1 ) = 1 % . Therefore, the strength of the
employees after 10 years will be
Example 7.13 : A company invested Tk. 50,000,00.00 as capital to purchase machineries . The invested
capital will be diminished due to depreciation . The depreciation cost is 25% in first year, 20% in second
year and 15% in successive years. Find the percentage of depreciation after 8 years. Also, find the scrap
value of the machineries after 8 years .
Solution : Let ri be the diminishing value of the machineries in i-th year ( i = 1, 2, 3, …….,8).Then the
average diminishing value is given by
r r r3 r4 r r 6 r7 r8 8
1
1 2
r=
=( 0.75 x 0.80 x 0.85 x 0.85 x 0.85 x 0.85 x 0.85 x 0.85)0.125 = 0.8304 or 83.04%
268
Example7.14 : A company invested some money which after 10 years became double at compound
interest rate. Find the rate of interest. If the rate prevails for future, at what time period the initial
investment will be trebled ?
Solution : Let P0 be the initial invested money . This amount became 2P0 after 10 years at a compound
interest rate, say r . Then, we have
2P0 = P0 ( 1 + r)10
2 = ( 1 + r ) 10
20.10 = 1 + r
Let N be the time period for P0 to become 3P0 at the rate 0.07177. Thus, we can write
3 P0 = P0 ( 1 + r )N
3 = ( 1 + 0.07177 )N
or N ( 0.0301 ) = 0.47712
0.47712
or N = 0.0301 = 15.8 5 or n = 16 years ( approx.)
Example 7.15 : At a compound rate of interest per annum the amount of Tk.10,000.00 became
Tk.15,000.00 in 3 years. Find the rate of interest.
We know, Pn = P0 ( 1 + r )n
or 150000 = 10000 ( 1 + r )3
or 1.5 = ( 1 + r )3
1
3
or r = (1.5) - 1 = 1.1447 – 1 = 0.1447 or 14.47%.
Example 7 16 : Find the compound interest rate on Tk.10,000.00 for 5 years at 8% per annum.
269
Solution : The initial capital is P0 = Tk. 10000.00. After one year it became
1 1
Pn 5
1
13604 .89 5
1
r= 0
P 10000 = 1.0635 – 1 =
0.0635
or 6.35%
Example 7.17 : Find the compound interest and rate of interest if a person invests Tk.5000.00 at the rate
of 8% interest when payment is made half-yearly .
Solution:
408
Therefore, the compound rate of interest is, 5000 = 0.0816 0r 8.16%.
270
Example 7.18 : A person receives Tk 8990.00 after 3 years. He invested the money at the rate of 9%
compound interest per annum. Find his capital.
Pn 8990
6942
P0 = (1 r )
n
1 0.09 )3
Example 7.19 : The compound interest rate is 8.5% per annum. A man needs Tk.10000.00 after 5 years.
How much money should he invest now?
Solution : Given Pn = 10000 , n = 5 , r= 0.085. We need P0 . We have already seen that [ Example 7.18]
Pn 10000
6650 .45
P0 =
1 r n
1 0.085 5
Example 7.20 :A machine is bought by an amount of Tk.50000.00 . Its depreciation cost is 10% per
annum. After 10 years what will be its scrap value ?
Example 7.21 : In buying a machine the initial investment was k.1,00,000.00 . The depreciation cost per
annum is 10% . The scrap value of the machine was Tk.40,000.00 . What was the effective life time of the
machine ?
Solution : The depreciation cost is at compound rate . The scrap value Pn is calculated as follows :
0.04= 0.9n
-n x 0.04576 = -0.39794
271
0.39794
n = 0.04576 = 8.69 or 9 years ( approx.).
Example 7.22: A man invested Tk.4000.00 at a certain compound interest rate per annum. After 4 years
the invested money became Tk.9000.00. What will be the amount after two years hence ?
We know that Pn = P0 ( 1 + r )n
9000 = 4000 ( 1 + r )4
9 = 4 ( 1 + r )4
9
(1 + r )4 = 4
3
( 1 + r )2 = 2 = 1.5
Again, we the value of Pn for future time period n = 2 years when P0 = 9000. This is given by
Pn = P0 ( 1 + r )2
We have ( 1 + r )2 = 1.5 ,or 1+r= 1.5 , or r = 1.2247 – 1 = 0.2247. Therefore , the rate of interest is
r = 22.47 %.
Example 7.23: A person invested some money to two different savings schemes at the rate of 15%
interest per annum. In one scheme the rate of interest is simple and in another scheme the rate of interest
is compound. After 4 years the person observed that the difference of income in two different schemes is
Tk.2000.00 . Find the invested amount.
Solution : Let us consider that Tk.100 is invested . Then by simple interest rate the interest becomes
I = Capital x time period x interest rate, where the rate of interest is r = 0.15.
= 100 x 4 x 0.15
= 60.
272
Pn = Po ( 1 + r )n . Here P0 = 100 , n =4 and r = 0.15.
The compound interest is ( 174.90 – 100 )= 74.90 . The difference between compound interest and simple
interest Is ( 74.90 – 60.00 ) = 14.90.
100
For one difference the invested money is 14.90 . Therefore ,for the difference of 2000 the invested
100
2000
money is 14.90 = 13,42.00
Example 7.24: Find the time period needed for an amount of capital to be 1.5 times at the compound
interest rate of 8% .
Solution : Let Pn be the amount which is 1.5 times of the invested money P0 .Then , we have
0.17609= n x 0.03342,
0.17609
n = 0.03342 = 5.27 years.
Example 7.25 : Find the difference between simple interest and compound interest if a person invests
Tk.10,000.00 for 5 years at the rate of 10%.
Solution: Here P0 = 10000 , r =0.10 , n = 5 . Now, if the interest rate is compound, then the accumulated
money after n = years is
The simple interest = P0 x r x n = 10000 x 0.10 x 5 = 5000 . The difference in the amount of simple
interest and compound interest = 6105.10 – 5000 = 1105.10.
Example 7.26 : A person invested Tk. 10000.00 for 5 years at a compound interest rate of 10 % per
annum. But interest was calculated quarterly . What was his accumulated money ?
Solution: Here P0 = 10000, r = 0.10, n= 5 years . As interest is calculated quarterly, the accumulated
money Pn is calculated by the formula
273
r 0.10
)
Pn = P0 ( 1 + 4 4n = 10000 ( 1 + 4 )4 x 5 = 16386.16 .
Example 7.27 : A person invested Tk.5000.00 for 10 years at a rate of compound interest 12% per
annum. But interest is calculated monthly . Find the accumulated money after 10 years.
Solution: Given P0 = 5000, r = 0.12 , n = 10 . We need Pn when interest is calculated monthly. The
calculation is done by the formula
r 0.12
Pn = P0 ( 1 + 12 ) 12 xn
= 5000 ( 1 + 12 )12 x 10 = 16501.93.
Example 7.28: A man purchased a machine at a price of Tk.100000.00. On use the rate of depreciation is
10%. What will be the scrap value of the machine after 10 years ?
Example 7.29: A machine is purchased at Tk.100000.00. On use the money is depreciated by 10% per
annum. For the first year the maintenance cost is Tk.1000.00. This cost is increased by 5% every year.
After some years the machine has been sold at Tk.30000.00. To avoid the loss what should be the
minimum annual return from the machine ?
0.3 = .9n
-0.52288 = -0.04576
0.52288
n = 0.04576 = 11.42 years or 11 years (approx.)
1000 + 1050 + 1052.50 +1105.12 + 1160.38 +1218.40 +1279.32 +1343.29 +1410.45 +1480.97 +1555.02
= 13655.45.
274
Therefore, per year the minimum return should be
83655 .45
11.42 = 7325.35.
Example 7.30 :In how many years the population of an area will be doubled when the growth rate is
1.5% per annum ?
Solution : Let P be the current population . It will be 2P when growth rate is r =0.015 . Here we can write
Pn = P0 ( 1 + r )n , where Pn = 2 P0 .
2P0 = P0 ( 1 +0.015 )n
2 = ( 1.015 )n
0.30103 = n x 0.006466
0.30103
n
0.006466 = 46.55 or 47 years ( approx.)
Example 7.31: A couple wanted to save money for their kid when he was born. They saved an amount of
Tk.5000.00. What will be the money at the 20th birth day of the kid when the money was saved at the rate
of 11.5% per annum ? What would be the money if saving could be done quarterly ?
r 0.115
Pn= P0 ( 1 + 4 )4 x n = 50000 ( 1 + 4 )4 x20 = 48,278.96.
Example 7.32: A machine is purchased at a price of Tk.50,000,00.00. The machine depreciates in first
year at the rate of 10%, the depreciation rates in second year, third year, fourth year and fifth year are
12%, 15% , 20% , and 25%, respectively. Find the depreciated value of the machine at the end of fifth
year.
Solution : Given the initial cost of the machine as P0 = 50,000,00 , r1 = 0.10 , r2 = 0.12, r3 = 0.15,
r4 = 0.20 and r5 = 0.25. As these rates are depreciation rates , ri values will be negative for all i = 1,2,3,4,5.
Pn = P 0 ( 1- r1 ) ( 1 - r2 ) ( 1 - r3 ) ( 1 – r4 ) ( 1 – r5 )
275
Example 7.33 : In an area the population size is 50,000,00 . The growth rate is 1.2 % . Find the
population of the area after 10 years.
Example 7.34: A machine can be purchased on down payment of Tk.500000.00 plus Tk.10,000,00.00 at
the end of 3 years. If the money is worth 15% , compounded annually , what should be the cost of the
machine ?
Solution: Let Pn = 10,000,00 be the payable amount after 3 years . Then, the total cost of the machine
will be ( Pn + 5,00,000). The value of Pn will be calculated on the basis of current payable P0 plus the
compound interest of 15% ( r = 0.15), where
Pn = P0 ( 1 + r )n
or P0 = Pn ( 1 + r )- n
= 1000000 ( 1 + 0.15 )- 3
= 657,516. 23
Example 7.35: The population of a town increases by 1.25% annually . In how many years the
population will be increased 50%.
Solution: Let P0 be the population of the study period . After n years this Po will be Pn = 1.50 P0 . Given
the rate of increase r = 0.0125. Then ,
Pn = P0 ( 1 + r )n
1.50 P0 = P0 ( 1 + 0.0125 )n
0.17609 = n x 0.005395
0.17609
Therefore, n = 0.005395 = 32.6 years, 33 years approx.
Example 7.36: An investment depreciates at the rate of 5% of its value at the beginning of a year. If the
investment is for Tk.50000.00 , what is the time period in completed years at the end of which the face
value of the investment will be less than or equal to half of the investment ?
276
Pn = 25000= final achievement, n = time period needed for the
25000.
n 0.022276 0.30103
0.03103
n 13.51
0.22276
Example 7.37:A saving scheme makes the investment double in 6 years. Find the rate of interest
accrued if compounded annually.
Pn = P0 ( 1 + r )n
2000 = 1000 ( 1 + r )6
( 1 +r )6 = 2
r = 20.16667 – 1
= 1.12246 – 1
=0.12246
or 12.22 %
Example 7.38: A man wants to invest Tk.100000.00 at a rate of 10% compound interest. But there are 3
schemes , viz. ( i ) interest accrued annually, ( ii ) interest accrued half-yearly , and ( iii ) interest accrued
quarterly . Which scheme is to be preferred if it is for 5 years?
r
)
Pn = P0 ( 1 + 2 2 n = 100000 ( 1 + 0.05 )2 x5 = 1,62,889.46
277
For quarterly scheme :
r 4n
)
Pn = P0 ( 1 + 4 = 100000 ( 1 + 0.025 )4 x 5 = 1,63,861.64.
Example 7.39: A man invested Tk.10000.00 on 01.01.2010 at a compound interest rate of 8.5% per
annum. He has invested Tk.15000.00 on 01.01.2012 at a compound interest rate 7% per annum, where
interest is payable half-yearly. Find his accumulated money after the end of 2020.
Solution: On 01.01.2010 the invested money is P0 = 10000 at the rate of r = 0.085. If money was in
invested condition for n = 11 year. We have Pn = P11, where
Again , P0 = 15000 was invested on 01.01.2012 at the rate of r = 0.07. This invested money was in
invested condition for 9 years. We need P9 , where
r 2 n
)
Pn = P0 ( 1 + 2 = 15000 ( 1 + 0.035 )2 x 9 = 27,862.34.
Example 7.40: A man can purchase a machine for Tk.500000.00 cash or for Tk.250000.00 as down
payment and Tk. 300000.00 at the end of one year. If money is worth 12% per annum, compounded half-
yearly , which plan should be chosen ?
Solution: The value of Tk.300000.00 can be considered as Pn as it is not paid during purchasing time.
We need the money value of this amount before n = 1 year when money value is decreased by compound
interest rate r = 0.12 payable half-yearly. Therefore, we need P0 , where
r 2 n
)
Pn = P0 ( 1 + 2
300000 = P0 ( 1 + 0.06 )2 x 1
300000
P0 = 1.06 = 266999.00
2
Under the second scheme the price of the machine becomes ( 250000 + 266999 ) = 5,16,999.00. on the
other hand, the price under first scheme is 500000.00. So, first scheme is to be chosen.
Example 7 41: An invested amount of money at compound interest, payable yearly, accumulated to
Tk.30000.00 at the end of second year and to Tk.36000.00 at the end of third year. Find the rate of
interest.
278
Solution: After n = 2 years the accumulated money became Pn = 30000. We need the initial capital
money P0 in the beginning of investment and the rate of interest r. Thus, we have
Pn = P0 ( 1 + r )n
30000 = P0 ( 1 + r )2 …………………………………………………………………. ( i )
Again, at the end of n = 3 years the accumulated money became Pn = 36000. We can write
36000 = P0 ( 1 + r )3 .....................................................................( ii )
Dividing ( ii ) by (I ), we get
36
1 + r = 30 = 1.2.
r = 1.2 – 1 = 0.20
30000
Therefore, P0 = 1.44 = 20,833.33
Example 7.42: Two partners A and B invested Tk.100000.00 in a savings deposit scheme at a rate of
compound interest 9.5% annually. But the final amount what A will get in 2 years the same amount B will
get in 4 years. What are their shares in the initial investment ?
Solution: Let A invested Tk.x and then B invested ( 100000 – x ). After n=2 years A will get PnA
amount at the rate of interest r =0.095 . This PnA is given by
= 119902.5 – 1.199025x
279
119902 .5
x = 2.199025 = 54,525.30 = The share of A.
Example 7.43: A man invested an amount at 12% simple interest rate per annum. Same amount of money
he invested at 11% compound interest rate per annum. From the second investment he received more
interest of Tk.763.10 after 3 years. How much money has he invested in both the schemes?
Solution: Let X be the money invested in the first scheme. Then the interest of the first scheme is
This X amount of money was again invested at 11% compound interest rate for 3 years. Then the interest
is I2 , where
I2 = X ( 1 + 0.11 )3 – X = 0.367631 X
or 0.00763 X = 763.10
763 .1
Therefore, X = 0.007631 = 1,00,000
So, the invested money in both the schemes is 1,00,000.00 + 1,00,000.00 = 2,00,000.00
Example 7.44: The difference between simple interest and compound interest at the same rate for 2 years
on a certain amount of money is 0.0025 times of the invested amount . Find the rate of interest.
Solution: Let x be the amount of invested money. Consider that the rate of interest is r . Thus the simple
interest for two years is
I1 = 2 x r
I2 = x ( 1 + r )2 – x
I2 - I1 = x ( 1 + r )2 – x – 2xr = 0.0025 x
or r2 = 0.0025
r = 0.05
Example 7.45: The difference in the simple interest and the compound interest on a certain amount of
money for 4 years at a rate of 10% is 641.00. What is the amount of invested money?
280
Solution: Let X be the invested money. Given r =0.10, n =4. Then the simple interest is
I1 = X x time x r = 0.40 X
I2 = X ( 1 + 0.10 )4 - X
or 0.0641 X = 641.0
641
Therefore, X = 0.0641 = 10,000.00
281
Chapter VIII
Annuity
8.1 Definition
An annuity is a financial product that pays out a fixed stream of payments to an individual, primarily used
as an income stream for retirees . Annuities are created and sold by financial institutions , which accept
and invest funds from individuals and then , upon annuitization , issue a stream of payments at a later
point in time. The period of time when an annuity is being funded and before payments begin is referred
to as the accumulation phase. Once payments commence , the contract is in the annuitization phase.
Here annuitization is the process of converting an annuity investment into a series of periodic income
payments. Annuities may be annuitized for a specific period for the life of the annuitants. Annuity
payments may only be maid to the annuitant and the surviving spouse in a joint life arrangement.
Annuitants can arrange for beneficiaries to receive a portion of the annuity balance upon their death.
Different annuities are ( i ) Life Annuity, ( ii ) Income Annuity, ( iii ) Wraparound Annuity, and ( iv )
Substand Health Annuity.
A life annuity is an insurance product that features predetermined periodic payment amount until the
death of the annuitant. They are commonly used to provide a guaranteed income in retirement that cannot
be outlived. Typically, the annuitants pay into the annuity on a periodic basis when he or she is still
working. Once funded and enacted, the annuity makes periodic payouts to the annuitant, usually monthly,
providing a reliable source of income. When a triggering event occurs, such as death, the periodic
payments from the annuity usually cease, though they may continue to pay out depending on o
Life annuities are essentially longevity insurance, as the risk of outliving one‟s savings is passed on to the
annuity issuer. These are used as a payment method for lottery winners and in structured settlements. One
of the form of life annuity is the definite benefit pension plans. It is based on an employee‟s salary , and
length of service.
The life annuity is funded by many retired persons to meet their recurring expenditure on housing, living,
health care, insurance premium or medical expenses. Annuities pay a benefit every month, but can also
pay quarterly, annually, or semi-annually depending on the needs of the annuitant. This type of annuity
pays a guaranteed income. This income is not indexed to inflation.
( b ) Guaranteed Annuity : It pays out for a certain period and payment is continued to a beneficiary
282
( c ) Fixed Annuity : It pays out a fixed percentage to the beneficiary.
( d ) Variable Annuity: The payment is based on the performance of a basket of investments or an index.
( e ) Joint Annuity: It pays until both spouse die. In case of death of first spouse it pays out sometimes at
a reduced rate.
Amount of an annuity is the total of all the installments left unpaid together with the compound interest of
each payment for the period it remains unpaid. For the calculation of the amount, let us consider that
Then Pn is measured by
P
[ (1 r ) n 1]
Pn = r ,
= P ( 1 + r )n – 1 + P ( 1 + r )n – 2 + ………. + P ( 1 + r )n – ( n – 1 )+ P
1
This Pn is a geometric progression [ G.P.] with common ratio [ C.R.] 1 r . Therefore,
n
1
1
1 r
1
1
n–1 1 r
Pn = P ( 1 +r )
On simplification, it gives
P
[(1 r ) n 1]
Pn r
In the above calculation it is assumed that payments are made annually. But payment may be made
quarterly, monthly or half-yearly. Let us assume that payment is made q times in a year. Then the
periodic payment of P is ( P/ q ) and if r is the rate of interest per unit per annum , then the amount Pn
283
nq
P r
[1
Pn =
r q - 1 ]
n4
4P r
[ 1 1]
Pn = r 4
12n
12 P r
[ 1 1]
Pn = r 12
If q =2 ( half-yearly ) , then
2n
2P r
[1 1]
Pn = r 2
Example 8.1: A man decided to deposit Tk.5000.00 at the end of each year in a bank. The bank pays 12%
compound interest per year for such a deposit. The man had in his mind to get the money back after 10
years without any installments. Find the total accumulated money after 10 years.
P
Pn = r [ ( 1 + r )n – 1 ]
5000
= 0.12 [ ( 1 + 0.12 )10 – 1 ]
= 87,743. 68
Example 8.2: A man invests Tk. 10,000.00 to a savings plan which offer 11% compound interest per
annum. The amount is deposited every year. The man wants his amount to be accumulated for 8 years.
What will be his accumulated money ? What will be his accumulated money after 2 years of his last
installment ?
Solution: Let Pn be the accumulated money after n = 8 years when r = 0.11. The invested money is
P = 10000. Then
284
[1 r 1]
P
Pn
n
r
10000
[(1 0.11)8 1]
0.11
10000
1.30454
0.11
1,18,594 .34
This amount will be increased for another two years at a compound interest rate r = 0.11. Then, after two
years the payment will be
Pn = P ( 1 +r )2
= 118594 ( 1 + 0.11 )2
= 1,46,120.09.
Example 8.3: Find the amount of immediate annuity of Tk.1000.00 per annum left unpaid for 12 years
allowing compound interest rate 12% per annum.
[1 r 1]
P
Pn
n
r
1000
[(1 0.12)12 1]
0.12
24,133 .13
Example 8.4: A man deposits Tk. 2000.00 every year in a savings scheme which allows 12% compound
interest. What will be the accumulated money after 10 years?
[1 r 1]
P
Pn
n
r
2000
[(1 0.12)10 1]
0.12
35,097 .48
Example 8.5: A man invests Tk.2000.00 at the end of each year in a savings account which allows 10%
compound interest. What will be the value of the deposits after 8th installment? Find the accumulated
money one year after 8th installment.
285
[1 r 1]
P
Pn
n
r
2000
[(1 0.10)8 1]
0.10
22,871 .78
After another one year this accumulated money will be increased to 22871.78 x 0.10 = 2287.18. Finally,
the invested money will be ( 22, 871.78 + 2278.18) = 25, 158.96 after 9 years.
Present value of an annuity is the sum of the present values of all payments ( or installments ) made at
successive annuity periods.
Let us consider that the present value is V of an annuity amount P to continue for n years with compound
interest rate r% per annum. Thus, if P is first installment, then the present value due one year hence is
P
Present value of the first payment due after one year hence = 1 r
P
Present value of the second payment due after 2 years hence = (1 r )
2
………………………………………………………………………………………………………………
P
= (1 r )
n
Present value of the n-th payment due after n years hence
The present value V is the sum of all the present values shown above. Hence
n
P 1
{1 }
P P P (1 r ) (1 r ) P
V ...... {1 (1 r ) n }
(1 r ) (1 r ) 2
(1 r ) n
1
1 r
1 r)
The
1
final value V stands in a simple form as V is a geometric progression with common ratio 1 r .
If the payments are made q times in a year and the annual investment is P , then the present value V of the
annuity P to continue for n years is given by
286
n q
P r
V [1 1 ]
r q
If the payment is made monthly , quarterly, or half-yearly, then the present value is given by, respectively
12 n 4 n 2 n
12 P r 4P r 2p r
V [1 1 ], V [1 1 ], V [1 1 ]
r 12 r 4 r 2
Example 8.6: A man agrees to take a loan of Tk.2,00,000.00 from a bank at a compound interest rate
13% per annum. The amount will be paid back within 5 years by yearly installment. Find the amount of
installment.
It is known that
P
V [1 (1 r ) n ]
r
P
20000 [1 (1 0.13) 5 ]
0.13
0.13 200000 P[1 1 0.13 ]
5
Example 8.7: Find the present value of an annuity of Tk. 5000.00 payable at the end of every year with
compound interest rate of 12% . The payment will be made within 5 years.
P
V [1 (1 r ) n ]
r
5000
V [1 (1 0.12) 5 ]
0.12
18,023.88 18,024
Example 8.8: Find the amount and present value of an annuity of Tk.150.00 for 12 years with
compound interest rate of 3.5% per annum.
287
P 150
[(1 r ) n 1] [(1 0.035)12 1] 2190 .29
Pn = r 0.035
P
V [1 (1 r ) n ]
r
150
V [1 (1 0.035) 12 ]
0.035
1449 .43 1449 .00
Example 8.9: A refrigerator is purchased on installment basis, such that Tk.1000.00 is to be paid on
signing the contract of purchase and the balance will be paid in 6 yearly installments of Tk.600.00
payable at the end of each year. Find the cash down payment if compound interest is charged at rate of
8% per annum.
Solution: Let V be the present value of the annuity of Tk.600 for 6 years at 8% interest. Then the cash
down of the refrigerator is Tk. ( V + 1000 ). Given P =600, r = 0.08, n =6. We have
V= r 0.08
Example 8.10: A man took loan of Tk.1,00,000.00 from a bank at a compound interest rate of 14% per
annum . He agreed to pay back the money within 5 years and payment will be made quarterly . Find the
amount of each installment.
Solution: Given V = 100000, r = 0.14, q =4, n = 5. We need the value of P, where the value is calculated
from the formula
q n
qP r
V [1 1 ]
r q
45
4P 0.14
100000 [1 1 ]
0.14 4
25000 0.14 P[1 (1.035) 20 ]
3500 P 0.49743
3500
P 7036 .17
0.49743
288
Example 8.11: A man took loan of Tk.50,000.00 from a bank at a compound interest rate of 14% per
annum and agreed to pay back the money by monthly installment within the period 3 years. Find the
amount of installment.
q n
qP r
V [1 1 ]
r q
123
12 P 0.14
50000 [1 1 ]
0.14 12
4166 .67 0.14 P[1 (1.01167 ) 36 ]
583 .3338 P 0.34143
583 .338
P 1708 .50
0.34143
Example 8.12: The price of a machine is Tk.1,00,000.00 currently. After 10 years the price is expected to
be increased by 25%. How much money should be invested yearly at 12% compound interest rate per
annum to replace the machine after 10 years?
Solution: The current value of the machine after 10 years will be V + 25% of V =
100000+25000=125000, where V is the present value. Given r =0.12, n =10. We need the value of P.
Here Pn = 125000=V.
P
Pn [(1 r ) n 1]
r
P
125000 [(1 0.12)10 1]
0.12
125000 0.12 P (2.105848 )
15000
P 7123 .02
2.105848
Example 8.13: A man took a loan of some money and returned the same in 3 quarterly installments of
Tk.5000.00 each. The rate of interest was 16% compound quarterly. Find the loan amount and the interest
charged for the principal.
Solution: Given P = 5000, r = 0.16 , n =3, ( payment is made in 3 quarters . It is a quarterly payment. So
q =4 ). We need V and the interest Paid , say I. Here
n 3
qP r 4 5000 0.16
V [1 1 [1 1 ] 125000 0.111 13875 .46
r q 0.16 4
289
This value is the principal amount. The man paid [5000 x 3 = 15000] . Therefore, the interest is
Example 8.14: A man borrowed Tk.50,000.00 at a compound interest rate of 15% per annum on the
condition that the amount [principal + interest ] would be paid within 20 years by yearly installment.
What would be the installment?
Solution: Let P= the amount of installment. Given r =0.15, n =20 , the present value V= 50000. We have
P
[1 1 r ]
n
V
r
P
[1 1 0.15 ]
20
50000
0.15
50000 0.15 P 0.9389
7500
P 7988 .07
0.9389
Example 8.15: A bank pays interest at the compound rate of 8% per annum. If Tk.100000.00 is
deposited in the bank at the beginning of each year, what will be the accumulated money at the end of 5
years?
(1 r )[1 r 1]
P
Pn
n
r
100000
(1 0.08)[(1 0.08) 5 1]
0.08
633592 .90.
A sinking fund is a part of a bond indenture or preferred stock character that requires the issuer to
regularly set money aside in a separate custodial account for the exclusive purpose of redeeming the
bonds or shares . If a company decides to take money from the market by issuing a bond, then the
company needs to think the ways and means to repay the money, sometimes with interest, or benefit, to
the issuer. In such a case the company needs to deposit periodically, to a trustee so that in need it can
repay the money to the bond purchaser. This fund is called sinking fund. For example, let us consider that
a company „C‟ issues bond of Tk.10 million that will mature in 20 years. If the bond has a sinking fund,
company „C‟ must deposit Tk. O.5 million every year into a sinking fund. This sinking fund is separate
from the operating fund of the company and it will be used to repay the matured value of a bond to the
purchaser.
The sinking fund is under a custodial account into which the fund payments will go. The bond issuer
makes payments to the trustee of the custodial account. This type of payments are usually fixed amounts.
290
However, the issuer sometimes does not have the scope for payment for several years. But in most cases,
the sinking fund requires the issuer to actually repay the portion of the debt on a prearranged schedule so
that all of the debts are repaid by the maturity date. So, whatever be the delay in payment to the bond
purchaser is not refused for repayment. Usually, sinking funds can either be in cash or in the form of other
bonds or preferred stock.
As the amounts of payment are typically fixed, there should be some criteria by the fund „s provisions,
although variable amounts of payment is allowed on earnings levels. If payment is not made properly,
there may be a case of failure to make scheduled principal and interest repayment which results in
defaulting on the loan.
Sometimes an issuer can establish one sinking fund for all of its issues rather than a separate sinking fund
for each issue. The sinking fund payments are generally a percentage of all outstanding debt, and the
issuer can apply the funds to one or more particular issues of its choice. These sinking fund are usually
called aggregate sinking funds.
A sinking fund can improve the credit worthiness of a bond issuer. It helps the issuer to pay a lower
interest rate to the investors. Thus there may be a savings of interest resulting more net income and cash
flow for funding operations. Also, business may deduct interest payments given to lenders from their
taxes, helping increased cash flow as well. The issuer may use the savings for covering sinking fund
payments or other obligations. In such a situation the investors appreciate the added protection a sinking
fund provides. This situation makes a congenial atmosphere for the investors to lend more money to a
company or to the issuer.
However, if interest rates decrease and bond prices increase, bonds may be called and investors may lose
some of their interest payments resulting in less long-term income. Also, investor‟s may have to put their
funds elsewhere at a lower interest rate which results a missing out long-term potential income.
A doubling option is a provision in a sinking fund that gives a bond issuer the right to redeem twice the
amount of debt when repurchasing callable bonds. It allows the issuer to retire additional bonds at the
sinking fund‟s call price. It works as follows:
Let us consider that a company issues Tk.4,00,000.00 worth of bonds those are is set to reach maturity in
10 years. The bonds issued have a sinking fund provision, which requires the company to set aside each
year TK.40,000.00 into a sinking fund for 10 years. The sinking fund provision also requires the bond
issuer to use those funds to retire a proportion of the debt each year by repurchasing bonds on the open
market. If the bond issue also has a doubling option, the bond issuer may opt to redeem up to
Tk.80,000.00 worth of bond issue per year.
The doubling option gives the bond issuer the right to double the sinking fund provision. The bonds for
repurchase are typically selected randomly, where random selection can be done by lottery , and the
repurchase will typically happen at the bonds‟ par value.
291
A doubling option will usually be exercised by the bond issuer as current interest rates more lower that
the bonds yield. In this circumstance, the bond issuer may be motivated to repurchase more debt through
the sinking fund option and refinance itself at the new, lower rates. For this reason, exercising the
doubling option reduce the return that investors will receive.
In general, sinking funds benefit investors in three ways. First, the redemptions leave less principal
outstanding, making final repayment more likely and thus lowering default risk. Second, if interest rate
increase, thereby lowering bond price, investors get some downside protection because the provision
forces the issuer to redeem at least some of those bonds at the sinking fund call price, which is usually
par, even if the bonds are selling below par at the time. Third, sinking fund provisions create liquidity for
the bond in the secondary market, which is specially good when interest rates are increasing and the
bonds are less valuable, the issuer is in the market as a buyer even when prices fall. Finally, it can be said
that sinking fund is created by investing annually a fixed sum of money at compound interest rate to pay
off a loan or a debenture stock on some future specified date or to accumulate to a desired amount for
replacing an asset, like plant and machinery, etc.
Let P be the sum of money set aside annually and Pn be the liability to be redeemed after n years. Then for
Pn the amount of annuity P payable for n years . The value of P can be calculated from the formula
P
Pn [(1 r ) n 1]
r , where r is the compound interest rate per annum.
Example 8.16: A company issued bond of Tk. 100.00 per bond . It issued 50,000 bonds on the condition
that compound interest rate of 10% will be paid and the bond will be matured after 20 years. What should
be the amount of sinking fund per year?
P
Pn [(1 r ) n 1]
r
P
5000000 [(1 0.10) 20 1]
0.10
500000 P (5.7275 )
500000
P 87298 .12
5.7275
Example 8.17: A refrigerator costs Tk.30,000.00 and its effective life is estimated 10 years. The
refrigerator is to be replaced after 10 years when its scrap value will be Tk.5000.00. Find the amount of
sinking fund annually which will be accumulated at compound interest rate 8% per annum. Assume that
the price will remain same after 10 years.
Solution: The price after n years will be Pn = 30000 – 5000 = 25000. Given n =10, r = 0.08 . We need P.
292
P
Pn [(1 r ) n 1]
r
P
25000 [(1 0.08)10 1]
0.08
2000 P (1.1589 )
2000
P 1725 .77
1.1589
Example 8.18: A sinking fund is to be created for the redemption of debentures of Tk.5,00,000.00 at the
end of 20 years. Find the amount of sinking fund per year if the investment can earn compound interest at
the rate of 5% per annum.
Solution: Given Pn= 500000, r =0.05, n = 20. We need the value of P. We know
P P P
Pn [(1 r ) n 1] [(1 0.05) 20 1] (1.65329 ) 33.06595 P
r 0.05 0.05
500000
500000 33.06595 P or P 15,121.30
33.06595
Example 8.19: The price of a machine is Tk.10,000,000.00 and its effective life is estimated 15 years.
After 15 years the machine may be sold at a price of Tk.1,00,000.00 . It is assumed that the price of the
machine may be increased 20% compared to the present price. Find the amount of payment of a sinking
fund per year if the fund gets a compound interest at the rate of 8% per annum.
Solution: From the given condition cost of the machine after 15 years will be Pn ,where
Pn= 1000000 + 200000 – 100000 = 11,00,000. Given n =15, r=0.08. We need P. Here
P P P
Pn [(1 r ) n 1] [(1 0.08)15 1] (2.17217 ) 27.15211 P
r 0.08 0.08
1100000
1100000 27.15211 P or P 40,512 .50
27.15211
Example 8.20: A sinking fund is to be created for replacing a T.V. worth Tk.35,000.00 after 20 years.
The scrap value of the T.V. is expected to be Tk.5000.00. How much money is to be deposited per year in
the sinking fund when the fund will get a compound interest of rate 12% per annum. It is assumed that the
price of T.V. will be increased by 10% of the present price.
Pn = 35000 + (35000) 0.10 – 5000 = 33,500. We need P, given n= 20, r= 0.12. Now
293
P P P
Pn [(1 r ) n 1] [(1 0.12) 20 1] (8.64629 ) 72.05244 P
r 0.12 0.12
33500
33500 72.05244 P or P 464.94 465.00
72.05244
A delayed annuity is an annuity in which the first installment is not paid immediately, as in an immediate
annuity. This type of delayed annuity is known commonly as deferred annuity and it is a type of life
annuity that guarantees a reliable stream of cash payments to an annuitant until death at which point the
benefit may be transferred to a beneficiary or estate depending on the options chosen by the buyer. The
way a delayed annuity differs from most of other annuities is in how premiums are paid into it and how
and when withdrawals are made.
Let the present value be V of a deferred annuity P to begin at the end of m years and to continue for n
years with compound interest rate r. Then the value of V is calculated by
P (1 r ) n 1
V { m n }
r (1 r )
P 1 P 1
V {1 m n
} {1 }
It can also be written as r (1 r ) r (1 r ) m
It is fixed series of cash flows received at a future date. In this scheme payment is received from some
time in future. It differs from annuity that the payment is received for infinite period but annuity payment
is received for a fixed period.
P 1
V
r (1 r ) m , where payments starts after m years.
Let P be the first installment made in the beginning of a year ( period) and annuity is continued for n
years. Then P earns compound interest for n years and hence P amounts to P (1 +r )n . The second
installments earns P (1 + r )n – 1 and so on. The last installment P earns interest for one year and hence it
becomes P ( 1 + ). Finally, the total accumulated money becomes
294
{(1 r ) n 1 P
(1 r )[(1 r ) n 1]
= P( 1 +r) (1 r ) 1 r
qP r r
(1 )[(1 ) q n 1]
Pn = r q q , where q = 2 , 3, 4, ……..m=12.
2P r r
(1 ) [(1 ) 2 n 1]
If q =2, then Pn = r 2 2 [half-yearly payment]
4P r r
(1 )[(1 ) 4 n 1]
If q = 4, then Pn = r 4 4 [ quarterly payment]
12 P r r
(1 )[(1 )12 n 1]
If q =12,then Pn = r 12 12 [ monthly payment]
In this case the payments are made at the beginning of each year (period) and present value (v) is given by
P p P
V P .........
1 r (1 r ) 2
(1 r ) n 1
P
(1 r ) [1 (1 r ) n ]
r
However, the payment may be made q times, where [q=2, 4, 12] ,q is half-yearly, or q is quarterly or q is
monthly]. In such a case the present value (V) is given respectively, by
2P r r
V (1 ) [1 (1 ) 2 n ]
r 2 2
4P r r
V (1 )[1 (1 ) 4 n ]
r 4 4
12 p r r
V (1 )[1 (1 ) 12 n ]
r 12 12
Example 8.21: A business organization took loan from a bank at compound interest of 14% per annum
on the understanding that the loan would be paid back in 15 yearly installments of Tk.5000.00.If the first
installment is paid at the end of three years from the date of loan issued and the other payments are made
regularly, what is the amount of loan?
295
P (1 r ) n 1 5000 (1 0.14)15 1 6.137938
V [ m n
] [ 3 15
35714 .29[ ] 20,728 .94
r (1 r ) 0.14 (1 0.14) 10.575169
rate of12% on the understanding that the money would be paid back in 20 years . But first installment was
paid after 2 years. How much was the amount of loan? The half-yearly installment of loan was
Tk.6000.00.
Solution: Given P = 6000, m = 4[ after2 years means after 4 installments], n = 10 [ due to 2 installments
in a year], r = 0.12. We need the value of V.
r
(1 ) 2 n 1
2P 2
V { m 2 n }
r (1 r )
2
0.12 210
(1 ) 1
2 6000 2 2.207135
{ } 100000{ } 54,511.50
0.12 0.12 4 210 4.048935
(1 )
2
Example 8.23: A bank pays interest at the rate of 9% per annum compounded half-yearly. A person
planned to have an amount of Tk. 30,000.00 from the bank after 5 years. How much money should be
deposited in the bank in the beginning of every half-year to get the expected money of Tk.30,000.00?
Solution: The expected money after n years is Pn=30000, n =5, r=0.09. We need the value of P, where
2P r r
Pn (1 ) [(1 ) 2 n 1]
r 2 2
2P 0.09 0.09 25
30000 (1 )[(1 ) 1]
0.09 2 2
P
15000 (1.045)(0.552969 )
0.09
1350
1350 0.577853 P P 2336 .23
0.577853
Example 8.24: A co-operative society pays interest at yearly compound rate of 15%. A man wants to take
a loan of amount Tk. 50,000.00 after 10 years. How much money should be deposited by the man at the
beginning of every year?
Solution: Given Pn= 50000, n= 10, r =0.15. We need the value of P, where
296
Example 8.25: A man deposits Tk.1000.00 per annum in the beginning of every on the understanding
that the amount will be accumulated at the yearly compound interest of 12% for 5 years. What will be the
accumulated money?
P 1000
V (1 r )[1 (1 r ) n (1 0.12)[1 (1 0.12) 5 ] 1000 (1.12)(3.604775 ) 4037 .35
r 0.12
Example 8.26: A loan is to be paid in 10 yearly installment of Tk.5000.00. The payment will be made at
the beginning of each year. For the loan the yearly compound interest rate is 14%. How much money is to
be taken as loan ?
P
Pn (1 r ) [1 (1 r ) n ]
r
5000
(1 0.14) [1 (1 0.14) 10 ]
0.14
35714 .2857 (1.14)(0.730256 ) 29,731 .86.
Example 8.27: Find the present value of an annuity due to Tk.1000.00 payable half-yearly at the
beginning of the half year for 5 years with compound interest rate of 10% compounded half-yearly.
2p r r
V (1 ) [1 (1 ) 2 n ]
r 2 2
2 1000 0.10 0.10 2 5
(1 ) [1 (1 ) ]
0.10 2 2
20000 (1.05) (0.386086 ) 8107 .82.
Example 8.28: A loan is to be paid in 8 equal half-yearly installments, each of Tk.4000.00 to be paid in
each of the beginning of half-year. If interest rate is charged 10% payable half-yearly, what is the loan
amount?
297
2p r r
V (1 ) [1 (1 ) 2 n ]
r 2 2
2 4000 0.10 0.10 2 4
(1 ) [1 (1 ) ]
0.10 2 2
80000 (1.05) (0.32316 ) 27145 .50
8.6.4 Amortization
Amortization is an accounting technique used to lower the cost value of a finite life or intangible assest
incrementally through scheduled charges to income. It is referred to the paying off of a debt, overtime, in
regular installments of interest and principal adequate enough to repay the loan in full by maturity. Thus,
it is the paying off of debt with a fixed repayment schedule in regular installments overtime.
Amortization works in the following manner. For example, a person took loan of Tk.20,000.00at 6%
interest for 5 years under the condition of yearly payment. Then the installment one year becomes
P
V {1 (1 r ) n }
r , where V = 20000, r=0.06, n =5
P
2000 {1 (1 0.06) 5 }
0.06
1200
1200 P(0.25274 ) P 4747 .96
0.25274
This P is the first installment out of which interest is Tk,120.00. In this installment 4627.96 is the
principal and interest is 120.00. In this way the break down of payments can be calculated. We can
explain it by example.
Example 8.29: A loan of Tk.5000.00 is to be paid in 10 equal yearly installments. The yearly compound
interest is 10% and first installment is paid after a year. Analyse the payment with break down of interest
and principal.
Solution: Given V =5000, r = 0.10, n = 10. We need the value of P, where P is calculated as follows:
We have
P P
V {1 (1 r ) n } {1 (1 0.10) 10} P(6.1445)
r 0.10
5000
5000 6.1445 P P 813.74
6.1445
Thus, the first installment is 813.74 out of which interest is ( 5000 X 0.10) = 500. The amortization is
(873.74- 500 ) = 373.74. The principal left for second year is (5000 – 373.74 ) = 4686.26 and its interest
is (4686.26 x 0.10 )= 468.63. The amortization after second installment is (813.74 – 468.63 ) =345.11.
The other values of amortization are shown below:
298
Amortization Table
Example 8.30: A loan of Tk.2000.00 is to be paid in 3 equal installments annually with compound
interest rate 9% . If the first payment is made after a year, analyse the payments into those on account of
the interest and on account of amortization of the principal.
P P
V {1 (1 r ) n } {1 (1 0.9) 3 } 2.531295 P
r 0.09
2000
2000 2.531295 P P 790.10
2.531295
Amortization Table
Example 8.31: A loan of Tk.5000.00 is to be paid in 2 equal installments annually with compound
interest rate 12% . The first installment is paid after one year. Find the installments and the interest after
two years.
P P
V {1 (1 r ) n } {1 (1 0.12) 2 } 1.69005 P
r 0.12
5000
5000 1.69005 P P 2958 .49
1.69005
299
After the first installment the interest paid is ( 5000 x 0.12)=600.00. The amortization amount is (
(2958.49 – 600.00) = 1758.49. The principal left for interest is (5000 – 1758.49 )=3241.51. The interest
for this left principal is ( 3241.51 x 0.12)= 388.98. The total interest after two years is
Example 8.32 A bank pays interest at the rate of 10% per annum compounded quarterly. A man deposits
Tk.5000.00 in the beginning of each quarter of a year and the process of deposit is continued for 5 years.
Find his accumulated amount of money after 5 years.
qp r r
Pn (1 )[(1 ) qn 1]
r q q
4 5000 0.10. 0.10 45
(1 )[(1 ) 1] 200000 1.025 0.638616 1,30,916.37
0.10 4 4
Example 8.33: A bank pays interest at the rate of 8% per annum compounded quarterly. A man needs an
amount of Tk.20,000.00 in 5 years. How much money should he deposit in the bank at the beginning of
each quarter of a year?
qp r r
Pn (1 )[(1 ) qn 1]
r q q
4 P 0.08. 0.08 45
20000 0.08 (1 )[(1 ) 1] 4 P 1.02 0.485947
0.08 4 4
400
400 P 0.495666 P 807.00
0.495666
Example 8.34: A business organization takes loan from a bank at 12% compound interest rate and agrees
to pay back the principal and interest in 10 equal yearly installments of Tk.2000.00 each. If the first
installment is paid after 3 years from the date of loan received and the other installments are paid
regularly at the end of the subsequent years, find the loan amount.
Solution: Since the first installment is paid at the end of 3 years ,the annuity in this case is deferred by 2
years. The present value of deferred annuity is given by
300
P (1 r ) n 1
V {
r (1 r ) m n
2000 (1 0.12)10 1 2000 2.105848
{ }
0.12 (1 0.12) 2 10 0.12 3.895976
2000 4.5043 9008 .65
Example 8.35: A professor retires at the age of 65 years. He will get pension of an amount of
Tk.36,000.00 per month for the rest of his life. According to expectation of life professor may survive for
another 7 years. If the interest rate is 12% payable monthly, what single amount is equivalent to his
pension?
Solution: Let V be the single amount of pension for n=7 years at compound interest rate of r =0.12.
qP r
V { 1 (1 ) qn }
r q
12 36000 0.12 127
{1 (1 ) } 12 36000 4.720704 20,39,344 .30
0.12 12
Example 8.36: A loan is to be paid in 10 half-yearly installments each of which is of Tk.5000.00 and
payment is to be made at the beginning of every 6 months. If interest is charged at the rate of 12% per
annum and payable half-yearly, find the amount of the loan.
Solution: Given P=5000, r =0.12 , n=5 [ as there are 10 half-yearly installments],q=2.We need V, where
qP r
V { 1 (1 ) qn }
r q
2 5000 0.12 25
{1 (1 ) } 2 5000 3.68004 36,800.40
0.12 2
Example 8.37: A bus is purchased on installment basis on a condition that Tk.50,000.00 is to be paid on
signing the contract. The remaining money will be paid by installment of Tk.5000.00 each payable at the
end of first, second, third and fourth years. If the interest is charged at 8% per annum, what would be the
cash down price of the bus?
Solution: Let V be the present value of the annuity of Tk.5000 for 4 years at 8% compound interest rate
per annum. Then the cash-down price of the bus is (V+5000). We know
p 5000
{1 (1 r ) n } {1 (1 0.08) 4 } 56,546.26
r 0.08 . Here P=5000,r=0.08,n =4.
301
Example 8.38: Find the present value of an annuity of Tk.5000 per annum for 6 years at 10% compound
interest rate.
p 5000
V {1 (1 r ) n } {1 (1 0.10) 6 } 21,776 .30
r 0.10
Example 8.39: A man wants to buy a government flat at a price of Tk.60,000,00.00 .He agrees to pay
50% of the price at signing the contract as down payment and the remaining money will be paid in 20
equal installments with compound interest rate of 10% per annum. Find the value of each installment.
Solution: As the price is Tk.6000000, the man needs to pay as down payment of
Tk.6000000 x 0.5 = Tk.3000000 . This amount is the present value of the flat and it is V. The other given
values are r =0.10, n = 20. We need the annuity money P , where
p P
V {1 (1 r ) n } {1 (1 0.10) 20} P 8.51356
r 0.10
3000000
3,52,379 .02
or 3000000 = 8.51356 P , Therefore, P= 8.51356
Example 8.40: A bank pays interest at the rate of 8% per annum compounded yearly. If a man deposits
Tk.5000.00 to the bank at the beginning of each year , what will be the total accumulated money after the
end of 10 years?
P 5000
Pn (1 r ) {(1 r ) n 1} (1 0.08){(1 0.08)10 1}
r 0.08
5000 15.64548 78,227.44
Example 8.41: A bank pays interest at the rate of 8% per annum compounded half-yearly. A man
deposited Tk.5000.00 at the beginning of each year for 5 years. Find the accumulated money after 5
years.
302
qP r r 2P r r
Pn (1 ) [(1 ) qn 1} (1 ) [(1 ) 2 n 1}
r q q r 2 2
2 5000 0.08 0.08 25
(1 ) [(1 ) 1]
0.08 2 2
125000 1.04 0.48024 62,431.76
Example 8.42: A loan of Tk.10,000.00 is to be paid in 5 equal installments with compound interest rate
of 15% per annum and first installment is paid after first year. Find the installment and the interest for the
capital.
P P
V [1 (1 r ) n ] [1 (1 0.15) 5 ] P 0.50282
r 0.15
1500
10000 0.15 P 0.50282 , P 2983 .17
0.50282
The first installment is Tk.2983.17. This amount includes the interest of Tk. (10000 x 0.15)=1500.00. So
the amortization is ( 2983.17 – 1500.00)= 1483.17. The principal left for the interest is
The interest of this amount is ( 8516.83 x 0.15)= 1277.52. The other intersts are shown below:
Amortization Table
End of year Annual payment, Tk. Interest due, Tk. Amortization, Tk. Principal left for
interest
1st 2983.17 1500.00 1483.17 10000.00
2nd 2983.17 1277.52 1705.65 8516.83
Example 8.43:The accumulation in a provident fund is invested at the end of every year to earn 12%
compound interest per annum. A person contributes 10% of his salary to which employer adds another
10% every month. Find how much the accumulation of money will be there after 45 years of service.
Solution: Let us consider that the monthly salary of the person is Tk.100.00. Then the contribution to the
provident fund is Tk.10.00 and along with employer‟s contribution this amount becomes (10+10)=20
monthly. The yearly contribution is 20 x 12 =240. Therefore, the initial annuity is P = 240, r=0.12 and n
=45 years. We need Pn , where
303
P
Pn { (1 r ) n 1}
r
240
{(1 0.12) 45 1} 240 1358 .230032
0.12
3,25,975 .20
This is the accumulation of the provident fund for every taka 100 of monthly salary of a person.
Example 8.44: The monthly salary of a person is TK.1,20,000.00. The person contributes 10% of his
salary in a provident fund for accumulation of money at the compound interest rate of 10% per annum.
The employer also contributes 10% . The total money is deposited at the end of every year. If the person
contributes for 40 years ,what will be his total accumulated money?
Solution: The yearly contribution to the provident fund is P= 2(120000 x 0.10) x 12 =2,88,000, n =40,
r=0.10 . We need Pn , where
P
Pn { (1 r ) n 1}
r
288000
{(1 0.10) 40 1} 2880000 44.259255
0.10
12,74,66,654 .44
Example 8.45: A man retires at the age of 62 years. During retirement he earns pension Tk.7,56,000.00
per year. But he needs to commute 50% of his pension to get ready cash money .The commuted money is
assumed to be increased by 10% by compound interest rate of 10% up to his life. The expectation of life
is 72 years. What will be the value of the accumulated money , if it is assumed that the pension is paid at
the end of each year?
Solution: 50% of the pension amount is ( 756000 x 0.5) = 378000, n = 72 – 62 =10, r = 0.10. We need
the value of V, where
P
V {1 (1 r ) n }
r
378000
{1 (1 0.10) 10}
0.10
378000 0.614456 23,22,643 .68
304
Supplementary Problems
1 . A man decides to deposit Tk.5000.00 at the end of each year in a deposit scheme which pays 8%
compound interest per annum. If the installment is allowed to accumulate , What will be the total
2 . A man invests Tk.10,000.00 at the end of each year in a deposit scheme which pays 7.5%
compound interest per annum. If the installments are allowed to accumulate for 10 years , what
will be the accumulated money one year after the 10 installments ? Ans.Tk.1,52,081.19.
3 . A person took loan of Tk.1,00,000.00 at the compound interest rate of 12% per annum and
agreed to pay back the money in 10 installments. The payment will be made at the end of each
4 . A two-wheeler is purchased on installment basis ,such that Tk.5000.00is to be paid after signing
the contract of the purchase and four-yearly installments of Tk.3000.00 each, payable at the end
first, second, third and fourth years. If interest is charged at the rate of 5% per annum, what
purchase and agreed to pay the remaining amount with compound interest rate of 12% per
annum compounded half-yearly in 20 equal installments. If the first installment is paid after the
end of six months from the date of purchase , find the amount of each installment. Ans.Tk.8719.66.
6 . Find the present value of an annuity of Tk.300.00 for 5 years at 4% compound interest.
Ans.Tk.1335.58
7 . What is the amount and present value of an annuity certain of Tk.500.00 for 15 years with
8 . A person bought a wrist watch worth of tk. 1561.00 by paying a cash of Tk. 300.00 on the
understanding that the balance will be paid with compound interest rate of 10% per annum
305
compounded half-yearly. The payment will be made in 3 equal half-yearly installments. Find the
9 . A portable computer costs a man Tk. 32,000.00 and its effective life is estimated to be 10 years.
After 10 years the selling price of the computer is expected to be Tk.8000.00 only. Find the sum of
money to be invested every year at 10% compound interest rate per annum. After 10 years the
expected price of the portable computer is 25% more of the current price. This sinking fund is to buy
10. A bank pays interest at the rate of 7.5% per annum compounded half-yearly. Find the amount of
money which is to be deposited in the bank beginning of each half-year in order to get an
11. A bank pays interest at the rate of 10% per annum compounded yearly. If Tk.2000.00 is deposited
to the bank at the beginning of every year , what will be the accumulated money at the end of 5
12. What will be the value of perpetuity of Tk.250.00 at 8.33% interest per annum? AnsTk.3000.00.
13. A loan is to be paid in 8 half-yearly installments , each of Tk.4000.00 to be paid at the beginning
of every 6 months . If interest rate is charged at 10% per annum payable half-yearly , find the loan
14. If a machine is purchased on installment basis, such that Tk.5000.00 is to be paid on the signing of
the contract and four yearly installments of Tk.3000.00 each, payable at the end of first, second,
third and fourth years. If the interest rate is charged at 5% per annum, what would be the cash down
15. A man purchased a flat valued at Tk.1,25,00,000.00. He paid Tk.10,000,000.00 at the time of
purchase and agreed to pay the balance with interest at 15% per annum compounded half-yearly in
20 equal half-yearly installments. If the first installment is paid after the end of six months from the
16. A man took loan at 8% compound interest per annum and agreed to pay both the principal and
306
interest in 10 equal yearly installments of Tk. 1200.00 each. If the first installment is to be paid at the
end of 5 years from the date of taking loan and the other yearly installments are paid regularly at the
17. A refrigerator costs a family Tk.65,000.00 and its effective life is estimated as 25 years. A sinking
fund
is to be created for replacing the refrigerator at the end of its life time when its scrap value will be
18. A person purchase a machine worth Tk.70,000.00 on a hire purchase scheme. At the time of gaining
possession the person needs to pay 40% of the cost of the machine and the rest is to be paid in 20
equal installments. If the compound interest rate is 7.5% , what should be the installment?
Ans.Tk.4120.00.
19. A person retires at the age of 60 years and earns a pension of Tk.10.000.00 per month. He needs to
commute 50% of his pension for ready money. The expectation of life is 72 years. If the commuted
money is allowed to accumulate at 10% compound interest, what will be the accumulated ready
money? It is assumed that pension is paid at the end of the year. Ans. 3,68,674.03.
20. The accumulation in a provident fund is invested at the end of every year. If a person‟s monthly
salary is Tk.50,000.00 if he and his employer contribute 10% of his salary to provident fund account
to be accumulated at compound interest rate 8% per annum, what will be the accumulated money
21. A loan of Tk. 50,000.00 is to be paid in 5 years in 5 equally installments with compound interest rate
12% per annum. First installment is paid after one year. What will be the installment and what will be
22. A man purchased a motorcycle worth Tk.3,00,000.00. He paid Tk.2,00,000.00 at the time of
purchase and agreed to pay the balance with compound interest of 12% per annum. But payment
will be made half-yearly in 10 years. He paid the first installment after the end of 6 months of data
307
23. A loan is paid in 6 equal annual installments, where each installment is of Tk.2500.00. The loan is
sanctioned at yearly compound interest rate of 8%. Find the amount of loan. Ans. Tk.12,474.00.
24. A T.V. set is purchased on installment basis such that 60% of its price is to be paid as cash-down
payment on signing the contract of purchase. The balance will be paid in six equal installments
payable half-yearly at 10% compound interest rate , compounded half-yearly. If the cash-down
25. A man invested Tk.2000.00 every year with a company which pays compound interest rate of 10%
per annum. Find his accumulated money after the payment of 8th installment. Ans.Tk.25,168.00
26. A man purchased a house worth Tk.70,00,000.00. At the time of purchased he paid 40% of the price
On the understanding that the balance will be paid in 20 equal yearly installments with 12%
27. A person took loan and paid it back in 3 equal installments of Tk. 4630.50. The loan is paid with 20%
compound interest rate compounded quarterly. What is the loan amount and what is the interest?
28. An industrialist bought a garment house worth Tk. 3,00,000,00.00 by cash down payment of
Tk. 2,00,000,00. He had an understanding with the seller that the balance will be paid in 20
installments with compound interest rate of 15% . Find the amount of installment. Ans.15,97,614.74.
29. A man borrowed Tk.10,000.00 from his relative on condition that the amount will be paid back with
compound interest rate of 5% by installment of Tk.1000.00 in each year. In how many years the loan
308
Chapter IX
Discounting
9.1 Definition
It is a financial mechanism in which debtor obtains the right to delay the payments to a creditor, for a
defined period of time, in exchange for a charge or fee. Essentially, the part owes money in the present
purchases the right to delay the payment until the future date. For example, a person buys a flat at Tk.
60,00,000.00 if payment is made cash. The purchaser may request the seller for the delayed payment for
six months. During this time the compound rate of interest may be 10% per annum. If the seller agrees to
consider the delayed payment up to six months at current value , then there is a discount of price of the
flat up to a definite period. Again, the seller may agree to sell the flat at 10% discount rate if the payment
is made currently. It means that there may be discount in ( a ) time period, and ( b ) price. Another
example, we can mention that the price of a scrap is Tk.5000.00 if anybody purchases it instantly , there
may be 10% discount in price. In that case the real selling price becomes as follows:
If 10% discount is announced on selling price of scrap worth Tk. 5000.00, then discount becomes
Tk.5000.00 x 0.10 = Tk.500.00. The selling price then becomes Tk. 50000.00 – Tk.500.00 = Tk.4500.00.
Again, it may happen that both the purchaser and seller agree with the selling price of the scrap, but the
purchaser may request the seller for the delayed payment by 6 months. The seller may agree
unconditionally or without fee. For example, the interest rate of the money is 10% per annum. Then for
Tk.5000.00 the interest for 6 months becomes 5000 x ( 0.10/2) = 250. The seller may agree to consider
the delayed payment for 6 months, then there is a discount of price for 6 months or in money value it is
250. Here the discount or charge is the difference between the original price to be paid in the present
period and the amount that has to be paid in future to settle down the marketing. It is noted that the
discount is usually associated with a discount rate, which is usually called discount yield [ D.Y]. The
discount yield is the proportional share of the initial amount ( current price, liability, owed ) that must be
paid to delay the payment for a certain period ( usually ,say one year). Then the D.Y is given by
Amount for delayed payment for certain period , say one year
D.Y
Current price (debt liabilty)
For example, the current price of an item is Tk.5000.00. A purchaser is supposed to pay the money after
one year with a charge of Tk.100.00. Then
100
D.Y 0.2
500 or 20%.
Since a person can earn a return on money invested ( say the current price) over some period ( say one
year) of time, most economic and financial models assume the discount yield is the same as the rate of
309
return the person could receive by investing this money elsewhere over the given period of time covered
by the delay in payment. The concept is associated with the opportunity cost of not having use of the
money for the period of time covered by the delay in payment.
Let P be the original payment currently due, and the debtor wants the delay in the payment for t years.
Consider that the market rate of return is r on a similar investment of asset during that period. Then the
future value of P becomes P( 1+r)t and the discount will be calculated as
where r is the discount yield. If F is the payment which will be made after t years in future, then the
present value P becomes
F
F (1 r ) t
P = (1 r )
t
Example 9.1: The current price of a machine is Tk.2,00,000.00. But the payment will be made after 2
years without fee. The current rate of interest is 12% per annum. Find the discount value of machine.
F 200000
1,59,438 .78
PV = (1 r ) (1 0.12) 2
t
Discount = P ( 1+ r)t – P
The discount factor, D.F.( T ), is the factor by which a future cash flow must be multiplied in order to
obtain the present value. For a zero-rate ( also called the spot rate ) r, taken from yield curve and a time to
cash flow T ( in years ) , the discount factor is
D. F.( T )= (1 + r T )- 1.
In the case where the only discount rate one has is not a zero rate but an annually –compound rate and
only has its yield to maturity, then annually compounded discount factor ca be used and is given by
310
D.F.( T ) = ( 1 + r )- T.
However, when operating in a bank, where the amount the bank can lend is linked to the value of its
assets, trader usually use daily compounding to discount cash flow. In that case discount factor for time
period T is calculated by
D.F.( T ) = e- r T
Thus, it is clear that the discounting is the process of determining the present value of a payment or a
stream of payments that is to be received in the future.
Example 9.2: A person borrows an amount from an organization which will be paid after 3 years. The
future payment is Tk.5,00,000.00. The compound interest rate is 10% per annum. What is the present
value of the loan?
Solution: Let P be the present value of the loan. The future payment is F= Tk.5,00,000.000 , r = 0.10,
t= 3. Then
Example 9.3: The price of a machine is Tk. 5,00,000.00 after a discount of 25%. Find the original price.
Solution: Let the original price be Tk.X. If X =100, after 25% discount the price becomes 75. Now
For Tk.500000 the original price is Tk. 500000 ( 100 / 75) =Tk. 6,66,666.67
Example 9.4: The price of a laptop is Tk.32,000.00. The distribution center announces 10% discount.
What will be the selling price?
The discount is 10% and hence the discounted amount is 32000 x 0.10 = 3200.
= 32000 – 3200
= 28, 800.
311
Example 9.5: A publisher marked the price of a text book at Tk.750.00. The salesman of the publisher
sells five copies of book at price Tk.2500.00. What is the rate of discount?
Solution: The marked price is Tk.750.00 per copy. The selling price per copy is Tk. (2500/ 5)=Tk.
500.00. Hence, the discount per copy is Tk.750 –Tk.500.00 = Tk.250.00. Now,
For marked price 100 the discount is 100 ( 250 / 750) = 33.33.
Example 9.6:The manufacturer‟s marked price of an item is Tk.500.00. The manufacturer announces
20% discount for wholesaler and wholesaler announces 10% discount for the retail seller. What is the
selling price of the item?
Example 9.7:The price of a flat is Tk. 90,00,000.00. The payment will be made after 3 years. The
compound interest rate is 8% per annum. Find the present value of the flat and the amount of discount.
Solution: The future payment is F= 9000000, compound interest rate r=0.08, n = 3 years. We need the
present value ( discounted value), PV, where
F 9000000
71,44,490 .17
PV= (1 r ) (1 0.08) 3
t
Example 9.8: The present price of a machine is Tk.50,00,000.00. A man purchases the machine by cash
down payment of Tk. 30,00,000.00. The balance of payment will be paid after 3 years. Find the present
value of the machine if there is a compound interest rate of 10% per annum.
Solution:As 30,00,000 has been paid, the remaining 20,00,000 [50,00,000 – 30,00,000 = 20,00,000 ] will
be paid after 3 years with compound interest of 10%. The present value of the future payment is given by
312
Finally, the present value of the machine becomes 30,00,000 + 15,02,629.60 = 45,02,629.60.
Example 9.9: The market price of a laptop is Tk.35,000.00. The distributor buys it at 10% discount and
he sells it at 5% profit. What is the selling price?
Solution: The distributor pays Tk. 35,000.0 – Tk.35,000.00 x 0.1 = Tk. 31,500.00.
Example 9.10: A retail seller purchases a laptop for Tk.30,000.00. He wants to make a profit of 10%
after allowing a discount of 15%. What should be the marker price of the laptop?
Solution: Let the market price of the laptop be Tk.x . On market price 15% is discount. So the discount
amount is 0.15 x. Thus the selling price becomes ( x – 0.15 x ) = 0.85 x. The profit of the retail seller is
0.85 x 30000
0.10
30000
0.10 30000 30000
x 38,823.53
0.85
Example 9.11: A businessman marked the price of a shirt Tk.2800.00. He allows two successive
discounts of 20% and 10% to a retail seller. The retail seller‟s cost is 10% and he wants a profit of 15%.
What should be the selling price of the shirt?
After first discount the price stands = Market price – First discount = 2800 – 560 = 2240
Thus, the purchasing price of the retail seller stands= 2240 – 224 = 2016
The retail seller‟s total cost for the shirt = 2016 + 201.60 = 2217.60
The price of the shirt with 15% profit stands = 2217.60 + 2217.60 x 0.15 = 2550.24.
Example 9.12: The list price of an article is 20% above the selling price and the cost price of the article
is 25% below the list price. Find the percentage of the discount on list price and profit on cost price.
313
The List price [ L.P.]= 100 + 100 x 0.20 = 120
The cost price[C.P.] is 25% below the list price and hence
20
100 16.67
120 %
Percentage of profit is .
Example 9.13:A manufacturer offers 15% discount on all his goods to the wholesaler and a further
discount of 10% on the balance. Find the selling price to the wholesaler who buys in cash. If the
purchasing price is Tk.2000.00, what is the discount rate?
Solution: As the marked price is Tk.2000.00, and it deserves the 15% discount, the price becomes
( 2000 – 2000 x 0.15 ) = 1700. It deserves another 10% discount. So the price becomes
( 1700 – 1700 x 0.10 ) = 1530. Total discount = 300 + 170 = 470. Percentage of discount becomes
470
100 23.50
2000
Example 9.14: A manufacturer allows 25% discount to the retail seller. The retail seller sells at 10%
below the list price. If the customer pays Tk.500.00 for an item, what is the percentage of profit of the
retail seller and what is the amount of profit?
100
500 555.56
90
314
83.33
100 20
416 .67
Example 9.15: The marked price of a book is Tk.700.00. The publisher allows 35% discounts to the retail
seller. The retail seller allows 20% discounts on the marked price to the customer. What is the percentage
of the retail seller?
Solution: The marked price is 700.00. After 35% discount the retail seller pays ( 700 – 700 x 0.35 ) =
455.
The retail seller sells at ( 700 – 700 x 0.20 )= 560. The retail seller‟s profit becomes
560 100
100 23.08
455 %
Example 9.16: The current price of a machine is Tk.60,00,000.00 if a down payment of Tk.40,00,000.00
is made during the contract of purchase. The remaining amount will be paid after 3 years at the compound
interest rate of 12%. What is the present value of the machine?
Solution: The future payment, F= 20,00,000.00 with compound interest rate of r=0.12 in t = 3 years. The
present value ( P.V.) of this future payment is given by
315
Chapter x
Geometry
10.1 Introduction
10.1.1 Coordinate System: A coordinate system is a system which uses one or more numbers, or
coordinates to uniquely determine the positions of the points or other geometric elements on a manifold
such as Euclidean space. The order of the coordinates is significant , and they are sometimes identified by
their position by an ordered tuple and sometimes by a letter, as the X-coordinate. The simplest example
of a coordinate system is the identification of points on a line with real numbers using the number line. In
this system, an arbitrary point O ( the origin) is chosen on a given line. The coordinate of a point P is
defined as the signed distance from O to P, where the signed distance is the distance taken as positive or
negative depending on which side of the plane lies. Each point is given a unique coordinate and each real
number is the coordinate of a unique point.
_______________________________________________________
-10 -9 -8 -7 -6 -5 -4 -3 -2 -1 0 1 2 3 4 5 6 7 8 9 10
Figure 1
10.1.2 Cartesian Coordinate System: The simple example of a coordinate system is a Cartesian coordinate
system. This system is a coordinate system that specifies each point uniquely in a plane by a set of
numerical coordinates , which are the signed distances to the points from two fixed perpendicular oriented
lines measured in the same unit or length. It is used to pinpoint where we are in a map or graph. In the
plane, two perpendicular lines are chosen and the coordinate of a point is taken to be signed distance to
the lines. The two lines are called axes of a coordinate and specifically they are called Rectangular axes.
Both the lines cut at a point. This point is named as origin. One of the two lines is horizontal line and it is
identified by X O X/. another line is vertical one and it I identified by YOY/.The two axes divide the plane
into four parts. Each part is known as quadrant . The quadrants are
Figure 2
XOY, YOX/, X/OY and Y/OX and these are known as first, second, third and fourth quadrant,
respectively.
316
Let us consider that P is a point at the first quadrant of the axes. The coordinate of P is say (a,b). From
this point we can draw a perpendicular to X- axis as follows:
Figure 3
The perpendicular cuts Xaxis at the Point A ( =4). Again, we can draw a line parallel to the X-axis which
cuts the Y axis at the point B( = 3 ). Here OA = a = 4, AP = b=3, PB= a = 4, BO = b = 3, where ( a=4,
b=3 ) is the coordinate of the point P.
In the first quadrant the values of the coordinates are positive, in the second quadrant the value of x is
negative but the value of y is positive, i.e. the coordinate of any point Q is ( - a, b ). In the third quadrant
the value of both x and y are negatives, i.e. the coordinate of any point R is( - a , - b ). The coordinate of
any point S in fourth quadrant in S( a, - b ).
10.1.3 Polar Coordinate System : The polar coordinate system is a two-dimensional coordinate system in
which each point in a plane is determined by a distance from a reference point and an angle from a
reference direction. The reference point is called pole, and the ray from the pole in the reference direction
is the polar axis. In this system a point O in the plane is chosen, where O is in the pole and a ray from
this point is taken as the polar axes. For a given angle , there is a single line through the pole whose
angle with the polar axis is ( counterclockwise) from the axis to the line. Then there is a unique point on
this line whose signed distance from the origin is r for given number r. For a given pair of coordinates ( r ,
) there is single point , but any is represented by many pairs of coordinates. For
317
Figure 4
example , ( r, ) , ( r, + ) and ( - r, ) are polar coordinates for the same point. The pole is
represented by (0, ) for any value of . Here r is generally called Radius Vector and is the vectorial
angle .The radius vector moves in anticlockwise direction and makes an angle , where positive angle is
formed. But if the radius vector move clockwise , the negative is considered.
There are two common methods for extending the polar coordinate system to three dimensions. In the
cylindrical coordinate system, a z- coordinate with the same meaning as in Cartesian coordinate system is
added to the r and polar coordinates giving a triple ( r, , z ).
Coordinate systems are often used to specify the position of a point, but they may also be used to specify
the position of more complex figures such as lines, planes, circles, or spheres . Foe example, Plucker
coordinate system.
10.1.4 Spherical Coordinate System : A spherical coordinate system is a coordinate system for three-
dimensional space where the position of a point is specified by three numbers: the radius or radial
distance of that point from a fixed origin, its polar angle measured from a fixed zenith direction, and the
azimuthal angle of its orthogonal projection on a reference plane that passes through the origin and is
orthogonal to the zenith, measured from a fixed reference direction on that plane. It is similar to the three-
dimensional version of the polar coordinate system. The radial distance is also known as radius or radial
coordinate. The polar angle may be called colatitude, zenith angle normal angle or inclination angle.
To define a spherical coordinate system, one must choose two orthogonal directions, the zenith and the
azimuth reference, and an origin point in a space as shown in the following figure: These choices
determine a reference plane that contains the origin and is perpendicular to the zenith. The spherical
318
Figure 5
319
Figure 6
The perpendicular is PN. Let us consider that the Cartesian coordinate of P is ( x, y) and polar coordinate
is (r, ). Now, from the figure, we can write
PN y
Sin , or Sin , or y = r Sin
OP r
……………………………………………..(i)
ON x
Again, Cos , or Cos , or x = r Cos
OP r
…………………………………………..(ii)
Squaring and adding both sides of (i) and (ii) , we get
r 2 ( Sin 2 Cos 2 ) x 2 y 2
or r 2 x 2 y 2 Sin 2 Cos 2 1
r x2 y2 ..............................(iii )
y PN
and tan
x ON
y y
tan 1 ( ); x 0, tan 1 ( ); x 0 }(iv )
x x
rSin y y
Here or tan [(i )to (iv )]
rCos x x
The equations (iii) and (iv) help us to show the relation between Cartesian and polar coordinates.
320
Example 10.1: (a) The polar coordinate of a point P is ( 3 , ). Find the Cartesian coordinate of the point
4
P.
(b) Polar coordinate of a point P is ( 2 , 45 0 ).Find the Cartesian coordinate of the point P.
Solution: (a) Let the Cartesian coordinate be ( x, y),
where x = r Cos , and y = r Sin
= 3 Cos = 3 Sin
4 4
= 3 x 0.71 = 3 x 0.71
= 2.13 = 2.13
4
(b) Let the polar coordinate of the point is ( r, ), where
r= x 2 y 2 = (4) 2 (2) 2 = 20 , as x = 4 and y = - 2.
y 2
Again , tan = = 0.5 , tan 1 0.5 26.560.
x 4
Therefore, the polar coordinate is ( 20, 26.560).
Example 10.3: Express the following polar coordinates in Cartesian coordinates.
( i ) r = 4 Sin , (ii) r= 3Cos , (iii) r= 2 Sin
Solution:
( i ) . Given r = 4 sin (ii) Given r = 3 Cos (iii) Given r = 2 Sin
r2= 4 r Sin r2 = 3r Cos r2= 2 r Sin
2
But r = x +y 2 2
= 3x , As x = r Cos =2y
Again, x = r Cos 2 2 2
But r = x +y = 3x 2 2
x +y = 2y
y = r Sin x2+y2- 3x = 0 or x2+y2 – 2y =0
Bur 4 r Sin = 4 y
Hence, x2+ y2= 4y
Or x2 + y2 – 4y = 0
Example 10.4: Express the following in polar coordinates:
321
( i ) x2+ y2=25, ( ii ) x2+ y2 – 2x = 0, ( iii ) ( x - 2 )2 = 32,
Solution:
( i ) We have ( ii ) We have (iii) We have
x2 + y2 = 25 x2 + y2= 2x ( x - 2) 2= 32
or x2+ y2 = 52 or x2 + y2= 2 r Cos or x2 + 22 = 4x + 32
r=5 = r2 if r = 2 Cos or x2 + y2 = 4(x + 8 )
The polar coordinate is ( 5, ). The polar coordinate is ( 2 Cos , ). = r2,
where r = 2 (x+8)
=2
(r Cos 8 )
The polar coordinate is
(2 (r Cos 8 , ); y=2
Example 10.5: Express the following in polar equation
( i ) y2= 4 x + 4, ( ii ) x2= 1 – 2y , (iii) y2 = 4 ( 1 – x ), ( iv ) ( x - 3 )2 = 6 – 5 x
Solution:
( i ) We have ( ii ) We have ( iii ) We have
or x2 + y2 = x2 + 4x + 4 x2= 1 – 2y y2 + x2 = x2 +4 – 4x
2 2
or x + y = ( x + 2 ) 2
or x +y = y + 1 – 2y
2 2 2
or x2 + y2= (2 – x )2
2
=r , where r = x + 2 or x +y = ( 1 – y )
2 2 2
x2 + y2= r2 , where
or r = r Cos +2 2 2 2
x + y = r , where r = 2 – x = 2 – r Cos
or r ( 1 – Cos )= 2 r = 1 – y = 1 – r Sin or r ( 1 + Cos ) = 2
or r ( 1 + Sin ) = 1
( iv ) We have
x2 - 6x + 9 = 6 – 5x or x2+ y2 = x – 3, or x2 + y2= r2, where r = ( x 3 or r = r Cos 3
10.1.6 Distance Between Two Points
Let there be two points P ( x1, y1) and Q( x2, y2) on a plane as shown in Figure 7. Let us draw a
perpendicular from P on X- axis and another perpendicular from Q. From Q let us draw another
perpendicular on the line PA . This line is QR as shown in the figure given below:
Figure 7
322
It is seen that there is a right triangle PQR [ right-angled triangle] for which the angles
PQR = RPQ
So, we can write
( PQ )2 = (QR)2+ ( PR)2 = ( x1 – x2)2 + ( y1- y2)2
PQ = ( x1 x2 ) 2 ( y1 y 2 ) 2 , where
PQ is the distance between two points P and Q.
If P( x, y) is only a point, the distance of this point from the origin O ( 0,0 ) is given by
OP = x 2 y 2
Example 10.6: Find the distance between each of the two points given below:
( i ) ( - 3 , - 2 ), ( 3, 2 ); ( ii ) ( 2,4 ),(-2 , 7 ); ( iii ) (- 1 , 2 ), ( 3, - 2 ) ; ( iv ) ( 2, 8 ), ( -6, 2 ) ; ( v ) ( 2,4
), (-2,1 )
Solution:
( i ) The given points are P ( - 3, - 2 ) and Q ( 3, 2 ). The distance between P and Q is given by
PQ = ( x1 x2 ) 2 ( y1 y 2 ) 2 (3 3) 2 (2 2) 2 36 16 52 2 13.
( ii ) The given points are P ( 2, 4) and Q ( -2, 7). The distance between P and Q is given by
PQ = ( x1 x2 ) 2 ( y1 y 2 ) 2 {2 (2)}2 {4 7) 2 16 9 25 5.
( iii ) The given points are P ( -1,2 ) and Q ( 3, -2 ). The distance between P and Q is given by
PQ = ( x1 x2 ) 2 ( y1 y 2 ) 2 (1 3) 2 {2 (2)}2 16 16 32 4 2
( iv) The given points are P ( 2,8) and Q( -6,2). The distance between P and Q is given by
PQ = ( x1 x2 ) 2 ( y1 y 2 ) 2 ({2 (6)}2 (8 2) 2 64 36 100 10
( v ) The given points are P ( 2,4 ) and Q ( -2,1). The distance between P and Q is given by
Pq = ( x1 x2 ) 2 ( y1 y 2 ) 2 ({2 (2)}2 {4 1}2 16 9 25 5.
Example 10.7: Find the distance of each of the points from origin:
( i ) ( 3,4); ( ii ) 6,8); ( iii ) (2,4 ); ( iv) ( 5,5 ); ( v ) ( -4, -6 ); ( vi) ( -3,- 4 ); ( vii ) ( -5, 3 ); ( viii ) (
2, 6 ).
Solution:
( i ) The point of origin is O(0.0) and another point is P( 3,4 ). The distance between O and P is given by
OP = x 2 y 2 32 4 2 9 16 25 5
( ii ) The point of origin is O(0,0 ) and another point is P ( 6.8). The distance between O and P is given by
OP = x 2 y 2 6 2 8 2 36 64 100 10
( iii ) The point of origin is O(0,0) and another point is P(2,4). The distance between O and P is given by
OP = x 2 y 2 2 2 4 2 4 16 20 2 5
( iv ) The point of origin is O (0.0 ) and another poni is P( 5,5). The distance between O and P is given by
OP = x 2 y 2 5 2 5 2 25 25 50 5 2 .
( v ) The point of origin is O(0.0 ) and another point is P(-4,-6). The distance between O and P is given by
OP = x 2 y 2 (4) 2 (6) 2 16 36 52 2 13 .
( vi) The point of origin is O(0.0) and another point is P(-3,-4). The distance between O and P is given by
OP = x 2 y 2 (3) 2 (4) 2 9 16 25 5 .
( vii )The point of origin is O(0.0) and another point is P( -5,3). The distance between O and P is given by
OP = x 2 y 2 (5) 2 32 25 9 34 .
323
( viii) The point of origin is O(0,0 ) and another point is P(2,6). The distance between O and P is given by
OP = x 2 y 2 2 2 6 2 4 36 40 2 10 .
Example 10.8: The coordinates of the points P, Q, and R are (-7,-1), (-3, 2) and (x,5), respectively.
Given the condition that PQ = PR. Find the value of x.
Solution: According to the coordinates of the given points, we have
PQ = ( x1 x2 ) 2 ( y1 y 2 ) 2 {(7) (3)}2 (1 2) 2 16 9 25 5
QR= ( x2 x3 ) 2 ( y2 y3 ) 2 (3 x) 2 (2 5) 2 9 x 2 6 x 9 x 2 6 x 18
Given PQ= QR
5 = x 6 x 18 , or 25 = x2+6x +18, or x2 + 6x – 7 =0 , or( x + 7) ( x – 1 ) = 0 or x = - 7,
2
or
x =1
Example 10.9: Show that the three points P ( 1,2 ), Q ( 2,4 ) and R ( 3, 6 ) fall on a line.
Solution: If we draw the graph of these 3 points, it takes the following shape:
Figure 8
Now, we can find the distances PQ, QR and PR to show that PQ + QR = PR. If this relation of the
distances prevails, we can conclude that the three given points fall on a line. We have
PQ = ( x1 x2 ) 2 ( y1 y 2 ) 2 (1 2) 2 (2 4) 2 1 4 5
QR = ( x2 x3 ) 2 ( y 2 y3 ) 2 (2 3) 2 (4 6) 2 1 4 5
PR = ( x1 x3 ) 2 ( y1 y3 ) 2 (1 3) 2 (2 6) 2 4 16 20 2 5
It is seen that PQ + QR = 5 5 2 5 PR. So, the points P , Q and R fall on a line.
Example 10.10: Find the value of k if the given three points P ( k, -1 ), Q ( 0,1 ) and R ( 2,3 ) fall on a
line.
Solution: If the given three points fall on a line, the shape of the graph will be as follows:
324
Figure 9
According to the given condition the relation PQ + QR = PR should hold good. Let us now calculate the
distances PQ, QR and PR. These distances are
PQ = ( x1 x2 ) 2 ( y1 y 2 ) 2 (k 0) 2 (1 1) 2 k 2 4
QR = ( x2 x3 ) 2 ( y2 y3 ) 2 (0 2) 2 (1 3) 2 8
PR = ( x1 x3 ) 2 ( y1 y3 ) 2 (k 2) 2 (1 3) 2 k 2 4k 20
Now, according to the condition of falling 3 points on a line, we can write
PQ + QR = PR
Or k 4 8 k 4k 20
2 2
On simplification, it gives k = -2 .
Alternative
If the given 3 points fall on a line, then the area of the triangle PQR formed should be zero. Here the area
of the triangle is given by
x1 y1 1 k 1 1
1 1 1
Area of the PQR = x2 y2 1 0 1 1 [k (1 3) 1(0 2) 1(0 2)] 2k 4
2 2 2
x3 y3 1 2 3 1
We can write – 2k – 4 = 0 . It gives that k = -2.
Example 10.11: Find the area of the triangle formed with the points O( 0,0 ) , A ( -3, 4) and B ( 3,4).
Solution: Let the triangle formed be AOB. Its area is given by
x1 y1 1 3 4 1
1 1 1
Area of the AOB = x2 y 2 1 0 0 1 [3(0 4) 4(0 3) 1(0 0)] 12 square
2 2 2
x3 y 3 1 3 4 1
units.
325
Example 10.12 The coordinates of three points A, B, and C are A( 1,2 ), B ( 3, -4 ) and C ( 5, -6).These 3
points form a triangle. Find the area of the triangle.
Solution: Let the triangle be ABC . Its area is given by
Area of ABC =
x1 y1 1 1 2 1
1 1 1 1
x2 y2 1 3 4 1 [1(4 6) 2(3 5) 1(18 20)] [2 4 2] 4 square units.
2 2 2 2
x3 y3 1 5 6 1
Example 10.13 Show that the three points A, B and C having the coordinates A( 4,-1 ), B (2,1) and C (
1,2) fall on a line.
Solution: If the three points fall on a line , then the area of the triangle formed with ABC will be equal to
zero. Here the area is given by
x1 y1 1 4 1 1
1 1 1 1
x2 y2 1 2 1 1 [4(1 2) 1(2 1) 1(4 1)] [4 1 3] 0
2 2 2 2
x3 y 1 1 2 1
Area of ABC = 3
Figure 10
326
It is seen that the straight line AB cuts the Y –axis at the point D and forms a positive angle with the
positive side of X- axis. On the line AB , let P ( x, y) indicates a point with coordinate ( x, y) . From the
point P the perpendicular line on X- axis is PM. Also, let us draw another perpendicular line DN on PM.
Let BAM = BDN = [ as the angles are similar ]. Also, it is seen that OD = c = MN.
Here PND = 900 . Thus, from the triangle PDN , we have PN = PM – NM = y – c .
The base line of the angle is DN =OM = x. Therefore, we have
PN yc
tan , or m, y = mx +c is the straight line equation and m= tan . If c= 0 ,
DN x
dy
the line passes through the origin. Here m = . It measures the change in y for unit change in x and it is
dx
called slope of the straight line equation.
The equation (ii) as shown above is an equation of a straight line which passes through a point P( x, y),
say. Then the slope of the equation is given by
y y1
m=
x x1
and hence the equation stands as
y – y1 = m ( x – x1)
If we consider another point Q( x1, y1) on the line as shown in Figure 11, then the slope of PQ is given by
y y1
Slope of PQ = =m.
x x1
Figure 11
The equation shown in ( iii) is a straight line equation which passes through two points P ( x1, y1) and
Q ( x2,y2) as shown below: The slope of PQ line is
y y1
x x1
y1 y 2
and the slope of QR line is m= m
x1 x 2
327
Figure 12
Figure 13
Here AB is the straight line and P(x, y ) is a point on this line. The slope of AP becomes
328
y0 y
Slope of AP =
xa xa
y b y b
The slope of BP is given by
x0 x
As the line passes through the points A, P and B , the slop of AP = the slope of BP. So, we have
y y b
xa x
x y
Or (x – a ) (y – b) = x y , or ab = ay + bx or 1.
a b
The equation (v) as shown above is an equation of a straight line AB which cuts the X-axis at the point A
and Y – axis at the point B. The figure is shown below:
Figure 14
Here the distance of the cut point A on X –axis from origin is OA and same at the cut point B on Y-axis is
OB. Let ON be the perpendicular on the line AB from the origin O (0,0 ). Let us consider that the distance
ON = p and AON = . Here AOB is a right angled triangle. Again , OAN is also a right angled
triangle. Therefore, BON = 90 . Now, the in the ONA OA = ON Sec pSec . Similarly,
in the OBN , we have OB = ON Sec ( (90 ) pCosec .Therefore, the equation of the straight
line is
x y
1
OA OB
On simplification it gives
x Cos + y Sin =p
This is a perpendicular form of the straight line equation.
Example 10.14: The equation of a straight line is 2 x – 3y + 9=0. Find the slope and y-intercept of the
equation.
Solution: We have 2x – 3y +9 =0
329
Or – 3y = - 2x – 9
2
Or y= x+3
3
2
Therefore, the slope is m = , and y-intercept, c = 3.
3
Example 10.15: The following are the equations of straight line
( i ) x – 4 y – 8 = 0 , (ii) 3 x + 6 y – 8 =0 , ( iii ) 4 x + 5 y – 13 = 0, ( iv) x – y + 2 = 0
Find slope and y – intercept for each of the equation.
Solution:
1 1
( i ) We have, x – 4y – 8 = 0 , or - 4 y = - x +8, or y = x – 2 . So, slope, m = and y-intercept, c
4 4
=- 2.
3 8 4
( ii ) We have, 3x + 6y – 8 = 0, or 6y = -3x +8, or y = - x 0.5 x .
6 6 3
4
So, slope ,m = -0.5 , y-intercept, c = .
3
4 13 13
( iii ) We have, 4x + 5y -13 = 0 , or 5y = - 4 x +13, or y = - x = - 0.8 x + . So, slope, m = -
5 5 5
13
0.8 and y-intercept, c = .
5
( iv ) We have, x – y + 2 + 0, or -y = -x – 2 , or y = x + 2, So, slope, m = 1 and y-intercept, c = 2.
Example 10.16: Using the following information find the equation of the straight in each case;
( i ) The line passes through the point ( 3, 6 ) and forms an angle 45o with the positive side of X-axis,
( ii ) The line passes through the point ( - 2, 5 ) and forms an angle 60o with the positive side of X-axis.
( iii ) The line passes through the point ( 3 ,5) and form an angle 1350 with the X-axis.
Solution:
( i ) The equation of the straight line is
y – y1 = m ( x – x1), where m = tan = tan 450 = 1
Therefore, we have y – 6 = 1 ( x – 3 ), or y = x + 3 is the required straight line.
( ii ) The equation of the straight is
y – y1 = m ( x – x1) , where m = tan = tan 600= 3
Therefore, we have y – 5 = 3 ( x + 2 ) or y = 3 x + ( 5 + 2 3 ) is the required equation.
( iii ) The equation of the straight line is
y – y1 = m ( x – x1 ) ,
where m = tan = tan 1350 = tan ( 180 – 45 )0= - tan 450=-1. Therefore, the equation of the straight line is
y – 5 = -1 ( x – 3 ) , or x + y – 8 = 0.
Example 10.17: Find slope and intercept of each of the following equations:
( i ) 6x – 5y + 30 = 0 , ( ii ) 4x + 9y – 36 = 0 , ( iii ) 5x - 7y – 35 = 0 , ( iv ) 2x – 3y +6 = 0 ,
Solution:
6x 5 y x y
( i ) We have 6x – 5y = - 30, or 1 , or 1.
30 30 5 6
6
Again, -5y = -6x – 30, or y = x6.
5
6
Therefore, the slope of the equation is, m= , and intercepts are - 5 and 6.
5
330
4x 9 y x y
( ii ) We have 4x +9y = 36 , or 1 , or 1.
36 36 9 4
4
Again, 9y = - 4x + 36, or y = x 4.
9
4
Therefore, the slope of the equation is m = and intercepts are 9, 4.
9
5x 7 y x y
( iii ) We have 5x – 7y = 35 , or 1 , or 1.
35 35 7 5
5
Again, - 7y = -5x + 35, or y = x 5 .
7
5
Therefore, the slope of the equation is m = and intercepts are 7, -5 .
7
2 3 x y
( iv ) We have 2x – 3y = -6 , or x y 1 , or 1.
6 6 3 2
2
Again, - 3y = - 2x – 6 , or y = x 2 .
3
2
Therefore, the slope of the equation is m = and the intercepts are - 3, 2.
3
Example 10.18: Find the equation of the straight line which passes through the points
( i ) ( 1,4) and (3,6); ( ii ) ( -2, 3 ) and ( 3, -3 ); ( iii ) ( 4, -3 ) and ( -3, 2 ); ( iv ) ( 0,4 ) and ( 4, 0 ). Also,
find the slope of each of the equation.
Solution:
( i ) The equation of a straight line which passes through two points ( x1, y1 ) and ( x2, y2 ) is given by
y y1 x x1 y 4 x 1
, or , or y = x + 3, where the slope is m = 1.
y 2 y1 x2 x1 6 4 3 1
The slope is also found out from the relation
y1 y 2 4 6
m= 1
x1 x2 1 3
( ii ) The equation of the straight line which passes through two points ( x1, y1 ) and (x2, y2 ) is given by
y y1 x x1 y 3 x (2)
, or , or 5(y-3) = -6 (x+2), or 6x + 5y -
y 2 y1 x2 x1 3 3 3 (2)
3=0
The slope of the equation is given by
y1 y2 3 (3) 6
m=
x1 x2 23 5
( iii ) The equation of the straight line which passes through two points ( x1, y1 ) and (x2, y2 ) is given by
y y1 x x1 y (3) x4
, or , or -7 y – 21 = 5x – 20 , or 5x +7y+1=0
y2 y1 x2 x1 2 (3) 3 4
The slope of the equation is
y1 y 2 3 2 5
m= .
x1 x2 4 (3) 7
( iv ) The equation of the straight line which passes through two points ( x1, y1 ) and (x2, y2 ) is given by
331
y y1 x x1 y4 x0
, or , or 4y – 16 = -4x , or y + x – 4 = 0
y 2 y1 x2 x1 04 40
The slope of the equation is
y1 y2 4 0
m= 1 .
x1 x2 0 4
Example 10.19: Find an equation of the straight line which cuts both axes and the line is divided equally
at the point ( 2, 3 ).
Solution: Let the equation of the straight line which cuts X-axis at the point A( a,0 ) and Y-axis at the
point B( 0,b ) be
x y
1
a b
Here the line is AB as shown below:
Figure 15
From the given condition the mid-point of the this line is ( 2 , 3). Hence, we can write
a0 0b
2 and 3 ; or a = 2 and b = 6.
2 2
Therefore, the equation of the straight line is
x y
1 ; or 3 x + 2y = 12.
4 6
Example 10.20: ( a ) In a garments industry there are two groups of workers. One group, on an average ,
prepares 5 shirts in 3 hours and another group prepares 6 shirts in 4 hours. If it is assumed that the time
linearly depend on number of shirts , find the equation for time on number of shirts prepared. How much
time is needed to prepare 10 shirts ?
( b ) In a super shop 5 customers are served in 10 minutes time during pick business hour by one
salesman. Another salesman served 15 customers in 25 minutes time. It is assumed that time of service is
linearly dependent on number of customers served. Find the equation for time on number of customers
served. Estimate time to serve 20 customers.
( c ) The queue time for 10 customers is 8 minutes during the banking hour 10:00 to 10:30 A.M. The
same time for 12 customers is 18 minutes during the banking hour 10:30 to 11:00 A.M. It is assumed that
332
queue time is linearly dependent on number of customers . Formulate this linear relationship of time on
number customers and estimate the time if the number of customers is 15.
Solution:
(a ) We have two points to study the linear relationship of y ( time in hours to prepare shirts) on x
( number of shirts prepared. The points are ( 5,3 ) and ( 6, 4 ). As time depends on number of shirts
prepares, the relationship of y on x will be straight line which passes through these, two coordinates
( 5 , 3 ) and ( 6,4 ). The equation of this type of linear relationship is a straight line which is given by
y y1 x x1 y 3 x 5
, or , or y – 3 = x – 5 , or x –y – 2 = 0 .
y 2 y1 x2 x1 43 65
The value of y when x = 10 is y = 10 – 2 = 8 hours.
( b ) We have two points to study the linear relationship of y ( time in minutes to serve customers ) on x
( number of customers served. The points are ( 5,10 ) and ( 15, 25 ). As time depends on number of
customers served, the relationship of y on x will be straight line which passes through these, two
coordinates ( 5, 10) and ( 15,25 ). The equation of this type of linear relationship is a straight line which is
given by
y y1 x x1 y 10 x 5
, or , or 10 y – 100 = 15x – 75 , or 3 x – 2y +5 = 0.
y 2 y1 x2 x1 25 10 15 5
The value of y when x = 20 is - 2 y = - 3x – 5 , or 2y = 3 x 20 – 5, or y = 27.5 minutes.
( c ) We have two points to study the linear relationship of y ( time in minutes to serve customers) on x
( number of customers served ). The points are ( 10,8 ) and ( 12, 18 ). As time depends on number of
customers served, the relationship of y on x will be straight line which passes through these, two
coordinates ( 10 , 8 ) and ( 12,18 ). The equation of this type of linear relationship is a straight line which
is given by
y y1 x x1 y 8 x 10
, or , or 2y – 16 = 10x – 100 , or 5x – y – 42 = 0.
y 2 y1 x2 x1 18 8 12 10
The value of y when x = 15 is y = 5 x 15 – 42 =33 minutes.
Example 10.21: ( a ) A company purchased a machine in 2012 by an amount of Tk. 500000.00. After 7
years the salvage value of the machine became Tk. 150000.00. Assuming the depreciation rate uniform
find the depreciation rate and estimate the salvage value for 2020.
( b ) A businessman invested Tk. 500000.00 in the year 2015 and he was earning a profit at the rate of
Tk.10000.00 per year. What will be his capital in 2025 ?
( c ) A television was bought in first January 2010 by paying Tk. 30000.00. In December 31st , 2019 it
was sold . The selling price was Tk. 2000.00. Assuming the rate of depreciation uniform , find the rate of
depreciation. What could be the selling price if it could be sold in 31st December , 2015 ?
( d ) A man took loan from a cooperative society at a simple interest rate of 11 % per annum in 2015. The
amount of loan was Tk.5000.00. How much amount of money the will have to pay in 2022 ?
Solution:
( a ) Let 2012 be the initial investment period . After 7 years the invested money became Tk.150000.00.
The depreciation was uniform over the period. The depreciation of the capital is linearly related to time.
So, depreciation can be shown by a straight line, where the line passes through the points A( 0, 500000 )
and ( 7, 150000). The equation of the straight line is given by
y y1 x x1 y 500000 x0
, or , or 7y – 3500000 = - 350000 x,
y 2 y1 x2 x1 150000 500000 7 0
Or y = - 50000 x + 500000 , So the depreciation rate is m = Tk. 50000.00. The salvage value in
2020 ( x = 8 ) will become y = - 50000 x 8 + 500000 =100000. The salvage value is Tk. 100000.00.
( b ) From the given condition the rate of profit is m = 10000 per year. It indicates that the capital
increases linearly. The invested money is linearly related to time. From the given condition the linear
relation is a straight line given by
333
y = mx +c
where the line passes through the point (0, 500000 ) with m= 10000. The line cuts Y-axis at the point
c=500000. Therefore, the required equation is
y = 10000 x + 500000
In 2025 ( x = 10 ) the capital will be y = 10000 x 10 + 500000 = 600000. The capital will be Tk.
600000.00
( c ) Let us consider that the initial value of the television in 1st January was Tk.30000.00 and selling price
( salvage value ) in 31st December, 2019 was Tk.2000.00. The rate of depreciation is uniform through out
the period, i.e. There was a linear relationship of price of television and time. This linear relationship can
be expressed by a straight line which passes through the points ( 0, 30000 ) and ( !0, 2000 ) . The
equation of the line is given by
y y1 x x1 y 30000 x0
, or , or 10 y – 300000 = -28000 x,
y 2 y1 x2 x1 2000 30000 10 0
Or y = -2800 x + 30000. Therefore, the rate of depreciation is m = Tk.2800 per year. The selling
price in 2015 ( x = 5 ) could be y = -2800 x 5 + 30000 = 16000. The salvage value is Tk.16000.00.
( d ) From the given condition the rate of interest is 11%, i.e. in each year the man needs to pay the
money 5000 x 0.11 = 550 as interest. This amount is 550 is the rate of interest for the taken loan and as
interest is simple, this money is m = 550. As interest rate is simple, the capital loan is increasing linearly
over time. This linear relationship can be expressed as an equation of a straight line, where the line is
y=mx+c
This line passes through the point ( 0, 5000) and it cuts the Y-axis at the point c, where c = 5000.
Therefore, the line is
y = 550 x + c. or y = 550 x + 5000.
After 7 years [ 2022 – 2015 = years ], the man will have to pay y = 550 x 7 + 5000 = 8850.
d= 2
m 1 m 1
2
334
Example 10.22: Find the distance between the two lines given below:
( a ) y = x + 2, y = x – 3 ; ( b ) 4x + 2y = 9, 2x + y = 3; ( c). 3x +2y = 15, 4y + 6x = 3
Solution:
( a ) Given y = x + 2 and y = x – 3 .So, m=1 for both the lines and the lines are parallel and there is
distance between the lines. We have c1= 2 and c = -3. Then the distance is measured by
c1m c2 m c2 c1
2 2
2 1 ()3 1 3 2
2 2
25 25
d= 2 2 12.5 =3.54
m 1 m 1 12 1 1 1
2
4 4
units
( b ) We have 4x + 2y = 9 or y = -2x + 4.5; So, m = -2 and c1= 4.5;
Again, 2x + y = 3 or y = - 2x + 3; So m = -2 and c2= 3. Then the distance is measured by
2 2
c1m c2 m c2 c1 4.5 (2) 3 (2) 3 4.5
2 2
9 2.25
d= 2 2.25
m 1 m 1 (2) 1 (2) 1
2 2 2
5 5
= 1.5 units.
( c ) We have 3x + 2y = 15, or y = - 1.5 x + 7.5, it gives m = -1.5 and c1 = 7.5.
Again, 4y + 6x = 3 , or y = - 1.5 x +0.75, it gives m = -1.5 and c2 = 0.75.
Here the slopes of both the equations are same, m = 1.5. So, the equations are parallel and they have some
distance between them. The distance is given by
2 2
c1m c2 m c2 c1 7.5 (1.5) 0.75 (1.5) 0.75 7.5
2 2
d= 2
m 1 m 1
2 2 2
( 1.5) 1 ( 1.5) 1
11.25 1.125 6.75 10.125 6.75
2 2 2 2
= = 3.1154 2 2.077 2
3.25 3.25 3.25 3.35
= 9.7056 4.3136 14.0192 3.744 units.
Example 10.23: Find the distance of each set of two lines given below:
( a ) 2x + 3y +4 =0, 2x + 3y – 6 = 0 ; ( b ) 4x – 3y + 2 = 0, 4x – 3y + 3 = 0;
( c ) 8x + 6y – 5 = 0, 8x + 6y +15 =0.
Solution:
( a ) We have 2x + 3y + 4 = 0 and 2x + 3y – 6 = 0. In each equation a = 2, b= 3; c1=4 and c2 = - 6.
The distance between the these two lines is given by
c1 c2 4 (6) 10
d= 2.77 units
a2 b2 2 2 32 13
( b ) We have 4x – 3y + 2 = 0 and 4x – 3 y + 3 = 0. In each equation a = 4 , b = -3; c1= 2 and c2 = 3.
The distance between these two lines is given by
c1 c2 23 1
d= 0.20 unit
a b
2 2
4 (3)
2 3 5
( c ) We have 8x + 6y – 5 = 0 and 8x + 6 y +15 = 0. In each line a = 8 , b= 6; c1= -5, c2 = 15.
The distance between these two lines is given by
c1 c2 5 15 20
d= 2 units.
a b 2
8 6
2 2 10
10.2.2 Conditions for two Parallel and Perpendicular Lines.
Let y = m1 x + c1 and y = m2x + c2 be two straight line equations. These two lines will be parallel if the
angle between these two lines is zero [ tan =0 ], where is the angle between these two lines.
Here m1 = tan 1 and m2 = tan 2 .
335
Let us consider that the angle between these two lines is as shown in the following figure, where
Figure 16
2 1 1 2 ……………( i )
If AC and AB lines form angles 2 and 1 with X-axis , respectively, then
2 1 (1 2 ) ………………( ii )
Thus, from ( i ) and ( ii ) , we have (1 2 ) . Therefore,
tan 1 tan 2 m m2 1 m1m2
tan =±tan( 1 2 ) tan 1 or cot
1 tan 1 tan 2 1 m1m2 m1 m2
Thus, two lines are parallel if m1= m2.
But if the equations are of the forms
a1x +b1y + c1=0 and a2x +b2y +c2=0
and if these two lines form an angle between them , where the angle is , then
m1 m2 a b a b
tan 2 1 1 2
1 m1m2 a1a1 b1b2
a c a a c a
Here y = 1 x 1 , hence m1 1 . Again, y= 2 x 2 , hence m2 2 .
b1 b1 b1 b2 b2 b2
a a
The two lines will be parallel if 1 2 , a1b2 a2 b1 , or a1b2 a2 b1 0
b1 b2
The two lines will be perpendicular, if m1m2= - 1 and a1a2 +b1b2=0.
Example 10.24 Find equation of a straight line in each case using the following information:
(a ). The line will be perpendicular to the line 2x - 3y +4 = 0 and passes through the points (2,1).
( b ). The line will be perpendicular to the line 3x + 2y – 5 = 0 and passes through the point (2,4).
( c ).The line will be perpendicular to the line 4x – y + 9 = 0 and passes through the point ( -3,2).
Solution:
336
2 4 2
( a ). We have 3y = 2x + 4 or y = x . Here the slope of the line, m = = m1(say).
3 3 3
3
A straight line will be perpendicular to this given line if the slope of new line becomes m= m2 so
2
that m1m2 = -1. This new line will pass through the point ( 2.1). So, its equation becomes
3
y – y1 = m (x – x1), or y – 1 = - ( x -2), or 2y – 2 = - 3x +6, or 2y + 3x – 8 = 0.
2
3 5 3
( b ). We have 2y = - 3x + 5, or y = x , where the slope of the line is m =m1(say)= . A new
2 2 2
2
line will be perpendicular to this line if it slope becomes m2= so that m1m2= -1. Therefore, the required
3
line will be
2
y – y1 = m ( x – x1) , or y – 4 = ( x 2) , or 3y – 12 = 2x – 4 , or 2x – 3 y +8 = 0.
3
( c ). We have 4x – y +9 = 0 , or y = 4x + 9. The slope of this line is m = m1(say) = 4. The new line will
1
be perpendicular to this given line if its slope becomes m2= so that m1m2= -1. The new line will pass
4
through the point ( -3 , 2). So, its equation becomes
1
y – y1 = m ( x – x1) , or y – 2 = {x ( 3)} , or 4y – 8 = - x – 3, or x +4y – 5 = 0.
4
Example 10.25: (a) Find an equation of a straight line which passes through the intersection of the
straight lines 3x + 2 y + 6 = 0 and 2x + 3y – 11 = 0 and perpendicular to the straight line 4x +6y +15 = 0.
(b ) Find an equation of a straight line which passes through the intersection of straight lines AB and AC,
where the equations of the lines are y = 2x +1 and y = 4x – 1 , respectively and perpendicular to the line
AB.
( c ) Find an equation of a straight line which passes through the intersection point of two lines which
pass through ( - 3, 2) and ( 2, -3) and ( 3,2) and ( 2, 4 ) and perpendicular to the straight line
x – y + 2 = 0.
Solution: (a) The intersection point of the given lines will be available from the solution of the
simultaneous equations
3x+2y+6=0 (i)
2x + 3y – 11 = 0 (ii)
Multiplying (i ) by 2 and (ii) by 3, we get
6x + 4 y + 12 = 0 (iii)
6x + 9y – 33 = 0 (iv)
(iii) – (iv) gives - 5 y = 45 or y = -9. Putting the value of y in (i) , we get 3 x – 2 x 9 +6 = 0, or x = 4.
Therefore, the line will pass through the point (4, -9 ).
2 5
To find the slope of the given line 4x +6y +15 = 0 , we can write 6y = -4 x – 15 , or y = x .
3 2
2
Therefore , the slope of the line is m = m1 ( say) . The new line will be perpendicular to this given
3
3
line if its slope m2 , so that m1m2= -1. Hence , the line is
2
3
y – y1 = m2(x – x1) , or y – (-9) = ( x – 4 ), or 2y +18= 3x – 12 . or 3 x – 2y – 30 = 0.
2
337
( b ) We have
y = 2x +1 …… (i)
y = 4x – 1 …… (ii)
(i)- (ii), gives – 2x +2 = 0 , it gives x = 1 . Putting x = 1 in (ii), we have y = 4 x 1 – 1 = 3. Therefore, the
intersection point of the lines AB and AC is (1, 3). Again, the slope of the line (i) is m= 2. So, we need a
1
line for which the slope will be m1= so that m m1 = - 1. Hence, the equation of the new line will be
2
1
y – y1 = m1 ( x – x1), or y – 3 = ( x 1) , or 2y – 6 = -x +1, or x + 2y – 7 = 0.
2
( c ) The line which passes through the points ( -3, 2) and ( 2, -3) is
y y1 x x1 y 2 x (3)
, or , or 5y – 10 = - 5x – 15, or x + y +1 = 0
y 2 y1 x2 x1 3 2 2 (3)
…….. (i)
Again, the line which passes through the points ( 3,2) and ( 2, 4) is
y y1 x x1 y 2 x 3
, or , or – y +2 = 2x – 6 , or 2x +y – 8 = 0,
y 2 y1 x2 x1 42 23
………….(ii)
Solving (i) and (ii), we have x = 9, y = - 10. The new line will pass through the point (9,-10) and will be
perpendicular to the line x – y +2 = 0. The slope of this line is m = - 1. We need an equation of a line the
slope of which is m1 = 1 so that m m1 = -1. Hence, the new equation of the line is
y – y1 = m1 (x – x1), or y – ( -10) = 1(x – 9 ), or x – y – 19 = 0.
Example 10.26: ( a ) Find an equation of a straight line which passes through the point ( -3, -4) and
parallel to the line 3x + 6y – 10 = 0.
( b ) Find an equation of a straight line which passes through the points ( 2, 5 ) and parallel to the line
8x – 4y + 15 =0.
Solution: ( a ) The equation of the given line is 3x +6y – 10 = 0. The slope of this line is
1 3 10
m1 (say) = as the equation becomes y = x .
2 6 6
We need an equation which is parallel to this equation and passes through the point (-3,-4). To become
1
parallel line it needs m1=m2 = . Therefore, the required equation of the line will be
2
1
y – y1 = m2 ( x – x1), or y – (-4) = { x – (-3)}, or 2y +8 = - x – 3 , or x + 2y +11 = 0.
2
(b) The equation of the given line is 8x – 4y + 15 = 0. The slope of this equation is m1(say) = 2 as the
15
equation becomes y = 2x + . This line will be parallel to a line if its slope is m2= 2. Hence, the
2
equation parallel to this equation will be
y - y1 = m2 (x – x1) , or y – 5 = 2 ( x -2 ) , or 2x – y + 1 = 0.
Example 10.27: Find an equation of a straight line which passes through the intersection of the lines
x + y – 5 = 0 and 2x – y – 7 = 0 and is parallel to the line 4x + 3y + 12 = 0.
Solution: The given two equations are
x + y – 5 = 0 ……… (i)
2x – y – 7 = 0 ……… (ii)
Multiplying (i) by 2, we get 2x + 2y – 10 = 0 …….(iii)
Subtracting (iii ) from (ii), we have - 3 y +3 = 0 . It gives y = 1. Putting the value of y in (i), we get x =
4.
338
The point of intersection is (4,1). The required line will pass through (4,1) and will be parallel to the line
4 4
4x+3y +12 = 0. It gives y = x 4 . The slope of this equation is m = . The required equation will
3 3
be of this slope as it should be parallel to the given line. Therefore, the equation of the line is
4
y – y1 = m (x – x1), or y – 1 = ( x 4) , or 3y – 3 = -4x +16, or 4x + 3y – 19 = 0.
3
Example 10.28: A man purchased two wrist watches the value of one was Tk. 5000.00 and the value of
second one was Tk. 3000.00. The first one he sold by an amount of Tk.800.00 after years 7 years and the
second one he sold by an amount of Tk. 1200 after 3 years. Are the rate of depreciation in both the cases
same ? If the rate of depreciation is same throughout the period.
Solution: As the price of the watch was declining over time period, there is a linear relationship of price
of the watch ( y ) and time (x). This linear relationship can be expressed by a straight line which passes
through the two points ( x1,y1) and ( x2,y2). For the first case these points are (0,5000) and (7,800) .For the
second watch these points are ( 0, 3000) and ( 3, 1200 ).
Using the first pair of points the equation of the straight line is
y y1 x x1 y 5000 xo
, or , or 7y – 35000 = -4200 x, or y= - 600x + 5000.
y 2 y1 x2 x1 800 5000 7 0
For this equation , the slope m1 ( rate of deprecation ) = - 600.
For the second pair of points the equation of the straight line is
y 3000 x0
, or 3y – 9000 = - 1800 x , or y = - 600 x + 3000.
1200 3000 3 0
For this equation , the slope is m2 = -600. The two equations are parallel. The rate of depreciation for
both the wrist watches was same.
Example 10.29: The two straight lines pass through the point ( -1, 2) and they form an angle 450 with the
line 3x – y + 7 = 0. Find the equations of these two straight lines.
Solution: Let the line be y – y1 = m ( x – x1) , where
y – 2 = m [ x – (-1)] , or y – 2 = m (x+1) , or y = m ( x +1 ) +2.
For the given line , we have y = 3 x + 7 which gives the slope m = 3= m1 (say). Again, the line which
passes through the point ( -1 , 2) has the slope m and it forms an angle = 450 with the given line. Thus,
we can write
m m1 m3
tan 450 = , or 1= , or 1 + 3m = (m 3) . In the right had side there is
1 m m1 1 3m
signs. If we consider +ve sign, we have 1 + 3m = m – 3 , or 2m = - 4 giving m = -2.
1
If we consider the - ve sign, then 1 + 3m = - (m – 3) ,or 4m = 2, or m = .Therefore, the two
2
required equations of straight lines are
y – 2 = - 2 ( x + 1 ), or y + 2x = 0
1
and y–2= (x + 1), or 2y – 4 = x +1 , or x – 2y +5 = 0.
2
Example 10.30: ( a ) The two lines 2x – y +3 = 0 and 4x + y – 3 = 0 intersect at a point. Find the point of
intersection and the angle form at the point of intersection.
( b ) The two lines 3x – 5y + 4 = 0 and 5x + 3y – 16 = 0 intersect at a point. Find the point of
intersection of the lines and find the angle between them.
Solution:
( a ) We have 2x – y +3 = 0 ………………….( i ) and 4x + y – 3 = 0 …………….( ii)
Multiplying (i) by 4 and (ii) by 2, we get
8 x – 4y + 12 = 0 ……………….(iii) and 8x + 2y – 6 = 0 ……………..(iv)
339
Subtracting (iv) from (ii), we get - 6y +18 = 0. It gives y = 3.
Putting the value of y in (i), we get 2x – 3 + 3 = 0 . It gives x = 0.
Therefore, the point of intersection is (0,3)
From (i), we have y = 2x + 3, hence ,the slope of this equation is m1 = 2. Again, from (ii), we have
Y = -4x +3. It gives m2 = - 4.
The angle between these two lines is given by
m1 m2 2 (4) 6 6
tan .
1 m1m2 1 2 (4) 7 7
1 6 6
Hence tan 40.6 0 , or tan 1 ( ) .
7 7
( b ) We have 3x – 5 y + 4 = 0 ……………..( i ) and 5x + 3y – 16 = 0 ………………. .( ii)
Multiplying ( i ) by 5 and 9 ii ) by 3, we get
15 x – 25 y + 20 = 0 ………(iii) and 15 x + 9y – 48 = 0 ……………….( iv)
Subtracting ( iv) from ( iii ), we get - 34 y + 68 = 0 . It gives y = 2.
Putting the value of y in (i), we get 3x – 5 x 2 +4 = 0, or 3x – 6 = 0, or x = 2. Therefore, the point of
intersection is ( 2,2 ).
3 4 3 5 16 5
From (i), we have y = x ,. Here m1= . From (ii), we have y= x . Here m 2 .
5 5 5 3 3 3
3 5
As m1 m2 = ( ) 1 , the two lines are perpendicular to each other. They form an angle of =900
5 3
Example 10.31: ( a ) Find the equation of a straight line which passes through the point ( - 1, 3 ) with
slope 2.
( b ) A man invested Tk.25000.00 in a simple interest scheme in 2014 . This money is increased to
Tk. 37000.00 in 2019. What is the rate of interest?
( c ) A straight line passes through the points ( -2, -3 ) and ( 2,4). Find the slope and y – intercepts of this
equation.
Solution:
( a ) The equation of a straight line is
y– y1 = m ( x – x1). Given the slope of this equation m = 2 and it passes through the point
x1=-1 and y1=3. Hence, the equation becomes
y -3 = 2 { x –(- 1)}, or 2x – y + 5 = 0
( b ) As the investment is made under simple interest scheme, the invested money is increased linearly
over time, i.e. the invested money y1 = 25000 in initial time x1= 0 is linearly increased to y2 = 37000
after x2 = 5 years. This linear relationship is given by
y y1 x x1 y 25000 x0
, or , or 5y – 125000= 12000 x
y 2 y1 x2 x1 37000 25000 5 0
Or 2400 x – y + 25000, y = 2400 x + 25000. Here m = 2400 is the rate of change of 25000 per year.
Hence , the rate of interest is 9.6%.
( c ) The equation of a straight line which passes through two points is give by
y y1 x x1 y (3) x (2)
, or , or 4 y + 12 = 7x +14
y 2 y1 x2 x1 4 (3) 2 (2)
7 1 7 1
Or y = x . Therefore, the slope of the equation is m = , and y – intercept, c = .
4 2 4 2
Example 10.32: The following are the 3 simultaneous equations
x – 3y + 1 = 0
x – 6y + 4 = 0
x+ay=0
340
For what value of a the above 3 equations meet at a point ?
Solution: The equations are
x – 3y + 1 = 0 ……… . (i)
x – 6y + 4 = 0 ……… .(ii)
x + ay = 0 ………. .(iii)
Now, (i) – (ii) gives 3 y – 3 = 0, or y =1. Putting the value of y in (i), we get x – 3 x 1 +1 =0, or x =
2.
As the lines meet at a point, the values x and y will satisfy the equation (iii). So, we have
2 + a x 1 = 0 or a = -2.
Example 10.33: A man invested Tk.5000.00 in 2008 at a simple interest rate and in 2013 he received
Tk.7000.00. This money was invested at a simple interest rate and in 2019 he received Tk. 10380. What
was the average interest rate throughout the invested period ?
Solution: As interest rate is simple, the initial (x1 = 0) invested money y1 Tk.5000.00 is linearly
increasing over time up to y2= Tk.7000.00 after x2= 5 years. This linear relationship is given by
y y1 x x1 y 5000 x0
, or , or 5y – 25000 = 2000 x
y 2 y1 x2 x1 7000 5000 5 0
Or y = 400 x + 5000; the slope of this equation is m1 = 400. It indicates that at every year Tk.5000.00 is
400 100
increased by Tk.400.00. Therefore, the rate of interest is 8% .
5000
Again, the amount TK.7000.00 = y1 is invested at simple interest rate and it is increased up to
Tk. 10380.00 ( y2) in x2= 6 years. This linear relationship is given by
y y1 x x1 y 7000 x0
, or , or 6y – 42000 = 3780 x. or y = 630 x +
y 2 y1 x2 x1 10780 7000 6 0
7000; Here the slope of this line is m2 = 630. It indicates that at every year the amount 7000 is increased
630 100
by an amount 630. Therefore, the rate of interest is 9% .
7000
The average rate of interest is 0.08 0.09 0.0072 0.0849 , or 8.49% . Here average rate is
calculated by geometric mean as we have calculated mean of rates.
10.3 Circle
A circle is a simple closed round shape figure, where each point in a plane is at a given distance from a
particular point known as Center. Equivalently, it is the curve traced out by a point that moves in a plane
so that its distance from a given point is constant. This constant distance is called radius. It is denoted by
„r‟.
Figure 17
Some Important Terms Related to a Circle
341
1.Arc : Any connected part of the circle is called an arc.
Figure 18
2.Centre: The point which is equidistance from the points of a circle is called centre.
Figure 19
3.Chord: A line segment whose end points lie on the circle is called chords.
Figure 20
342
4.Circumference: The circumference of a circle is the distance around it. That is circumference would be
the length of the circle if it were opened straightened out to a line segment. Since a circle is the edge of a
disc, circumference is a special case of perimeter.
Figure 21
5. Diameter: A line segment whose endpoints lie on the circle and which passes through the centre. It is a
chord which passes through the centre. It is the line which have largest distance between any two points
on the circle. It is twice the radius of the circle.
Figure 22
6. Radius: It is a line segment joining the centre of the circle to any point on the circle itself. Thus, it is
the fixed distance from the centre to any point on the circle. It is half of the diameter of a circle.
Figure 23
343
7. Tangent: Tangent to a plane curve at a given point is the straight line that just touches the curve at that
point. Leibniz defined it as the line through a pair of infinitely close point on the curve. It is the straight
line which touches the curve y = f(x) at a point x = c on the curve if the line passes through the point {c,
f(c) } on the curve and has the slope f/(c) , where f/ is the derivative of f.
Figure 24.
344
Thus, for circle the general condition is
g2+ f2 – c ≥ 0
However, as r = g 2 f 2 c and r is always positive, we can take ( g2+f2 – c ) > 0.
f) If the centre of a circle is at the origin ( 0,0 ), it cuts X-axis and Y-axis twice. One cut point is in the
left-hand side and another one is at the right-hand side of X-axis . The two cut points in Y-axis are in the
upper side and lower side of the axis. This is shown below:
In that case the distance between centre and any of the cut point is the radius „r‟ of the circle and area
above the X-axis within the circle is equal to half of the area of the circle and it is given by
1
Half of the area of the circle = r2
2
Same area is also measured when the circle cuts twice in Y-axis.
If the centre of the circle is not at the origin , it can cut X- axis and or Y-axis twice [Figure 27]. In that
case the area within the circle under the cut point is given by 2 f 2 c . The similar area under the cut
point in X – axis is measured by 2 g 2 c. G)
Conditions for Two Circles to Touch Each Other: Let the centers of the two circles be C1 and C2. The two
circles can touch each other outside the circle as shown below:
Figure 28
345
Here r1 is the radius of the first circle having centre C1 and r2 is the radius of the second circle having
centre C2 such that r1 < r2 . The distance between two circles is given by the line C1C2 as shown in the
figure, where
C1C2 = r1 + r2 , if the two circles touch each other outside, and
C1C2 = r2 – r1 , if a smaller circle is drawn inside the bigger one as shown in the figure 29.
Figure 29
Example 10.34: Find the coordinate of the centre and the radius of each of the circle given below:
( i ) x2+y2 – 8x + 6y + 9 = 0, ( ii ) 4 x2 +4 y2 +24x – 4 y – 27 = 0, (iii ) x2+y2 – 6x – 8 y + 21 =0,
( iv ) x2 + y2 + 4x + 8y – 30 = 0, ( v ) x2 + y2 – 9 = 0.
Solution:
( i ) The general equation of a circle is
x2 + y2 + 2 g x + 2 f y + c = 0, where centre of the circle is ( -g, -f) and radius,r = g2 f 2 c
In our case, the equation is x2 + y2 – 8x + 6y + 9 = 0
or x2+y2 + 2( -4 ) x + 2( 3 ) y + 9 = 0
where g = - 4, f = 3 and c = 9. But , radius, r = g 2 f 2 c (4) 2 32 9 16 4 .
Therefore , the centre is ( 4, -3 ) and radius = 4.
( ii ) We have, 4x2 + 4y2+ 24 x – 4 y – 27 = 0
27
or x2 +y2 + 6 x – y - = 0.
4
1 27
or x2+y2+ 2 ( 3) x + 2 ( - ) y + ( ) =0.\,
2 4
1 27 1 2 27
Here g = 3, f = and c = . Hence, radius, r = g f c 3 ( ) ( )
2 2 2
2 4 2 4
1 27 36 1 2 64 1
= 9 = 16 4 .Therefore, the centre is ( -3, ) and radius, r =4.
4 4 4 4 2
( iii ) The standard equation of a circle is given by
( x – a )2+ ( y – b )2 = r2 , where ( a, b ) is the coordinate of the centre of the circle and r is the
radius.
We have the equation of the circle
x2 + y2 -6 x – 8 y +21 = 0
or x2 – 2 (3)x + 32 + y2 – 2(4)y + 42 = 4
or ( x – 3)2 + ( y – 4)2 = 22
346
Therefore, the centre is ( 3 , 2 ) and radius, r = 2.
( iv ) The given equation is
x2 + y2 + 4x + 8y – 30 = 0
or x2 + 2 ( 2)x + 4 + y2 + 2 ( 4)y + 16 – 30 – 20 = 0
or ( x + 2 )2 + ( y +4)2 = 50
or { x – (-2) }2 + { y – ( -4) }2 = ( 50 )2.
Therefore, the centre is ( - 2, -4 ) and radius , r = 50 .
( v ) The given equation is
x2 + y2 – 9 = 0
or ( x – 0 )2 + ( y – 0 )2 = 32
Therefore, the centre of the equation is ( 0 , 0 ) and radiu, r = 3.
Example 10.35: ( a ) A shopkeeper sells round shape wedding plate containing round shape make-up
box. The price of each plate is Tk.2000.00 and the price of each make-up box is Tk. 100.00. Another
shopkeeper sells the similar items at different prices. The price of the plate is Tk.1500.00 and the price of
price of each make-up box is Tk. 150.00. A customer, on investigation, observed that the total cost of the
wedding plate including make – up box remains same. How many make-up boxes are given in a plate?
( b ) In a circular shape land 11 pillars each at a distance of 12 meters are constructed at the boundary of
the land. What are the radius and diameter of the land? What is the area of the land?
( c ) In a circular shape land of radius 28 meters some pillars are constructed in the boundary of the land
each at a distance of 16 meters. A pillar costs Tk. 4000.00. What is the total cost in constructing pillars ?
Solution:
( a ) Let r be the number of make-up boxes in a wedding plate . Total cost of plate of shopkeeper – 1 is
Tk. (2000 + 100 r). The same for shopkeeper-2 is Tk.( 1500 +150 r). But, total costs remain same in each
case, i.e. 2000 + 100 r = 1500 + 150 r.
or 2000 – 1500 = 150 r – 100 r
or 50 r = 500
500
or r= 10 .
50
( b ) As 11 pillars each at a distance of 12 meters are constructed, the circumference of the circular land is
( 11 x 12 ) = 132 meters. The circumference of a circle is given by
22
Circumference = 2 π r = 2 x x r. Given circumference = 132 meters.
7
22 132 7
Therefore, 132 = 2 x x r , or r = 3 7 21 meters.
7 22 2
The radius of the circle is r = 21 meter and the diameter = 2 r = 2 x 21 = 42 meters. The area of the circle
is
22
Area = π r2 = 212 = 1386 sq.meter.
7
( c ) Given the radius of the circular land r = 28 meters. The circumference of the land is
22
Circumference = 2 π r = 2 x 28 = 176 meters.
7
176
A pillar is constructed at a distance of 16 meters. Total constructed pillars are 11 . Each pillar
16
costs Tk.4000.00. So, the total cost in constructing 11 pillars is ( 4000 x 11 ) = 44,000.00. Tk. 44,000.00.
Example 10.36: A rice mill owner has a circular land , the radius of which is 63 meters. He has a plan to
develop the land by cement- concrete works. An engineer estimated that for development the cost per
square meter will be Tk. 2000.00. What will be the total cost for this development? If in the boundary
347
pillars are constructed at a distance of 4 meters and each pillar costs Tk. 2000.00, what will be the total
cost to construct pillars ?
Solution: The area of the circular land is given by
Area = π r2 , given the radius, r = 63 meters. So
22
Area = 63 2 12,474 sq.meters.
7
Each sq.meter needs Tk.2000.00. Therefore, total cost will be ( 12, 474 x 2000 ) = 2,49,48 000.00 .
The circumference of the circular land is 2 π r. Given r = 63 meters. So, the circumference is
22
Circumference = 2 x 63 396 meters.
7
396
Pillars will be constructed at a distance of 4 meters. Total pillars will be 99 . Each pillar costs
4
Tk.2000.00. Therefore, total cost in constructing pillar is ( 99 x 2000 ) = 1,98,000. Tk.1,98,000.00.
Example 10.37: ( a ) The centre of a circle is on Y – axis and it passes through the points ( 4,0) and ( -
4,2). Find the equation of the circle and its radius.
( b ) The centre of a circle is at X- axis and it passes through the points ( 0,5 ) and ( -2, 3 ). Find the
equation of the circle and its radius.
Solution:
(a ) The general equation of the circle is
x2 + y2 + 2gx + 2 fy + c = 0, where centre of the circle is ( -g , -f ) and radius r =
g2 f 2 c
As the centre is on Y-axis, g= 0. Hence the equation stands
x2 + y2+ 2fy + c = 0
Again, the circle passes through one of the point ( 4, 0 ) . From this point , we have
42 + c = 0.
It gives c = -16 ……… (i).
The circle also passes through the point ( -4, 2). From this point, we have
(-4)2 + 22 + 2 f x 2 + c = 0, or 20 +4f – 16 =0 , as c = - 16
It gives 4 f +4 = 0 …..(ii)
348
Example 10.38: A circle passes through three points ( 2, 1 ), ( -6 , 5 ) and ( - 3 , -4 ). Find the equation
of the circle.
Solution: The general equation of the circle is
x2 + y2 + 2 gx + 2 f y + c = 0
The circle passes through 3 points. For the first point ( 2,1 ), we have
22+12 + 2(g) 2 + 2 f ( 1 ) + c = 0 , or 4g + 2f + c = - 5 …… ( i )
Using the second point ( -6 , 5 ), we have
( -6)2 + 52 + 2 g ( -6) + 2 f ( 5 ) + c = 0 , or 12 g - 10 f - c = 61 ……. ( ii )
Using the third point ( - 3 , - 4 ), we have
( - 3 )2 + ( - 4 )2 + 2g ( -3 ) + 2f ( - 4 ) +c = 0, or 6g + 8 f – c = 25 ……. (iii)
Multiplying (i) by 12 and (ii) by 4, we get
48 g +24 f +12 c = - 60 … …(iv)
48 g – 40 f – 4 c = 244 ……. (v)
Subtracting (v) from (iv), we get 4 f + c = - 19 …… (vi)
Multiplying (ii) by 6 and (iii) by 12, we get
72 g – 60 f – 6 c = 366 ……. .(vii)
72 g + 96 f – 12 c = 300 ……… (viii)
Subtracting (viii) from (vii), we get 26 f – c =-11 …….. ( ix )
Adding ( vi) and ( ix), we get 30 f = - 30 , or f = -1
Putting the value of f in ( vi), we get 4 ( - 1 ) +c = - 19 , or c = - 15.
Putting th value of f and c in (i), we get 4 g + 2(-1) – 15 = -5 , or 4 g = 12 , or g = 3.
Therefore , the centre of the circle is ( - 3, 1) and the radius is
r = g 2 f 2 c 32 (1) 2 (15) 25 5
The equation of the circle is
x2 + y2 + 2 gx + 2 fy+ c = 0
or x2 + y2 + 2 (3 )x +2 (-1) y + (-15)= 0
or x2 + y2 + 6x – 2y – 15 = 0.
Example 10.39: ( a) Find the equation of the circle the centre of which is ( 3,4) and touches the circle x2
+y2 = 9 externally.
(b) Find the equation of the circle the centre of which is ( -3, 4 ) and touches the circle x2 + y2 = 4
internally.
Solution:(a ) Let us consider that the equation of the circle is
( x – a )2 + ( y – b )2 = r2 , where the coordinate of the centre is ( a, b) and radius r.
The coordinate of the centre is ( 3,4 ) but the radius r is not known. The equation can be written as
( x – 3 )2 + ( y – 4 )2 = r12
This circle touches another circle
x2 + y2 = 32
The centre of this circle is (0,0 ) and radius r2 = 3. The positions of these two circles are shown in Figure
30.
349
Figure 30
As two circles touch each other externally, there is a distance between the centres of these circles. Let C1
be the centre of circle-1 and C2 be the centre of circle -2. Then the distance between two points C1 and C2
is given by
C1C2 = r1 +r2 = r1 + 3.
Again, the distance between two points C1(3,4 ) and C2 ( 0, 0 ) is given by
C1C2 = (3 0) 2 (4 0) 2 5
Thus, we have r1+3 = 5, it gives that r1 = 5 – 3 = 2.
Therefore, the required equation of the circle is
( x – 3 ) 2 + ( y – 4 ) 2 = 22
or x2 + y2+ 6x – 8y + 21 = 0.
( b ) Let the equation of the circle be
( x – a )2 + ( y – b)2 = r12 , where the centre point C1 ( a , b) and r1 is the radius.
or { x – (-3)}2 + ( y – 4 )2 = r12
or ( x +3 )2 + ( y – 4 )2 = r12
This circle touches another circle
x2 + y2 = 22 = r22. Here r2 = 2 is the radius of the second circle which is touched by the first
circle internally .
The radius r1 of the first circle is ( r2 + the distance C1C2). The distance C1C2 is the distance between two
points ( 0,0 ) and ( -3, 4). It is given by
C1C2 = ( x1 x2 ) 2 ( y1 y 2 ) 2 {0 (3)}2 (0 4) 2 5
Therefore, the radius r1 = r2 + 5= 2 +5 = 7. Hence, the equation of the required circle is
( x + 3 ) 2 + ( y – 4 ) 2 = 72
or x2 + 6x + y2 – 8y – 24 = 0.
Example 10.40: Find an equation of a circle which passes through to points ( -2 , -3 ) and ( 2 , 3 ) and
centre of which lies on the straight line x + 4 y + 5 = 0.
Solution: Let ( -g , -f ) be the centre of the circle. As the centre is on the line x + 4y +5 =0, the coordinate
of the centre satisfies the line. Hence, we have
. – g – 4 f + 5 = 0 , or g + 4 f = 5 ……. … (i)
The equation of the circle is
x2 + y2 + 2gx + 2 fy + c = 0
The circle passes through the points ( -2 , -3) and ( 2, 3 ). For the first point, we have
( - 2 )2 + (-3)2 + 2g ( -2) + 2 f ( -3) + c = 0
or 4g + 6f – c = 13 ……………(ii)
For the second point, we have
350
22 + 32 + 4g + 6f + c = 0
or 4g + 6f + c = -13 ………… … (iii)
Adding (ii) and (iii ), we get 8g + 12 f = 0 , or 2g + 3 f = 0 ………..(iv)
Subtracting ( iv) from { 2(i)}, we get 5 f = 10, or f = 2. Putting the value of f in (iv),
we get 2 g + 3 (2) = 0, or g = - 3. Putting the values of g and f in (ii), weget
4(-3) + 6 (2 ) – c = 13
or 3 = - 13
The radius of the circle is r = g 2 f 2 c (3) 2 2 2 (13) 26
The centre of the circle is ( 3, - 2 ). Hence, the equation of the circle is
x2 + y2 +2 (-3)x + 2 ( 2 )y + c = 0
or x2 + y2 – 6x + 4 y + 26 = 0.
Example 10.41: ( a ) Find an equation of a circle the coordinate of the centre of which is ( - 5, - 4 ) and
radius is 5.
( b ) Find an equation of a circle the coordinate of centre of which is ( -2 , 4 ) and radius is 7. Also, find
the length of circumference of this circle.
Solution: The general equation of a circle is
x2 + y2 + 2gx + 2f y +c =0,
where the centre of the circle is (-g ,-f). From the given condition g = 5, f = 4 and radius, r = 5. Again,
r2 = g2+ f2 – c , or 52 = 52 + 42 – c , or c = 16. Therfore, the the equation of the circle is
x2 + y2 + 2(5) x + 2 ( 4 ) y+ 16 = 0
or x2 + y2 + 10 x +8y + 16 = 0.
( b ) The equation of a circle when the coordinate of the centre is ( a, b ) and r is the radius is
( x – a )2 + ( y – b )2 = r2
Given a = -5, b= -4 and r = 7. Hence, we have
{ x ( -5)}2+ {y-(4)}2 = 72
or x2 + y2 – 10x – 8y – 8 = 0.
22
The length of the circumference is 2 πr = 2 7 = 44 units.
7
Supplementary Problems
1.(a) Find the equation of a straight line which passes through the points ( 5,7 ) and ( 0, -4).
Ans. 11x – 6y -24 = 0
(b) Find the equation of the straight line which passes through the point ( 3, 5) and form an angle of
1350 with X –axis. Ans. x + y – 8 = 0.
2.(a) Find the slope and y-intercept of each of the following straight lines:
4 3
(i) 4x – 3y – 12 = 0, Ans. m = and c = -4; (ii) 3x + 4y + 16=0, Ans. m= , c = -4 .
3 4
(b) Are the above two lines perpendicular to each other ?
4 3
Ans. yes; as m1 = and m2= , then m1m2= -1. It indicates that the two lines are perpendicular.
3 4
3.(a) Find the point of intersection of the two lines given below:
4x – 3y – 12 =0 and 3x + 4y + 16 = 0, Ans. ( 0, -4).
(b) The equation of two straight lines are given below:
2x + y = 16 and 4x + 2y = 33
Are the two lines parallel? If so, find the distance between these two lines.
Ans. yes, distance = 0.224.
4.(a) Three straight lines are given below :
351
3x + 2y – 5 = 0, 4x – y +3 = 0 and ax + 3y – 6 = 0. If these lines pass through the same point, find
the
11
value of. Ans. .
6
(b) The straight line equation is 6x – 12 y + 11 =0, find slope and y-intercept of the equation.
11
Ans. m = 0.5, c = .
12
5.(a) Find the equation of the straight line which passes through the point ( - 2 , 3 ) and is perpendicular
to the straight line 3x – y + 6 = 0. Ans. x + 3y – 7 = 0.
(b) Find the equation of the straight line which passes through the point ( 3, 4 ) and is parallel to the
line 6x – 3y + 5 = 0. Ans. 2x – y – 2 = 0.
6. Find the equation of a straight line which passes through the points ( -2 , 3) and (4 , -1). Also, find an
equation which is perpendicular to this line. Ans. 2 x + 3y – 5 =0 ; 3x – 2y + 12 = 0.
7. Find the slope and cut point of the axes by each of the straight lines (a) 6 x – 5 y + 5 = 0., and
(b) x + 2 y – 6 = 0, (c) 3x – 4 y + 15 = 0.
6 5 1 3 4
Ans. (a) m= ; a = , b = 1; (b) m= ; a=6, b=3; (c) m= ;a=-5,b= .
5 6 2 4 15
8.(a) Find the distance between the points P( 1, - 3 ) and Q ( -2 , 4). Ans. 58 .
(b) Find the distance between the points P ( 0, 4 ) and Q ( 4, 0). Ans. 4 2 .
(c) Find the distance between the Points P ( 6,4 ) and Q( 0, 12 ). Ans. 10.
9.(a) A man purchased a machine at a price Tk. 50000.00 in a year. After 10 years the price of the
2
machine was reduced to Tk.5000.00. Find the rate of depreciation of the price. Ans. m = .
9
(b) A machine was bought in a year at a price Tk.100000.00. After 10 years the price was reduced to
Tk. 5000.00. Express this depreciation of the price over time. Ans. 19 x + 2 y - 200 = 0.
10.(a) Find the equation of the circle the radius of which is 10 and coordinate of the centre is ( 11, 2).
Ans. x2 +y2 – 22 x – 4 y + 25 = 0.
(b) Find the coordinate of the centre and radius of the given circle
x2 + y2 – 4x + 6y – 14 =0. Ans. ( 2 , - 3 ); r = 1.
11.(a) A circular plot of radius 21 meters is to be developed by bricks works at a cost of Tk. 6000.00 per
Square meter. Find the total cost of development of the plot. Ans. Tk.83,16,000.00
(b) In the boundary of a circular plot of radius 14 meter some pillars are to be constructed each at a
distance of 11 meter. Each pillar will cost Tk. 2500.00. What will be the total cost to construct the
pillars ? Ans. Tk. 20.000.00.
12.(a) Find the distance between each pair of Points (i) P ( 4, -2 ) and Q ( - 3, 6); (ii) P ( 3,6 ) and Q( -
4,2).
Ans. (i) 113 , (ii) 65 .
(b) Find the equation of the straight line which touches the points ( 4, -2 ) and (- 3, 6).
Ans. 8 x + 7y - 18 = 0.
13.(a) Find an equation of the straight line which passes through the point ( -3 , 8) and cuts the axes at
equal distance. Ans. x + y – 5 = 0.
(b) Find an equation of the straight line which passes through the intersection point of the straight
lines x – 2y – 1 = 0 and 2x + 3y + 2 = 0 and the slope of the line is tan 450. Ans. 7x – 7 y – 11
=0.
14. Show that the three points P ( 4 , -1 ), Q ( 2 , 1 ) and R ( 1 , 2 ) lie on one straight line.
15. Let P be a point on X –axis and its distances from the point ( 0.3) and ( 5 , -2) are same. Find the
coordinate of P. Ans. ( 2 , 0).
16. Find the area of the triangle formed with the points ( - 4, 0), ( 4, 0) and (0,4). Ans. 16 units.
352
17. Find the value of x for which the 3 points A ( x , -1 ), B ( 2 , 3 ) and C( 0, 1 ) lie on a straight line.
Ans. x = - 2.
18. A television was bought at a price of Tk. 30,000.00 in a year. After 25 years it had no sale value.
What is the rate of depreciation ? Ans. –Tk.1200.00
19. From a distribution centre of television 5 televisions are sold in 3 hours. Again, 6 televisions are
sold in 4 hours. How much time is needed to sell 8 televisions ? Ans. 6 hours.
20. In a cyber café 10 customers are served in 5 hours. Another 12 customers are served in 8 hours. If
time of service is linearly related to number of customers, what will be the time to serve 15
customers? Ans.12.5 hours.
353
Chapter XI
Calculus
11.1 Introduction
Calculus is a branch of mathematics of counting and calculations to study the continuous change
simultaneously in two or more variables, where one variable is a dependent one which depends on
another one or more variables and variables are either linearly or non-linearly related. Depending on a
specified mathematical function one may need to measure the rate of change of dependent variable for
unit change in independent variable or one may need to measure the total area under the curve expressed
by functional form of the variables. It indicates that there are two branches of calculus, namely ( a )
Differential Calculus, and ( b ) Integral Calculus. Whatever be the type of calculus, both the branches
do follow some fundamental rules of calculation and hence both the methods are related to each other.
Both the branches make use of the fundamental notions of convergence of infinite sequences and finite
series to a well defined limit.
Calculus is usually developed in working with very small quantities. Historically, the first method to do
so was infinitesimals involved in real numbers but which are infinitely small. For example, an
infinitesimal number could be greater than zero but less than any number in the sequence 1, 0.5, 0.33,
0.25 , …….. and thus any positive real number. From this point of view, calculus is a collection of
techniques for manipulating infinitesimals. As a symbol of infinitely small change in x the term dx and
dy
infinitely small change in y the term dy are used and their ratio is calculated. This ratio is the
dx
derivative of y with respect to x which is usually used to measure the change in y for unit change in x.
Let us consider that X is a variable and x is a value of X. There may be an increase or decrease in the
value of x. Assume that h= x be the change in the variable X and it occurs when the variable value
increases or decreases from one value x = x0 to another value x = x1 in its domain. Here h= x = x1 –
x0 and we may write
x1 = x 0 + x or x1= x0 + h
354
y f ( x0 x ) f ( x0 ) h
lim lim lim 1
x x h
h x 0 h x 0
This limit is known as Derivative of y with respect of x. Thus, the derivative of the function y = f( x )
with respect to x at the point x = x0 is given above , provided the limit exists. This limit is also called the
instantaneous rate of change y with respect to x = x0 . Thus, calculus is a collection of techniques for
manipulating certain limits infinitesimals get replaced by very small numbers and the infinitely small
behavior for smaller and smaller numbers. The limits were the first way to provide rigorous foundations
for calculus.
It is the study of the definition, properties, and application of derivative of function. The process of
finding the derivative is called Differentiation . Given a function y =f ( x ) at a point in the domain , the
derivative at that point is a way of encoding the small- scale behavior of the function near that point. By
definition the derivative of a function at every point in its domain , if it is possible to produce a new
function, called the derivative function or just the derivative of the original function in formal terms. The
derivative can take the squaring function as an input. This means that the derivative takes all the
information of the squaring function – such as that two is sent four , three is sent to nine, and so om.
An integral assigns numbers in a way that can describe displacement, area, volume, and other concepts
that arise by combining infinitesimal data. Integration is one of the two main operations of calculus, with
its inverse operation, differentiation, being the other. Given a function f (x ) of a real variable X and an
interval [ a, b ] of the real line, the definite integral
b
a
f ( x ) dx
Is defined informally as the signed area of the region in the xy – plane that is bounded by the graph of f(x)
, the x –axis and the vertical lines x = a and x =b. The area above the x – axis adds to the total and tht
below the x – axis subtracts from the total.
The operation of integration , up to an additive constant, is the inverse of the operation of differentiation.
For this reason , the term integral may also refer to the related notion of the anti –derivative , a function
F(x) whose derivative is the given function f(x). In this case, it is called an indefinite integral and is
written as
F(x ) = f ( x) dx
Here the symbol dx is used to represent an infinitesimally „Small Value‟ of the indefinite variable x to be
multiplied by the integral and summed up in an infinite sense.
355
The fundamental principles related to calculus are :
( i ) Let f be a continuous real-valued function defined on a closed interval [ a , b ]. Let F be the function
defined , for all x in [ a , b ] by
x
F ( x) f (t ) dt
a
F/ ( x) = f(x)
For all x in ( a , b ).
( ii ) Let f be the real-valued function defined on a close interval [ a , b ] that admits an anti-derivative F
on [ a , b ] . That is f and F are functions such that for all x in [ a , b ]
f( x ) = F/(x )
b
a
f ( x) dx = F ( b ) – F ( a ).
11.4 Rules for Differentiating Functions and Some Standard Results of Derivatives.
We have already discussed that a function f is said to be differentiable at x = x0 if the derivative f/( x0)
Now, the problem is to find the derivative . The process of finding the derivative of a function is called
differentiation.
356
d n d 1 n d d 1 1 d
1. ( x ) n x n 1 , 2. ( n ) n 1 , 3. ( x) 1, 4. ( x ) , 5. (e x ) e x
dx dx x x dx dx 2 x dx
d 1 d 1 d d
6. (log x ) , 7. [log a ( x) ] log a e , 8. (a x ) a x log e a , 9. (sin x) cos x ,
dx x dx x dx dx
d d d d 1
10. (cos x) sin x , 11. (tan x) Sec 2 x , 12. (cot x) cos ec 2 x , 13. (sin 1 x)
dx dx dx dx 1 x2
d d d 1
14. (sec x) sec x tan x , 15. (cos ec x) cos ec x cot x , 16. (cos1 x)
dx dx dx 1 x2
d 1 d 1 d 1
17. (tan 1 x) ,18. (cot 1 x) ,19. (cos ec 1 x)
dx 1 x 2
dx 1 x 2
dx x x2 1
d 1
20 . (sec 1 x)
dx x x2 1
21. n th Derivative of Some Functions :
m!
(a) D n ( x n ) n ! , (b) D n ( x m ) x m n , where m n and it is positive int egar.
( m n) !
1 (1) n n !
(c ) D n (e x ) e x , (d ) D n (e ax ) a n e a x , (e) D n ( )
xa ( x a) n 1
(1) n 1 (n 1) !
( f ) D [log ( x a)]
n
( x a) n
x2 x3 x4
ex 1 x ....................
2! 3 ! 4!
x2 x3 x4 xn
log(1 x) x ..............(1) n 1 ...... [ -1< x < 1. ]
2 3 4 n
x2 x3 x4
log(1 x) x .............
2 3 4
1
1 x x 2 x 3 x 4 .......[1 x 1]...
1 x
1
1 x x 2 x 3 x 4 ...........[1 x 1]
1 x
11.5 Differentiation
357
Differentiation is a mathematical technique of exceptional power versatility. It is one of the central
concept of calculus and it has a variety of applications. The most important applications of differentiation
are drawing of curve , optimization of function, analysis of rates of changes in dependent variables for
changes in explanatory [ independent ] variables.
Optimization Problem
In business there is an attempt to maximize the profit. For example, let us consider that a manufacturer of
cement wants to maximize his profit from the sale of cement. The profit depends on monthly sale, where
each bag of cement is sold for x taka. Also, consider that the profit is a function of x, where
f ( x ) = 100 ( 5 –x )
Figure 1
From the graph it is seen that there is an optimal selling price x at which the profit of the manufacturer
will be maximum. This optimal profit is the x coordinate of the peak of the graph and at this point the
slope of the curve is m of the equation and this m is the derivative of y with respect to x, i.e.
dy df ( x)
m
dx dx
Thus to identify the optimum profit ( any function ) we can apply the technique of differentiation of
calculus.
Rate of Change
dy dy
For a linear function the rate of change is measured by dx and dx is the derivative of the function
358
X 1 2 3 4
Y 2 4 6 8
dy
Here dx 2 indicates that if x increases by I unit , y increases by 2 units. This is observed also from
the set of x and y values shown above. The graph of y = f(x ) with the above 4 pairs of values or with any
other pairs of values will depict a straight line as shown below:
Figure 2
Again, let us consider another function of y = f ( x ) = 8 – 2x such that for different values of x the y
values are in decreasing trend, i.e.
x 1 2 3 4
y 6 4 2 0
where the value
dy
2
dx
indicates that if x increases by I unit, y decreases by 2 units. The change in y for unit change in x is called
the rate of change of y for change in x. The graph of this function is shown below:
359
Figure 3
In trigonometric language this rate of change of y for unit change in x is known as slope of the straight
line , where both the functions represent equations of the straight lines. Here the rate of change of f(x)
with respect to its independent variable is equal to the slope ( steepness) of its straight-line graph and this
slope is constant.
If the function of x is not linear, its rate of change with respect to its independent variable is still the
steepness of its graph and is measured , in this case, by the slope of the line that is tangent to the graph at
the point in question. However, if the graph is not a straight line, the slope is not constant. It varies from
point to poin
Example 11 1: It is estimated that the production of garments x months from now will be
f(x ) = x2 + 50 x +1000
( a ) at what rate will the production be changing with respect to time 12 months from now ?
Solution:
The rate of change of production 12 months from now will be f/(12) = 2 x12 +50= 74 per month.
( b ) The actual change in the production during 18 month is the difference between the production at the
end of 18 months and the production at the end of 12 months, i.e.
Example 11.2: It is assumed that x months from now , the production of cement bags of a certain
industry follows the function f (x ) = x2 + 40 x + 1200.
360
( a ) At what rate will the production be changing with respect to time 12 months from now?
( c ) If the current monthly production is 10000 bags , what is the percentage rate of change per month ?
(d) What is the percentage rate of change with respect to time 12 months from now?
Solution:
The rate of change of production 12 months from now is f/(12 ) = 2 x 12 + 40 = 64 per month.
( b ) The actual change in the production during the 13th month is the difference between the production
at the end of 13th month and at the end of 12th month. It is measured by
= 1889 – 1824 = 65
( d ) Here f/( 12 ) = 64 and f ( 12 ) = [ (12)2 + 40 x 12 + 1200 = 1824. Therefore, the percentage change
with respect to tie 12 months from now is
f / (12) 65
100 100 3.56
f (12) 1824
Example 11.3: The gross national product ( GNP ) of a countryis a function of time t and is given by
( b ) At what percentage rate is the GNP changing with respect to time in 2019?
Solution:
361
N/( t ) = 2 t + 1
N / (9) 19
Percentage rate of change 100 100 5.28 % per year
N (9) 360
Example 11.4: It is assumed that x years from 2011, the population of a country will be
P ( x ) = x2 + 2x + 52 million
( b ) At what percentage rate will the population be changing with respect to time 2019?
Solution:
( a ) The rate of change of population is given by P/( x ) = 2 x + 2. The rate of change in 2019 after 2011
is given by P/( 8 ) = 2 x 8 + 2 = 18 million.
( b ) The percentage rate of change of population with respect to time 2019 will be
P / (8) 18
100 100 13.64 %
P (8) 132
The rate of change of any quantity is measured by f/( x ) , where f( x ) is a function of x, i.e y = f( x ).
Here
dy
f / ( x)
dx
We have already discussed this rate of change and have applied in calculating some rates of change. The
percentage rate of change is then given by
362
The Slope of a Tangent:
It has already been discussed that using calculus we can solve optimization problem and we can compute
the change of dependent variable for unit change in independent variable provided there
exists a function which admits the derivative at a given point . The change of the dependent variable for
change in independent variable can be calculated if there exists a procedure for finding slope of the
tangent to the curve at a given point. Let us now discuss what does the word Tangent indicate.
The tangent line or simply the tangent to a plane curve at a given point is the straight line that „just
touches‟ the curve at that point. Leibniz defined it as the line through a pair of infinitely close points on
the curve. Moreover, a straight line is said to be a tangent of a curve y = f(x) at a point x = c on the curve
if the passes through the point { c , f ( c )} on the curve and has slope f/ (c ), where f/( c ) is the derivative
of f. Here slope or gradient of a line is a number that describes both the direction and steepness of the
line. From the geometrical point of view , if a straight line passes through two points A ( x1, y1) and
B(x2, y2), the slope of the line passing through the points is given by
y 2 y1 y
Slope, m
x 2 x1 x
But, if we know only one point of tangency { c, f ( c )} , the slope can be calculated directly as shown by
y
the above formula . However, it is possible if it is managed to show that x 0 so that x tends to a
number . The method can be applied by an example.
Given f(x) = x2 at a point ( 2,4 ) . We need to find the slope of the line that is tangent to the given curve.
Here the x coordinate of the given point is 2, it follows that the x coordinate of the neighboring point is
2 x and that the y coordinate of this point is (2 x) 2 . Hence
(2 x) 2 4 x 2 4x 4
Slope of the sec ant x 4
x x
Let x 0 ,then the slope tends to 4. This means that at the given point ( 2 , 4 ) , the slope of the tangent
approaches 4. Again , let us consider the point of tangency as ( x , x2 ) and the neighboring point ( x+ x
), ( x + x )2 . If the curve of y = x2 is drawn, then the slope of the secant through ( x + x2 ) is given by
( x x) 2 x 2 x 2 x 2 2x x 2
Slope of the sec ant x 2 x
x x
This slope approaches 2x if x 0. This means that at the point ( x , x2 ) , the slope of the tangent is 2x.
Now, if the point is ( 2 , 4 ) , the slope is 2 (2 ) = 4.
363
Here we have considered a function f ~ y =x2 and related to this function we have derived the slope of its
tangent in terms of x coordinate of the point of tangency. This derived function is known as Derivative
and it is denoted by f/.Thus, if y = f( x ) = x2, then f/( x ) = 2 x and this derivative f/(x) expresses the slope
of the tangent of the curve y = f( x ) as a function of x coordinate of the point of tangency .
f ( x h) f ( x )
h
This expression is Newton‟s difference quotient [ also known as first order divided difference ]. The slope
of this secant line differs from the slope of the tangent line by an amount that is approximately
proportional to h.
Step – 3 : As h approaches zero, the slope of the secant line approaches the slope of the tangent line.
Therefore, the true derivative of f at x is the limit of the value of the difference quotient as the secant lines
get closer and closer to being a tangent line:
f ( x h) f ( x )
f / ( x) lim
h
h0
This result is the derivative and is denoted by f/(x ) or simply f/ Since immediately substituting 0 for h
results in division by zero . Calculating the derivative directly can be unintuitive of the difference
quotient. Equivalently , the slope could be estimated by employing ( x – h ) and x. This method is known
as Method of Derivative by First Principle.
Example 11.5: Let us consider that the profit of an owner of a garments industry depends on how many
shirts are sold in a day and this profit is a function of x , where x is the price of a shirt. The function is
f = f( x ) = y = 500 ( 10 – x ) ( x – 3 )
364
f ( x h) f ( x ) 500( x h) 2 6500 ( x h) 1500 500 x 2 6500 1500
f / ( x ) lim lim
h h
h0 h0
Now, if x = 6 , f/( x ) = 500. If we think that the slope of the tangent is zero, we have
f/( x ) = 0
It means that x = 6.5 is the x coordinate and the optimum selling price is 6.5 unit.
Example 11.6: A bag of cement is produced at a cost of Tk.600.00. The selling price of a bag is Taka x.
If the selling price is x taka , the consumer will buy ( 800 – x ) bag per day. Find the value of x so that the
producer‟s profit becomes maximum.
Solution: As the selling price is x taka per bag, the profit per bag is Tk.( x – 600 ). If ( 800 – x ) bahs are
sold , total profit per day becomes ( 800 – x ) ( x – 600 ). Hence
f ( x h) f ( x )
f / ( x) lim
h
h0
( x h) 2 1400 ( x h) 480000 ( x 2 1400 x 480000 )
lim
h0
h 2 1400 h 2 xh
lim
h
h0
1400 2 x
The profit will be maximum if the slope of the tangent becomes zero. Then
1400 – 2x = 0
Or x = 700
365
That means if selling price is Tk. 700.00, the profit will be maximum.
( b ) Find the equation of the line that is tangent to the graph of the above function when x = 2.
Solution: ( a ) We have
1
f ( x)
x
f ( x h) f ( x ) 1 1 1 x ( x h)
f / ( x) lim lim [ ][ ]
h h x h x x( x h
h0 h0
x xh
x ( x h) x ( x h) h 1 1
lim lim 2 , h 0
h x ( x h) h x ( x h) x
h0 h0
y – y1 = m ( x – x1 )
or y – ½ = - ¼ ( x – 2 )
or y = -( 1/4) x +1 or x – 4 y – 4 =0
Example 11.8: Find the derivative of each of the following functions and find the slope of the line that
is tangent to the graph for the specific value.
( a ) y = f( x ) = 5 x – 4 , ( b ) y = f( x ) = 2 x2- 3 x + 5, ( c ) y =f( x ) = ( 2 / x ) , x = ½ ,
Solution: ( a )
366
f ( x h) f ( x ) 5 ( x h) 4 5 x 4 5 x 5h 4 5 x 4
f / ( x) lim lim lim
h h h
h0 h0 h0
5h 4
lim 5.
h
h0
(b)
f ( x h) f ( x ) 2 ( x h) 2 3 ( x h) 5 2 x 2 3 x 5
f / ( x) lim lim
h h
h0 h0
2 h 2 4 xh 3h
lim lim 2h 4 x 3 4 x 3
h
h0 h0
Slope of the tangent when x = 0 is 5. The equation of the line which passes through the point ( 0, 5) and
the slope of the line – 3 is y + 3x-5 = 0.
(c)
2 2 2 x 2( x h)
f ( x h) f ( x ) 2h 2
lim x h x lim xh 1
f / ( x) lim lim 2
h x h h x ( x h) x
h0 h0 h0 h0
The slope is
1 2
m f /( ) 8
2 1
( )2
2
Therefore, the y coordinate of the tangency when f( 1 / 2 ) = 4. The tangent line is
y – y1 = m ( x – x1)
( y – 4 ) = - 8 ( x – 0.5 0)
Or y +8x – 8 = 0
( d)
367
f ( x h) f ( x ) ( x h) 2 1 x 2 1 x 2 h 2 2hx x 2
f ( x) lim
/
lim lim 2x
h h h
h0 h0 h0
The slope of the tangent line is m = f/(-1) = -2, when x = -1. The y coordinate when x = -1 is y= (-1)2-1=0
( y –y1 ) = m ( x – x1)
Or y – 0 = -2( x + 1 )
Or 2x + y +2 = 0.
(e)
1 1 x xh
f ( x h) f ( x ) xh x x xh x xh
f / ( x) lim lim lim
h h h x h
h0 h0 h0
xxh 1 1 1 1
lim
h x xh x xh x 2 x 2x x
h0
1 1
Here m f / (4)
2 4 4 16
1 1
f (4) y,
4 2
Solution:(a)
f ( x h) f ( x ) e xh e x e x [e h 1] 1 h 2 h3
f / ( x) lim lim lim e x lim [ 1 h ........ 1]
h h h h 2! 3 !
h0 h0 h0 h0
1 h 2 h3
ex lim [1 h ..........] e x
h 2! 3!
h0
368
b)
f ( x h) f ( x ) ( x h) n x n 1 h 1 h
f / ( x) lim lim lim [ x n (1 ) n x n ] lim x n [( 1 ) n 1}
h h h x h n
h0 h0 h0 h0
1 n h h2 h3 h
lim x [ 1 n c1 n c 2 2 n c3 3 ............ n c n ( ) n
h x x x n
h0
1 h
lim x n [ n c1 n c 2 2 .......] n x n 1
x x
h0
1 2x 3
(a) f ( x) , (b) f ( x) , (c ) f ( x) x 2 3x 5
x2 3x 4
Solution: ( a )
1 1
f ( x h) f ( x ) x2 xh2 h
f / ( x) lim lim x 2 h x 2 lim lim
h h ( x 2) ( x h 2) ( x 2) ( x h 2)
h0 h0 h0 h0
1
( x 2) 2
(b)
369
f ( x h) f ( x ) 2 ( x h) 3 2 x 3
f / ( x) lim lim
h 3 ( x h) 4 3 x 4
h0 h0
1 { 2 ( x h) 3}{3x 4} { 3( x h) 4}{3x 4}
lim
h (3x 4) { 3( x h) 4}
h0
1 3( x 4) (2 x 3 2h) (3x 4) (3x 4 3h)
lim
h 3 ( x 4) {3( x h) 4}
h0
(6 x 8 6 x 9)h 17 17
lim lim
h (3x 4) { (3x 4 3h)} (3x 4) (3x 4) (3 x 4) 2
h0
(c)
f ( x h) f ( x ) 1
f / ( x) lim lim ( x h) 2 3( x h) 5 x 2 3x 5
h h
h0 h0
1 2
lim ( x h 2 2 xh 3x 3h 5 x 2 3x 5)
h
h0
1
lim {h(2 x 3) h 2 } 2 x 3
h
h0
( a) f ( x) 2 x 1 , (b) f ( x) 3 x , (c) f ( x) | x |
Solution: (a)
370
f ( x h) f ( x ) 1
f / ( x) lim lim [ 2 x 2h 1 2 x 1 ]
h h
h0 h0
2 x 2h 1 2 x 1 2 x 2h 1 2 x 1
lim
h 2 x 2h 1 2 x 1
h0
1 2 x 2h 1 2 x 1 2 2 1
lim lim
h 2 x 2h 1 2 x 1 2 x 2h 1 2 x 1 2 2 x 1 2x 1
h0 h0
1 1 1
Here the function f ( x) 2 x 1 does not exist if x 2 as f ( 2 ) 2(
2
) 1 0.
So, this function has right-hand continuity at x = -1/2. At x= -1/2 f1(x) is infinite.
(b)
f ( x h) f ( x ) ( x h)1 / 3 ( x )1 / 3
f / ( x) lim lim
h h
h0 h0
( x h)1 / 3 x 1 / 3 ( x h) 2 / 3 x 2 / 3 x 1 / 3 ( x h)1 / 3
lim
h ( x h) 2 / 3 x 2 / 3 x 1 / 3 ( x h)1 / 3
h0
xhx 1 1
lim 2/3 2/3
h { ( x h) 2/3
x ( x h) x
1/ 3 1/ 3 2/3
x x x
2/3 2/3
3x
h0
Here f/(0) does not exist but f(x) is continuous at x = 0. This means that if the derivative of a function
exists at x = 0 , the function is continuous . But the converse is not true.
(c)
f ( x h) f ( x ) ( x h) ( x )
f / ( x) lim lim 1
h h
h0 h0
Example 11.12: Find the derivative of each of the functions given below
( a ) f(x ) = 2x – 3 , ( b ) f( x ) = x2 +4x
371
and find the value of f( x ) when x changes from 3 to 4 for f( x ) given in (a) ,and from 0.5 to 0.8 for f(x )
given in (b).
Solution: ( a )
f ( x h) f ( x ) 2( x h) 3 (2 3) 1
f / ( x) lim lim lim [2 x 2h 3 2 x 3] 2
h h h
h0 h0 h0
(b)
f ( x h) f ( x ) 1
f / ( x) lim lim [( x h) 2 4( x h) x 2 4 x]
h h
h0 h0
1 1
lim [ x 2 h 2 2 xh 4 x 4h x 2 4 x] lim [h 2 2 xh 4h] 2 x 4
h h
h0 h0
Here f( 0 .5) = 0.25 +4 x 0.5= 2.25, f(0.8) = 0.64 + 4 x 0.8 = 3.84. Hence,
Example 11.13: The cost of production of a television set is Tk. 15000.00. If the selling price is x taka,
then there will be a demand of ( 20000 – x ) pieces. Express the producer‟s profit as a function of price.
Estimate the optimal selling price.
f( x ) = ( x – 15000 ) ( 2000 – x ).
35000 = 2x
372
Or x = 17500.
Example 11.14: The selling price of a 20 watts energy bulb is Tk. X. The cost of production of a bulb is
Tk. 75.00. The demand of bulb per day is ( 90 – x ). Express the producer‟s profit as a function of rice.
Find the optimal selling price.
Solution: As the selling price is x taka, the profit from a bulb is ( x – 75 ) taka. As demand is ( 90 – x ),
the profit function is
f( x ) = ( 90 – x ) ( x – 75 ) = - x 2 + 165x -6750.
Now,
The optimal selling price is found out from the equation f/( x ) = 0. Here the equation is
165 – 2x = 0
Or x = 82.5.
f ( x h) f ( x ) 3( x h) 2 3x 2 3( x 2 h 2 2 xh x 2 )
f / ( x) lim lim lim lim (h 6 x) 6 x
h h h
h0 h0 h0 h0
In section 11.4, we have discussed some rules of differentiation. Let us now investigate the results of
those rules:
dy
Let C be a constant and f(x ) = C. We need f/( x) i.e. we need dx when y = C. Here the value iof f/( x
)=0.
373
f ( x h) f ( x ) C C
f / ( x) lim lim 0
h h
h0 h0
f ( x) u v
is
f / ( x) f / (u ) f / (v).
f ( x) u v .
Let us give an increment h to x and the corresponding increment in u and v be hu and hv , respectively.
Then
f ( x h) f ( x ) 1
f / ( x) lim lim [ ( u hu ) (v hv ) (u v) ]
h h
h0 h0
d h h du dv
f / ( x) [ f ( x)] lim u lim v f / (u ) f /( (v)
dx h h dx dx
h0 h0
h 0, hu 0 and hv 0
Rule – 3 : The derivative of the product of derivable functions f( x )= u v, where u and v are derivable
functions. The result is given by
First function x derivative of second function + second function x derivative of first function.
Proof: Let f ( x ) = u v. As u and v are derivable functions, consider that the increment in x is hx . The
corresponding increments in u and v are hu and hv , respectively. Then
374
d f ( x h) f ( x ) (u hu ) (v hv ) uv
f ( x) f / ( x) lim lim , hx 0, hu 0, hv 0
dx hx hx
hx 0 hx 0
hu h h
lim [ v v u u hv ]
hx hx hx
hx 0
hu h hu du dv du du dv
v lim u lim v lim lim hu v u 0 v u
hx hx hx dx dx dx dx dx
hx 0 hx 0 hx 0 hx 0
d f ( x h) f ( x ) (u h) n u n
[ f ( x)] f / ( x) lim lim
dx h h
h0 h0
1
lim [ u n n c1 u n 1 h n c 2 u n 2 h 2 ....... h n u n ] n c1u n 1 n u n 1
h
h0
1 d d
2
[ Deno min ator ( Numerator) Nummerator ( Deno min ator )]
{ deno min ator} dx dx
Proof: Let f( x ) = u / v , where u and v are derivable functions. Let us consider that there is an increment
hx in x and the corresponding increment in u and v are hu and hv , respectively. Then
d f ( x h) f ( x ) u hu u h v u hv
f / ( x) [ f ( x) ] lim lim[ ] lim u
dx hx v hv v v(v hv )
hx 0 hx 0 hx 0
du dv
v u
h 1 h u du 1 dv u dx dx .
lim [ u . ] lim[ v . ] . .
hx v hv hx v(v hv ) dx v dx v 2 v 2
hx 0 hx 0
375
Let u = ∂(x) be a derivable function of x in the closed interval a ≤ x ≤ b , the range of the u being [ α, β ].
Then if f( x ) = f[ ∂( x ) ]=f( u ) is a derivable function of u in the closed interval α ≤x ≤ β, the composite
function f( x ) = f[∂ ( x ) ] is derivable with respect to x and the derivative f/ ( x ) is given by
dy dy du
dx du dx
Proof: Let hx be an increment of x and hu be the increment of u in u = ∂(x). Suppose that the
corresponding to the increment hu of u hy be the increment in y determined from y = f( u ).
hu hu h hy hu hy hy dy dy du
or lim lim lim ; lim .
hx h y hx hx hy hx hx dx du dx
hx 0 hx 0 hx 0 hx 0
dy hy du hy dy dy du
lim 0 and lim 0; Here also the result is
dx hx dx hx dx du dx
hx 0 hx 0
dy
Example 11.15: Find dx of each of the following functions:
4 2 6
4 2
( i ) y = x1/3 , ( ii ) y = 4 x1/4 + 5 x2 -3x , ( iii ) y = x , ( iv ) y = x , ( v ) y = x
Solution:
1
dy d 1 / 3 1 d 3 1 1 1
(i) (x ) ( x ) x 2 / 3 2 , dy d 16
dx dx 3 dx 3 (iii ) (4 x 4 ) 4(4) x 41 5
3x3 dx dx x
dy d 4 dy d 6
(iv ) (2 x 2 ) 2 (2) x 21 3 , (v) (6 x 1 ) 6 x 11 2
dx dx x dx dx x
Example 11.16: Find the derivative of each of the following functions with respect to independent
variable.
376
3x 2
( vii ) y = 4 x4 – 3 x3 + 2 x2 – 5 , ( viii ) y=e4x + 5 , ( ix ) y = 6x2e2x ( x ) y 4x 1 .
dy
Example 11.17: Find dx of each of the following functions:
x2 2 2x
(vii ) y x x , (viii ) y 2 x 3 , (ix) y 4 x 2 a 2 , ( x) y , ( xi ) y
x
x 1 1 x2
( xii ) y (log x) 2
Solution:
log y = x log x
1 dy d d x
log x ( x) x (log x) x11 log x 1 log x
y dx dx dx x
dy
y (1 log x) x x (1 log x)
dx
dy d d d
(3x sin x) 3[ x (sin x) sin x ( x)] 3[ x cos x sin x ( x11 )]
( ii ) dx dx dx dx
3[ x cos x sin x] 3 cos x [ x tan x].
dy d d
(iii ) y sin 2 x , (sin 2 x) 2 sin 21 x (sin x) 2 sin x cos x
dx dx dx
dy d 2 d d
(iv ) y x 2 sin 3 x, ( x sin 3 x) x 2 (sin 3 x) sin 3 x ( x 2 )
dx dx dx dx
d
x 2 3 sin 31 x (sin x) sin 3 x 2 x 21 3x 2 sin 2 x cos x 2 x sin 3 x
dx
x sin x{3x cos x 2 sin x}
2
dy d 1 d 2 1 2
(v) y log x 2 , (log x 2 ) 2 (x ) 2 2x
dx dx x dx x x
(vi ) y 2 x ,
377
x log y = x log 2
1 dy dy
log 2 ( x11 ) log 2 or y log 2 2 x log 2.
y dx dx
(vii ) y x x
x
log y = xx log x
1 dy d x d d xx
( x log x) x x (log x) log x ( x x ) log x { x x (1 log x)}
y dx dx dx dx x
dy 1 1
y [ x x { (1 log x) log x} x x x x { (1 log x) log x}
x
dx x x
1 1
dy 1 1 d 2 1
(viii ) y 2 x 3 (2 x 3) , (2 x 3) 2
2
(2 x)
dx 2 dx 2 2x 3 2x 3
(ix) y 4 x 2 a 2 (4 x 2 a 2 )1 / 2
4 2x
1 1
dy 1 1 d 1 d 4x
(4 x 2 a 2 ) 2 (4 x 2 a 2 ) 2 (4 x 2 a 2 ) (4 x 2 )
dx 2 dx 2 4 x 2 a 2 dx 2 4x 2 a 2 4x 2 a 2
x 2 2 dy d x 2 2 1 d d
( x) y , ( ) [ ( x 1) ( x 2 2) ( x 2 2) ( x 1)]
x 1 dx dx x 1 ( x 1) 2
dx dx
1 x 2 2x 2
[( x 1) (2 x) ( x 2) 1]
2
( x 1) 2 ( x 1) 2
1
2x dy d 2x 1 d d
( xi ) y , ( ) [ 1 x2 (2 x) 2 x (1 x 2 ) 2 ]
1 x2 dx dx 1 x 2 { 1 x 2 }2 dx dx
1
1 1 1 d
[ 2 1 x 2
2 x (1 x 2 2
) ( x 2 )]
1 x 2
2 dx
2
1 x 2
[2{ 1 x 2 }] .
1 x 2
1 x2
3
(1 x 2 ) 2
dy d d 2 log x
( xii ) y (log x) 2 , (log x) 2 2 log x (log x) .
dx dx dx x
378
Derivaative of implicit function is a function that is defined implicitly by an implicit equation, by
associating one of the variables with others. For example, if y is a variable which is related linearly or
non-linearly with the variables x1 , x2, ……….., xk ( argument) ,is called implicit function. Specifically,
the equation of a circle
x2 + y2 = r2
for which the centre is in the origin is an example of implicit function which is the relation of 3 variables.
A common type of implicit function is an inverse function. For example, if g is a function of x , ten the
inverse function of g is g-1 , where g-1 gives a solution of the equation y = g( x ) for x in terms of y. This
solution is x = g-1 (y). Here g-1 is an implicit function. It is also an inverse function of g. In general, an
algebraic function
can be written as y = f( x ) ,where the coefficients a(x) are the polynomial functions of x. Here y = f(x) is
a multivariate implicit function. The special feature of this function is that there is no dependent variable
to express it as a function of independent variables.
Implicit Differentiation:
This differentiation makes use of the chain rule to differentiate implicitly defined functions. For example,
let
y = e4x - 5
dy
be an implicit function. We need dx . Here, we can write y = ez, where z = 4x – 5. Then , the value of
dy dy dz d d
e z (4 x 5) e z ( 4 x ) 4 e z 4 e 4 x 5
dx dz dx dx dx
dy
This rule of finding dx is known as Chain Rule.
Y = f( x ) = 3x2+ 4y2 - 7 =0
dy
We need dx . Here, we can have 6x + 8y = 0. Then
dy 6 x 3 x
dx 8y 4y .
379
General Formula for derivative of Implicit Function.
dy
Let f(x,y) = 0 be an implicit function . We need dx of f( x, y)=0. Here
f
x 1
dy f ( x)
dx f f1 ( y )
y
Here f1(x) and f1(y) indicate the partial derivatives of f(x,y) with respect to x and y, respectively.
The above formula from the use of generalized chain rule to obtain the total derivative with respect to x of
both sides of f(x,y)=0. Thus, we can write
f dx f dy f f dy
0, 0
x dx y dx x y dx
dy
This equation can be solved for dx .
dy
Example 11.18: Find dx of each of the following functions:
( viii ) x2y3 – 2x y = 6x + y +1 , ( ix ) x2 = y3 , ( x ) x y = 2 , ( xi ) x + ( 1 / y ) = 5
Solution:
d 2 d d dy dy x
(i ) x 2 y 2 9, (x ) ( y 2 ) (9) or 2 x 2 y 0,
dx dx dx dx dx y
(ii ) x 2 y 2 2 gx 2 fy c 0,
d 2 d d d d
( x ) ( y 2 ) (2 gx) (2 fy ) (c) 0
dx dx dx dx dx
dy dy dy
or 2 x 2 y 2 g 2 f 0 0 or 2 ( y f ) 2 ( x g )
dx dx dx
dy xg
or
dx y f
380
d 2 d d
(iii ) x 2 y 2 2 xy 0,( x ) ( y 2 ) (2 xy ) 0,
dx dx dx
dy dy d dy dy
or 2 x 2 y 2 x 2 y ( x) 0, or 2 x 2 y (2 x 2 y ), or 1
dx dx dx dx dx
d 2 dy d dy dy 2 x
(iv) x 2 y 3 12, (x ) 3y 2 (12), or 3 y 2 2 x, or
dx dx dx dx dx 3 y 2
(v) x 3 3x 2 y 3xy 2 y 3 0,
d 3 d dy d dy dy
( x ) 3 y ( x 2 ) 3x 2 3y 2 ( x) 3 x 2 y 3 y 2 0
dx dx dx dx dx dx
dy dy dy
or 3x 2 6 xy 3x 2 3 y 2 6 xy 3y 2 0
dx dx dx
dy dy x 2 2 xy y 2
or [3x 2 6 xy 3 y 2 ] 3x 2 6 xy 3 y 2 , or 1
dx dx x 2 2 xy y 2
d 4 d d dy
(vi ) x 4 3x 3 2 x 2 6 y 0, (x ) 3 (x3 ) 2 (x 2 ) 6 0
dx dx dx dx
dy dy 1 x
or 4 x 3 3 3x 2 2 2 x 6 0, or [ 4 x 3 9 x 2 4 x] [4 x 2 9 x 4]
dx dx 6 6
dy d dy d dy
(vii ) x 2 y 2 y 3 3x 2 y, x 2 y (x 2 ) 2 3y 2 3 ( x) 2
dx dx dx dx dx
dy 2 dy 3 2 xy
or [ x 6 y 2 2] 3 2 xy, or 2
dx dx x 6 y 2 2
(viii ) x 2 y 3 2 xy 6 x y 1,
dy d dy d d dy d
x2 3y 2 y 3 ( x 2 ) 2 x 2 y ( x) 6 ( x) (1)
dx dx dx dx dx dx dx
dy dy 2 (3x xy 3 y )
or [3x y 2 x 1] 6 x 2 xy 2 y, or
2 2 3
dx dx 3x 2 y 2 2 x 1
d 2 d 3 dy dy 2 x
(ix) x 2 y 3 , (x ) ( y ) , or 2 x 3 y 2 , or
dx dx dx dx 3 y 2
d d d dy dy y
( x) xy 2, y ( x) x ( y ) (2), or y x 0, or
dx dx dx dx dx x
1 d d d dy dy 1
( xi ) x 5, ( x) ( y 1 ) (5), or 1 y 2 0, or 2 y 2
y dx dx dx dx dx y
381
11.9 Derivative of Parametric Function
Let x = f(t) and y = g(t) be two functions of t. Here t is a third variable. In this case the two relations
together can be used to define a function of x and y. This new function is known as parametric function
dy
with a parameter t. In such a situation, we need to calculate dx . Here
dx dy
f / (t ) g /
(t )
dt and dt
dy dy dt dy dx
/ .
dx dt dx dt dt
dy
Example 11.19: Find dx of each of the following parametric functions:
( i ) x = 3 t 3 , y = 4 t2 ; ( ii ) x = 2 t2 – 3 t , y= 3 t3 +4 t2 – 2 t; ( iii ) x= ( 1 – t ) ( 1 +t )-1, y= t2 – 1;
5t 4 x et 1 3x 2 2 x
( vii ) y ,t ; ( viii ) y , t .
5t 4 x e 2t 1 2 x 2 3x
Solution:
dy
dx dy dy dt 8t 8
(i) x 3t 3 , y 4t 2 ; 3 3t 31 9t 2 , 4 2t 21 8t; 2
dt dt dx dx 9t 9t
dt
dx dy
(ii ) x 2t 2 3t , y 3t 3 4t 2 2t ; 2 2t 21 3t 11 4t 3, 3 3t 31 4 2t 21 2t 11
dt dt
9t 2 8t 2
dy
dy dt 9t 2 8t 2
dx dx 4t 3
dt
382
d d
(1 t ) (1 t ) (1 t ) (1 t )
1 t dx dt dt (1 t )( 1) (1 t )(1) 2
(iii ) x , y t 2 1;
1 t dt (1 t ) 2
(1 t ) 2
(1 t ) 2
dy
dy dy dt 2t
2t 21 0 2t ; t (1 t ) 2
dt dx dx 2
dt (1 t ) 2
dx dy
(iv ) x 3t 2 e t , y 2t 2 3t ; e t 3 2t 21 3t 2 e t 3t e t (2 t ); 2 2t 21 3t 11 4t 3
dt dt
dy
dy dt 4t 3
dx dx 3te t (2 t )
dt
dx dy
(v) x e 3t t 2 , y t 3 2t 2 t ; 3e 3t 2t 21 3e 3t 2t , 3t 31 2 2t 21 t 11
dt dt
= 3t2 – 4t - 1
dy
dy dt t (3t 4) 1
.
dx dx 3e 3t 2t
dt
dx
(vi ) x t 2 2t 1, y t 3 3t 2 3t 1; 2t 21 2t 11 0 2(t 1),
dt
dy
3t 31 3 2t 21 3t 11 0 3t 2 6t 3 3(t 2 2t 1) 3(t 1) 2
dt
dy
dy dt 3(t 1) 2 3
(t 1).
dx dx 2(t 1) 2
dt
d d
(5 t ) (5 t ) (5 t ) (5 t )
5t 4 x dy dt dt (5 t ) (5 t ) 10
(vii ) y ,t ;
5t 4 x dt (5 t ) 2
(5 t ) 2
(5 t ) 2
d d
(4 x) (4 x) (4 x) (4 x)
dt dx dx (4 x) (4 x) 8
dx (4 x) 2
(4 x) 2
(4 x) 2
dy dy dt 10 8 80
dx dt dx (5 t ) 2
(4 x) 2
(5 t ) (4 x) 2
2
383
11.10 Successive Differentiation
Let y = f(x) be a function of x which is differential up to n times. The first derivative of f(x) is f/(x). If
f/(x) is differentiable , we can write the differentiated result as f//(x). This f//(x) is the second derivative of
f(x). For example , let y = f(x) = x3 – 3 x2 + 4x. Then
dy d d d
f/( x ) = dx dx ( x ) 3 dx ( x ) 4 dx ( x) =3x2 - 3 x 2x + 4= 3x2 – 6x +4
3 2
This is the first derivative of the given function. The second derivative of the function is f//(x), where
d2y d 2 d d
f ( x ) = dx 2 3 dx ( x ) 6 dx ( x) dx (4) 6 x 6
//
In a similar way, we have f///(x) = 6. Here f//(x) is differentiable. The first, second , third, …….(n-1)th
and n-th derivative can also be written by f(1), f(2), ………..,f(n-1), f(n). Here f(n) is available if f(n-1) is
differentiable. In the above example, we have f(3) as f(2) is differentiable. This type of differentiation up to
n –th time is known as Successive Differentiation provided (n – 1)th derivative is differentiable on an
interval I.
dny
f (n)
dx n
f (h h) n f ( n 1) ( x)
f ( n ) lim
h
h0
The result of this derivative , sometimes, takes a special form depending on the function of x. Let us
Observe the result of the n-th derivative of the function y = f(x) = f = (ax+b)m , ( ax+b) > 0 and m lies in
the interval I. Let us consider that
n N
then, we have
However, if
384
( i ) m < n , f(n)( x ) =0, ( ii ) m = n , f( n )(x)= n! an
m!
(iii ) m n, f ( n ) ( x) (ax b) mn a n
( m n) !
If
1 (1) n n !a n
f ( x) y, yn
ax b (ax b) n 1
y = f(x) = ax +blog x
d d d b
y (1) f / ( x) (ax b log x) a ( x) b (log x) a a bx 1
dx dx dx x
d d d b
y ( 2) f // ( x) (a bx 1 ) (a) b ( x 1 ) 0 bx 11 2
dx dx dx x
Example 11. 20: Find the values as indicated below using the given functions:
( i ) Find y3 , if y = x2log x
Solution:
dy d d 2 x2
y1 x2 (log x) log x (x ) 2 x log x x 2 x log x
dx dx dx x
d2y d d d 2x
y2 2 ( x) 2 x (log x) 2 log x ( x) 1 2 log x 3 2 log x
dx dx dx dx x
d3y d d 2
y3 3 (3) 2 (log x) .
dx dx dx x
Solution:
dy d 3 d d d
y1 ( x ) 6a ( x 2 ) 12 ( x) (8) 3x 2 6 2ax 12 3x 2 12 ax 12
dx dx dx dx dx
2
dy d y d d d d
y2 1 2 (3x 2 12 ax 12) 3 ( x 2 ) 12 a ( x) (12) 6 x 12 a
dx dx dx dx dx dx
3
d y d d d
y3 3 (6 x 12) 6 ( x) (12) 6
dx dx dx dx
385
( iii ) Find y2, if y = 3x sin x
Solution:
dy d d
y1 3x (sin x) 3 sin x ( x) 3x cos x 3 sin x 3{x cos x sin x}
dx dx dx
d2y d d d
y 2 2 3[ x (cos x) cos x ( x) (sin x)] 3[ x sin x 2 cos x ]
dx dx dx dx
(i v ) Find y2, if y = (x – a )2 ( x – b )2
Solution:
dy d d
y1 ( x a) 2 ( x b) 2 ( x b) 2 ( x a ) 2 2 ( x a ) 2 ( x b) 2 ( x b) 2 ( x a )
dx dx dx
2
d y dy d d d d
y 2 2 1 2 [( x a) 2 ( x b) ( x b) ( x a) 2 ] 2 [( x b) 2 ( x a ) ( x a ) ( x b) 2 ]
dx dx dx dx dx dx
2 [( x a) 2( x b)( x a)] 2 [( x b) 2( x a) ( x b)]
2 2
Solution:
d d
(1 x) (1 x) (1 x) (1 x)
1 x dy d 1 x dx dx 1(1 x) (1 x)
y , y1 ( ) 2(1 x) 2
1 x dx dx 1 x (1 x) 2
(1 x) 2
d2y d 4
y 2 2 2 (1 x) 2 (2)(2)(1 x) 21 1
dx dx (1 x) 3
11.11 Maxima and Minima
The words Maxima and Minima ( plural of maximum and minimum, respectively ) of a function , known
collectively as extreme , are the largest and smallest value of a function, either within a given range or on
the entire domain of the function.
A real valued function y = f( x ) = f defined on a domain X has a global maximum point at x* if f(x*) ≥ f(
x ) for all x in X. Similarly, the function has a global minimum point at x* if f( x*)≤ f( x )for all x in X.
The value of the function at a maximum point is called the maximum value of the function and the value
at a minimum point is called the minimum value of the function.
Let us consider that y = f( x ) be a function of x. Then , we need to follow the following steps
dy
( i ) Calculate f/( x ) = dx
( ii ) Put f/( x ) = 0 and solve for the values of x.
386
( iii ) Put the value(s) of x in y= f( x ) to get the y-coordinate . At this stage the pair of values x and y will
give the stationary point of x and y coordinate(s).
d2y
( iv ) Calculate f//( x ) = dx 2 .
( v ) Put the value(s) of x in f//( x ) . If f?/( x ) >0 under x, then the function y= f(x ) is minium and the
local minimum is at the point ( x1 , y1) of f( x ). If f//( x ) < 0 , the function f ( x ) is maximum at the point (
x1,y1).
The above method of investigation of maxima and minima of a function is done using the second
derivative of the given function.
Let y = f ( x ) be a real valued function of x defined on a domain X, where X has the global maximum
point at x* if f( x*) ≥ F( x ) for all x in X. Here
dy
/ /
y1= f ( x ) =f = dx
is the first derivative of the function y=f( x ). From this derivative the critical value(s) of x are found out
fro the equation
f/( x ) = 0.
Let the critical values of x be x0 and x1. When x < x0 ( sufficiently near to x0 ) and f/ ( x ) is –ve for x0 ,
then the function f ( x ) has a minimu value. Again, if f/( x ) ix +ve for x < x0 , then the function has a
maximum value.
y = f( x ) = x2 – 6 x + 9
dy d 2 d d
f / ( x) ( x ) 6 ( x) (9) 2 x 6
dx dx dx dx
Thus f/( x ) changes sign from negative to positive as we pass through x = 3 from left to right . The curve
y = f( x ) falls and then rises. Thus,
387
Example 11.21: Examine the maxima and minima of each of the following functions:
Solution:
dy d d d
(i ) y 3x 3 6 x 2 16, f / ( x) 3 ( x 3 ) 6 ( x 2 ) (16) 9 x 2 12 x
dx dx dx dx
dy 4
0 or 9 x 2 12 x 0, or 3x 4 0, or x 0, x
dx 3
2 2
d y d d d y
2
9 ( x 2 ) 12 ( x) 18 x 12, 12, when x 0
dx dx dx dx 2
The second derivative is (– 12 ) if x = 0. It implies that the function is maximum. Again, for x = 4/3, the
second derivative becomes 18(4/3) – 12 =12, which is +ve. Therefore, the function is minimum.
for x = (4/3), f(x) = 3 ( 4/3 )3 – 6 ( 4/3 )2 + 16 = 12.44 . The function f( x ) is minimum when x = (4/3)
and the minimum value is 12.44.
The problem is solved using the second derivative of the function. Let us investigate the phenomenon
using the first derivative of the function. The result of the first derivative is ( 9x2 – 12 ). Equating this
result to zero , we get x = 0 and (4/3) and f( x = 4/3) = 12.44 and for x= 0, we get f( x=0) =16. It indicates
that , when x=0, f(x) is maximum and when x + (4/3), f( x ) is minimum.
(ii ) y 4 x 4 4 x 3 2 x 2 7
dy d d d d
y1 f / ( x) 4 ( x 4 ) 4 ( x 3 ) 2 ( x 2 ) (7) 16 x 3 12 x 2 4 x
dx dx dx dx dx
dy
0, 16 x 3 12 x 2 4 x 0, or x (4 x 2 3x 1) 0, x 0, or 4 x 2 3x 1 0.
dx
388
4 x 2 3x 1 0, or 4 x 2 4 x x 1 0, or 4 x( x 1) 1( x 1) 0
1
or ( x 1) (4 x 1) 0, x 1, or x .
4
2
d y d d d d
2
(16 x 3 12 x 2 4 x) 16 ( x 3 ) 12 ( x 2 ) 4 ( x) 48 x 2 24 x 4
dx dx dx dx dx
2
d y
If x 0, 2 48 0 24 0 4 4, the function is max imum.
dx
d2y
If x 1, 2 48(1) 2 24 1 4 20, the function is min imum.
dx
1 d 2 y 1 1
If x , 2 48( ) 2 24( ) 4 5, the function is min imum.
4 dx 4 4
Now, let us see the values of f( x ) for different values of x.
(iii ) y x 3 9 x 2 15 x,
dy d d d
y1 f / ( x) ( x 3 ) 9 ( x 2 ) 15 ( x) 3x 2 18 x 15
dx dx dx dx
dy
0, 3x 2 18 x 15 0, or x 2 6 x 5 0, or x 2 5 x x 5 0
dx
or x ( x 5) 1 ( x 5) 0, or ( x 1) ( x 5) 0 or x 1, x 5
d2y d d d d
2
(3x 2 18 x 15) 3 ( x 2 ) 18 ( x) (15) 6 x 18
dx dx dx dx dx
2
d y
If x 1, 2 6(1) 18 12, the function is min imum.
dx
d2y
If x 5, 2 6(5) 18 12, the function is max imum.
dx
389
(iv ) y x 5 5 x 4 1
dy d 5 d 5 d d
y1 f / ( x) ( x 5 x 4 1) ( x ) 5 ( x 4 ) (1) 5 x 4 20 x 3
dx dx dx dx dx
dy
0, 5 x 4 20 x 3 0, or 5 x 3 ( x 4) 0, x 4, x 0
dx
d2y d d d
2
(5 x 4 20 x 3 ) 5 ( x 4 ) 20 ( x 3 ) 20 x 3 60 x 2
dx dx dx dx
2
d y d2y
If x 0, 2
0, the function is max imum . If x 4, 2
20(4) 3 60(4) 2 320
dx dx
the function is min imum
The value of f( x ) = 1, and for x = 4, f( x ) = (4)5 – 5 (4)4 +1= -255, the function is minimum.
Example 11.22: Find the maximum and minimum values of each of the following functions :
( iv ) y = x + ( 1/x) , (v) y = 4 ex + 9 e- x .
Solution:
(i ) y x 5 5 x 4 5 x 3 1,
dy d 5 d d d
( x ) 5 ( x 4 ) 5 ( x 3 ) (1) 5 x 4 20 x 3 15 x 2 5 x 2 ( x 2 4 x 3)
dx dx dx dx dx
5 x ( x 3x x 3) 5 x {x( x 3) 1( x 3)} 5 x 2 ( x 3) ( x 1)
2 2 2
dy d2y d d d
0, x 0, x 1, x 3., 2 (5 x 4 ) (20 x 3 ) (15 x 2 ) 20 x 3 60 x 2 30 x
dx dx dx dx dx
2
d y
If x 0, 20 0 60 0 30 0 0, the function is min imum [ the test is inclusive]
dx 2
d2y
If x 1, 20(1) 3 60(1) 2 30(1) 10, the function is max imum.
dx 2
d2y
If x 3, 2
20(3) 3 60(3) 2 30(3) 90 0, the function is min imum.
dx
390
d2y
Again, when x = 0, y = -1, the function is minimum. However, dx 2 0 indicates that the test is
inclusive and f(x) may be maximum or minimum or there is no relative extremum at x for a particular
value.
(ii ) y x 4 2 x 3 3x 2 4 x 4
dy d 4 d d d d
( x ) 2 ( x 3 ) 3 ( x 2 ) 4 ( x) (4) 4 x 3 6 x 2 6 x 4
dx dx dx dx x dx
dy
0, 4 x 3 6 x 2 6 x 4 0,
dx
It is seen that, if x = 1, the equation is satisfied. Hence ( x – 1 ) is a factor of the left hand side of the
equation. Hence, we can write
d2y d d d d d
2
(4 x 3 6 x 2 6 x 4) 4 ( x 3 ) 6 ( x 2 ) 6 ( x) (4) 12 x 2 12 x 6
dx dx dx dx dx dx
2
d y
If x 1, 12 1 12 1 6 18 0, the function is min imum
dx 2
d2y
If x 2, 2 12(2) 2 12(2) 6 18 0, the function is min imum
dx
1 d2y 1 1
If x , 2 12( ) 2 12( ) 6 9 0, the function is max imum
2 dx 2 2
When x=1, y = 1+2 – 3 – 4 +4=0 is the minimum value of the function. Again, when x = - 2 ,
1 4 1 1 1 81
y( ) 2( ) 3 3( ) 2 4( ) 4
2 2 2 2 16 .
391
9 dy d d dy
(iv ) y 4e x
x
4e x 9e x , 4 (e x ) 9 (e x ) 4e x 9e x , 0 x 0.4054
e dx dx dx dx
d2y d d d
2
(4e x 9e x ) 4 (e x ) 9 (e x ) 4e x 9e x 12 0, if x 0.4054
dx dx dx dx
Hence, the function is minimum and its minimum value is 12 when x = 0.4054.
1 dy d d d 1
x x 1 ,
(v ) y x ( x) ( x 1 ) 1 x 2 , 0, 1 2 0, or x 2 1, or x 1
x dx dx dx dx x
2 2
d y d 2 d y
2
(1 x 2 ) (2) x 3 3 2, if x 1. For x 1, 2 2
dx dx x dx
It implies that for x = 1, the function is minimum and the minimum value of y =2. If x = -1, the function
is maximum and the maximum value of y =0. Note that, in this case maximum value is less than the
minimum value.
Optimization means finding an alternative with the most cost effective or highest achievable performance
under the given constraints , by maximizing desired factors and minimizing undesired ones. In
comparison, maximization means trying to attain the highest or maximum result or outcome without
regard to cost or expense. Practice of optimization is restricted by the lack of full information and the lack
of time to evaluate what information are available.
Classical optimization techniques are useful in finding the optimum solution or unconstrained maxima or
minima of continuous and differentiable functions. In most optimization problems , the objective is to
identify the absolute maximum or absolute minimum of a particular function on some relevant interval.
The identification is done using the following steps:
( a ) The absolute maximum of the function is the largest value of the function on the interval. For this
the value of the variable is found out from the solution of the equation
dy
f / ( x) 0
dx
where y = f ( x ) is the given function . The value of x falls within the specified interval. Let us consider
that f/( x ) is a quadratic function . Then we have two values of x , say, x1 and x2 . Then we have first-
order critical points of f( x ) as { x1 , f( x1 ) } and another critical point is { x2 , f(x2)}.
( b ) Find
d2y
f // ( x)
dx 2
And evaluate f//(x) for x1 and x2 . If f//( x ) <0, the critical point [ x1 , f( x1)}is a relative maximum. If
f//(x)>0, the critical point is a relative minimum. Similar is the case about the critical point [ x2 , f(x2)}.
392
( c ) The critical points found out according to the above procedure will be used to decide the extremum.
For example , let y = f( x ) = x2 + 54 x -1, where x>0. Then
dy d 2 d 1 54 dy 2 x 3 54
f / ( x) ( x ) 54 ( ) 2 x 2 . Let f ( x)
/
0
dx dx dx x x dx x2
It gives x3=27, or x = 3. Here the interval has no right-hand end point and does not contain its left-hand
end point at x =0, the only possible extremum is
f( 3) = 32 + 54/3= 27
Here f/(x) <0 , if 0 < x < 3 and f/( x )>0, if x >3 , the graph of f( x ) is decreasing for 0 < x < 3 and
increasing for x > 3.
Example 11.23: The regular air fare from Dhaka to Kolkata and back is Tk. 12000.00. An air line uses a
plane with a capacity of 250 passengers on this route. However, on an average, 200 passengers run every
day. There is a suggestion fro Market Research organization that if fare is reduced up to Tk.500.00 ,on an
average , 10 passengers may be increased in each flight. What should be the fare to maximize the
revenue?
Solution: Let R = revenue function from the flight = No. of passengers x Ticket price.
dy dy dy
R / (n) 20000 10000 n , 0n2
dx dn dn
d2y
R // (n) 10000 0
dn 2
It indicates that for n = 2 the revenue function becomes maximum. Thus the maximum revenue is
The best fare will be 12000 – 500 x 2 = 11000. Number of passengers will be 200 +2 x 20 =220.
393
Example11.24: If q units of shirt are produced , the total cost of production is C(q) = 2q2+6q+72 taka.
For what level of production the average cost will be minimum?
C (q) 1 72
A(q ) { 2q 2 6q 72} 2q 6
q q q
As q is the number of units of production , it cannot be negative. The range of q will be greater than zero
( q > 0). So, we need to find absolute minimum value of A(q) on the bounded interval q>0. Here
dA(q) d d d 72 dA(q) 72
A / (q) 2 (q) (6) 72 (q 1 ) 2 2 , 0 2 2,
dq dq dq dq q dq q
or q = 6 and -6 . But q>0, hence q=6. This value of q is the only first-order critical value in the interval
q>0. Here A(6) = 2 x 6 + 6 +72/6= 30. Now
d 2 A(q) 2 72 2 72 144
2
A // (q) , A // (6) 0
dq q3 63 216
Therefore, the function A(q) is minimum and its minimum value is 30. On an averge, the minimum
production cost is 30 taka.
Example 11.25: Suppose x years after its establishment in 2000 a university had a number of students
f( x ) = 50 [ 2 x3 – 36 x2+ 192 x]
( a ) At what time between 2000 to 2017 the number of students became largest ?
( b ) At what time between 2000 to 2020 the number of students will be minimum?
( c ) At what time between 2000 to 2020 the number of students will be maximum?
Solution: ( a ) The range of x will be 0 to 18, and x cannot be negative. From the given function, we have
f//( x ) = 50[ 12 x – 72 ]
For x = 4, f//( x ) = 50 [ 12 x 4 – 72] = - 1200 < 0. For x =4, the function of number of students became
largest. After 4 years of establishment, the number of students became largest and the largest number of
students were
394
( b ) The range of x is from 0 to 21 and from the given function, the values of x are 4 and 8. But for x = 4
the function becomes maximum. Again, for x = 8, f// ( x ) = 50 [ 12 x 8 – 72 ]=1200>0. For x =8, the
function became minimum. After 8 years of establishment, the number of students became minimum and
the minimum number of students were f(8)= 50 [ 2(8)3 – 36 (8)2 + 192(8)]=12800.
Example 11.26: A survey on residents residing in a city centre has been conducted to estimate the
number of people who have listening habit of television programmes after x hours starting from 7:00
P.M. up to mid-night 12:00 P.M. The survey indicated that the percentage of the adult population who are
used to enjoy television programmes x hours after 7:00 P.M. follows the function
At what time after 7:00 P.M. up to 12:00 P.M. the percentage of people who will enjoy television
programmes will be maximum?
Solution: The range of time x is 7:00 – 12:00 P.M. i.e. 0< x < 5 . Moreover, the value of x cannot be
negative and it will not exceed 5. From the given function of percentage of people, we have
f/( x ) = 0.5 [ 3 x2 – 36 x + 81 ]
f// ( x ) = 0.5 [ 6x – 36 ]
When x = 3, f//( x ) = 0.5 [ 6 x 3 – 36 ] = -9 <0. Thus, the function is maximum and the maximum value of
x after 3 hours of 7:00 P.M. is
Example 11.27: The cost of production of a refrigerator is Tk.15000.00. If the selling price of the
refrigerator is x taka, there may ba a demand of ( 17000 – x) pieces of refrigerator. What should be the
selling price of the refrigerator so that the producer‟s profit becomes maximum?
Solution: The selling price is x taka. The profit per refrigerator is ( x – 15000 ). The profit from the sale
of ( 17000 – x ) refrigerators is ( x – 15000 ) ( 17000 – x ). So, the profit function is
The profit will be maximum if this profit function becomes maximum. Here
Now, f//( x ) = -2 <0 when x = 16000. The profit function becomes maximum when x = 16000 and the
maximum profit becomes
395
F ( 16000 ) = ( 17000 – 16000 ) ( 16000 – 15000 )=1,00,0 000.
Example 11.28: A worker in a garments industry usually arrives at 8:00 A.M. After t hours of his /her
arrival he /she can produce
P ( t ) = - t3 + 9t2 +24t
units of shirt. At what time during the morning shift [ up to 2 P.M.] the worker can perform most
efficiently?
Solution: As P (t) is the production function to produce garments by the worker, the rate of production is
given by P/ (t ), where
d
P / (t ) P(t ) 3t 2 18t 24
dt
The rate of production will be maximum if P//(t)<0 for certain value of t within the interval 8:00 – 2 :00
P.M. [ 0 < t < 6 ]. Now,
or ( t – 4 ) ( t – 2) = 0 , or t=2,4
Now, P//( t ) = -6t +18. When t = 2, P//( t ) = - 6 x 2 +18 = 6, the function becomes minimum. Again,
when t = 4, P//( t ) = -6 x 4 +18 = - 6, the function becomes maximum and the maximum value of P ( t) is
P( t ) = -( 4)3 + 9(4)2 + 24 x 4 = 176. The worker can perform at 12: P.M. most efficiently.
Example 11.29: A cement factory starts working at 8 : 00 A.M and continues the production up to
2:00P.M. The factory can produce
Bags of cement within 0 < t < 6. At what time during the working hour the production will be maximum?
Solution: As P( t ) is the production function of cement , the rate of production will be given by P/(t) t).
Here P/( t ) = - 3 t2 + 18 t + 12
The rate of production will be maximum if P//( t ) < 0. The value of t lies between 0 to 6. Again,
P//( t ) = -6t + 18
This value will be negative if t> 3 but not exceeding 6. Now, if t = 3 , the production rate [ P/( t )]
becomes P/( t = 3 ) = -3 ( 3 )2+18 x 3 +12 = 39. For t = 0, we have P/( t =0) = -(0)3+18 ( o ) + 12 = 12.
For t = 5, P/(t=5) = 27 and for t = 6, P/( t =6) = 12. It indicates that fot t= 3, the rate of production
becomes maximum. Hence at 11:00 A.M. [ t = 3 ] the production will be optimum.
396
Example 11.30: A bus company will charter a bus that holds 50 people to groups of 35 or more. If a
group contains exactly 35 people , each person pays Tk. 600.00. In larger groups, every body‟s fare is
reduced by Tk.10.00 for each person in excess of 35 . determine the size of the group for which the bus
company‟s revenue will be greatest.
Let x be the number of people in excess of 35. Total number of passengers is ( 35 + x ). Fare per person is
Here x is excess of 35 and total number of passengers is 50. So, 0 ≤ x ≤ 15. Therefore,
R/( x ) = - 20 x + 250
Now, if R/( x ) = 0, we have x = 12.5. Thus, we can say that absolute maximum of R( x ) will occur when
x = 12.5. on the interval 0 ≤ x ≤ 15. Now,
Thus the revenue o the company will be greatest if the number of passengers becoe 47 or 48.
Example 11.31: A shoe manufacturer buys 5000 shoe packets per week from a distributor. The ordering
fee for each lot is Tk.50.00, the storage cost of the packets is Tk.0.25 per packet. Cost of each packet is
Tk.2.00. The use of packets s at constant rate throughout the week and supply of the packets is available
just as the preceding supply is used up. How many packets should the manufacturer order each time to
minimize cost.
Let x be the number of packets in each order and C( x ) be the total cost. Now,
Since 5000 packets are ordered per week and total packets are x in each order , the number of orders is
(5000/x). So, the order cost is
Purchase cost = Total packets ordered x cost per packet = 5000 x 2.00 = 10000.00
397
After the arrival of each lot it is stored and daily some are used. When one lot is arrived x packets are
replaced, on an average, in the store. Therefore, the storing cost is
The value of C( x ) will be minimum subject to the condition 0 < x ≤ 5000. Here
d 25000 2 25000
C / ( x) (C ( x)) 0.125 2
, C // ( x) 0.00056 0, for x 447
dx x x3
The value of C( x ) will be minimum if C/( x ) = 0 and C//( x ) > 0. This implies that
25000 25000
2
0.125 , or x 2 200000 , or x 447
x 0.125
But x cannot be negative. Therefore, x = 447 and for this x the value of C( x ) becomes
25000
C ( x) 0.125 447 10000 10,111 .80
447
Example 11.32: It is assumed that the total cost of manufacturing x units of shirt in a garments industry is
C( x ) = 9 x 2+ 424 x + 3600
( b ) At what level of production is the average cost per shirt equal to the marginal cost?
( c ) On the same set of axes , graph the average cost and marginal cost functions.
3600
A / ( x) 9
x2
3600 3600
A / ( x) 0, or 9 , or x2 400, x 20
x2 9
398
2 3600
A // ( x)
x3
For x = 20, A//( x ) = 7200 / ( 20)3 = 0.9 > 0. So, the function A(x ) is minimum when x = 20 i.e. when the
level of production is x = 20 , the average cost id minimum.
( b ) The marginal cost is derived from the cost function C( x )by calculating C/(x ), where marginal cost
is equal to the average cost. Here C/( x ) = 18 x +424 and this value is equal to A( x ). Thus, we have
This is the same level of production as is observed in ( a ) for which the average cost is minimum.
Here A/( x ) is positive if x = 20 or x > 20. But A/( x ) < 0 when x < 20 , i.e. when 0 < x < 20. Hence A(x
) is decreasing for 0 < x < 20 and is increasing when x>20 and it has a relative minimum at x = 20.
Elasticity of Demand
Let p be the price and q be the demand of a commodity. Then the elasticity of demand is given by
p dq
q dp
This measure of elasticity measures the percentage change in demand due to a percent change in price.
We can explain it by an example.
Example 11.33: Consider that the demand q and price p are related by the linear equation q = 350 – 5p,
where 0 ≤ p ≤70.
( b ) Calculate the elasticity of demand if the price p = 60. Interpret the answer.
( c ) Calculate the elasticity of demand if the price is p = 35 and interpret the result.
p dq p 5p p
(5)
q dp q 350 5 p 70 p
399
60
6
70 60
35
1
70 35
If price is 35, an one percent increase in price leads to one percent decrease in demand.
p
3
70 p
or 210 -3p = p. Hence, p = 52.5, an one percent increase in price results in 3 percent decrease in demand
when p = 52.5.
Example 11.34: ( a ) A manufacturer produces woolen trouser x units per hour, where the total cost of
production is
1 3
C ( x) x 5 x 2 75 x 10
3
At what level of production of units will be such that the marginal cost is minimum.
( b ) The sum of two positive numbers is 16. Find the maximum value of their product.
This marginal cost will be minimum for a value of x so that C//( x ) > 0. Here C//( x ) = 2x – 10 , or x = 5.
It indicates that at a level of production of x = 5 units, the marginal cost will be minimum.
( b ) Let x be one of the positive numbers . Their sum is 16. So another one is (16 – x ). The product of
these two numbers is f ( x )= x( 16 – x). This product will be maximum under the condition that f/( x ) =0.
Here f/( x ) = 16 – 2x and f?( x ) =0 gives x = 8. Hence the other positive number is ( 16 – 8 ) =8.
Therefore, the product is 8 x 8 =64.
Example 11.35: The total cost function for producing x units of an item is given by
C( x ) = ( x2 – 16 x +400) taka.
400
Find the average cost function and the level of output so that the average cost function becomes
minimum.
Solution: The cost function for producing x units is C( x ) = x2 – 16 x + 400. The average cost function is
C ( x) 400
A( x) x 16
x x
This function will be minimum or maximum depending on the value of x from the equation
400
A / ( x) 1
x2
x2 = 400 or x = ±20
But the value of x cannot be negative as it is the number of units produced. Again
2 400
A // ( x)
x3
Under x = 20 , A//( x )= 800 / ( 20 )3 = 0.1>0. As A//( x ) > 0, A( x ) is minimum . Therefore , the level of
output is x = 20 for the minimum average cost.
Example 11.36: The telephone directorate finds that there is a net profit of Tk.50.00 per instrument if an
exchange has 2000 or less subscribers. But if there are over 2000 subscribers the profit per instrument
decrease by Tk. 0.01 for each subscriber above 2000. How many subscribers would give maximum net
gain?
Solution: Let the number of subscribers above 2000 be x. Then the net profit per instrument becomes
x
(50 )
100
P ( x ) = ( 2000 + x ) ( ( 50 – x/100 )
x2
10000 30 x
100
The function will be maximum or minimum under the condition P//( x ) < 0 or P//( x ) > 0, respectively for
a value of x such that P/( x ) = 0. Here
401
2x x
P / ( x) 30 30
100 50
Thus P/(x) =0 gives x = 1500. Hence, the number of subscribers will be ( 2000 + 1500 ) =3500.
Example 11.37: A manufacturer can sell x units of his/her product per month at a price of taka
P( x ) = ( 302 – 2 x ) per unit. The cost of production of x units is C( x ) = ( 2x + 1000 )taka. Find the
number of units to be produced per month to get maximum profit.
S ( x ) = x P ( x ) = x ( 302 – 2 x ).
The cost of production of x units is C ( x ) = 2x + 1000 . Hence, the profit in selling x unit is
The profit will be maximum for a value of x such that F/( x ) =0 . Here
dF ( x)
F / ( x) 4 x 300
dx
Now, F/( x ) = 0 gives - 4 x +300 = 0, Here x = 75. Therefore, 75 units will give maximum profit.
Example 11.38: From the data of traffic flow at the exit point of a city it is observed that most of the
vehicles go out during 6:00 P.M. to 10:00 P.M. on a normal working day. The speed of the traffic at the
exit point follow the function
1
S (t ) t 3 8t 2 63t 125
3
Miles per hour, where t is the number of hours past 12:00 noon. At what time between 12:00 noon to
10:00 P.M. is the traffic moving the fastest and at which time is it moving the slowest?
Solution: We need to find the absolute maximum and absolute minimum of the function S ( t ) on the
interval 0 ≤ t ≤ 10, We get
S/( t )= t2 – 16 t + 63
This S/(t) will be maximum or minimum subject to the condition S/(t)= 0.i.e. t2 – 16 t+63 =0. The
equation
402
gives ( x – 9 ) ( x – 7 ) = 0, or x = 7, x = 9. These two values lie in the interval 0 ≤ t ≤ 10. Now, for these
two values , we have
S ( 7 ) = (1/3) ( 7 )3 – 8 ( 7 )2+ 63 ( 7 ) – 125 = 38, and S( 9 ) = ( 1/3)(9)3 – 8 (9)2 +63(9) – 125 =37.
Here S(9 ) is minimum than S(7). The minimum and maximum property of the function can also be
investigated from the result of S//( t ) .where S//( t ) = 2t – 16 .
For x = 7, we see that S//( t ) = 2 x 7 – 16 =-2 <0. So, the function is maximum and the maximum speed is
38 miles after 7:00 P.M. Again, for x =9, we have S//( t ) = 2 x 9 – 16 = 2 >0. So, the function is minimum
and the minimum speed is 37 miles after 9:00 P.M.
Example 11.39: A car oves the distance S at time t , where S is a function of t and is given by
1
S (t ) t 3 1.5t 2 10t
3
Find the speed of the car after t =6 seconds . Also, find the time at which the car will stop.
dS (t ) 1 2
S / (t ) 3t 2(1.5)t 10 t 2 3t 10 (t 2)(t 5)
dt 3
The car will stop if S/(t) = 0. From this equation, we have t = -2 and t = 5. But the speed or the time
cannot be negative. So, t = 5. This implies that the car will stop after 5 seconds.
dS (t )
6 2 3 6 10 8 units per sec ond
dt 6
Example 11.40: A train moves S(t)=(3t + 0. 125 t2) distance in t seconds. What will be the speed of the
train after 5 seconds?
dS (t ) 1
S / (t ) 3 2t 3 0.25t
dt 8
Now, after 5 seconds this speed will be ( 3 + 0.25 x 5 )= 4.25 units / t seconds.
Supplementary Problems
403
Ans. ( i ) 2x – 3, ( ii ) 2 ( x +2 ), ( iii ) -12 ( 3 + 2x )-2.
Ans. ( i ) ( 2x +1 )- (1/2) , ( ii ) 2x – 3 .
Ans. At x = 2 , m = 1 and at x = 3, m = - 1 .
5. The profit in selling a product at a price x taka is given by f ( x ) = ( x – 100 ) ( 120 – x ). Find the
6. Find the value of f/(x ) by first principle of each of the following functions and find the equation of
the line that is tangent to their graphs for the specified value of x.
( i ) y = 3 x3 – 2 x2 + 3 x – 5 , ( ii ) y = 4 x4 – 3 x2 + 4. ( iii ) y = 4 x3 – 3x + 2, ( iv )y = 5 x 3 –
6x+2
dy
8. Compute dx of each of the following functions using chin rule of differentiation:
1 1 1
(c ) y , u 3 2 ; x ; Ans. 16
u x 2
404
dy
9. Find dx of each of the following functions and simplify the results.
1 10 x 6 1 4x
(c ) y ; Ans. ; (d ) y ; Ans.
5x 6 x 5
2
(5 x 6 x 5) 2
2
4x 2 1 (4 x 2 1) 3 / 2
3 24 x
( e) y ; Ans. .
(1 x 2 ) 4 (1 x 2 ) 5
10. The gross annual earnings of a certain company is shown by the equation
f ( x) 10 x 2 x 236
thousand taka t years after company‟s formation in January ,2015. At what rate is the gross annual
earnings of the company growing in January 2019? Ans. 2.025 per year.
11. It is estimated that t years from now the population of a certain community will be
6
P(t ) 12
t 1 thousand
An environmental study indicates that the average daily level of carbon monoxide in the air will be
C ( p) 0.6 p 2 2 p 24
units when the population is p thousand. At what percentage rate will be the level of carbon
monoxide be changing with respect to time2 years now? Ans.5.09% per year.
dy
12. Find dx of each of the following functions:
x2 x( x 6) 2 4x x 1
(i ) y ; Ans. , (ii ) y , Ans. , (iii ) y , Ans.
x3 ( x 3) 2
(1 x )
2
(1 x )
2 2
x 1 ( x 1) 2
x 2 3x 2
(iv ) y , Ans.1 ; (v) y x 4 4 x 3 10, Ans.4 x 3 12 x 2 4 x 2 ( x 3)
x 1 ( x 1) 2
13. Use second derivative test to find the relative maxima and minima of each of the following functions.
405
1 x2
(iii ) y x , Ans.Min(1.2); Max(1,2); (iv ) y , Ans.Min (4,8); Max (0,6).
x x2
14. In every week day during 1:00 P.M to 6:00 P.M. the speed of traffic flow in an area of a city follows
the speed function S ( t ) = t3 – 10.5 t2+30 t + 20 miles per hour , where t is the number of hours after past
noon. At what time between 1: 00 P.M. to 6:00 P.M.is the traffic moving fastest , and at what time is it
moving the slowest?
Ans. At 2:00 P.M the traffic is moving fastest with speed 46 miles per hour. The lowest speed at 5 :00
P.M. and the speed is 32.5 miles per hour.
15. Find the absolute maximum and absolute minimum of the function
16
y x2
x
16. ( a ) Let the price p and demand q of a commodity is related by the equation q = 300 – p2, where
( b ) Let the price p and demand q of a commodity be linearly related by the equation q = 240 – 2p .
( 0 ≤ p ≤ 120 ).
p
( i ) Express the elasticity of demand as a function of p. Ans . 120 p .
17.( a ) The total cost of production of q units of a certain commodity is given by C(q)= 3q2 + 4q +108.
( i ) At what level of production is the average cost per unit the smallest? Ans. q = 6.
( ii ) At what level of production is the average cost per unit equal to the marginal cost? Ans. q = 6.
( b ) A bicycle manufacturer buys 6000 tires a year from a distributor. The total cost of purchasing ,
120000
C ( x) 0.48 x 1500
x
At what value of number of tires the total cost will be minimal? Ans. x = 500.
406
( c ) A farm dealing electronic items uses 600 cases of transistors each year. The cost of storing, one
case for a year is Tk. 0.90 and ordering cost is Tk. 30.00 per order. How many cases should the
farm ordr each time to keep total cost at a minimum? [ Assume that the number of cases
ordered is reached when the previous ordered cases are finished ] Ans. 200.
11.14 Integration
We have already discussed about the integral calculus. Integral calculus deals with integration of a given
function f when this f has an anti-derivative function F. It indicated that the original function f is available
from the derivative of function F. An integral assigns numbers to functions in a way that can describe
displacement , area, volume, and other concepts that arise by combining infinitesimal data.
Given a function f of a real variable x and an interval [a, b] of the real line, the definite integral
b
a
f ( x) dx
Is defined informally as the signed area in the xy-plane that bounded by the graph of f , the x-axis and th
vertical lines, x=a and y=b. The area above the x-axis adds to the total and that below the x –axis
subtracts from the total. If
F ( x) f ( x) dx
without the specification of the values of x, then this integration is known s indefinite integral. The
technique of integration is used in doing the following aspects:
( i ) Probability theory to determine the probability of a value of random variable or the probability of a
function of random variable subject to the condition that
f ( x) dx 1
( iii ) to compute the volume of a three-dimensional object that has a curved boundary.
The integration is f two types, viz. ( a ) Definite Integral, and ( b) Indefinite Integral. The indefinite
integral is an anti-derivative of a function plus a constant. Let us consider that F and G are two ant-
derivatives of a function f. There exists a constant C such that
G( x ) = F(x ) +C
For example, let us consider that G( x)= x3 + 2 and F(x) = x3. Then, for both the functions the derivatives
are
407
d {F ( x)}
d {G ( x)} 3x 2
3 x 2 and dx
dx
But, from the anti-derivative of the first function, we can write x3+2 and from the second result we can
write x3. In both the cases, we have done the anti-derive of the function f(x) = 3 x2 . Here
f ( x) dx G( x)
Again, f ( x) dx F ( x) C
Here both the results are the anti-derivatives of the same function. But the results differ. In the second
result there is a plus constant quantity C. This means that any two anti-derivatives of the same function
must differ by a constant. If F is an anti-derivative of f, then we can write
d{F ( x)}
F / ( x) f ( x)
dx
It implies that for each value of x f(x) is the slope of the tangent to the graph of F(x). If G is another anti-
derivative of f, then also the slope of the tangent is f(x). This means that the graph G is parallel to the
graph of F. As F and G are parallel , there exists a constant C such that G(x) = F(x) + C.Let us explain the
fact for the function f(x) = 3 x2 . T here are several anti-derivatives of this function. For example, F(x) =
x3+2, F(x) = x3 , F(x)= x3 – 3 .In every case the derivative of F(x) is 3 x2. But three graphs of three
functions of F(x) are different as shown in Figure
408
Figure 4
Therefore, anti derivative of f(x) is F(x) plus a constant. Hence, we can write
f ( x) dx F ( x) C
f ( x) dx 3 x dx x 3 C
2
For example,
The symbol is called “Integral Sign” and any one needs it to find the most general form of the
anti-derivative of the function. Here f(x) is called the “Integrand” and C is the constant of integration. The
symbol dx has a role to indicate that x is the variable with respect to which the integration is to be
performed. Thus , the indefinite integral is
f ( x) dx F ( x) C
If and only if F/(x)=f(x) and it has no specific value as C is not specified. On the other hand , if F(x) is
expressed as the difference between the values of the integral at specified upper and lower limits of the
independent variable, then the integral is called “Definite “ integral. For this integral there is no C in the
final result of the integration.
Let f(x) = x n +1 . The derivative of this f(x) is ( n + 1) x n . The anti-derivative of this result is written as
x n 1
f ( x) dx
n 1
As the above case is to integrate the function without any limit, it is a case of indefinite integral. Hence,
we have the result of the integration which is given by
x n 1
x dx C.
n
n 1
xn
Again, consider f(x) = x n–1
, then f ( x ) dx
n
C.
x n 1
Also, consider that f(x) =x n – 2, then f ( x) dx
n 1
C , provided n 1
It is seen that in integrating a function having power in the independent variable x , the result is found out
by increasing the power by 1 and divide the function by the new power.
409
Rule - 2
If a function has a constant term in multiplied form with independent variable, then that constant will
x5
remain as it is. For example, let f(x) = 5 x4, then f ( x) dx 5 C x 5 C.
5
x n 1
n
If f(x) = a x , then f ( x) dx a
n 1
C.
Rule – 3
The result of integration of a constant is the independent variable x plus the positive constant C. Because
the function f(x) = a can be written as f(x) = a x1 – 1. Then
x 111
f ( x) dx a
(1 1 1)
C a x C.
Rule- 4
If a function f(x) is the sum or difference of two ( or more) functions, then the result of integration of f(x)
is the sum or difference of the results of separate integration of the functions plus a positive constant. Let
f ( x) g ( x) h( x), then
f ( x)dx g ( x)dx c h( x) dx c
1 2 g ( x) dx h( x) dx C
For example, let f( x ) = 4 x3+ 3 x2 – 2 x, then
There are number of general rules for integration of products of two or more functions and quotients of
two functions. Usually, the product or quotient can be rewritten in simple forms and then the rules of
integration discussed above can be applied. For example, let us consider a product of two functions f(x)
and g(x) as follows: g( x ) = 4 x3 + 3 x2 – 5 x and h(x) = 1 / x3 . Then , we can form another function f(x)
which is the quotient of two functions h(x) and g(x). That is
1 x3 x2 x 3 5
f ( x) 3 (4 x 3x 5 x) 4 3 3 3 5 3 4 2 4 3 x 1 5 x 2
3 2
x x x x x x
Now, the integration of f(x) can be done applying the rules discussed above. Therefore, we have
410
x111 x 21
f ( x) dx 4 x11 dx 3 x 1 dx 5 x 2 dx 4
(1 1 1)
c1 3 log x c 2 5
(2 1)
c3
5 1
=4x + 3logx + x +C. Here x dx log x is a standard result.
However, even after simplification the functional form remains in quotient form or product form. In such
a case the integration of product form can be done using the technique known as “Integration by parts”.
The procedure of integration by parts is discussed in the Section 11.15.
d g ( x)
Let f(x) = g (x ) h(x ), where h( x) dx H ( x) and dx
g / ( x) . Here f(x) is the product of two
functions g(x) and h(x). Both the functions are suitable for differentiation and integration. Then,
integration of f(x) is done according to following formula:
f ( x)dx g ( x) H ( x) g
/
( x) H ( x)dx
It indicates that when a function is the product of two functions, then the result of integration is the
[product of one function] x [ the result of integration of other function] – [integration of the result of
differentiation of the function which is kept constant] x[ the result of integration of other function]. For
example, let f(x) = ex x2, h(x ) = ex and g(x) = x2. Here
dg ( x)
H ( x) h( x)dx e x , g / ( x) 2x
dx
d ( x) x
f ( x)dx g ( x) H ( x) g ( x) H ( x)dx x 2 e x 2 xe x dx x 2 e x 2[ xe x
/
Then e dx]
dx
= x e 2[ xe e dx] x e 2 xe 2e e [ x 2 x 2] C
2 x x x 2 x x x x 2
The method deals with the integral version of „Chain Rule‟. Let us first explain the chain rule by an
example. For this ,let us consider the function f(x) = ( x2 + 3x +5)9. The derivative of this function is
d ( f ( x) d ( x 2 3x 5) 9 8 d ( x 3 x 5)
2
9 ( x 3x 5)
2
9 ( x 2 3x 5) 8 (2 x 3)
dx dx dx
411
df ( x) du
9 u 91 .
dx dx
du
g (u )
Let 9u8=g(u). Then the derivative is in a product form, where the form is dx .
We need to integrate any function which is a product as shown above. We can integrate many products of
the above form by applying the chain rule in reverse. Let G is an anti-derivative of g. Then
du
g (u) dx dx G(u) C
since by chain rule we can write
d du du
[G (u )] G / (u ) g (u )
dx dx dx
du
g (u) dx dx g (u) du
The following steps are useful in performing integration:
For example, let us find the integral value of the function 9 (x2+3x+5)8 (2x+3), i.e
du
9 (x 3x 5) 8 (2 x 3) dx g (u ) g (u)du 9u du u 9 C
2 8
,
dx
= ( x2+3x+5)8+C
2
y x 3e x
4
Solution: We need
d x4 2
f ( x) dx x e
x4 2
) 4 x 3 e x 2
4
3
dx. Here (e
dx
Thus, we have
412
1 1
f ( x)dx 4 4 x e
x4 2
dx 4 x 3 e x 2 dx
4
3
Here the integrand is in product form and 4 x3 is the derivative of (x4 +2). Therefore, we can write
2 du
g (u )
4
4 x 3e x
dx
where g(u) = eu and u = x4+2 . Hence, by the integration form of the chain rule, we can write
1 1 du 1 u 1 4
x e 4 x 3 e x 2 dx g (u )
x4 2
dx e C e x 2 C
4
3
4 4 dx 4 4 .
The integral form of the chain rule may be thought of as a technique for simplifying an integral by
changing the variable. For example, in Example 11.41 given above we changed x to u , where u = x4 +2.
In that example, the function to be integrated is
2
y f ( x) x 3 e x
4
.
du
f ( x)dx x e dx g (u )
3 x4 2
dx
Here the variable of integration x has been changed to u and the integrand is transformed to
g (u )du.
In
the expression of transformed one, the expression ( du/dx)dx in the original integral is replaced in the
simplified integral by the symbol du. Here ( du / dx)dx is in the form of quotient and it is simply du.
However, the method is suitable if the integrand is in a form of product or quotient of two functions ,
where one function is a constant multiple of the derivative of other function. For example, let us consider
the integration of the function
3x d 2
f ( x) , where ( x 1) 2 x
x 1
2
dx
Here 3 x is a constant multiple of derivative of ( x2 – 1 ) and the multiple is (3/2). Now, we can make the
transformation u = x2, then we have
3x 3 2 x dx 13 3 du 3 3
f ( x)dx ( x 2
1)
dx
2 ( x 1)
2
u2
du
2 u
ln u C ln( x 2 1) C.
2 2
413
11.15: Some Important Results of Integration
Exponential ex ex + C
Other ax ax
C
ln( a )
ln (x) x ln( x ) – x +C
Trigonometric function cos x sin x +C
sin x -cos x +C
Sec2x tan x + C
f (u ) dx 1 x dx 1 xa
(i ) F (u )dx du, (ii ) 2 tan 1 C , (iii ) 2 ln C
u /
x a 2
a a x a 2
2a x a
1 1 1 n 1
(ix ) sin 2 x dx x sin 2 x C , ( x) sin n xdx sin n 1 x cos x sin n 2 x dx,
2 4 n n
1 n 1
( xi ) cosn x dx
n
cosn 1 x sin x cosn 2 x dx. ( xii ) cos ec 2 x dx cot x C
n
1 1
cos ec x dx ln cosec x cot x C , ( xvi ) cos xdx x sin 2 x C.
2
(xv) 2 4
414
Results of Integration of Exponential and Logarithmic Functions:
ax
(i ) e x dx e x C , (ii ) a x dx C , (iii ) ln xdx x(ln x 1) C
ln a
x ea x
(iv ) log a x dx (ln x 1) C , (v) x e a x dx (ax 1) C ,
ln a a2
ea x (ax) i x 2 2x 2
(vi ) dx ln x C , (vii ) x 2 e a x dx e a x ( 2 3 ) C ,
x i 1 i i ! a a a
1 n a x n n 1 a x x n 1
(viii ) x n e a x dx x e x e dx, (ix) x n ln xdx [(n 1) ln x 1] C ,
a a (n 1) 2
ea x 1 ea x ea x
( x) dx [ a x n1 dx] C
xn (n 1) x n 1
1 2 1
(i ) y x 8 , (ii ) y
4
, (iii ) y 10, (iv ) y e 3 x , (v) y 3 x 3 , (vi ) y x , (vii ) y 4 e 4 x ,
x x x
1 1
(viii ) y e x x x , (ix) y 2 ( x 2 2 x 1), ( x) y (2 x 1) 2
2 x
Solution:
1 81 1 9 dx 1 1
(i ) x 8 dx x x C. (ii ) x x 4 dx x 41 3 C
8 1 9 4
(4 1) 3x
1 01 1
(iii ) 10 dx 10 x 0 dx 10 x 10 x C , (iv ) e 3 x dx e 3 x C ,
0 1 3
du 1 1 1 1
[Alternatively: Let u 3x dx 3, or dx 3 du, e dx 3 e du 3 e C 3 e C
3x u u 3x
1 1
2 1 1 3 1 1
(v) (3 x 3 )dx 3 x 2 dx 2 x 3 dx dx x2 2 x 31 log x C
x x x 1 3 1
1
2
415
1
1 1 3
2
x 2 4
(vi ) x dx 2
C x 2 C , (vii ) 4 e 4 x dx 4 e 4 x dx e 4 x C e 4 x C
1 3 4
1
2
3 3 3 5
1 1 1 1 1 2
(viii ) [ e x x 2 ]dx e x dx x 2 dx e x x 2 C ex x 2 C
2 2 2 3 2 5
1
2
2
1 x x 1 1
(ix ) 2 ( x 2 2 x 1)dx 2 dx 2 2 dx x 2 dx dx 2 dx x 21 C
x x x x 2 1
1
x 2 log x C
x
3x 3x 6 (ln x) 2 x
(i ) y , (ii ) y , (iii ) y , (iv ) y , (v) y (2 x 6) 5
x 1
2
[ 2 x 8 x 3]
2 x x 1
2x 4 x
(vi ) y (4 x 1), (vii ) y x c x , (viii ) y x( x 2 1) 5 , (ix ) y , ( x) y
2
,
x 1
5
x 1
6x 3 x
( xi ) y 3x 2 ( x 3 1) 5 , ( xii ) y , ( xiii ) y ( x 1) ( x 2 2 x 5)12 , ( xiv ) y .
4x 4x 1
2
( x 5) 6
Solution:
3x 3 du 3 3
(i ) dx ln x C ln x 2 1 C
x 1
2
2 u 2 2
3x 6 3( x 2)dx
(ii ) dx
[2 x 8 x 3]
2
[ 2 x 2 8 x 3]
416
It is observed that
d (2 x 2 8 x 3)
4 x 8 4( x 2)
dx
Here 3(x +2) is a constant multiple of the derivative of (2x2+ 8x +3) . This value is the other term in the
function of y. Then
du du 3
4 x 8, or du 4( x 2)dx, or ( x 2)dx , or 3( x 2)dx du
dx 4 4
3x 6
1 1 1
3 du 3 2 3 1 3 3
dx u du u 2 u2 (2 x 2 8 x 3) C
[2 x 8 x 3]
2 4 u 4 1 2 2
4( 1)
2
(ln x) 2 1 21 1 1
(iii ) dx u 2 du u C u 3 C ((ln x) 3 C
x 2 1 3 3
x dx (u 1) 1
(iv ) du du du u ln u C ( x 1) ln ( x 1) C
x 1 u u
1 5 1 1 1
(v) (2 x 6) 5 dx u du u 51 C u 6 C (2 x 6) 6 C
2 2(5 1) 12 12
1 1 3 3
1 1 1 1 1 1
(vi ) (4 x 1) dx u du u 2 du u 2 C u 2 C (4 x 1) 2 C
4 4 1 6 6
4( 1)
2
Here the transformation made is u = 4x +1. It gives du = 4dx. Therefore dx= (1/4) du.
1 u 1 1 2
(vii ) x e x dx e du e u C e x C
2
2 2 2
1 5 1 1 1 1
(viii ) x ( x 2 1) dx u du u 51 C u 6 C ( x 2 1) 6 C.
2 2 (5 1) 12 12
417
2 x4 1 1 2 2
(ix ) 5 dx 2 du ln u C ln x 5 1 C
x 1 5u 5 5
x u 1 1
( x) dx du du du u ln u C x ln ( x 1) C1
x 1 u u
1 1
( xi ) 3x 2 ( x 3 1) 5 dx. u 5 du u 51 C ( x 3 1) 6 C
(5 1) 6
6x 3 2x 1 3 du 3 3
( xii ) dx 3 2 dx ln u C ln 4 x 2 4 x 1 C
4x 4x 1
2
4x 4x 1 4 u 4 4
Therefore, ( 2 x – 1 ) dx = ( ¼) du.
1 12 1 1 1 2
( xiii ) ( x 1) ( x 2 2 x 5)12 dx u du u 121 C ( x 2 x 5)13 C
2 2 (12 1) 26
Here the transformation made is u = ( x2+2x +5).It gives du = (2x +2) dx= 2(x+1) dx.
Therefore, ( x + 1 ) dx = ( 1/ 2 ) du.
x u5 1 1
( xiv ) dx 6 du u 5 du 5 u 6 du u 51 5 u 61 C
( x 5) 6
u (5 1) (6 1)
1 1 1
C
4 ( x 5) 4
( x 5) 5
f ( x)dx g ( x) h( x) dx g ( x) H ( x) g
/
( x) H ( x)dx,
d g ( x)
g / ( x)
Here f(x) = g (x ) h(x ) is the product of two functions, where h( x) dx H ( x) and dx .
To find this value of integration of the function f(x) it needs different steps. These steps are
418
1. Select one of the factors which are in product form so that the factor is easy to differentiate and
gives simple result.
2. The other factor ( which is easier for integration ) is integrated and the integrated result is
multiplied by the factor which is selected in step – 1 for differentiation.
3. The factor selected for differentiation is to be differentiated and the result is multiplied by the
integrated result of other factor. This product is to be integrated.
4. The result obtained in step – 3 is to be subtracted from the result obtained in step – 2.
As a simple example, let us consider the integration of the product (1 – x ) ex. Let g(x) = 1 – x and
d
g / ( x) g ( x) 1, and h( x) e x so that h( x) dx e x dx e x
dx
(1 x) e x dx (1 x) e x (1) e x dx (1 x) e x e x C e x (2 x) C.
Example 11.44: Integrate the following functions in product form with respect to x.
x
(i ) y x e x , (ii ) y x c 2 x , (iii ) y x ln 2 x, (iv ) y x ( x 6), (v) y x( x 1) 8 , (vi ) y
x2
x
(vii ) y x 2 e x , (viii ) y (3 2 x) e x , (ix ) y , ( x) y x 7 ( x 4 5) 8 , ( xi ) y x 3 ( x 2 1)10
2x 1
( xii ) y x x 5
Solution:
(i ) x e x dx,
d{g ( x)}
g / ( x) , H ( x) h( x)dx. x e x dx g ( x) H ( x) g / ( x) H ( x)dx xe x e x dx
dx
xe x e x (1) C e x ( x 1) C.
(ii ) xe 2 x dx
Let g(x) = x and h(x) = e2x, then g/(x)=1 and H(x)= (1 / 2) e2x. Here
x 2x 1 2x
xe 2 x dx g ( x) H ( x) g / ( x) H ( x)dx e e dx
d{g ( x)} 2 2
g / ( x) , H ( x) h( x)dx. 2x
dx x 2x 1 1 2x e 1
e e C ( x ) C.
2 2 2 2 2
419
(iii ) x ln 2 x dx
Let g(x)=ln2 x and h(x) = x, then g/(x)=2 / 2x=1/x and H(x)=x2/2. Here
x2 1 2
x2 1 g /
( x ) H ( x ) dx ln 2 x x dx
H ( x) xdx , g ( x) x ln 2 xdx g ( x) H ( x)
/
2 2x
2 x
x2 1 x2 1
ln 2 x x 2 C (ln 2 x ) C
2 4 2 2
(iv ) x x 6dx Let g(x) = x and h(x) = (x – 6 )1/2. Here g/(x) = 1 and H(x) = (2/3) (x – 6)3/2.
3 3
2x 2
x ( x 6dx g ( x) H ( x) g / ( x) H ( x) dx
( x 6) 2 x 2 dx
3 3
2x
3
2 1
3
1 2x
3
4
5
( x 6)
2
x C
2
( x 6) ( x 6) 2 C
2
3 33 3 15
1
2
(v) x( x 1) dx
Let g(x ) =x and h(x)= (x+1)7 , then g/(x) = 1 and H(x) = (1/8)x8.
dx 1
g / ( x) 1, H ( x) ( x 1) 7 dx ( x 1) 8
dx 8
1 1 1 1
x( x 1) 7 dx g ( x) H ( x) g / ( x) ( x 1) 8 dx x ( x 1) 8 ( x 1) 81 C
8 8 8 (8 1)
1 x 1
( x 1) 8 [ x ] C.
8 9
x
(vi ) dx
x2
420
1 1
d ( x) 1 1
g / ( x) 1, H ( x) ( x 2) 2 dx ( x 2) 2
dx 1
1
2
x
dx g ( x) H ( x) g / ( x) H ( x)dx 2 x x 2 2 x 2dx
x2
1
1 1
2x x 2 2 ( x 2) 2 C
1
1
2
3
4
2 x x 2 ( x 2) 2 C
3
(vii ) x 2 e x dx
d (x2 )
g / ( x) 2 x, H ( x) e x dx e x
dx
x 2 e x dx g ( x) H ( x) g / ( x) H ( x)dx x 2 e x 2 xe x dx x 2 e x 2e x ( x 1) C
(viii ) (3 2 x) e x dx
d (3 2 x)
g / ( x) 2, H ( x) e x dx e x
dx
(3 2 x) e x dx g ( x) H ( x) g / ( x) H ( x)dx
x
(ix ) dx
2x 1
u 1
1 u 1
1 1
x 1 1
dx du du [ u du u 2 du]
2 2
2x 1 u 2 4 u 4
421
1 1
1 1 1 1 1
[ u 2 C1 u 2 C2
4 1 1
1 1
2 2
3 1 3 1
1 2 2 1 1
[ u 2 u 2 C (2 x 1) 2 (2 x 1) 2 C
4 3 1 6 2
2(2 x 1) ( x 1)
C.
6 2x 1
( x) x 7 ( x 4 5) 8 dx
Let u = x4, du = 4x3 dx; x3 dx = (1/4) du. Again, let g(u)= u, then
d {g (u ) 1 1
g / (u ) 1, h(u ) (u 5) 8 ; H (u ) h(u )du (u 5) 81 (u 5) 9
du 8 1 9
1 1
x ( x 5) dx x x ( x 5) dx 4 u (u 5) du 4 [ g (u) H (u) g (u) H (u)du
7 4 8 3 4 4 8 8 /
1 1 1 1 1 1 1
[u (u 5) 9 (u 5) 9 du] [ u (u 5) 9 (u 5) 91 ] C
4 9 9 4 9 9 9 1
1 1 (u 5) 1 1 4 ( x 4 5)
[ (u 5) {u
9
}] C [ ( x 5) {x
9 4
}] C
4 9 10 4 9 10
1 1 9x 4 5
[ ( x 4 5) 9 { }] C.
4 9 10
( xi ) x 3 ( x 2 1)10 dx
Let u = x2, then du = 2 x dx; it gives x dx = (1/2) du. Consider that g(u) =u and h(u) =( u – 1 )10.
Now
dg (u ) 1 1
g / (u ) 1, H (u ) h(u )du (u 1)101 (u 1)11
du 10 1 11
1 1
x ( x 2 1)10 dx x 2 ( x 2 1)10 x dx u (u 1)10 du [ g (u ) H (u ) g / (u ) H (u )du
3
2 2
1 1 1 1 1 1 1
[ u (u 1)11 (u 1)11 du] [ u (u 1) (u 1)111 ] C
2 11 11 2 11 11 11 1
422
1 1 1 1 1 11u 1
[ (u 1)11{u (u 1)} C [ ( x 2 1)11{ }] C
2 11 12 2 11 12
1 1 11x 2 1
[ ( x 2 1)11 ] C.
2 11 12
( xii ) x x 5 dx ,
1 1 3
dg( x) 2
Let g ( x) x and h( x) ( x 5) 2 , g / ( x) 1, H ( x) ( x 5) 2 dx ( x 5) 2
dx 3
1 3 3
2 2
x( x 5) dx g ( x) H ( x) g ( x) H ( x)dx x( x 5) 2 ( x 5) 2 dx
2 /
3 3
3x 10
3 3 3 3
2 2 1 1 2 2 2
x( x 5) 2 ( x 5) 2 C ( x 5) 2 [ x ( x 5)] C ( x 5) 2 ( )C
3 33 3 5 3 5
1
2
x3 1 x3 4 1
(i ) dx, (ii ) dx, (iii ) ( x 2 2 )dx, (iv ) x ( x 3)dx, (v) ( x 2) 3 dx.
x 1 x 2
x
Solution:
x3 1 ( x 1) ( x 2 x 1) 1 1
(i ) dx dx x 2 dx xdx dx x 3 x 2 x C
x 1 ( x 1) 3 2
x3 4 x2 1 x2 4
(ii ) dx x dx 4 x 2
dx 4 x 2 1
C C
x2 2 2 1 2 x
1 2 1 1 5 x 3
(iii ) ( x 2 ) dx ( x 4
2 ) dx x 4
dx 2
dx x 4
dx x 2 x C
x2 x4 5 3
1 5 1
x 3 2x C
5 3x
3 1 3 1 5 3
1 1 1 1 2 2
(iv ) x ( x 3)dx x 2 dx 3 x 2 dx x2 3 x2 C x 2x 2 C
3 1 5
1 1
2 2
(v) ( x 2) 3 dx
423
Let u = x – 2 , then du = dx. Therefore,
1 31 1
u du u C ( x 2) 4 C.
3
3 1 4
27 x 1 91 x 2 x 2 14 x 24 x3 x
(i ) 4 dx, (ii )
x
dx, (iii ) dx, (iv ) dx, (v) dx.
3 x
x3 x 1 x 1
Solution:
1 dy
(i ) 4 x dx. Let y 4 x , ln y x ln 4, dy ln 4 dx, or y ln 4 4 x ln 4
y dx
d 4x 4 x ln 4 4x
( c) 4 x . 4 x dx C.
dx ln 4 ln 4 ln 4
32 x 9 3 3 x 9 3 x 3 3 x
27 3 2 x dx 9 3 3 x dx 27 C [ 3 ] C.
2 ln 3 3 ln 3 ln 3 2 3
2 x 2 14 x 24 2 x 2 6 x 8 x 24 2 x( x 3) 8( x 3)
(iii ) dx dx dx (2 x 8)dx
x3 x3 x3
x2
2 xdx 8 dx 2 8 x C x( x 8) C
2
x3 x3 1 1 x3 1 1
(iv ) dx dx dx dx ( x 2 x 1)dx ln x 1
x 1 x 1 x 1 x 1
x3 x2
x 2 dx xdx dx ln x 1 x ln x 1 C.
3 2
x x 11 1
(v ) dx dx dx dx x ln x 1 C.
x 1 x 1 x 1
424
Supplementary Problems
3 3
x x
xe e 1
16. y , Ans. C , 17. y e 2 x 3 , Ans. e 2 x 3 C ,
(1 x) 2
1 x 2
3
2
18. y (2 x 3) x 3 x 1, Ans. ( x 2 3 x 1) 2 C ,
2
3
3
2
19. y 3 x 2 , Ans. (3 x 2) 2 C ,
9
3
xdx 1 2
1. 2 , Ans. C , 2. 3x 1dx, Ans. (3x 1) 2 C ,
( x 4) 3
4( x 4)
2 2
9
2
dx 3 1
3. 1
, Ans. (a bx) 3 C , 4.. (1 x 3 ) 2 dx, Ans. (1 x 3 ) 3 C ,
2b 9
(a bx) 3
3x dx 3 dx
5. 2 , Ans. ln x 2 2 C , 6. , Ans. 2 x 3 C ,
x 2 2 x3
x 2 2x
3
x2 1
7. , Ans. C , 8. 3x 2 1 2 x 3 dx, Ans. (1 2 x 3 ) 2 C ,
( x 1) 2
x 1 3
425
dx 1 ( x 1)
9. , Ans. C , 10, dx, Ans. x 2 2 x 4 C ,
( x 1) 3
2 ( x 1) 2
x 2x 4
2
1
3 x2 1
e 1 2
11. x (3 5 x)dx, Ans. 2 x (1 x) C , 12. 3 dx, Ans. e x C ,
2
x 2
1 1 1 dx 1 2x 3
13. (e x x ) 2 dx Ans. e 2 x 2 x 2 x C , 14. 2 , Ans. ln C,
e 2 2e 4x 9 12 2 x 3
dx
15. , Ans. ln( x x 2 4 ) C.
x 4
x3 1 xe x ex
(iii ) y x 2 ln x, Ans. [ ln x ] C , (iv ) y , Ans. C,
3 3 (1 x) 2 1 x
x2 2 1 x
(v ) y , Ans. (3x 2 4 x 8) 1 x C , (vi ) y 3
, Ans. 1
C,
1 x2 15
(4 x ) 2 2
4(4 x )
2 2
x(5 3 x ) 3 1 x
2 x x
1
(vii ) y , Ans. ln C , (viii ) y xe 5
, Ans. 5( x 5)e 5
C,
(1 x )
2 3
8(1 x 2 ) 2 16 1 x
3
ln x 1 1
(ix ) y 2
, Ans. (ln x 1) C , ( x) y x x 2 1, Ans. ( x 2 1) 2 C ,
x x 3
x x
x 1 1 e xe ex
( xi ) y e ( 2 ), Ans. C , ( xii ) y , Ans. C,
x x x (1 x) 2 1 x
1 1 3 x dx
( xiii ) y , Ans. C , ( xiv ) y , Ans. 3x 2 1,
(2 x 3) 2
2(2 x 3) 3x 1
2
x2 x 1 2x 5 1
( xv ) y , Ans. ln 2 x 3 C , ( xvi ) y 2 , Ans. ,
2x 3 2 4 ( x 5 x) 3
2( x 5 x) 2
2
4x 3 1
( xvi ) y , Ans. 2 x 1(4 x 3).
2x 1 3
426
(d). Integrate the following functions using the technique of substitution:
3
1 1
(i ) y 4 x 1, Ans. (4 x 1) 2 C , (ii ) y x( x 2 1) 5 , Ans. ( x 2 1) 6 C ,
6 12
4
2x 2 ln 5 x 1
(iii ) y 5 , Ans. ln x 5 1 C , (iv ) y , Ans. (ln 5 x) 2 C ,
x 1 5 x 2
3x 4 12 x 3 6 3
(v ) y , Ans. ln x 5 5 x 4 10 x 12 C ,
x 5 x 10 x 12
5 4
5
1 1 x 1
(vi ) y , Ans. C , (vii ) y , Ans. ( x 5) 4 ( x 5) 5 C ,
x(ln x) 2
ln x ( x 5) 6
4
5 3
1 4
(viii ) y (2 x 3) 2 x 1, Ans. (2 x 1) 2 (2 x 1) 2 C.
5 3
Example 11.47: It is assumed that x months from now the population of a certain area will be changing at
the rate 4+ 12 x per month. The current population of the area is 10000. What will be the population 16
months now?
Solution: Let P(x) be the population of the area x months from now. Then the rate of change of
population with respect to time is the derivative
d P( x)
4 12 x
dx
1
d P( x)
P( x) dx (4 12 x )dx 4 dx 12 x 2 dx
dx
3
2 2
4 x 12 x C
3
The current population is 10000, i.e. when x= 0, the value of P(x) is 10000. Therefore, we have
P(0) = 4 x 0 + 8 x (0)3/2+C=10000
Example 11.48: It is assumed that the customer of a bank will be changing x months from now at the rate
of (3+5x) customer per month. The present number of customers is 8000. Find the number of customers
after one year from now.
Solution: Let N(x) be the number of customers of the bank x months from now. The rate of change of
change of customer with respect to time is the derivative of N(x) with respect to x, i.e.
427
dN ( x)
3 5x
dx
Now, N(x) can be calculated from the integration of the above derivative, i.e.
dN ( x) x2
N ( x) dx (3 5 x)dx 3 dx 5 xdx 3 x 5 C
dx 2
5 2
8000 = 3 x 0 + (0) +C, it gives C=8000.
2
Example 11.49: It is assumed that the production of trousers of a company x months from now will be
changing at the rate of (10 + 2x + 3x2) per month. The present number of production of trousers in month
is 150000. Find the production of trousers after 6 months from now.
Solution: Let N(x) be the number of production of trousers of the company x months from now. The rate
of change of production with respect to time is the derivative
dN ( x)
(10 2 x 3x 2 ).
dx
dN ( x)
dx
dx (10 2 x 3x 2 )dx 10 dx 2 xdx 3 x 2 dx
=10 x + x2+x3+C
The current number of production of trousers in a month is 150000 when x =0, i.e.
C = 150000
After 6 months from now the number of production of trousers will be N(6), where
Example 11.50: A manufacturer observes that the marginal cost in producing per unit electronic
calculator when q units are produced is ( 5q2 – 82q +650 ) . The total cost of production of first 2 units is
Tk.2000.00. Find the total cost of production of first 10 units of calculator.
Solution: Let C(q) be the total cost of production of q units. Given the marginal cost of production as
428
C/(q) = 5q2– 82 q +650
5 3
C (q)dq 5 q 2 dq 82 qdq 650 dq q 41q 2 650 q C
/
Therefore, C(q)=
3
5 3
Given C(2) = 2000, 2000 = 2 41 2 2 650 2 C . It gives C= 6000 – 40 +492 – 1350= 5102.
3
5
Hence, C(q) = C(10)= (10) 3 42(10) 2 650 (10) 5102 9168 .67 .
3
Example 11.51: A retailer receives 1000 mobile phone set to sell over a 4 months period at a constant
rate of 250 sets per month. If handling cost during selling is Tk.25 per set per month, how much the
retailer will pay as selling cost with respect to time?
Solution: Let S(t) be the selling cost over t months. Since the set is sold at a constant rate of 250 sets per
month, the number of sets left in the hand after t months is 1000 – 250 t. Selling cost per unit is Tk. 25.
The rate of change of selling cost is
dS (t )
Cost per set Number of sets 25(1000 50t )
dt
Example 11.52: The marginal cost in producing per bag of cement when q bags are produced is given by
0.3q2- q +500. The total cost of production of first two bags is Tk. 1000.00. Find the total cost of
production of first 50 bags of cement.
Solution: Let C(q) be the total cost of production of q bags of cement. Given the marginal cost of
production as C/(q), where C/(q)= 0.3q2 – q +500. Hence
429
C(q)= 0.1 x 23 – 0.5 x 22+ 500 x 2 + C; or 1000 = 0.8 – 2.0 +1000 +C, Therefore, C = 1.2.
Hence C(q) = 0.1q3 – 0.5 q2+500q+C; and C(50)= 0.1 ( 50)3- 0.5 ( 50)2 +500 x 50 +1.2=36251.2
Example 11.53: The rate of circulation of a news paper from t months now is given by 200t+400.
Currently there is circulation of 50000 in a month. Find total circulation for 12 months from now.
Solution: Let C(t) be the total circulation t months now. Given C/(t) = 200t+ 400, where
50000= 100(0)2+400(0)+C
Example 11.54: The rate of change of number of containers handling t months from now is
[100 - 5 ln t 1 ] in a sea port. The current number of containers handling per month is 60000. What
will be the number of containers handling one year from now?
Solution: Let N(t) be the number of containers handling t months from now. Given the rate of change of
number of containers handling is N/(t)=100 - 5ln t 1 . Here
5
N(t) = N (t )dt 100 dt 5 ln t 1 dt 100t t 1 C
Given N(0) = 60000= 100(0) - 5+C. Here C = 60005. Therefore, number of containers handling one year
(12 months) from now will be
5
N(12) = 100 x 12 - C =1200 – 0.38 + 60005=61,204.62.
12 1
Example 11.55: The rate of change of employment of labor in garments industries t months from now is
given by [ 400 – 3 ln t 1 ]. The current employed labors are 1500000. What will be the number of
labors 6 months now?
Solution: Let N(t) be the number of employed labors t months from now. The rate of change of
employment is given by
430
3
N (t )dt 400 dt 3 ln t 1 dt 400t C . But given N(0) = 1500000
/
N(t) =
t 1
when t = 0. Therefore, 1500000 = 400 x 0 – 3 + C or C = 1500003. We need N(6), where
3
N(6) = 400 x 6 - + 1500003=15,02,402.5 15,02,403.
7
Example 11.56: The rate of change of carbon monoxide in air per month is ( 0.1t+0.1) from t months
now. If the carbon monoxide is currently 3.5 part per million, what will be the amount of carbon
monoxide in air 6 months from now.
Solution: Let A(t) be the amount of carbon monoxide in air t months from now. The rate of change of
carbon monoxide per month is given by (0.1t+ 0.1 ) from now. That is
A (t )dt 0.2 t dt 0.1 dt 0.1t 0.1t C . Given A(o) = 3.5 when t=0.
/ 2
A(t) =
Therefore, 3.5 = 0.1 x (0)2+0.1(0)+C, or C=3.5. We need A(6) when t =6. Thus , we have
Example 11.57: The marginal cost of production of a shirt when q units of shirt are produced is 6q+1
Taka. The total cost of production of first unit of shirt is Taka 150.00. Find the total cost of production of
first 10 units of shirt.
Solution: Let c(q) be the total cost of production of first q units of shirt. Given c/(q) = (6q+1). Then
Example 11.58: It is assumed that t months from now the demand of edible oil in a super market will be
increased at the rate ( 3+ 2 t2 ) kg per month. If the current demand of edible oil is 5000 kg per month,
what will be the demand after 6 months from now?
Solution: Let D(t) be the demand of edible oil t months from now. Given D/(t ) = 3 + t2. Then
2
D (t ) dt 3 dt 2 t dt 3t t 3 C
/ 2
D(t)=
3
2
Given D(0)=5000, and hence 5000 = 3 x 0 + (0)+C. It gives C= 5000. We need D(6), where
3
2 2
D(6) = 3 x 6 + (6) +5000=5042.
3
431
Example 11.59: It is assumed that the demand of smart phone t months from now will be increased at the
rate of (500 - 3et) per month. If the current demand of smart phone is 1000 in a month, what will be the
demand 6 months from now ?
Solution: Let N(t) be the number of smart phone t months from now. Given N/(t) = 500 – 3 et . Then
Given N(0) = 1000, where 1000 = 500(0) – 3 e0+C, where C= 503. We need N(6), where
Example 11.60: The resale value of a television decreases at a rate – 10000 e-0.4t Taka per year, where
change occurs with change of time t. The current price of television is Taka 30000.00. What will be its
resale value after 10 years?
Solution: Let P(t) be the price of a television t years from now. Given P/(t) = - 1000e-0.4 t. Then
10000 0.4t
P(t) = -10000 e 0.4t dt
0.4
e C = 25000 e-0.4t +C. Given P(0) = 30000, then
Supplementary Problems
1.It is estimated that t months from now the population of a certain area will be changed at the rate
2
( 4 + 5 t 3 ) people per month. Find the population of the area 8 months from now if the current population
is 10000. Ans. 10128.
2. An objective is moving so that its speed after t minutes is v(t) = 3 + 2t + 6 t2 meters per minute. How
much distance the object will cover during the second minute ? Ans. 26 meters.
3. A television producer expects that x months from now consumers will be buying f(x) = 5000 - 60 x
Television sets per month at the price p(x) = 80 + 3 x taka per television set. What is the total revenue
the producer can expect from the sale of television over the next 16 months ? Ans. 7,267,840 taka.
4.The marginal profit of a certain company is( 100 – 2q ) taka per unit when q units are produced. If the
company‟s profit is taka 700.00 when 10 units are produced, what is the company‟s maximum possible
profit ? Ans. Tk.2300.00.
432
5. The resale value of a certain machine decreases at a rate that change with time. When the machine is t
t
5
years old, the rate at which its value is changing is ( -960 e ) taka per year. If the machine was bought
new for Tk. 5000.00, how much will it be worth 10 years later ? Ans. 849.61.
6. A manufacturer has found that the marginal cost is { 0.5 ( q+1) e 0.39 ) taka per unit when q units are
produced. The total cost of production of 100 units is Tk. 200.00 . What is the total cost of production of
first 20 units? Ans. Tk.667.70.
7. The demand of an industrial product is in increasing at the rate 0.20 e0.02t in t months from now. The
current demand of the product in a month is 5000 units. What number of units will be demanded after one
year from now? Ans. 5064.
8. The price of a currently bought television is in decreasing trend at the rate ( - 1000 e-0.05t)after every t
years. If the current price of the television is Tk.25000.00, What will be the price after 25 years?
Ans.Tk.5606.53.
9. The marginal cost of production of shirts when q units of shirt is produced is given by 0.4q3 – 0.5+ 20.
If the cost of production of first 10 units of shirt is Tk.1500.00, what will be the cost of production of first
100 units of shirt ? Ans. Tk. 10,002.28.
10. The marginal cost of production of portable computer is 3q2 – 50q+20000 when q units are produced.
Find the total cost of production of 10 units of portable computer. Ans. Tk. 1,98,500.00.
11. It is assumed that the production of an industry will be changing at the rate (t + 3t2 – 5) over t years
from now. The current production is 5000000 units. What will be the number of units of production after
5 years from now ? Ans.5000112.5.
5
12. The workers in a garments industry is changing over time t . The rate of change is given by . If
t 1
the number of workers is currently 500, what will be the number of workers after 4 years ? Ans. 504.
13. The population of a slum area is increasing at the 12+3 x from x years now. The current slum
population is 10000. What will be the slum population after 10 years ? Ans. 10,183.
14. The demand of an item is decreasing over time from now. The rate of decrease is (10 – 3t). If the
current demand of the item is 500 units, what will be the demand of the item after 10 year from now ?
Ans. 455.
15. The marginal cost of getting x services from a service point is given by (x2 – 10x+ 75) . The total cost
of 5 services is Tk.1000.00. Find the total cost 0f 10 services. Ans. Tk. 1246.63.
16. The rate of cost of production of x items is ( 2x – 16 ). If the total cost of production of 10 items is
Tk.340.00, what will be the total cost of production of 20 units ? Ans. Tk.480.00.
433
17. The rate of profit in selling x units is ( - 4x +300). The total profit from 5 units is Tk.1500.00. What is
the profit in selling 20 units ? Ans. Tk.2850.00.
18. The marginal cost of buying q units of an item is ( 6 q +4). If total cost of buying 10 units is Tk.
400.00 what will be the total cost of buying 20 units ? Ans. Tk. 1340.
Let F(x) be function of x which is changing with respect to x, where the rate of change is given by
dF ( x)
f(x)=
dx
df ( x)
[ If function of x is f(x) , then the rate of change is measured by f/(x)= . For example,
dx.
df ( x) d ( x 2 2 x 1)
let f(x) = x2 – 2 x +1, then f/(x) = 2( x 1) ]. Here F(x)= 2( x – 1 ).At this
dx dx
stage any one may be interested to evaluate the change F(x) when the values of x lie between two values
a and b ( a <b ) . This change is measured by
The numerical value of above integration is the result of definite integral of the function f(x) . This
process of finding the value of integration is called definite integral. More clearly, the definite integral is
defined to be exactly the limit and summation that is seen in the result given above to find the net area
between a function and the x- axis.
1 1 0 9
1
(i ) ( x 3x 1) dx , (ii ) 6 x( x 1) dx, (iii ) (3x 3x 2 x 1) dx, (iv ) ( t
4 3 2 2 5 2
) dt ,
0 0 1 1 t
1
t 1
ln 2 2 1 1
x2 6x
(v) (e t e t ) dt , (vi ) dx, (vii ) dx , (viii ) 3 dt , (ix ) x 2 e 2 x dx
1 ( x 1) x 1
3 2 2
1 0 3 t 0
ln
2
e2
1
( x)
e
x ln x
Solution:
434
1 1 1 1
x5
(i ) ( x 4 3x 3 1)dx x 4 dx 3 x 3 dx dx 10 3 1 x 4 10 x 10 ( 1 0) 3( 1 0) (1 0)
0 0 0 0
5 4 5 4
1 3 9
1 .
5 4 20
1 2
3
(ii ) 6 x( x 2 1) 2 dx 3u 2 du u 3 12 (2 3 13 ) 7 . Here, we have considered x2+1=u, it gives
0 1
3
2x dx = du. It is seen that when x =1,u=2 and when x = 0, u=1 and hence the limit of the integration.
0 0 0 0 0
(iii ) (3x 3x 2 x 1)dx 3 x dx 3 x dx 2 xdx dx
5 2 5 2 3 6
x 01 3 x 3 01 2 x 2 01 x 01
1 1 1 1 1
6 3 2
0.5(0 1) (0 1) (0 1) (0 1) 3.5
9 1 1 9 1 9 1 3 1 3 1
2 2 2 40
(iv ) (t t )dt t dt t dt t 2 2t (9 2 1) 2(9 2 1) 26 4
2 2 2 2 9 2 9
1 1
1 1 13
3 3 3 3
ln 2 ln 2 ln 2 1 1
lnln 12 e t lnln 12
ln ln
(e e )dt e dt e dt e
t t
(v ) t t t
(e ln 2 e 2
) (e ln 2 e 2
)
1 1 1 2 2
ln ln ln
2 2 2
1 1
(2 ) (2 ) 0.
2 2
2
x2 du
(vi ) dx, Let x3+1=u, it gives 3x2 =du, Or x2dx = . Here, when x=1,u=2 and when x =2,
1 ( x 1)
3 2
3
u=9. Therefore,
9 9
du 1 2 1 1
2 3 u 2 3 2 u du 3 u 92 1 [ 1 1 ) 7
.
3 9 2 54
1
6x
(vii ) dx , Let x2+1=u, 2x dx = du. Here , when x = 1, u=2 and when x=0, u=1. Therefore, we
0 x 1
2
have
2
3du
3 ln u 1 3[ ln 2 ln 1] 3 0.6931 2.0794 .
2
1
u
435
1 1
t 1
(viii )
1 1
3 t 3 dt 3 t 2 dt t 3 dt t
1
13 1 t 2 13 (1 1 ) 1 (1 1 ) 2 .
2 3 2 9 9
1
(ix ) x 2 e 2 x dx,
0
1 2x
e dx
2x
Let g(x) = x2, then g/(x) = 2x. Consider h(x) = e2x, then H(x) = e . So, we can write
2
1 1 1
x e
2 2x
dx g ( x) H ( x)
1
0
/ 1
g ( x) H ( x)dx x e 2 x 2
2 x 1 e 2 x dx
1
0
0 0
2 0
2
1 1
1 1 1
(e 2 0) xe 2 x dx e 2 xe 2 x 10 1 e 2 x dx 1 e 2 1 (e 2 0) 1 e 2 x 10
2 0
2 2 20 2 2 4
1 2
(e 1).
4
e2
1 1
( x) x ln x dx .
e
Let u = ln x, then du=
x
dx . Here , when x = e2, then u =2 and when x = e, then u=1.
e2 2
Thus , we can write
1 du
e x ln x dx 1 u ln u 12 ln 2 ln 1 ln 2.
Example 11.62: The demand of edible oil in area is increasing from t months now at the rate ( 5 + 9 t )
ton per month. How much will be the increase in demand in the area during the next 6 months ?
Solution: Let C(t) be the demand of edible oil in the area for t months now. Given C/(t)= (5+ t ). We
need
6 6 6 3 3
C
/
(t ) dt 5 dt 9 t dt 5 t
6
o
2
9 t2
6
0 5[6 0] 6[ 6 0] 30 6 14.70 118.20 tons
2
0 0 0
3
1 3 3
2
C (t )dt (5 9 t )dt 5 dt 9 t 2 dt 5t 9 3 t 2 C 5t 6t 2 C . We need
/
[Here C(t) =
the difference of the demand of initial time to6 months time,i.e. C(6) – C(0), where
436
Example 11.63: Due to new recruitment of labors in January every year the rate of change of number of
labors for t months starting from January is given by ( 14 + 6 t ). If there are 500 labors in January of a
year, what will be the number of labors in June 30 of that year?
Solution: Let N(t) be the number of labors after t months. Given N/(t) = 14 +6 t. Here
1
N(t) = N / (t ) dt 14 dt 6 tdt 14t 6 t 2 C 14t 3t 2 C
2
So, N(0) = 500. But From the function N(t) we have N(0) = C= 500. Therefore, the number of labors in
June becomes N(6) = 192 +500=692.
[ Alternatively: The change in the number of labors from January to June can be measured by
6 6 6
This amount is the change in number of labors from January to June. From January to June totalnumber
of labors becomes (500+192 ) = 692.]
Example 11.64: The marginal cost of production of a body spray container when q units of container are
produced is given by 3 q2 – 24 q + 48. What will be the increase in total production cost if number of
containers is increased from 5 to 10?
Solution: Let C(q) be the total cost of production of q containers. Given C/(q)= 3 q2-24 q +48. We need
10 10 10 10
C (q)dq 3 q dq 24 q dq 48 dq
/ 2 3 21
q 105 24 11
q 105 48 q 105
5 5 5 5
2 1 11
=[ (10)3 – ( 5)3] – 12[( 10)2 – ( 5)2]+ 48[ 10 – 5 ]= (1000 – 125) – 12(100 – 25) + 48( 10 – 5 )
= 215.
Example 11.65: It is assumed that the demand of clothes ( in million pieces) in a community is increasing
exponentially at the rate of 10% per year. If the current demand of clothes I that community is 5 million
pieces, What will be the demand of clothes in the next 5 years ?
Solution: Let C(t) be the total demand of clothes from now up to t years. The rate of change of demand
per year is C/(t), where
C/(t) = 5 e0.1 t
5 0.1t
C (t )dt 5 e 0.1t dt e C 50 e 0.1t C .
/
Here C(t) =
0.1
437
Total demand during the next 5 years is given by C(5) – C(0), where
C(5) = 50 e0.1x5+C= 82.44, C(0) = 50 e0.1x0+C=50 +C. Therefore, C(5) – C(0) = 82.44+C – 50 – C =32.44.
C
/
(t )dt
5 0.1t
e 50 50 [e 0.5 1] 50 0.648 32.44 ]
0
0.1
Example 11.66: Marginal cost of production of q ( in 000) bags of cement is given by 15 q2 – 12 q +65.
What will be the production if the product is raised from 10 to 20 thousands?
Solution: Let C(q) be the production cost of q thousands bags of cement, where C/(q) = 15 q2 – 12 q + 65.
Therefore,
15 3 12 2
C (q) dq 15 q 2 dq 12 qdq 65 dq q q 65q C
/
C(q) =
3 2
Example 11.67: The rate of change of gross national product (GNP) of a country t years after 2012 is
given by Tk.( 3t2 +8t+120) millions. The current GNP is Tk. 20000.00 millions. What will be the GNP in
the next 5 years ?
Solution: Let G(t) be the GNP of the country t years from a particular time. Given the rate of change of
G(t) as G/(t) = 3t2+8t+120, where
G (t ) dt 3 t 2 dt 8 t dt 120 dt t 3 4t 2 120t C
/
G(t) =
The current GNP ( in 2020) is Tk.20000.00 million. After ( 2020 – 2012 )=8 years GNP is
On the assumption that the G(t) is true after 5 years of 2020, the total period of becomes (8+5) years from
2012. Thus, we need G(13),where
13 13
438
Example 11.68: The marginal profit of a company is given by ( - 40+ 2q ) if q units are produced. If the
company‟s profit is Tk. 700.00 when 10 units are produced, then (a) what is the company‟s maximum
profit ? ,(b) what is the company‟s profit if the production is increased from 10 to 40 units ?.
Solution: Let P(q) be the profit of the company if q units are produced. Given P/(q) = - 40 + 2q. Then ,
the profit P(q) is calculated from the function
P(q) = - 40 q + q2+1000 . The profit P(q) will be maximum if P/(q) = 0. It implies that [ -40 + 2q =0]. It
gives q=20. Therefore,
(a). the maximum profit of the company is P (20) = -40 x 20 + (20)2+ 1000=600.00 taka.
(b). We have P(40) = - 4o x 40 + ( 40)2+1000= 1000. We have P(10) = 700 and P(40) = 1000. Hence, the
profit of the company when production is increased form 10 to 40 is given by
Example 11.69: There are different investment plans in the economic market. A man invested money in a
plan which generates profit at the rate R1(x) = 150 + x2 for x years from now. He also invested money in
another plan of investment which generates profit R2(x) = 250 + 15 x.
(a). Which plan do you think profitable and for how many years?.
(b). Compute net excess profit if the man invests money in more profitable plan.
Solution: (a) Let us see the profitable position due to two investment plans
The second plan is initially more profitable than the first plan. But at one stage both the plans provide
same amount of profit. Thus, we can write R1(x) = R2(x), which gives
But x cannot be negative as it indicates time for investment (year). Therefore, x = 20. It is seen that the
second plan is profitable if investment is made for 20 years from now.
(b). The net profit from R2(x) after 20 years is calculated as follows:
20 20 20
0 0 0
x2 1
{15 100 x x 3 } 020 7.5(400 0) 100(20 0) 1 (8000 0) 2333 .33
2 3 3
Example 11.70: An industrialist observed that a machine when x years old generate revenue at the rate
439
R(x) = 7000 – 25 x2 taka per year and results in cost that accumulate at the rate C(x) = 3000 + 15x2 taka
per year.
(b). What are the net earnings generatedby the machine during the profitable period ?
Solution:
(a). The use of machine will be profitable as long as the rate at which the revenue generated is greater
that the rate at which the cost accumulated. If not, use of machine will not be profitable. Thus, the
machine can be used until R(x) = C(x), i.e.
(b). The rate of net earnings from the use of machine is R(x) – C (x). As use of machine will be profitable
for 10 years, the net earnings will be calculated for 10 years from now, i.e.
Net earnings =
10 10 10
0 0 0
10 10
4000 dx 40 x 2 dx [4000 x
40 3
x ] 100 4000 (10 0) 40 (10 3 0) 2666 .67taka.
0 0
3 3
Example 11.71: The consumer‟s demand function of a commodity is given by D(q) = 18 ( 30 – q2) taka
per unit of a commodity when there is a demand of q units. What will be the total cost if a consumer
wants to have 5 units of the commodity?
Solution: As D(q) is the demand function per unit of commodity, the total cost for 5 units can be
evaluated from the integration
5 5 5
Supplementary Problems
5 3 2
1 1 8 16
.1. (3x 2 x 2)dx, Ans. 144, 2. (1 2 )dx, Ans. ln 3, 3. (2 x 4) 5 dx, Ans.
2
2 1
x x 3 1
3
440
4 1 e 1
dx 4 7 x
4. , Ans. , 5. ( x 3 1) x 4 2 x 2 1 dx, Ans. . 6. dx, Ans.e
0 6x 1 3 0
6 2
x 1
e2 2 10
7. ln x dx, Ans.e 1, 8. 2
xe
x
dx, Ans. 3e 2
e ,
2
9. (20 x) e 0.1x dx, Ans.152.85
1 2 0
1 2
4 2 3
10. ( x 2 2 x)dx, Ans. , 11. (1 3x)dx, Ans. 4 2 .
0
3 1 x 2
e2
1 dx x
12. (e 3x
5 x )dx,4
Ans. (e 6 e 3 ) 31, 13. , Ans.2, 14. dx, Ans. 2 1.
1
3 1
x 1 x2
1 1
ln 2 x 4 2
e 3 x 16 dx
15. 1 e x
dx, Ans. ln ., 16.
2 1
dx, Ans. ln 81, 17.
3 3 2 x
, Ans. 3 2
0
1 x 4 0
e2
1 1 1
xe x dx 3
18. , Ans. , 19. x ln (1 2 x)dx, Ans. ln 3, 20. (3x 2 5)dx, Ans.6,
0 ( x 1)
2
2 0
8 0
x2
2 2 15
4 1
21. dx, Ans.2 ln 3, 22. x log x dx, Ans. ln 4 , 23. x 2 e 3 x dx, Ans. (5e 3 2)
0
x 1 1
3 8
29
2 3 3
dx ln 2 9
24. , Ans. , 25. (3 2 x x 2 )dx, Ans.9, 26. (1 x 2 )dx, Ans.
1 x (1 ln x )
2
1 ln 2 0 1
4
4 3 8
116 dx
27. (1 x) x dx, Ans. , 28. , Ans.2, 29. 1 3x dx, Ans.26,
1
15 0 1 x 1
2 8 11
40 xdx 98
30. x ( x 1)dx,
2 3
Ans. , 31. , Ans.6, 32 2 x 3dx, Ans. ,
0
3 4 x 15
2
3
3
2 1 4
1 4 dx
33. x e dx, 3 x2
Ans. (e 2 1), 34. (2 x x )dx,
2 3
Ans. , 35. , Ans.2.
0
2 1
3 1 x
36. It is assumed that x months from now the production of an industry will be increasing at the rate
(5 + 3x2/3) ton per month. How much amount of production will be increased over the next 8 months?
Ans. 98.
441
37. The resale value of a certain industrial machine decreases over 10- year period at a rate that changes
with time. When the machine is x years old, the rate of change is 220( x - 10 ) taka per year. How much
the machine is depreciated during the second year? Ans. Tk. 1870.00.
38. The marginal cost of production of q lots of 10 trousers is 6( q – 5 )2 taka per lot. Find the total
production cost if the lot of production is increased from 10 to 13. Ans. Tk.774.00.
39. It is estimated that the demand for an industrial item is increasing exponentially at the rate 2 percent
per year. If the current demand is 5000 units per year and if the price remains fixed at Tk.400.00 per unit,
how much revenue will the industrialist receive from the sale of the product over the next two years?
Ans.Tk. 4,081,077.00.
40. The gross national earnings of a certain company were A(t) = 0.1 t2+10 t +20 thousand taka t years
after its formation in 2010. At what rate are the gross national earnings of the company growing with
respect to time in 2020? Ans. Tk. 12.0
41. It is estimated that t years from now, the production of an industry will be Q(t) = t2 +200 t + 10000
when Q(t) was studied in 2015. (a) What will be the rate of production per unit time in 2020?
(b) What will be the percentage change of production in 2020? Ans. (a) 210, (b) 4.2%
42. The gross national product (GNP) of a certain country is N(t) = t2+5t+ 106 billion taka for t years
from the study year 2015. At what rate is the GNP changing with respect to time in 2020 ? Ans. 15
billion.
43. The resale value of a certain industrial machine decreases at a rate with respect to time t year, where
the rate is – 1000 e-0.1t taka per year. If the machine is bought new for Tk. 55000.00. What will be the
price of this machine after 10 years ? Ans. Tk. 13,678.9.
44. It is observed from a study that t months from now the demand of a product will be increasing at the
rate ( 4 + 3t0.7) unit per month. How much unit will be demanded after one year? Ans. 169.
45. It is estimated that x days from now a farmer‟s from crop will be increasing at the rate
( 0.3x2+0.6x + 1) tons per day. How much will be the value of the crop increase during the next 5 days if
the market value of the crop remain fixed at Tk. 500.00 per ton? Ans. Tk. 12,500.00.
442
Chapter XII
Linear Programming
12.1 Definition: Linear programming is a method to arrive at the best conclusion for maximum profit or
minimum or lowest cost in any field of inquiry, specially in commerce and industry. The method is so
designed that the achievement of the outcome can be expressed by a mathematical model, where the
requirements of the model are presented by the linear relationships. As mathematical model is involved in
achieving the best conclusion, the technique is also called mathematical optimization. More formally,
linear programming is a technique for the optimization of a linear objective function , subject to linear
equality and linear inequality constraints. For example, let us consider that an owner of a cyber café has
two wings to serve the C number of customers in a day, where x1 customers use internet for foreign
transaction and x2 customers use internet for local transaction. There is a restriction that x1 + x2 C
[ limit on the number of customers ]. From each customer of foreign transaction he can earn T 1 taka and
from each customer of local transaction he can earn T 2 taka. So, his total earnings will be T1x1 + T2x2 .
The owner needs to maximize his earnings. The total time limit to use the cyber café is ,say, t hours. Let
us consider that x1 type of customers have the scope to utilize t1 hours time and this time for x2 type of
customers is t2 hours. The cost of the owner per customer of x1 type is , say, p2 and this cost for x2 type of
customer is p2 such that the total cost becomes x1p1 + x2p2 p. Here x1 and x2 indicate number of
customers , these values can be not negative ,i.e. x1 0 and x2 0. For this example the mathematical
model is to be evolved so that T1x1 + T2x2 becomes maximum subjects to the restrictions
x1 + x2 C
t1x1 + t2x2 t
p1x1 + p2x2 p
x1 0 , x2 0
This model with restriction can be written in matrix notation as
1 1 C
x1 x1 x1
[ T1 T2 ] subject to
t 1 t 2 x t
where 0
x2 p1 p 2 2 p x2
It can be written as C/ X , A X b, and X 0, where
1 1 C
/
t , X x1
A= t1 t 2
C T1 T 2 , b x
p1 p 2 p 2
The problem is to maximize C/ X subject to the restrictions A X b and X 0.
The solution of the above problem is to find the values of x1 and x2 such that the sum ( T1x1 + T2x2 )
becomes maximum subject to the restrictions x1 0 and x2 0 and AX b. In the problem, there are
two unknowns x1 and x2 and 5 restrictions. All the restrictions are inequalities and they are all linear in
the sense that each involves an inequality in some linear function of the variables. Here x1 0 and
x2 0 are special type of restrictions. These are known as non-negativity restrictions ( constraints ) and
are often found in linear programming problems. The other restrictions are called main restrictions
(constraints). The function to be maximized ( minimized ) is called Objective Function . In the avobe
example the objective function is T1x1 + T2x2 .
For the standard maximization problem, we can write
b = [ b1 b2 b3 …. bm ]/ , C = [ c1 c2 c3 ….. cn ]/ and
443
a11 a12 a13 ..........a1n x1
a a 22 a 23 ..........a 2 n x
A= 21
X 2
..................................... .....
a m1 a m 2 a m 3 ..........a m n xn
Thus, we have
C/X= c1x1+ c2x2 + ………..+ cn xn subject to the restriction
ai1x1 + ai2 x2 + …………. + ain xn bj , j = 1 , 2 , ……., m and xi 0 , i = 1 , 2 , ……, n.
Here the problem is to find the values of the elements n- vector X = [ x1 x2 ……. Xn ] to maximize C/ X.
The Standard Minimum Problem:
This problem deals with the minimization of the function Y/ b = y1b1 + y2 b2 + ……….+ ym bm subject to
the restriction
ai 1 y1 + ai 2 y2 + …… + ai n yn ≥ cj ; i = 1 , 2, …., m; j = 1 , 2 , ….,n and yi ≥ 0 .
The problem is to find the values on n elements of the m – vector Y = [ y1 , y2, …….ym ]/ .
It is to be noted that for standard maximization problem the restrictions are ≤ and for the standard
minimization problem the restrictions are ≥.
Example 12.1: Solve the linear programming problem and maximize 5 x1 + 6 x2 subject to the
restrictions
x1 – x2 ≥ 3, 5 x1 + 4 x2 ≤35 and x1≥ 0, x2≥ 0.
Solution: It is simple that the maximum value will occur at the intersection of the two equations
5 x1 + 4 x2 = 35 … (a) and x1 – x2 = 3 ….. (b)
For the solution of these equations , we can use the value of x1 = 3 + x2 from (b) and replace the value of
x1 in (a). It gives
20
5 ( 3 + x2 ) + 4 x2 = 35, 9 x2 = 20 Or x2 = = 2.222
9
Hence , from the equation x1 = 3 + x2 , we have x1 = 3 + 2.222= 5.222.
Therefore, the maximum value is
5 x1 + 6 x2
Or = 5 x 5.222 + 6 x 2.222
Or = 39.442.
Example 12.2: An individual garments producer can produce shirts and trousers. Each produced shirt is
sold can earn a profit of Tk.100.00 and each trouser can earn a profit of Tk. 200.00. The producer can
work 60 hours per week and he needs 3 hours to make a shirt and 4 hours to make a trouser. On demand
of the customers, the producer can prepare at least 2 times as many shirts as trousers. Shirts need two
times as much storage space as trousers and the room is sufficient for at most 10 shirts each week.
Formulate these information as a linear programming problem.
Solution: Let xs= Number of shirts, xT = Number of trousers. Constraints for total working time is
xS
3 xS + 4 xT ≤ 60; customer‟s demand is xS ≥ 2 xT . Storage space is xT 10, x S 0, xT 0
2
Objective function is 100 xS + 200 xT .
The solution lies at the intersection of the equations
xS
xT 10 ……. (a) , and 3 xS + 4 xT = 60 ………(b)
2
x
From (a), we can write XT = 10 - S . Putting this value of xT in (b), we have
2
444
xS 20
3 xS + 4 ( 10 - ) =60. It gives xS = 20; Hence xT = 10 - =0. Now, putting the value of xS
2 2
and xT in the objective function, the profit is calculated as
100 xs + 200 xT = 100 x 20 + 200 x 0 = 2000.
Example 12.3: Solve the following linear programming problem and maximize the objective function
(x1+ x2 ) subject to the constraints
x1≥ 0, x2≥ 0 and x1 + 2 x2 ≤ 4
4x1 +2 x2 ≤ 12
x1 x 2 1
Solution: The maximum will occur at the intersection of
x1 + 2 x2= 4 ….. (i)
and 4x1 + 2 x2 = 12……(ii)
The solution of these two simultaneous equations are found out as follows:
From equation (i) , we have x1 = 4 – 2 x2 . Putting this value of x1 in equation (ii), we have
4( 4 – 2 x2) + 2x2 = 12
Or - 6 x2 = - 4
2
Therefore , x2 = . Putting the value of x2 in equation (i), we have
3
2 8
x1 = 4 – 2 ( ) =
3 3
Hence, the maximum value of the objective function is
8 2 10
x1 +x2 = .
3 3 3
As an alternative method, the problem can be sloved graphyically, where the graph is shown below:
Figure 1
When the graph is drawn the set nof points in the plane that satisfies the constraints becomes clear and it
helps to identify the points of the set which maximizes the value of the objective function. Each inequality
constraint is satisfied by a half-plane of points , and the constraint set is the intersection of all the half-
planes. The constraint set is shown below:
Here the optimal point is the intersection of the constraints
x1 + x2 = 4 and 4x1 + 2x2 = 12
445
This optimal point helps in achieving the maximum of x1 + x2 as ( x1 , x2 ) ranges over this constraint set.
The function x1 + x2 is constant on line with slope -1, such line is x1 + x2 = 1 and as we move this line
further from the origin up and to the right, the value of x1 + x2 increases. Therefore, we seek the line of
slope -1, i.e. furthest from the origin and still touches the constraint set. This occurs at the intersection of
the lines x1 + 2 x2 = 4 and 4 x1 + 2 x2 = 12.
Example 12.4: A company produces two types of loaf L1 and L2 using two different machines M1 and
M2. The processing time in producing each unit of L1 is 40 minutes when M1 is used and this time is 25
minutes when M2 is used. Each unit of L2 needs 20 minutes when the loaf is produced using M1 This time
in using M2 is 35 minutes.
In the beginning of a week there are 20 units of L1 and 100 units of L2 in the stock of the company. The
forecasted processing time in using M1 is 60 hours and the same time in using M2 is 40 hours.
The forecasted demand for L1 in the current week is 80 units and the forecasted demand for L2 is 150
units. The problem is to maximize the function L1 + L2 at the end of the week. Formulate this linear
programming problem and find the Maximum value of L1+ L2 .
Solution: The constraints are
40 L1 + 20 L2 ≤ 60 x 60 [ Time for M1 ]
25 L1 + 35 L2 ≤ 40 x 60 [ Time for M2 ]
L1 ≥ 80 – 20 = 60
L2 ≥ 150 – 100 = 50
The objective is to maximize the function L1 + ( 20 – 80 ) + L2 + (100 – 150 ), Or ( L1+L2 – 110 ).
This amount should be in stock at the end of the week. The maximum will occur at the intersection of L 1
= 60 and 40 L1 + 20 L2 = 3600. Thus, we have L1 = 60 and L2 = 60. Then the stock becomes 10.
Example 12.5: In an industry two kinds of oil O1 and O2 are produced using two types of machine M1
and M2 . If O1 is produced using M1 , it needs 50 minutes processing time . This processing time is 30
minutes when M2 is used. The processing times in producing O2 using M1 and M2 are 24 and 33 minutes,
respectively. Available processing time forecast to be 40 hours and 35 hours for M 1 and M2, respectively.
In the beginning of the week, the stock of O1 and O2 are 30 and 90 units, respectively. The expected
demand of O1 is 75 units and that is for O2 is 95 units. The owner of the industry has a plan to maximize
O1 + O2 in the stock at the end of the week.
Formulate the linear programming problem to find the maximum value of the objective function. Also,
solve this problem graphically.
Solution: According to the given information the following constraints can be formulated:
50 O1 + 24 O2 ≤ 40 x 60 [ Machine time of M1]
30 O1 + 33 O2 ≤ 35 x 60 [ Machine time of M2 ]
O1 ≥ 75 – 30 =45
O2 ≥ 95 – 90 = 5
The objective is to maximize ( O1 + 30 – 75 ) + ( O2 + 90 – 95) or O1 + O2 – 50
The maximum of O1 and O2 will occur at the intersection of the equations O1= 45 and
50 O1 + 24 O2 = 2400. It gives O1 = 45 and O2 = 6.25. The maximum of the objective function
( O1 + O2 – 50 ) = 45 + 6.25 – 50 = 1.25.
The optimum point O1 and O2 can be found out from the following graph.
446
Figure 2
Example 12.6: Solve the linear programming problem to maximize the function (20 x +30y – 10 ) under
the constraints
13 x + 19 y ≤ 2400
20 x + 29 y ≤ 2100
x ≥ 10
x, y ≥ 0
and find the maximum value.
Solution: The function will be maximum at the point x and y when this point will satisfies the
simultaneous equations
13 x + 19 y = 2400 and 20 x + 29 y = 2100
As x ≥ 10, the maximum will occur at the intersection of x = 10 20 x + 29 y =2100. It gives x = 10 and
y = 65.22. Hence the maximum value of the objective function is
(20 x + 30 y – 10) or ( 20 x 10 + 30 x 65.22 – 10 ) = 2146.6.
Example 12.7: Find the maximal and minimal values of the objective function ( 3 x + 4 y ) subject to the
constraints
x + 2y ≤ 14
3x–y≥0
x–y≤2
Solution: The restrictions are equivalent to , i.e.
x
y 7
x 2 y 14 2
3 x y 0 y 3
x y 2 y x 2
Let us draw these restrictions in the graph paper to find the corner points. The corner points will give the
maximal and minimal values of the function 3x + 4y. However, the identification of corner points is not
always clear. To avoid the problem it needs the solutions of pairs of simultaneous equations. For clear
idea of corner points, let us solve the system of linear equatuins.
447
Figure 3
x x
y=7- y=7- y = 3x
2 2
y = 3x y=x–2 y=x–2
2y = 14 – x 2 y = 14 – x 3x=x–2
Y = 3x 2y = 14 – 2 – y 2x = - 2 , or x = -1
7x = 14 , or x = 2 y=4 y=-3
Y=6 x=6 corner point is { - 1 , - 3 }
Hence { 2,6 } is a corner point {6,4 } is a corner point
According to these points, we have the 3 values of the function 3 x + 4 y . These values are
3 x 2 + 4 x 6 = 30, [ for 2.6 ]
3 x 6 + 4 x 4 = 32, [ for 6,4 ]
3 ( -1) + 4 ( - 3 ) = - 15 , [ for -1, -3 ]
It is seen that for the point ( 6,4) the function becomes maximum and for the point (- 1 , - 3 ) the function
becomes minimum.
Example 12.8: Solve the linear programming problem and maximize the function 3 x – y subject to the
restriction
x+y≥5
4x – 3 y ≤ 6 , x ≥ 0 and y≥ 0.
Solution: The maximum value of the function will be attained at the intersection point of the
simultaneous equations x + y = 5 and 4x – 3y = 6. It gives x = 5 – y. Putting this value of x in the second
equation , we have
4 ( 5 – y ) – 3 y = 6, or y = 2. Hence , x = 5 – 2 = 3.
Therefore, maximum of the function will be attained at the point {3, 2). The maximum value of the
function is (3x – y) = 3 x 3 – 2 = 7.
448
x y x y
Here (a) can be written as 1 . This inequality gives a straight line of the type 1 with
6 4 a b
coordinate (a , b) . Therefore, (a) can give a straight line equation with coordinate (6,4). In a similar way,
8
(b) can give a straight line equation with coordinate ( ,4) as (b) can be written as
3
3x y
1.
8 4
Now, let us draw these two equations as follows:
Figure 4
Both the lines intersect at a point ( 0,4 ). This point is a solution of the inequalities give in (a) and (b).
With reference to XOX/ and YOY/ as the axes of coordinates , the graph AB and CD of (a) and (b)
are drawn in the above figure and it is seen that a point ( 0, 0) not lying on (a) satisfies the inequality
2x + 3 y ≤ 12. The origin side of the line(a) including all points lying on AB is the graph or solution set
of (a) . Again, a point ( 0, 0) not lying on (b) including all points lying on CD is the graph or solution set
of (b). The common solution set or graph of (a) and (b) is shaded in the above figure.
The above technique of drawing graph is used to solve the linear programming problem for maximization
or minimization of an objective function. The maximum value of the objective function is available using
the intersection point of the lines (a) and (b)
The above mentioned technique is applied in solving the problem given in Example 12.7.
12.3 Different Areas of Applications of Linear Programming Problem (LPP):
1. Manufacturing Problem : In industrial sector the industrialist wants to maximize his / her profit by
selling industrial goods. But production of goods depend on raw materials , machine hours, labor hours,
manpower, etc. The industrialist will try to maximize his profit ( or minimize the cost ) under some
restrictions in using the different inputs.
2. Investment Problem : There are different investment schemes in the national economy . Most of the
schemes give fixed income. An investor needs a decision for the amount to be invested and in which
scheme it is to be invested so that profit becomes maximum.
449
3. Diet Problem : Every suggested diet has some constituents or nutrients. The diet producer needs to take
decision so that preparation cost of the diet becomes minimum under some restrictions. The minimum
amount of constituents may reduce the preparation cost subject to the restriction of standard quality of the
diet.
4. Transportation Problem: The main transportation problem arises in business and economics when a
producer needs to take a decision to supply the industrial products or other business items to the
consumers or wholesalers so that the transportation cost becomes minimum under the restriction of
minimum time needed for transportation and the requirement of minimum manpower.
5. Blending Problem: Many industrial products need different types of raw materials to produce a set of
items. The items are produced in such a way that optimum quantity of each of raw materials is used.
6. Selection of Media for Familiarity of Industrial Product: Every industrialist or businessman wants that
his / her product should be well known to the consumers. The familiarity of the product depends, now a
days, on mass media. Thus, a problem of allocation of advertisement in different media needs a careful
thought so that the sze of the effective number of customers becomes maximum.
Let x be the number of units of R and y be the number of units of A which are to be manufactured. The
profit will be ( 1000 x + 1500 y ) and it will be maximized under the constraints
3 x + 2 y ≤ 150 , x + 2y ≤ 120, x≥ 0 and y ≥0
We have
x y
1............(i )
50 75
x y
1...........(ii )
120 60
The graph of this LPP is shown below:
450
Figure 5
Here the optimum point is ( 52.5, 33.75) . But x and y are numbers of R and A, respectively. So, these
numbers cannot be fraction. The integer number of R can be taken as 53 and the integer number of A can
be taken as 34. Thus, the optimum point is (53,34). Therefore, the maximum profit is
1000 x + 1500 y = 1000 x 53 + 1500 x 34= 1,04,000.00.
12.5 Method of Solving Linear Programming Problem:
Any LPP may have two or more variables. The solution of LPP is done by (i) Graphical Method, and
(ii) Simplex Method. Graphical method is not suitable if the problem involves more than two variables. In
that case Simple Method is used to solve the problem. In this chapter we do not discuss the simplex
method. Here the solution of LPP is discussed for two variables problem.
The graphical method also utilizes two aspects of solution. These are (a) Corner Point Method, and (b)
Iso-profit and Iso cost method. We shall discuss only the corner point method.
Corner Point Method:
The method is based on the fundamental extreme point theorem. The different steps for this methodare as
follows:
1. Express the LPP in mathematical form if it is given in descriptive form,
2. Consider the equations given corresponding to given inequalities of all the linear constraints.
Draw the straight lines corresponding to the linear equations in one graph,
3. On the same graph paper determine the feasible region which is the region from the origin (0,0) to
the intersection of the lines and the cut point at the axes,
4. List the corner points ( i.e. the vertices) of the feasible region and find the values of x a nd y of
each corner point either by inspection of the graph or by solving the equations locating the corner,
5. Find the value of the objective function for each pair of (x , y ) values. The largest value of
objective function is the maximum value of the objective function and the smallest value is the
minimum value of the function,
6(a) If the feasible region is unbounded, then M ( the maximum value) of Z = ax + by when the open
half-plane determined by ax + by > M has no point common with the feasible region.
Otherwise, Z has no maximum value.
(b) Similarly, if M is the minimum value of z = ax + by < M , no point in common with the feasible
region. Otherwise, z has no minimum value..
Example 12.10: A dietician wishes to mix two types of food F1 and F2 in such a way that the vitamin
contents of the mixture contain at least 10 units of vitamin A and 12 units of vitamin B. Food F 1 costs Tk.
200.00/kg and F2 costs Tk. 220.00 / kg. Food F1 contains 3 units /kg of vitamin A and 5 units /kg of
451
vitamin B while food F2 contains 4 units/kg of vitamin A and 2 units/kg of vitamin B. Formulate the
problem as a linear programming problem to minimize the cost of mixture. What will be the minimum
cost?
Solution: Let the mixture contains x kg of food F1 and y kg of food F2. Then information can be shown
in the following table:
Resource Food ( in kg) Requirement ( in units)
F1(x)
F2(y)
Vitamin A ( units/kg) 3 4 10
Vitamin B ( units /kg) 5 2 12
Cost ( Taka/kg ) 200
220
Figure 6
Now, for (0,0), we have 200 x 0 + 220 x 0 = 0; for ( 0,3 ), we have 200 x 0 + 220 x 3 = 660;
for ( 2,o), we have 200 x 2 + 220 x 0 = 400; for (2,1), we have 200 x 2 + 220 x 1 = 620.
The cost will be minimum at the point ( 0,0).
Example 12.11: A linear programming model is as follows:
Maximize the objective function 22 x + 18 y subject to the constraints
3x + 2 y ≤ 48, x + y ≤ 20, x ≥0 and y ≥ 0.
452
Draw the graph of the constraints for the problem and indicate the feasible region. Also, find the
maximum value of the objective function.
Solution: From the constraints, we have
3 x + 2 y = 48 ……… (i) and x + y = 20 ………..(ii)
x y x y
From (i), we can write 1 . It gives a = 16 and b = 24,. Again, from (ii), we have 1
16 24 20 20
.
It gives a = b =20. The lines can be shown in the graph as follows:
Figure 7
Here OACD is the feasible region. The point for this region are ( 0, 0), ( 16, 0 ), ( 0, 20 ) and ( 8, 12 ).
Fr these points the objective function ( 22 x + 18 y ) gives the following values:
For ( 0, 0), we have 22 x 0 + 18 x 0 = 0
For ( 16,0), we have 22 x 16 + 18 x 0 = 352
For ( 0,20), we have 22 x 0 + 18 x 20 = 360
For ( 8,12 ), we have 22 x 8 + 18 x 12 = 392.
It is seen that the objective function becomes maximum for the point x =8 and y =12.
Example 12.12: maximize the objective function ( 5 x + 7 y ) subjects to the restrictions
3 x + 2 y ≤ 12 , 2x + 3 y ≤ 13, x > 0 and y>0.
Find the feasible region graphically for which the objective function may be maximum and minimum.
Solution: We can write the restrictions in linear equation form , where the simultaneous equations are
x y
3 x + 2 y = 12 or 1 ………….. (i)
4 6
x y
2 x + 3 y = 13 or 1 ……….(ii)
6.5 13
3
Let us draw these two straight lines to find the point of intersection and the region which will maximize
or minimize the objective function. The graph of the line is shown below:
453
Figure 8
The shaded area OAED is the feasible region for maximum. Here the coordinates of different points are
O( 0.0), A ( 4,0), D ( 0, 4 ) and E ( 2,3 ).
The values of the objective function ( 5 x + 7 y ) for these 4 points are
For (0,0), we have 5 x 0 + 7x 0 = 0, For ( 4,0), we have 5 x 4 + 7 x 0 = 20
For (0, 4), we have 5 x 0 + 7 x 4 = 28, For ( 2, 3), we have 5 x 2 + 4 x 3 = 31
Hence the objective function becomes maximum at the point ( 2,3 ).
The feasible function for minimum of the objective function is unbounded region of the point BEC. The
coordinates for these 3 points are B(0,6), E ( 2,3) and C ( 0,6.5). The values of objective function for these
3 points are
For ( 0,6) , we have 5 x 0 + 7x 6 = 42, For ( 2, 3), we have 5 x 2 + 7 x 3 = 31,
For ( 0,6.5), we have 5 x 0 + 7 x 6.5 = 45.5. The minimum of the objective function is 31 for the point
E(2,3).
Example 12.13: A shopkeeper sells only television and refrigerator. He has a capital of Tk. 3,00,000.00
to invest for this business and he has a space to accommodate 20 units of television and refrigerators. A
television costs him Tk. 25,000.00 and a refrigerator costs him Tk. 12,000.00. He can sell a television at a
profit of Tk. 1500.00 and a refrigerator at a profit of Tk. 600.00. It is assumed that whatever units he buys
those can be sold completely. Formulate the LPP problem and solve it graphically to maximize the
objective unction of profit.
Solution: Let us assume that the shopkeeper buys x units of television and y units of refrigerator. Then he
needs to maximize the objective function z = 1500 x + 600 y subject to the restrictions
25000 x + 12000 y ≤300000, x + y ≤20, x ≥0 and y ≥0
x y
The equation 25000 x + 12000 y = 300000 can be written as 1 ……………..(i)
12 25
x y
The equation x + y = 20 can be written as 1 …………….(ii)
20 20
These two lines can be shown graphically as follows:
454
Figure 9
From the figure it is seen that the feasible region is OAED and the points of these 4 bounded points are
60 200
O (0,0), A ( 12, 0), E ( , ) and D( 0, 20 ). For these points the valus of z are shown below:
13 13
For (0,0), we have z = 1500 x 0 + 600 x 0 = 0, For ( 12,0) , we have z = 1500 x 12 + 600 x 0 = 18000,
60 200 60 200
For ( , ) , we have 1500 x + 600 x = 16153.85,
13 13 13 13
For ( 0,20), we have 1500 x 0 + 600 x 20 = 12000.
It is seen that for the point (12,0) the objective function becomes maximum and it attained at horizontal
axis (12.0).
Example 12.14: A machine can produce either Product A or product B. In producing A it needs 3 units of
chemicals and 5 units of metals. The product B needs 7 units of chemicals and 5 units of metals. During
production period 150 units of chemicals and 280 units of metals are available. The profit from the sale of
the finished product A is 25 and the same from the sale of product B is 20. Formulate this LPP and find
out the maximum profit when the objective function z = 25 x + 20 y.
Solution: let x be the number of units of the product A and y be the number of units of the product B.
Then the objective [ profit function] becomes z = 25 x + 20 y under the restrictions
3 x + 5 y ≤ 150 , 7x + 5 y ≤ 280, x≥ 0 and y≥0.
Using the restrictions, we can form two simultaneous equations
x y
3 x + 5 y = 150 which gives 1 …………… (i)
50 30
x y
X + 5 y = 280 which gives 1 ……………..(ii)
40 56
These two straight lines are shown in the following figure:
455
Figure 10
The feasible region is bounded by OCEB. The coordinates of these 4 points are O(0,0), C( 40,0),
E( 32.5,10.5) and B( 0,30). The objective function takes the values for these 4 points which are as
follows:
For O (0,0) , we have z = 25 x 0 + 20 x 0 = 0 , For C( 40,0), we have z = 25 x 40 + 20 x0 = 1000
For E ( 32.5,10.5), we have z = 25 x 32.5 + 20 x 10.5= 1022.5,
For B(0,30),we have z = 25 x 0 + 20 x 30 =600.
Therefore, the profit becomes maximum for the point E ( 32.5, 10.5).
Example 12.15: A food processing unit prepares one kind of snacks using sugar, flour and oil. The
prepared snacks are of two types A and B. For A 5 units of flour, 2 units of sugar and 3 units of oil are
needed. For snacks B 3 units of flour, 4 units of sugar and one unit of oil are needed. In the very
beginning of the preparation process there are 90 units of flour, 60 units of sugar and 45 units of oil in the
stock. The snack A is sold at a price of Tk. 20 per unit and snack B is sold at a price of Tk.15 per unit.
How many units of A and B are to be prepared for maximizing revenue. Solve this LPP graphically.
Solution: Let x units of A and y units of B are to be prepared. For these two snacks the needed
components are shown below in tabular form
Snack No. of units to be Component needed Sale in
prepared Flour Sugar Oil Taka
A X 5 2 3 20
B y 3 4 1 15
Available stock 90 60 45
The objective function to be maximized is z = 20 x + 15 y, subject to the constraints
5 x + 3 y ≤ 90, 2 x + 4y ≤ 60, 3 x + y ≤ 45, x≥ 0 and y ≥0.
These restrictions can be used to find the simultaneous equations
5 x + 3 y = 90, 2 x + 4 y = 60 and 3 x + y = 45
These equations can be transformed into straight line equations, where the equations are
x y x y x y
1, 1, 1
18 30 30 15 15 45
These equations can be shown by drawing lines as follows:
456
Figure 11
The feasible region is OEGD. Th points are O( 0,0 ), E( 15,0 ), G( 12,9) and D( 0,15).
Solving the equations 2 x + 4y = 60 and 3 x + y = 45, we get the points of G( 12,9 ). Now,
For O(0,0) , z = 20 x + 15 y = 20 x 0 + 15 x 0 = 0, For E( 15,0), z = 20 x 15 + 15 x 0 = 300,
For D(0,15), z = 20 x 0 + 15 x 15 = 225, For G(12,9), z = 20 x 12 + 15 x 9 = 375.
It is seen that maximum earning is achieved at the point G(12,9).
Example 12.16 : A goldsmith has 9 gms of gold and 8 gms of silver in his stock. These amounts of
metals can be utilized to prepare two types of ornaments A and B. A requires 3 gms of silver and one gm
of gold and B requires one gm of silver and 2 gms of gold. Each unit of A ornament gives him profit of
Tk.150.00 and each unit of ornament B gives him profit of Tk.200.00. To maximize the profit how much
units of each type of ornament is to be prepared?
Solution : Let x units of gold and y units of silver are needed to prepare any type of ornament. From the
given condition the objective function is z = 150 x + 200 y subject to the restrictions
x + 2 y ≤ 8, 3 x + y ≤9, x ≥ 0 and y ≥ 0.
The different components of these restrictions are shown below:
Ornament Number of units to be Metals to be used Income(in taka)
prepared Gold
Silver
A x 1 3 150
B y 2 1 200
Available stock 8 9
The restrictions can be written in simultaneous equations from as follows:
x y x y
x + 2 y = 8, or 1 3x+y=9 or 1
8 4 3 9
The graph of the equation is shown below:
457
Figure 12
The feasible region is OCEB . The corner points are O(0,0), C( 3,0), E (2,3 ) and B ( 0,4). For these
corner points the values of the objective function are found out. These values are
For O(0,0), we have z = 150x + 200y = 150 x 0 + 200 x 0 = 0, For C(3.0), we have z= 150 x 3 + 200 x 0=
450
For E ( 2,3 ), we have z = 150 x 2 + 200 x 3 = 900, For B(0,4), we have z = 150 x 0 + 200 x 4 = 800.
It is seen that z ( max) = 900 and it is the value at the point E( 2,3),i.e. it occurs at x = 2 and y = 3.
Example 12.17: A garments industrialist has 30 units of labor and 17 units of capital which are to be
utilized to produce shirts and trousers. In the production process the shirts need 2 units of labor and 3
units of capital and I preparing trousers it needs 3 units of labor and one unit of capital. The price per unit
of shirt is Tk. 250.00 and the price per unit of trouser is Tk.300.00. Formulate this LPP and solve it
graphically to maximize the revenue.
Solution: Let x units of shirts and y units of trousers are to be prepared to maximize the revenue
x y
3 x + y = 17 or 1
17 17
3
The graph of the equations are shown below:
458
Figure 13
The feasible region is OCEB with points O( 0,0), C( 5.2, 0) E ( 3,8) and B( 0,10). For these points the
values of objective function are
For O(0,0),we have z = 250 x 0 + 300 x 0 = 0, For C ( 5.2, 0), we have z = 250x 5.2 + 300 x 0 = 1300
For E( 3,8), we have z = 250 x 3 + 300 x 8 = 3150, For B(0,10), e have z = 250 x 0 + 300 x 10 =3000.
It is seen that for the point E( 3,8) the objective function becomes maximum.
Supplementary Problems
1.a) Minimize z= 4 x + 2 y under the constraints 3 x + y ≥ 27, - x – y ≤ - 21 , x + 2 y ≥ 30, x≥ 0 and
y≥ 0.
Solve the problem graphically. Ans. Z(minimum) = 48 at the point (0,2).
b) Minimize z = 3x + 5 y under the constraints x+y ≥ 2, x + 3 y ≥3, x ≥0 and y ≥ 0. Ans. Z(min)= 7 at
the
3 1
point ( , ).
2 2
c) Maximize z = 5 x + 7 y under the constraints 3 x + 2 y ≤12, 2 x + 3 y ≤ 13, x ≥ 0 and y ≥ 0.
Ans. Z(max) = 31 at the point ( 2,3 ).
2.(i) Find graphically the feasible region of each of the given inequalities:
(a) 3 x + 8 y ≤ 24, 2 x + y ≤ 6, x ≥ 0 and y ≥ 0.
(b) 3 x + 5 y ≤ 10 , 5 x + 3 y ≤15, x ≥ 0 and y ≥ 0.
(c) 4 x + 3 y ≤14 , 3 x + 2 y ≤10, x≥ 0 and y ≥ 0.
(d) 5 x + 3 y ≤ 8 , x + y ≤ 2, x ≥0 and y ≥ 0.
(ii) Graphically find the feasible region of each of the LPP and find the maximum value of the
objective function.
(a) 3 x + 4 y ≤18, 2 x + y ≤ 7 , x ≥ 0 , y≥ 0; z = 5x + 9y; Ans. z(max) = 37.
(b) 2 x + y ≤ 5 , x + 2 y ≤ 4, x≥ 0 , y≥ 0; z = 6 x + 7 y . Ans. z ( max ) = 19.
(c) 2 x + y ≤ 80, x + 2 y ≤ 100, x ≥0 y≥ 0, z = 5x + 4y; Ans. z(max) = 260.
(d) 2 x = 3 y ≤ 600, 5 x + 2 y ≤1000, x ≥ 0, y ≥ 0; z = 11 x + 9 y ; Ans. z(max) = 2472.76.
459
3. (i) In an industry two types of cement C1 and C2 are produced. The production is processed in two
machines M1and M2. The maximum working capacity of M1 is 60 hours per week and this capacity
for M2 is 48 hours. C1 needs 4 hours for complete process and C2 needs 2 hours for complete
process. If the profit from C1 is Tk. 6.00 per bag and that is from C2 is Tk.8 per bag. Find the
number of bags of C1 and C2 to be produced per week the profit becomes maximum. Also, find the
maximum profit if the objective function is z = 6C1 + 8 C2.
(ii) An industry produces two types shaving cream A and B and sells these at a profit of Tk. 5.00 and
Tk.3.00 per unit , respectively. These creams are processed in two machines M 1 and M2. One unit of A
needs one minute processing time if M1 is used and 2 minutes processing time when M2 is used. For B the
same processing times are 1 minute and 1 minute when M1 and M2, respectively are used. The machine
M1 and M2 are available for 5 and 6 hours , respectively in a day. Find out the number of units of A and B
to be produced o that profit becomes maximum. Solve the problem graphically.
Ans. 60 ints of A and 240 units of B; Profit is Tk. 1020.00.
4. Make a graphical representation of all the constraints in the following LPP indicating the feasible
region and hence solve the problem:
(a) Minimize z = 3x + 2y subject to the restrictions 3 x + 4 y ≤ 12, x + y ≥2, x≥ 0 and y ≥ 0. Ans.
z(min)=4 at the point ( 0,2).
(b) Minimize z = 3 x – y subject to the constraints x + y ≤ 6, 2 x + y ≤ 8, 3x + 2y ≤ 6, x≥ 0 and y ≥ 0.
Ans. z(min) = - 6 at ( 0,6).
(c ) Maximize z = 2 x + 3y under the constraints 10 x + 5 y ≥ 1400, 6 x + 10 y ≥ 1400, x ≥ 0, y ≥ 0. Ans.
440.
(d ) Maximize z = 3 x + 5y subject to the restrictions 3x + 4 y ≤ 25, 2 x + y ≤10, x ≥ 0, y ≥ 0. Ans. 29.
( e ) Maximize z = 2 x + 3y subject o the restrictions 7 x + 5 y ≤31, 5 x + y ≤35, 4x + 3y ≤17, x≥ 0, y ≥
0.
Ans. 1.
5.( i ) In a ceramic industry two types of cup A and B is produced. The production process needs 3 types
of machines M1 , M2 and M3. The times ( in minutes) required by the machines for cup A and for cup
B during production process are shown below:
Types of cup Time required in Machines
M1 M2 M3
A 12 18 6
B 6 0 9
Each machine is available for a maximum period of 6 hours per day. If the profit from each cup of
type A is Tk.0.75 and that is from cup B is Tk.0.50, show that 15 cups of type A and 30 cups of type
B
should be manufactured to get the maximum profit. Find the maximum profit. Ans. z(max)=
Tk.26.20
(ii) In a refinery two types crude edible oil A and B are processed using two types of chemical C1 and
C2.
For edible oil A 2 units of C1 and one unit of C2 are used. For B one unit of C1 and 2 units of C2 are
used. For the production process the available units of C1 are 800 and the units of C2 are 1000. On
sell the profits from C1 and C2 become Tk. 30.00 and Tk.20.00 per unit respectively. Formulate the
LPP to maximize the profit. Also, find out the maximum profit.
Ans. 2 x + y ≤ 800, x + 2y ≤1000, x ≥ 0, y ≥ 0; z = 30 x + 20 y , z(max) = 14000 at the point
(200,400 ).
A retail seller sells two types wall painting A and B. He has Tk. 50000.00 to invest and has a space
toStore 60 pieces of wall painting. One unit of A costs him Tk. 2500.00 and one unite of B costs him
Tk.500.00 . The net profit from the sale of painting A is Tk. 500.00 and that from the sale of B is
Tk.150.00. If he can sell all the paintings what he purchases , how should he invest his capital to have
a maximum profit ? Ans. Maximize z = 500 x + 150 y subject to the restrictions 2500 x + 500 y
≤50000 , x + y ≤ 60, x≥ 0 and y≥ 0;z(max) = Tk.12,500.00 and it is attained at the point (10,15).
460
(iv) A baker has 90, 80, and 20 units of flour, sugar and oil to produce two types of cake A and B. A will
be sold at Tk. 25 per unit and B will be sold at Tk. 20 per unit. A needs 2 units of flour, 1 unit of sugar
and I unit of oil. For B the needs for these items are 1, 2, and 1 unit, respectively. Formulate the LPP
to maximize the revenue of the baker. What is the maximum revenue?
Ans. The objective function is z = 25 x + 20 y subject to the restrictions 2 x + y ≤90, x + 2y ≤ 80 and
x + y ≤ 50, x ≥ 0 and y ≥ 0; z(max) = 1200.
(v) A shoe manufacturer produces two types of shoe A and B. The shoe is manufactured using lather,
which is processed and then cut and assembled. The time needed for processing , cutting and
assembling for A are2, 1 , 1, respectively and for B these times are 3, 1, 3. The selling price of A is
Tk. 2000.00 and that for B is Tk. 2500.00. Suggest the LPP to maximize the revenue. The times
available for these works are 300, 300 and 240 hours, respectively.
Ans. The objective function is z = 2 x = 3y ≤ 300, x + y ≤300, x + 3y ≤ 240, x ≥ 0 and y ≥ 0;
Z(max) = 27,000.00 at the point ( 60 , 60).
(vi) One bread of a baker is made of bran and rice that contains at least 88 grams of protein and at least
36 milligrams of iron. It is known that bran contains 80 grams of protein and 40 milligrams of iron
per kilogram , and that rice bran contains 100 grams of protein and 30 milligrams of iron per
kilogram . Find the maximum cost of production of this new bread if bran costs Tk. 50.00 per
kilogram and iron costs Tk. 40.00 per grams.
Ans. 20 x + 25 y ≥22, 20 x + 15 y ≥ 18 , x ≥ 0, and y≥ 0. The objective function is z = 50 x + 40y.
(vii) A footpath shopkeeper sits with a table to sell pants and shirts. In his table he can keep 60 pieces of
items. A pant costs him tk. 400.00 and a shirt costs him Tk.250.00. On sale he can earn profit of
Tk. 100.00 from a pant and Tk. 75.00 from a shirt. He has a capital of Tk. 15000.00 for investment.
Formulate this LPP to maximize his profit.
Ans. 8 x + 5 y ≤ 300, x + y ≤60, x ≥0 and y ≥ 0; z = 100x + 75 y. z(max) =Tk.4250.00
(viii) In preparing two types of belts A and B of synthetic leather it needs 30 minutes to cut the leather
for belt of type A and 10 minutes to stich the belt. These time for belt B are 20 and 6 minutes,
respectively. The available total cutting time is 8 hours and total available time to stich is 2.5
hours. In selling a belt it makes a profit of Tk. 20 from belt A and Tk. 15 from belt B. Find the
maximum profit. Ans. ( 6,15 ); Tk.345.
(ix) Two types of loaf A and B are manufactured in a bakery. For loaf A it needs 200gms flour and 30
gms of Sugar. For loaf B it needs 100gms flour and 20 gms sugar. The owner of the bakery has 6
kg flour and 1 kg sugar. The owner makes a profit of Tk. 5.00 in selling loaf A and TK.4.00 in
selling loaf B. Find the maximum profit of the baker. Ans. ( 20,20); Tk. 180.00.
(x) A person makes a plan to send emails to his friends outside the country and within the country
from a cyber café. He can type a mail for foreign country within 5 minutes and can send it within 5
seconds. These times for local emails are 10 minutes and 5 seconds, respectively. He has a time
limit of 20 minute to type the email and 15 seconds to send the mail. For typing the email he needs
to pay Tk.20.00 per minute and Tk. 5.00 per mail for sending the email. How much money he needs
to pay for sending the email. Ans. ( 2,1); Tk. 45.00
(xi) A farmer has 4 acres of cultivable land. He utilizes 3 land for rice cultivation and jute cultivation in
the same season. He utilizes urea 60 kg/acre for rice cultivation and 40 kg per acre urea in cultivating
jute. He has 180 kg urea in his stock. He makes a profit of Tk. 500.00 / acre
rice cultivation and Tk. 200. 00/acre from jute cultivation. What will be his maximum profit in a
season ? Ans. ( 1,3); Tk. 1100.00.
(xii) A hawker sells underwear and shocks putting the items in table which can contain 45 pieces of two
Items. The buying price of an underwear is Tk.40.00 and that of a pair of shocks is Tk. 25.00. The
capital of the hawker is Tk. 1500.00. His profit from an underwear is Tk.10.00 and that from a pair of
shocks is Tk.5.00. For maximum profit how much pieces of underwear and how many pairs of
shocks the hawker should buy ? Ans. ( 25, 20 ); Tk. 350.00.
461
Bibliography
462